Remedies of Gov'T-statute of Limitations

Published on July 2016 | Categories: Documents | Downloads: 29 | Comments: 0 | Views: 765
of 194
Download PDF   Embed   Report

Comments

Content

Republic of the Philippines SUPREME COURT Manila THIRD DIVISION

G.R. No. 128315 June 29, 1999 COMMISSIONER OF INTERNAL REVENUE, petitioner, vs. PASCOR REALTY AND DEVELOPMENT CORPORATION, ROGELIO A. DIO and VIRGINIA S. DIO, respondents.

PANGANIBAN, J.: An assessment contains not only a computation of tax liabilities, but also a demand for payment within a prescribed period. It also signals the time when penalties and protests begin to accrue against the taxpayer. To enable the taxpayer to determine his remedies thereon, due process requires that it must be served on and received by the taxpayer. Accordingly, an affidavit, which was executed by revenue officers stating the tax liabilities of a taxpayer and attached to a criminal complaint for tax evasion, cannot be deemed an assessment that can be questioned before the Court of Tax Appeals. Statement of the Case Before this Court is a Petition for Review on Certiorari under Rule 45 of the Rules of Court praying for the nullification of the October 30, 1996 Decision 1 of the Court of Appeals 2 in CA-GR SP No. 40853, which effectively affirmed the January 25, 1996 Resolution 3 of the Court of Tax Appeals 4 CTA Case No. 5271. The CTA disposed as follows: WHEREFORE, finding [the herein petitioner's] "Motion to Dismiss" as UNMERITORIOUS, the same is hereby DENIED. [The CIR] is hereby given a period of thirty (30) days from receipt hereof to file her answer. Petitioner also seeks to nullify the February 13, 1997 Resolution 5 of the Court of Appeals denying reconsideration. The Facts As found by the Court of Appeals, the undisputed facts of the case are as follows: It appears that by virtue of Letter of Authority No. 001198, then BIR Commissioner Jose U. Ong authorized Revenue Officers Thomas T. Que, Sonia T. Estorco and Emmanuel M. Savellano to examine the books of accounts and other accounting records of Pascor Realty and Development Corporation. (PRDC) for the years ending 1986, 1987 and 1988. The said examination resulted in a recommendation for

the issuance of an assessment in the amounts of P7,498,434.65 and P3,015,236.35 for the years 1986 and 1987, respectively. On March 1, 1995, the Commissioner of Internal Revenue filed a criminal complaint before the Department of Justice against the PRDC, its President Rogelio A. Dio, and its Treasurer Virginia S. Dio, alleging evasion of taxes in the total amount of P10,513,671 .00. Private respondents PRDC, et.al. filed an Urgent Request for Reconsideration/Reinvestigation disputing the tax assessment and tax liability. On March 23, 1995, private respondents received a subpoena from the DOJ in connection with the criminal complaint filed by the Commissioner of Internal Revenue (BIR) against them.
1âwphi1.nêt

In a letter dated May 17, 1995, the CIR denied the urgent request for reconsideration/reinvestigation of the private respondents on the ground that no formal assessment of the has as yet been issued by the Commissioner. Private respondents then elevated the Decision of the CIR dated May 17, 1995 to the Court of Tax Appeals on a petition for review docketed as CTA Case No. 5271 on July 21, 1995. On September 6, 1995, the CIR filed a Motion to Dismiss the petition on the ground that the CTA has no jurisdiction over the subject matter of the petition, as there was no formal assessment issued against the petitioners. The CTA denied the said motion to dismiss in a Resolution dated January 25, 1996 and ordered the CIR to file an answer within thirty (30) days from receipt of said resolution. The CIR received the resolution on January 31, 1996 but did not file an answer nor did she move to reconsider the resolution. Instead, the CIR filed this petition on June 7, 1996, alleging as grounds that: Respondent Court of Tax Appeals acted with grave abuse of discretion and without jurisdiction in considering the affidavit/report of the revenue officer and the indorsement of said report to the secretary of justice as assessment which may be appealed to the Court of Tax Appeals; Respondent Court Tax Appeals acted with grave abuse of discretion in considering the denial by petitioner of private respondents' Motion for Reconsideration as [a] final decision which may be appealed to the Court of Tax Appeals. In denying the motion to dismiss filed by the CIR, the Court of Tax Appeals stated: We agree with petitioners' contentions, that the criminal complaint for tax evasion is the assessment issued, and that the letter denial of May 17, 1995 is the decision properly appealable to [u]s. Respondent's ground of denial, therefore, that there was no formal assessment issued, is untenable. It is the Court's honest belief, that the criminal case for tax evasion is already anassessment. The complaint, more particularly, the Joint Affidavit of Revenue Examiners Lagmay and Savellano attached thereto, contains the details of the assessment like the kind and amount of tax due, and the period covered:

Petitioners are right, in claiming that the provisions of Republic Act No. 1125, relating to exclusive appellate jurisdiction of this Court, do not, make any mention of "formal assessment." The law merely states, that this Court has exclusive appellate jurisdiction over decisions of the Commissioner of Internal Revenue on disputed assessments, and other matters arising under the National Internal Revenue Code, other law or part administered by the Bureau of Internal Revenue Code. As far as this Court is concerned, the amount and kind of tax due, and the period covered, are sufficient details needed for an "assessment." These details are more than complete, compared to the following definitions of the term as quoted hereunder. Thus: Assessment is laying a tax. Johnson City v. Clinchfield R. Co., 43 S.W. (2d) 386, 387, 163 Tenn. 332. (Words and Phrases, Permanent Edition, Vol. 4, p. 446). The word assessment when used in connection with taxation, may have more than one meaning. The ultimate purpose of an assessment to such a connection is to ascertain the amount that each taxpayer is to pay. More commonly, the word "assessment" means the official valuation of a taxpayer's property for purpose of taxation. State v. New York, N.H. and H.R. Co. 22 A. 765, 768, 60 Conn. 326, 325. (Ibid. p. 445)
From the above, it can be gleaned that an assessment simply states how much tax is due from a taxpayer. Thus, based on these definitions, the details of the tax as given in the Joint Affidavit of respondent's examiners, which was attached to the tax evasion complaint, more than suffice to qualify as an assessment. Therefore, this assessment having been disputed by petitioners, and there being a denial of their letter disputing such assessment, this Court unquestionably acquired jurisdiction over the instant petition for review. 6

As earlier observed, the Court of Appeals sustained the CTA and dismissed the petition. Hence, this recourse to this Court. 7 Ruling of the Court of Appeals The Court of Appeals held that the tax court committed no grave abuse of discretion in ruling that the Criminal Complaint for tax evasion filed by the Commissioner of Internal Revenue with the Department of Justice constituted an "assessment" of the tax due, and that the said assessment could be the subject of a protest. By definition, an assessment is simply the statement of the details and the amount of tax due from a taxpayer. Based on this definition, the details of the tax contained in the BIR examiners' Joint Affidavit, 8 which was attached to the criminal Complaint, constituted an assessment. Since the assailed Order of the CTA was merely interlocutory and devoid of grave abuse of discretion, a petition for certiorari did not lie. Issues Petitioners submit for the consideration of this Court following issues: (1) Whether or not the criminal complaint for tax evasion can be construed as an assessment.

(2) Whether or not an assessment is necessary before criminal charges for tax evasion may be instituted.
(3) Whether or not the CTA can take cognizance of the case in the absence of an assessment. 9

In the main, the Court will resolve whether the revenue officers' Affidavit-Report, which was attached to criminal revenue Complaint filed the Department of Justice, constituted an assessment that could be questioned before the Court of Tax Appeals. The Court's Ruling The petition is meritorious. Main Issue: Assessment Petitioner argues that the filing of the criminal complaint with the Department of Justice cannot in any way be construed as a formal assessment of private respondents' tax liabilities. This position is based on Section 205 of the National Internal Revenue Code 10 (NIRC), which provides that remedies for the collection of deficient taxes may be by either civil or criminal action. Likewise, petitioner cites Section 223(a) of the same Code, which states that in case of failure to file a return, the tax may be assessed or a proceeding in court may be begun without assessment. Respondents, on the other hand, maintain that an assessment is not an action or proceeding for the collection of taxes, but merely a notice that the amount stated therein is due as tax and that the taxpayer is required to pay the same. Thus, qualifying as an assessment was the BIR examiners' Joint Affidavit, which contained the details of the supposed taxes due from respondent for taxable years ending 1987 and 1988, and which was attached to the tax evasion Complaint filed with the DOJ. Consequently, the denial by the BIR of private respondents' request for reinvestigation of the disputed assessment is properly appealable to the CTA. We agree with petitioner. Neither the NIRC nor the regulations governing the protest of assessments 11 provide a specific definition or form of an assessment. However, the NIRC defines the specific functions and effects of an assessment. To consider the affidavit attached to the Complaint as a proper assessment is to subvert the nature of an assessment and to set a bad precedent that will prejudice innocent taxpayers. True, as pointed out by the private respondents, an assessment informs the taxpayer that he or she has tax liabilities. But not all documents coming from the BIR containing a computation of the tax liability can be deemed assessments. To start with, an assessment must be sent to and received by a taxpayer, and must demand payment of the taxes described therein within a specific period. Thus, the NIRC imposes a 25 percent penalty, in addition to the tax due, in case the taxpayer fails to pay deficiency tax within the time prescribed for its payment in the notice of assessment. Likewise, an interest of 20 percent per annum, or such higher rates as may be prescribed by rules and regulations, is to be collected form the date prescribed for its payment until the full payment. 12 The issuance of an assessment is vital in determining, the period of limitation regarding its proper issuance and the period within which to protest it. Section 203 13 of the NIRC provides that internal revenue taxes must be assessed within three years from the last day within which to file the return. Section 222, 14 on the other hand, specifies a period of ten years in case a fraudulent return with

intent to evade was submitted or in case of failure to file a return. Also, Section 228 15 of the same law states that said assessment may be protested only within thirty days from receipt thereof. Necessarily, the taxpayer must be certain that a specific document constitutes an assessment. Otherwise, confusion would arise regarding the period within which to make an assessment or to protest the same, or whether interest and penalty may accrue thereon. It should also be stressed that the said document is a notice duly sent to the taxpayer. Indeed, an assessment is deemed made only when the collector of internal revenue releases, mails or sends such notice to the taxpayer. 16 In the present case, the revenue officers' Affidavit merely contained a computation of respondents' tax liability. It did not state a demand or a period for payment. Worse, it was addressed to the justice secretary, not to the taxpayers. Respondents maintain that an assessment, in relation to taxation, is simply understood' to mean:
A notice to the effect that the amount therein stated is due as tax and a demand for payment thereof. 17 Fixes the liability of the taxpayer and ascertains the facts and furnishes the data for the proper presentation of tax rolls. 18

Even these definitions fail to advance private respondents' case. That the BIR examiners' Joint Affidavit attached to the Criminal Complaint contained some details of the tax liabilities of private respondents does not ipso factomake it an assessment. The purpose of the Joint Affidavit was merely to support and substantiate the Criminal Complaint for tax evasion. Clearly, it was not meant to be a notice of the tax due and a demand to the private respondents for payment thereof. The fact that the Complaint itself was specifically directed and sent to the Department of Justice and not to private respondents shows that the intent of the commissioner was to file a criminal complaint for tax evasion, not to issue an assessment. Although the revenue officers recommended the issuance of an assessment, the commissioner opted instead to file a criminal case for tax evasion. What private respondents received was a notice from the DOJ that a criminal case for tax evasion had been filed against them, not a notice that the Bureau of Internal Revenue had made an assessment. In addition, what private respondents sent to the commissioner was a motion for a reconsideration of the tax evasion charges filed, not of an assessment, as shown thus: This is to request for reconsideration of the tax evasion charges against my client, PASCOR Realty and Development Corporation and for the same to be referred to the Appellate Division in order to give my client the opportunity of a fair and objective hearing. 19 Additional Issues: Assessment Not Necessary Before Filing of Criminal Complaint

Private respondents maintain that the filing of a criminal complaint must be preceded by an assessment. This is incorrect, because Section 222 of the NIRC specifically states that in cases where a false or fraudulent return is submitted or in cases of failure to file a return such as this case, proceedings in court may be commenced without an assessment. Furthermore, Section 205 of the same Code clearly mandates that the civil and criminal aspects of the case may be pursued simultaneously. In Ungab v. Cusi, 20 petitioner therein sought the dismissal of the criminal Complaints for being premature, since his protest to the CTA had not yet been resolved. The Court held that such protests could not stop or suspend the criminal action which was independent of the resolution of the protest in the CTA. This was because the commissioner of internal revenue had, in such tax evasion cases, discretion on whether to issue an assessment or to file a criminal case against the taxpayer or to do both. Private respondents insist that Section 222 should be read in relation to Section 255 of the NLRC, 21 which penalizes failure to file a return. They add that a tax assessment should precede a criminal indictment. We disagree. To reiterate, said Section 222 states that an assessment is not necessary before a criminal charge can be filed. This is the general rule. Private respondents failed to show that they are entitled to an exception. Moreover, the criminal charge need only be supported by a prima facie showing of failure to file a required return. This fact need not be proven by an assessment. The issuance of an assessment must be distinguished from the filing of a complaint. Before an assessment is issued, there is, by practice, a pre-assessment notice sent to the taxpayer. The taxpayer is then given a chance to submit position papers and documents to prove that the assessment is unwarranted. If the commissioner is unsatisfied, an assessment signed by him or her is then sent to the taxpayer informing the latter specifically and clearly that an assessment has been made against him or her. In contrast, the criminal charge need not go through all these. The criminal charge is filed directly with the DOJ. Thereafter, the taxpayer is notified that a criminal case had been filed against him, not that the commissioner has issued an assessment. It must be stressed that a criminal complaint is instituted not to demand payment, but to penalize the taxpayer for violation of the Tax Code. WHEREFORE, the petition is hereby GRANTED. The assailed Decision is REVERSED and SET ASIDE. CTA Case No. 5271 is likewise DISMISSED. No costs. SO ORDERED. Vitug, Purisima and Gonzaga-Reyes, JJ., concur. Romero, J., abroad on official business. Republic of the Philippines SUPREME COURT Manila SECOND DIVISION G.R. No. 81446 August 18, 1988 BONIFACIA SY PO, petitioner, vs. HONORABLE COURT OF TAX APPEALS AND HONORABLE COMMISSIONER OF INTERNAL REVENUE,respondents.

Basilio E. Duaban for petitioner.

SARMIENTO, J.: This is an appeal from the decision 1 of the respondent Court of Tax Appeals, dated September 30,1987, which affirmed an earlier decision of the correspondent Commissioner of Internal Revenue in assessment letters dated August 16, 1972 and September 26, 1972, which ordered the payment by the petitioner of deficiency income tax for 1966 to 1970 in the amount of P7,154,685.16 and deficiency specific tax for January 2, 1964 to January 19, 1972, in the amount of P5,595,003.68. We adopt the respondent court's finding of facts, to wit: Petitioner is the widow of the late Mr. Po Bien Sing who died on September 7, 1980. In the taxable years 1964 to 1972, the deceased Po Bien Sing was the sole proprietor of Silver Cup Wine Factory (Silver Cup for brevity), Talisay, Cebu. He was engaged in the business of manufacture and sale of compounded liquors, using alcohol and other ingredients as raw materials. On the basis of a denunciation against Silver Cup allegedly "for tax evasion amounting to millions of pesos" the then Secretary of Finance Cesar Virata directed the Finance-BIR--NBI team constituted under Finance Department Order No. 13-70 dated February 19, 1971 (Exh- 3, pp. 532-553, Folder II, BIR rec.) to conduct the corresponding investigation in a memorandum dated April 2, 1971 (p. 528, Folder II, BIR rec.). Accordingly, a letter and a subpoena duces tecum dated April 13,1971 and May 3,1971, respectively, were issued against Silver Cup requesting production of the accounting records and other related documents for the examination of the team. (Exh. 11, pp. 525-526, Folder II, BIR rec.). Mr. Po Bien Sing did not produce his books of accounts as requested (Affidavit dated December 24, 1971 of Mr. Generoso. Quinain of the team, p. 525, Folder H, BIR rec.). This prompted the team with the assistance of the PC Company, Cebu City, to enter the factory bodega of Silver Cup and seized different brands, consisting of 1,555 cases of alcohol products. (Exh. 22, Memorandum Report of the Team dated June 5, 1971, pp. 491-492, Folder II, BIR rec.). The inventory lists of the seized alcohol products are contained in Volumes I, II, III, IV and V (Exhibits 14, 15, 16, 17, and 18, respectively, BIR rec.). On the basis of the team's report of investigation, the respondent Commissioner of Internal Revenue assessed Mr. Po Bien Sing deficiency income tax for 1966 to 1970 in the amount of P7,154,685.16 (Exh. 6 pp. 17-19, Folder I, BIR rec.) and for deficiency specific tax for January 2,1964 to January 19, 1972 in the amount of P5,595,003.68 (Exh. 8, p. 107, Folder I, BIR rec.). Petitioner protested the deficiency assessments through letters dated October 9 and October 30, 1972 (Exhs. 7 and 9, pp. 27-28; pp. 152-159, respectively, BIR rec.), which protests were referred for reinvestigation. The corresponding report dated August 13, 1981 (Exh. 1 0, pp. 355, Folder I, BIR rec.) recommended the reiteration of the assessments in view of the taxpayer's persistent failure to present the books of accounts for examination (Exh. 8, p. 107, Folder I, BIR rec.), compelling respondent to issue warrants of distraint and levy on September 10, 1981 (Exh. 11, p. 361, Folder I, BIR rec.).

The warrants were admittedly received by petitioner on October 14, 1981 (Par. IX, Petition; admitted par. 2, Answer), which petitioner deemed respondent's decision denying her protest on the subject assessments. Hence, petitioner's appeal on October 29,1981. 2

The petitioner assigns the following errors: I RESPONDENT INTENTIONALLY ERRED IN HOLDING THAT PETITIONER HAS NOT PRESENTED ANY EVIDENCE OF RELEVANCE AND COMPETENCE REQUIRED TO BASH THE TROUBLING DISCREPANCIES AND SQUARE THE ISSUE OF ILLEGALITY POSITED ON THE SUBJECT ASSESSMENTS. II RESPONDENT COURT OF TAX APPEALS PALPABLY ERRED IN DECIDING THE CASE IN A WAY CONTRARY TO THE DOCTRINES ALREADY LAID DOWN BY THIS COURT. III RESPONDENT COURT OF TAX APPEALS GRAVELY ERRED IN FINDING PO BEEN SING TO HAVE INCURRED THE ALLEGED DEFICIENCY TAXES IN QUESTION. 3 We affirm. Settled is the rule that the factual findings of the Court of Tax Appeals are binding upon this Honorable Court and can only be disturbed on appeal if not supported by substantial evidence. 4 The assignments of errors boils down to a single issue previously raised before the respondent Court, i.e., whether or not the assessments have valid and legal bases. The applicable legal provision is Section 16(b) of the National Internal Revenue Code of 1977 as amended. It reads: Sec. 16. Power of the Commissioner of Internal Revenue to make assessments.² xxx xxx xxx (b) Failure to submit required returns, statements, reports and other documents. When a report required by law as a basis for the assessment of an national internal revenue tax shall not be forthcoming within the time fixed by law or regulation or when there is reason to believe that any such report is false, incomplete, or erroneous, the Commissioner of Internal Revenue shall assess the proper tax on the best evidence obtainable. In case a person fails to file a required return or other document at the time prescribed by law, or willfully or otherwise, files a false or fraudulent return or other documents, the Commissioner shall make or amend the return from his own knowledge and from such information as he can obtain through testimony or otherwise, which shall be prima facie correct and sufficient for all legal purposes.

The law is specific and clear. The rule on the "best evidence obtainable" applies when a tax report required by law for the purpose of assessment is not available or when the tax report is incomplete or fraudulent. In the instant case, the persistent failure of the late Po Bien Sing and the herein petitioner to present their books of accounts for examination for the taxable years involved left the Commissioner of Internal Revenue no other legal option except to resort to the power conferred upon him under Section 16 of the Tax Code. The tax figures arrived at by the Commissioner of Internal Revenue are by no means arbitrary. We reproduce the respondent court's findings, to wit: As thus shown, on the basis of the quantity of bottles of wines seized during the raid and the sworn statements of former employees Messrs. Nelson S. Po and Alfonso Po taken on May 26, and 27,1971, respectively, by the investigating team in Cebu City (Exhs. 4 and 5, pp. 514-517, pp. 511-513, Folder 11, BIR rec.), it was ascertained that the Silver Cup for the years 1964 to 1970, inclusive, utilized and consumed in the manufacture of compounded liquours and other products 20,105 drums of alcohol as raw materials 81,288,787 proof liters of alcohol. As determined, the total specific tax liability of the taxpayer for 1964 to 1971 amounted to P5,593,003.68 (Exh. E, petition, p. 10, CTA rec.) Likewise, the team found due from Silver Cup deficiency income taxes for the years 1966 to 1970 inclusive in the aggregate sum of P7,154,685.16, as follows: 1966 P207,636.24 1967 645,335.04 1968 1,683,588.48 1969 1,589,622.48 1970 3,028,502.92 Total amount due. and collectible P7,154,685.16

The 50% surcharge has been imposed, pursuant to Section 72 * of the Tax Code and tax 1/2% monthly interest has likewise been imposed pursuant to the provision of Section 51(d) ** of the Tax Code (Exh. O, petition). 5 The petitioner assails these assessments as wrong. In the case of Collector of Internal Revenue vs. Reyes, 6 we ruled: Where the taxpayer is appealing to the tax court on the ground that the Collector's assessment is erroneous, it is incumbent upon him to prove there what is the correct

and just liability by a full and fair disclosure of all pertinent data in his possession. Otherwise, if the taxpayer confines himself to proving that the tax assessment is wrong, the tax court proceedings would settle nothing, and the way would be left open for subsequent assessments and appeals in interminable succession. Tax assessments by tax examiners are presumed correct and made in good faith. The taxpayer has the duty to prove otherwise. 7 In the absence of proof of any irregularities in the performance of duties, an assessment duly made by a Bureau of Internal Revenue examiner and approved by his superior officers will not be disturbed. 8 All presumptions are in favor of the correctness of tax assessments. 9 On the whole, we find that the fraudulent acts detailed in the decision under review had not been satisfactorily rebutted by the petitioner. There are indeed clear indications on the part of the taxpayer to deprive the Government of the taxes due. The Assistant Factory Superintendent of Silver Cup, Nelson Po gave the following testimony: Annexes "A", "A-1 " to "A-17" show that from January to December 1970, Silver Cup had used in production 189 drums of untaxed distilled alcohol and 3,722 drums of untaxed distilled alcohol. Can you tell us how could this be possible with the presence of a revenue inspector in the premises of Silver Cup during working hours? Actually, the revenue inspector or storekeeper comes around once a week on the average. Sometimes, when the storekeeper is around in the morning and Po Bein Sing wants to operate with untaxed alcohol as raw materials, Po Bien Sing tells the storekeeper to go home because the factory is not going to operate for the day. After the storekeeper leaves, the illegal operation then begins. Untaxed alcohol is brought in from Cebu Alcohol Plant into the compound of Silver Cup sometimes at about 6:00 A.M. or at 12:00 noon or in the evening or even at mid-night when the storekeeper is not around. When the storekeeper comes, he sees nothing because untaxed alcohol is brought directly to, and stored at, a secret tunnel within the bodega itself inside the compound of Silver Cup. In the same vein, the factory personnel manager testified that false entries were entered in the official register book: thus,
A ² As factory personnel manager and all-around handy man of Po Bien Sing, owner of Silver Cup, these labels were entrusted to me to make the false entries in the official register book of Silver Cup, which I did under the direction of Po Bien Sing. (Sworn statement, p. 512, Folder II, BIR rec.) 10 (Emphasis ours)

The existence of fraud as found by the respondents can not be lightly set aside absent substantial evidence presented by the petitioner to counteract such finding. The findings of fact of the respondent Court of Tax Appeals are entitled to the highest respect. 11 We do not find anything in the questioned decision that should disturb this long-established doctrine. WHEREFORE, the Petition is DENIED. The Decision of the respondent Court of Tax Appeals is hereby AFFIRMED. Costs against the petitioner. SO ORDERED.

Melencio-Herrera, Paras and Padilla, JJ., concur.

Republic of the Philippines SUPREME COURT Manila FIRST DIVISION

G.R. No. 96262 March 22, 1999 COMMISSIONER OF INTERNAL REVENUE, petitioner, vs. EMBROIDERY AND GARMENTS INDUSTRIES (PHIL.), INC., respondent.

PARDO, J.: The case is an appeal via certiorari from a decision of the Court of Appeals 1 affirming that of the Court of Tax Appeals 2 absolving respondent from liability for deficiency income tax and advance sales tax in the amounts of P2,756,241.68, and P3,500,798.47, respectively, for the years 1959 to 1961. The facts may be related as follows: On September 21, 1964, on the basis of a sworn report of an informer, the Courts of First Instance of Manila and Bulacan issued search warrants for the seizure of certain documents from the offices of respondent Embroidery and Garments Industries (Phil.), Inc. in Manila and Valenzuela, Bulacan. Armed with the warrants, agents of the Anti-Technical Smuggling Unit, Bureau of Internal Revenue, seized various business records and documents from respondent's offices. On January 4, 1966, petitioner assessed respondent the sum of P436,846.44, inclusive of 75% surcharge and penalty as advance sales tax for the years 1959 to 1961 and, on March 23, 1966, assessed deficiency income tax in the sum of P4,799,641.95, inclusive of 50% surcharge and 1/2% monthly interest for the years 1960 and 1961. Respondent protested the assessments, and on December 9,1970, petitioner issued to respondent a revised assessment requiring the latter to pay the amount of P2,756,241.68, inclusive of 50% surcharge and 1/2% monthly interest as deficiency income tax for the years 1959 to 1961. On December 22, 1970, petitioner required respondent to pay P3,500,798.47, as advance sales tax and 75% surcharge corresponding to the same years.

On January 7, 1971, respondent filed with the Bureau of Internal Revenue a protest disputing the revised assessments and requesting further investigation. On the same date, petitioner denied the protest. On January 20, 1971, respondent requested petitioner to reconsider the denial of its protest. On January 29, 1971, petitioner granted the request upon respondent's execution of a waiver of the statute of limitations. On September 14, 1971, petitioner denied respondent's protest on the disputed assessments. On October 14, 1971 , respondent filed with the Court of Tax Appeals a petition for review of the disputed tax assessments. On March 29, 1972, respondent filed its answer to the petition praying for its dismissal. On January 15, 1990, the Court of Tax Appeals rendered decision finding respondent not liable for deficiency income tax and advance sales tax assessed against it, and accordingly, reversed the BIR decision. In its decision, the Court of Tax Appeals held that the assessments were doubtful validity as they were based on the incompetent evidence consisting of an informant's report and the sworn statement of the disgruntled former general manager of respondent that in the years in question respondent sold all its dollar quotas to local Chinese textile traders at an overprice or premium on the dollar value of textile importation of 80% for suiting materials and 70.% for women's clothing materials and faked its invoices to reduce its costs of importation. On the other hand, respondent adduced evidence consisting of official records of the Bureau of Customs that its tax-free importation's had been re-exported to their suppliers in accordance with the Embroidery Law and cleared by the Bureau of Customs. The tax court ruled that the assessments must be based on actual facts and proved by competent evidence, not imposed based on unverified information supplied by an informant, or disputed presumptions. On June 13, 1990, petitioner filed with the Court of Appeals a petition for review of the decision of the Court of Tax Appeals. 3 On November 9, 1990, the Court of Appeals promulgated its decision affirming the appealed decision of the tax court. 4 On December 4, 1990, petitioner filed a motion for reconsideration of the Court of Appeals' decision. On February 7, 1991, the Court of Appeals denied the motion. 5 On March 18, 1991, within, the extended time granted, petitioner filed with the Supreme Court a petition for review on certiorari of the decision of the Court of Appeals. 6 In the petition, the Commissioner of Internal Revenue submits that the Court of Appeals erred: (1) in not holding that respondent is liable for deficiency income tax and advance sales tax in view of its failure to declare its income realized for the years 1959 to 1961 from the sales of its dollar quota to local Chinese textile dealers at a premium of 70% to 80% of the dollar value, which dollar quota rights were allocated by the Central Bank of the Philippines to enable respondent to import tax-free textile raw materials to be manufactured into finished products for re-export pursuant to the provisions of the Embroidery Law (R. A. No. 3137), and

(2) in not holding that the imposition of 50% surcharge for fraud was legal and justified. 7 The issues raised are clearly factual and must be resolved on the basis of the evidence adduced before the tax court. The case tarried too long in the tax court. In the meantime, the star witness had died, and the needed originals of documentary evidence could no longer be located. What is more, it is a, fundamental rule that on appeal via certiorari from a decision of the Court of Appeals to the Supreme Court may raise only questions of law, which must be distinctly set forth. 8 Findings of fact of the Court of Appeals and even of the tax court are final, binding or conclusive on the parties 9 and upon this Court, 10 which will not be reviewed 11 or disturbed on appeal unless these findings are not supported by evidence, 12 with certain well recognized exceptions, such as (1) when the conclusion is grounded entirely on speculations 13, surmises or conjectures; (2) when the inference made is manifestly mistaken, absurd or impossible; (3) where there is grave abuse of discretion; (4) when the judgment is based on a misapprehension of facts; (5) when the findings of fact are conflicting; (6) when the Court of Appeals, in making its findings, went beyond the issues of the case and the same is contrary to the admissions of both appellant and appellee; (7) when the findings of the Court of Appeals are contrary to those of the trial courts; 14 (8) when the findings of fact are conclusions without citation of specific evidence on which they are based; (9) when the Court of Appeals overlooked certain relevant facts not disputed by the parties, which, if properly considered, would justify a different conclusion; and (10) when the findings of fact of the Court of Appeals are premised on the absence of evidence and are contradicted by the evidence on record.15 This case does not come within any of the exceptions. WHEREFORE, the Court hereby AFFIRMS the appealed decision of the Court of Appeals in CAG.R. SP No. 20813. No costs. SO ORDERED. Davide, Jr, C.J., Melo and Kapunan, JJ., concur. Republic of the Philippines SUPREME COURT Manila EN BANC G.R. No. 502 January 29, 1946

BASILIA CABRERA, plaintiff-appellee, vs. THE PROVINCIAL TREASURER OF TAYABAS and PEDRO J. CATIGBAC, defendantsappellants. Lorenzo Sumulong for appellant. Jose W. Diokno for appellee. PARAS, J.:

On October 30, 1940, the provincial treasurer of Tayabas issued a notice for the sale at public auction of numerous, real properties forfeited for tax delinquency, including a certain parcel of land located in the barrio of Buenavista, municipality of Candelaria, Province of Tayabas, and assessed in the name of Nemesio Cabrera, said sale to be held "on December 15, 1940 at 8 a.m. and every day thereafter at the same place and hour until all the properties shall have been sold to the highest bidder." Copy of the notice was sent by registered mail to Nemesio Cabrera, but the envelope containing the same was returned with the remark "Unclaimed," undoubtedly because Nemesio Cabrera had already died in 1935. The land was actually sold on May 12, 1941, for the sum of P74.34 to the appellant Pedro J. Catigbac, in whose favor the final bill of sale was executed on September 23, 1942. Thereafter the appellee, Basilia Cabrera, filed a complaint in the Court of First Instance of Tayabas against the provincial treasurer and the appellant, attacking the validity of the tax sale on the grounds that she was not notified therefore and that although the land had remained in the assessment book in the name of Nemesio Cabrera, a former owner, she has become its registered owner, since 1934 when a Torrens title (No. 8167) was issued to her by the register of deeds of Tayabas. From a judgment favorable to the appellee, the present appeal was taken by Pedro J. Catigbac. Under the law (Commonwealth Act No. 470, section 35), the provincial treasurer is enjoined to set forth in the notice, among other particulars, the date of the tax sale. We are of the opinion that this mandatory requirement was not satisfied in the present case, because the announcement that the sale would take place on December 15, 1940 and every day thereafter, is as general and indefinite as a notice for the sale "within this or next year" or "some time within the month of December." In order to enable a taxpayer to protect his rights, he should at least appraised of the exact date of the proceeding by which he is to lose his property. When we consider the fact that the sale in favor of the appellant was executed on May 12, 1941, or nearly five months after December 15, 1940, the violation of the mandatory requirement becomes more obvious. Indeed, in his motion for reconsideration (seeRecord on Appeal, pp. 33-41), the appellant had admitted, unknowingly perhaps, that when he went to the office of the municipal treasurer after reading the notice of sale in December, 1940, to inquire about the advertised land, he was told to return on May 12, 1941. The implication that follows is that the tax officials had really adopted the view that they could sell any of the numerous forfeited lots on any date subsequent to December 15, 1940, without new notice, thereby making the resulting sale more private than public, likewise in violation of the law. It may be observed that as regards tax sales, unlike ordinary execution sales, the statute does not expressly authorize adjournment from day to day. The reminder may, however, be given that the tax officials will greatly be inconvenienced by following the law strictly, especially when numerous properties are, as in the present case (132 parcels), to be disposed of for tax delinquency. We will not venture to disagree, but it is believed that the officials who are ever solicitous in protecting private proprietary rights, shall have helped, to the same extent, in maintaining the solid foundation of the Government which they seek to serve and of which they themselves are a part. What has been said is sufficient to decide this appeal, although it will not altogether be amiss to refer to details that further support the judgment of the lower court. The appellee was admittedly not notified of the auction sale, and this also vitiates the proceeding. She is the registered owner of the land and, since 1934, has become liable for the taxes thereon. For all purposes, she is the delinquent taxpayer "against whom the taxes were assessed," referred to in section 34 of Commonwealth Act No. 470. It cannot be Nemesio Cabrera for the latter's obligation to pay taxes ended where the appellee's liability began. Neither the alleged receipt by the appellee of a copy of certificate of sale dated May 12, 1941, nor her failure to redeem thereafter, had the effect of validating the prior tax proceeding. The sale in favor of the appellant cannot bind the appellee, since the land purportedly conveyed was owned by Nemesio Cabrera, not by the appellee; and, at the time of the sale, Nemesio Cabrera had no interest whatsoever in the land in question that could have passed to the appellant.

The appellee may be criticized for her failure to have the land transferred to her name in the assessment record. The circumstance, nevertheless, cannot supplant the absence of notice. Of course, it is the duty of any person acquiring at the time real property to prepare and submit a tax declaration within sixty days (Commonwealth Act No. 470, section 12), but it is no less true that when the owner refuses or fails to make the required declaration, the provincial assessor should himself declare the property in the name of the defaulting owner (Commonwealth Act No. 470, section 14). In this case there is absolutely no showing that the appellee had deliberately failed to make the declaration to defraud the tax officials; and it may be remarked that there can be no reason why her Torrens title, which binds the whole world, cannot at least charge the Government which had issued it, with notice thereof. A little synchronization between the offices of the register of deeds and of the provincial assessor, with perhaps very negligible additional clerical work on the part of both, will surely result in a more efficient enforcement of the tax laws. Not having appealed, the appellee cannot now pretend that the judgement of the lower court is erroneous in so far as it failed to award damages in her favor for the sum of P500. While an appellee can on appeal make a counter-assignment of error, it must be with a view merely to sustaining the judgement, not to obtaining other affirmative relief. The appealed judgment is affirmed, with costs of both instances against the appellant. So ordered. Moran, C.J., Jaranilla, and Pablo, JJ., concur. Republic of the Philippines SUPREME COURT Manila EN BANC G.R. No. L-9061 November 18, 1957

RICARDO VELAYO, petitioner-appellant, vs. FERNANDO ORDOVEZA, ET AL., respondents-appellees. Liwag and Vivo for appellant. Benjamin Relova and Raul A. Aristorenas for appellees. CONCEPCION, J.: After publication of the corresponding notice, on October 29, 1949, the City Treasurer of Manila sold, at public auction, to Ricardo Velayo ² whose address is Gapan, Nueva Ecija ² for the sum of P185.95, representing the amount due by way of unpaid real estate taxes, the years 1948 and 1949, plus penalty and costs, on the property described in said notice as follows: LAND containing an area of 99.40 SQUARE METERS, more or less, with improvements thereon, located at 813 Lepanto, Sampaloc, Manila, and designated as lot 16, Block 35, Assessment No. 3305 in the list of taxable properties for the district of Sampaloc. The property located at said address ² 813 Lepanto, Sampaloc, Manila ² is covered by Transfer Certificate of Title No. 79178 for the City of Manila, in the name of Fernando, Ramon, Annie, Beatriz

and Isabelita, all surnamed Ordoveza, who had bought it sometime before 1948. However, said certificate of title contains the following description: 1. A PARCEL OF LAND (Lot No. 20 of Block No. 4002 of the Cadastral Survey of the City of Manila, situated in the District of Sampaloc. Bounded on the NE. by lot No. 22 of Block No. 4002; on the SE. by Lot No. 21 of Block No. 4002; on the SW. by Lot No. 18 of Block No. 4002. . . containing an area of NINETY FOUR SQUARE METERS AND EIGHTY SQUARE DECIMETERS (94.80), more or less. . . 2. A PARCEL OF LAND (Lot No. 21 of Block No. 4002 of the Cadastral Survey of the City of Manila), situated in the NW. line of Calle Lepanto, District of Sampaloc. Bound on the NE. by Lot No. 23 of Block No. 4002; on the SE. by Calle Lepanto; on the SW. by Lot No. 19 of Block No. 4002; and on the NW. by Lot No. 20 of Block No. 4002. . . containing an area of TWO SQUARE METERS AND THIRTY SQUARE DECIMETERS (2.30), more or less. . . When Fernando Ordoveza tried to pay the real estate tax thereon on November 20, 1959, a clerk in the office of the City Treasurer advised him of the sale aforementioned. On the same date, Ordoveza wrote to Velayo the letter, Exhibit A, offering to reimburse the amount paid by him with 15 per cent interest thereon. Instead of answering this letter, Velayo secured from the City Treasurer the corresponding certificate of sale, dated December 4, 1950. Upon Velayo's failure to reply, Fernando Ordoveza wrote to the City Treasurer, on December 11, 1950, the letter, Exhibit B, stating that he (Ordovesa) was surprised to hear about the sale, for he had not received any previous notice thereof or read in the newspapers about the public auction to be held in connection therewith, and enclosing a money order for P300.72, to cover the taxes due, plus interest and costs, with the request that said sum be accepted by way of redemption of the property in question. Four (4) days later, or on December 15, 1950, Velayo caused the certificate of sale in his favor to be filed with the Register of Deeds of Manila. On December 19, 1950, this officer replied to Ordoveza's letter, Exhibit B, with the information that the sum of P300.72 enclosed therewith had been accepted merely by way of deposit, upon the ground that the period to redeem the property had expired on October 29, 1950, and that it was, therefore, up to the Ordoveza's either to seek an extrajudicial settlement with Velayo, or to assail the validity of the tax sale (Exhibit 3). On January 17, 1951, the Register of Deeds of Manila demanded from the Ordoveza's the production of its owner's duplicate of said Transfer Certificate of Title No. 79178, for annotation of the memorandum relative to the certificate of sale above referred to. The record does not show whether this demand was heeded or not. Meanwhile, or on November 12, 1951, the City Treasurer executed the corresponding deed of sale (Exhibit C) to Velayo. On November 10, 1954, Velayo filed in the Cadastral Case of the Court of First Instance of Manila in which the registration of the property in dispute was effected, a petition stating that he had bought said Lot 16, Block 35 of the City of Manila, on October 29, 1949, for tax delinquency; that said property had not been redeemed on or before October 29, 1950; that the deemed of sale in his favor was executed by the city treasurer on November 12, 1951; that the property thus conveyed to him is, also, known as Lots Nos. 20 and 21, Block No. 4002, for the Cadastral Survey of Manila, which are more particularly described in Transfer Certificate of Title No. 79178 of said City; and that the corresponding certificate of title had not as yet, been issued to him, and praying that the Register of Deeds of Manila be ordered to register said deed of sale, Transfer Certificate of Title No. 79178 be cancelled and that a certificate of title be issued in the name of said petitioner, Ricardo Velayo.

The Ordoveza's opposed the petition, upon the ground, among others that the description in said deed of sale is different from the description appearing in Transfer Certificate of Title No. 79178; that the property in question, which had been supposedly sold for P185.92 is assessed at P4,910; that Velayo had refrained from filing the certificate of sale with the register of deed until over a year after the sale, with the obvious purpose of preventing redemption on or before October 29, 1950; and that prior to the registration of said certificate of sale, the Ordoveza's had already paid to the City Treasurer the amount due by way of real-estate taxes, with 15 per cent interest thereupon, plus the taxes for 1950; and that the officer charged by law with the duty of giving notice of the contemplated sale of properties for tax delinquency and of executing the corresponding deed of sale is the City Assessors, not the City Treasurer, and praying that Velayo's petition be denied and that the memorandum, is Transfer Certificate of Title No. 79178, of the sale in his favor, be ordered cancelled. After appropriate proceedings, on February 22, 1955, the Court of First Instance of Manila sustained the last ground of said opposition and, accordingly, denied Velayo's petition. The case is now before Us on appeal taken by Ricardo Velayo. We find no merit in the appeal. Section 65 of Republic Act No. 409 explicity provides that: Fifteen days after the (real estate) tax shall become delinquent the city assessor and collector shall prepare and sign a certified copy of the records of his office showing the persons delinquent in payment of their taxes and the amount of tax and penalty respectively due from them. He may thereupon proceed to seize personal property of each delinquent not exempt . . . and . . . to sell public auction, either at the main entrance of the City Hall or at the place where such property is seized. . . so much as the same as shall satisfy the tax, penalty and costs . . . (parenthetical words and emphasis supplied.). In addition thereto, "the city assessor and collector, may" ² pursuant to section 69 of said Act ² "advertise the real estate of the delinquent for sale. . .". . . . At any time before the day fixed for the sale, the taxpayer may discontinue all proceedings by paying the taxes, penalties, and costs to the city assessor and collector. If he does not do so, the sale shall proceed and shall be held either at the main entrance of the City Hall or on the premises to be sold, as city assessor and collector may determine. Within five days after the sale city assessor and collector shall make return of the proceedings and spread it in his records. The purchaser at the sale shall receive a certificate from the city assessor and collector from is records showing the proceedings of the sale, describing the property sold, . . (Emphasis supplied.) However, according to section 70 of said Act: Within one year from the date of sale the delinquent taxpayer, or anyone for him, shall have the right of paying to the city assessor and collector the amount of the public taxes, penalties, and cost together with interest . . . at the rate of fifteen per centum per annum . . . ; and such payment shall entitle the person in paying to the delivery of the certificate issued to the purchaser and a certificate from the city assessor and collector that he has thus redeemed the real estate and the city assessor and collector shall forthwith pay over to the purchaser the amount by which shall real estate has thus redeemed . . . (Emphasis supplied.) Again, section 71 and 72 of said Act read:

In the case the taxpayer shall not redeem the real property sold . . . within on year from the date of the sale,city assessor and collector shall . . . execute a deed in form and effect sufficient to convey to the purchaser so much of the real estate against which the taxes have been assessed as has been sold . . . (Section 71; emphasis supplied.) In case there is no bidder at the public sale of such reality who offers a sum sufficient to pay the taxes, penalties, and cost city assessor and collector shall declare the real estate forfeited for the city, and shall make, within two days thereafter, a return of his proceedings and the forfeiture, which shall be spread upon the records of his office. (Section 72; emphasis supplied). The foregoing provisions are clear, explicit and unequivocal. The city assessor and collector is the officer charged with the function of distraining personal property for the collection of delinquent real estate taxes. It is he who shall "advertise the real estate of the delinquent for sale." In order to suspend the same, payment of said taxes should be made to him prior thereto. The sale, if not suspended, shall be made under his authority. The proceedings relative to the sale shall be spread in the records of the city assessor. The certificate of the sale is to be issued by him. Redemption may be made by payment to him, within one year. A certificate of redemption, if effected, shall be issued by the city assessor. It is he who, in such event, refund the corresponding amount to the at the tax sale. He, too, shall execute the deed of sale in favor of said buyer, in the absence of redemption. Lastly, it is he who "shall declare the real estate forfeited to the city," in case there is no bidder at the public sale. In the case at bar, the notice of sale at public auction was given, and the sale was made, by the city treasurer, who, likewise, executed the certificate of sale and, later on, the deed of sale, although he had no authority therefor. Accordingly, said notice, sale, certificate and deed are insufficient to divest the Ordoveza's of their title to the property in question. It is urged, however, that the term "city assessor and collector" in the aforementioned sections 65, 69, 70, 71 and 72 of Republic Act. No. 409, refers to the City Treasurer, because: (a) Section 52 of said Act provides that the City Treasurer "shall perform in and for the city the duties imposed by law or regulation upon provincial treasurers," who, pursuant to Commonwealth Act No. 470, are in charge of the proceedings relative to the sale of property for the collection of real estate tax delinquencies; and (b) Section 73 of Republic Act No. 409 provides, that "within one year from the date" of the forfeiture of the property to the city of Manila-in case there is no bidder at the public sale of said property² . . . the taxpayer, or anyone for him may redeem said reality as above provided in cases where the same is sold. But, if the reality is not thus redeemed within the year the forfeiture shall become absolute and the city treasurer shall execute a deed, similar in form and having the same effect as the deed required to be made by him in case of sale, conveying the real estate to the city. The deed shall be recorded as required by law for other real estate titles and shall then be forwarded to the mayor for notation and return to the city treasurer who shall file the same and enter it in his records of city property. (Emphasis supplied.) Appellant stresses the statement in this provision to the effect that, if the property forfeited to the city is not redeemed, "the city treasurer shall execute a deed, similar in form and having the same effect as the deedrequired to be made by him in case of sale, conveying the real estate of the city." Based upon this sentence, it is urged that the deed of sale mentioned in section 71 must be understood as "required to be made," therefore, by the "city treasurer," not "the city assessor and collector."

As regards the first argument, suffice it to say that section 52 of Republic Act No. 409, imposes upon the city treasurer the duties vested upon provincial treasurers, "unless otherwise specifically provided by law or regulations" and that section 65, 69, 70, 71 and 72 of said Republic Act No. 409 specially provide otherwise. With respect to the second argument, it should be noted that the aforementioned provisions of said Act form part of Article XII thereof, which is entitled "Department of Assessment," and were meant, consequently, to govern the functions of the city assessor. Again, said sections 65 and 69 to 72 are a substantial reproduction of sections 2494 and 2498 to 2501 of the Revised Administrative Code, or the former Charter of the City of Manila, which likewise vested the functions under consideration in the "city assessor and collector." Thus, Republic Act No. 409 merely maintained the provisions of the old law, insofar as said sections 65 and 669 to 72 are concerned. Indeed, the last paragraphs of section 53 states: WHENEVER the words 'city assessor and collector' occur in the Article in relation to any matter pertaining t assessment, or property falling under such department, the same shall be deemed to mean the city assessor, and all the duties and powers heretofore devolving upon such officer shall hereafter be performed and exercised by the city assessor. (Emphasis supplied.) Although not indispensable to the disposition of this appeal, we cannot overlook the fact that the notice of sale published, in connection with the case at bar referred to a "land containing an area of 99.40 square meters, more or less . . . designated as Lot 16, Block 35" of the City of Manila, whereas the property of Ordoveza's, covered by Transfer Certificate of Title No. 79178, is described therein as Lots Nos. 20 and 21 of Block No. 4002 of the Cadastral Survey of said City, with an area of 94.80 and 2.30 square meters, or less, respectively. Although, both documents name Lepanto street and give the same house number, the difference in the number of the block (the notice said Block 35 and the property of the Ordoveza's is in Block 4002) and in that of the lot (the notice mentioned only one [1] lot, bearing No. 16, whereas Transfer Certificate of Title No. 79178 covers [2] lots, bearing Nos. 20 and 21) ² apart from the difference in area ² are such as to have a confusing misleading effect. The injurious consequences of such variance becomes more manifest when we consider that the lots of the Ordoveza's are registered under the Torrens System and that its owners are consequently, justified in relying upon the description given in their certificate of title as the one officially identifying said property. In other words, it is hardly possible to sanction the tax sale of a property with a description distinct and different from that which appears in its certificate of title, without impairing the full and credence with the same in meant to command and, hence, without effecting the essence of the Torrens Systems. This suggest the advisability or need of adopting means and ways tending to insure that the records of the assessment for purpose of real estate tax on registered properties contain a description thereof which dovetails with that of the records of the corresponding registration proceedings. Thus, instead of being merely a formality, which often does not really give the notice demanded by the requirements of due process, the advertisement of tax sales prescribed by law would furnished substantially the information and warning it is meant to convey to, among others, the owner and delinquent taxpayer, in order that he may either make payment before the sale, and thus suspend the same, or redeem his property within the statutory period. Wherefore, without prejudice the appellant's right to recover the price by him paid for the property in dispute, the order appealed from should be, as it is hereby, affirmed, and the Register of Deeds of Manila is directed to cancel the memorandum, appearing on Transfer Certificate of Title No. 79178, of the sale made to appellant Ricardo Velayo, with cost against the latter. It is so ordered.

Paras, C.J., Bengzon, Padilla, Reyes, A., Bautista Angelo, Labrador, Reyes, J.B.L., Endencia, and Felix, JJ.,concur. Republic of the Philippines SUPREME COURT Manila SECOND DIVISION

G.R. No. 120880 June 5, 1997 FERDINAND R. MARCOS II, petitioner, vs. COURT OF APPEALS, THE COMMISSIONER OF THE BUREAU OF INTERNAL REVENUE and HERMINIA D. DE GUZMAN, respondents.

TORRES, JR., J.: In this Petition for Review on Certiorari, Government action is once again assailed as precipitate and unfair, suffering the basic and oftly implored requisites of due process of law. Specifically, the petition assails the Decision1 of the Court of Appeals dated November 29, 1994 in CA-G.R. SP No. 31363, where the said court held: In view of all the foregoing, we rule that the deficiency income tax assessments and estate tax assessment, are already final and (u)nappealable-and-the subsequent levy of real properties is a tax remedy resorted to by the government, sanctioned by Section 213 and 218 of the National Internal Revenue Code. This summary tax remedy is distinct and separate from the other tax remedies (such as Judicial Civil actions and Criminal actions), and is not affected or precluded by the pendency of any other tax remedies instituted by the government. WHEREFORE, premises considered, judgment is hereby rendered DISMISSING the petition forcertiorari with prayer for Restraining Order and Injunction. No pronouncements as to costs. SO ORDERED. More than seven years since the demise of the late Ferdinand E. Marcos, the former President of the Republic of the Philippines, the matter of the settlement of his estate, and its dues to the government in estate taxes, are still unresolved, the latter issue being now before this Court for resolution. Specifically, petitioner Ferdinand R. Marcos II, the eldest son of the decedent, questions the actuations of the respondent Commissioner of Internal Revenue in assessing, and collecting through the summary remedy of Levy on Real Properties, estate and income tax delinquencies upon the estate and properties of his father, despite the pendency of the proceedings on probate of the will of the late president, which is docketed as Sp. Proc. No. 10279 in the Regional Trial Court of Pasig, Branch 156.

Petitioner had filed with the respondent Court of Appeals a Petition for Certiorari and Prohibition with an application for writ of preliminary injunction and/or temporary restraining order on June 28, 1993, seeking to ² I. Annul and set aside the Notices of Levy on real property dated February 22, 1993 and May 20, 1993, issued by respondent Commissioner of Internal Revenue; II. Annul and set aside the Notices of Sale dated May 26, 1993; III. Enjoin the Head Revenue Executive Assistant Director II (Collection Service), from proceeding with the Auction of the real properties covered by Notices of Sale. After the parties had pleaded their case, the Court of Appeals rendered its Decision 2 on November 29, 1994, ruling that the deficiency assessments for estate and income tax made upon the petitioner and the estate of the deceased President Marcos have already become final and unappealable, and may thus be enforced by the summary remedy of levying upon the properties of the late President, as was done by the respondent Commissioner of Internal Revenue. WHEREFORE, premises considered judgment is hereby rendered DISMISSING the petition forCertiorari with prayer for Restraining Order and Injunction. No pronouncements as to cost. SO ORDERED. Unperturbed, petitioner is now before us assailing the validity of the appellate court's decision, assigning the following as errors: A. RESPONDENT COURT MANIFESTLY ERRED IN RULING THAT THE SUMMARY TAX REMEDIES RESORTED TO BY THE GOVERNMENT ARE NOT AFFECTED AND PRECLUDED BY THE PENDENCY OF THE SPECIAL PROCEEDING FOR THE ALLOWANCE OF THE LATE PRESIDENT'S ALLEGED WILL. TO THE CONTRARY, THIS PROBATE PROCEEDING PRECISELY PLACED ALL PROPERTIES WHICH FORM PART OF THE LATE PRESIDENT'S ESTATE IN CUSTODIA LEGIS OF THE PROBATE COURT TO THE EXCLUSION OF ALL OTHER COURTS AND ADMINISTRATIVE AGENCIES. B. RESPONDENT COURT ARBITRARILY ERRED IN SWEEPINGLY DECIDING THAT SINCE THE TAX ASSESSMENTS OF PETITIONER AND HIS PARENTS HAD ALREADY BECOME FINAL AND UNAPPEALABLE, THERE WAS NO NEED TO GO INTO THE MERITS OF THE GROUNDS CITED IN THE PETITION. INDEPENDENT OF WHETHER THE TAX ASSESSMENTS HAD ALREADY BECOME FINAL, HOWEVER, PETITIONER HAS THE RIGHT TO QUESTION THE UNLAWFUL MANNER AND METHOD IN WHICH TAX COLLECTION IS SOUGHT TO BE ENFORCED BY RESPONDENTS COMMISSIONER AND DE GUZMAN. THUS, RESPONDENT COURT SHOULD HAVE FAVORABLY CONSIDERED THE MERITS OF THE FOLLOWING GROUNDS IN THE PETITION: (1) The Notices of Levy on Real Property were issued beyond the period provided in the Revenue Memorandum Circular No. 38-68.

(2) [a] The numerous pending court cases questioning the late President's ownership or interests in several properties (both personal and real) make the total value of his estate, and the consequent estate tax due, incapable of exact pecuniary determination at this time. Thus, respondents' assessment of the estate tax and their issuance of the Notices of Levy and Sale are premature, confiscatory and oppressive. [b] Petitioner, as one of the late President's compulsory heirs, was never notified, much less served with copies of the Notices of Levy, contrary to the mandate of Section 213 of the NIRC. As such, petitioner was never given an opportunity to contest the Notices in violation of his right to due process of law. C. ON ACCOUNT OF THE CLEAR MERIT OF THE PETITION, RESPONDENT COURT MANIFESTLY ERRED IN RULING THAT IT HAD NO POWER TO GRANT INJUNCTIVE RELIEF TO PETITIONER. SECTION 219 OF THE NIRC NOTWITHSTANDING, COURTS POSSESS THE POWER TO ISSUE A WRIT OF PRELIMINARY INJUNCTION TO RESTRAIN RESPONDENTS COMMISSIONER'S AND DE GUZMAN'S ARBITRARY METHOD OF COLLECTING THE ALLEGED DEFICIENCY ESTATE AND INCOME TAXES BY MEANS OF LEVY. The facts as found by the appellate court are undisputed, and are hereby adopted: On September 29, 1989, former President Ferdinand Marcos died in Honolulu, Hawaii, USA. On June 27, 1990, a Special Tax Audit Team was created to conduct investigations and examinations of the tax liabilities and obligations of the late president, as well as that of his family, associates and "cronies". Said audit team concluded its investigation with a Memorandum dated July 26, 1991. The investigation disclosed that the Marcoses failed to file a written notice of the death of the decedent, an estate tax returns [sic], as well as several income tax returns covering the years 1982 to 1986, ² all in violation of the National Internal Revenue Code (NIRC). Subsequently, criminal charges were filed against Mrs. Imelda R. Marcos before the Regional Trial of Quezon City for violations of Sections 82, 83 and 84 (has penalized under Sections 253 and 254 in relation to Section 252 ² a & b) of the National Internal Revenue Code (NIRC). The Commissioner of Internal Revenue thereby caused the preparation and filing of the Estate Tax Return for the estate of the late president, the Income Tax Returns of the Spouses Marcos for the years 1985 to 1986, and the Income Tax Returns of petitioner Ferdinand "Bongbong" Marcos II for the years 1982 to 1985. On July 26, 1991, the BIR issued the following: (1) Deficiency estate tax assessment no. FAC-2-89-91-002464 (against the estate of the late president Ferdinand Marcos in the amount of P23,293,607,638.00 Pesos); (2) Deficiency income tax assessment no. FAC-1-85-91-002452 and Deficiency income tax assessment no. FAC-1-86-91002451 (against the Spouses Ferdinand and Imelda Marcos in the amounts of P149,551.70 and P184,009,737.40 representing deficiency income tax for the years 1985 and 1986); (3) Deficiency income tax assessment nos. FAC-1-82-91-002460 to

FAC-1-85-91-002463 (against petitioner Ferdinand "Bongbong" Marcos II in the amounts of P258.70 pesos; P9,386.40 Pesos; P4,388.30 Pesos; and P6,376.60 Pesos representing his deficiency income taxes for the years 1982 to 1985). The Commissioner of Internal Revenue avers that copies of the deficiency estate and income tax assessments were all personally and constructively served on August 26, 1991 and September 12, 1991 upon Mrs. Imelda Marcos (through her caretaker Mr. Martinez) at her last known address at No. 204 Ortega St., San Juan, M.M. (Annexes "D" and "E" of the Petition). Likewise, copies of the deficiency tax assessments issued against petitioner Ferdinand "Bongbong" Marcos II were also personally and constructively served upon him (through his caretaker) on September 12, 1991, at his last known address at Don Mariano Marcos St. corner P. Guevarra St., San Juan, M.M. (Annexes "J" and "J-1" of the Petition). Thereafter, Formal Assessment notices were served on October 20, 1992, upon Mrs. Marcos c/o petitioner, at his office, House of Representatives, Batasan Pambansa, Quezon City. Moreover, a notice to Taxpayer inviting Mrs. Marcos (or her duly authorized representative or counsel), to a conference, was furnished the counsel of Mrs. Marcos, Dean Antonio Coronel ² but to no avail. The deficiency tax assessments were not protested administratively, by Mrs. Marcos and the other heirs of the late president, within 30 days from service of said assessments. On February 22, 1993, the BIR Commissioner issued twenty-two notices of levy on real property against certain parcels of land owned by the Marcoses ² to satisfy the alleged estate tax and deficiency income taxes of Spouses Marcos. On May 20, 1993, four more Notices of Levy on real property were issued for the purpose of satisfying the deficiency income taxes. On May 26, 1993, additional four (4) notices of Levy on real property were again issued. The foregoing tax remedies were resorted to pursuant to Sections 205 and 213 of the National Internal Revenue Code (NIRC). In response to a letter dated March 12, 1993 sent by Atty. Loreto Ata (counsel of herein petitioner) calling the attention of the BIR and requesting that they be duly notified of any action taken by the BIR affecting the interest of their client Ferdinand "Bongbong" Marcos II, as well as the interest of the late president ² copies of the aforesaid notices were, served on April 7, 1993 and on June 10, 1993, upon Mrs. Imelda Marcos, the petitioner, and their counsel of record, "De Borja, Medialdea, Ata, Bello, Guevarra and Serapio Law Office". Notices of sale at public auction were posted on May 26, 1993, at the lobby of the City Hall of Tacloban City. The public auction for the sale of the eleven (11) parcels of land took place on July 5, 1993. There being no bidder, the lots were declared forfeited in favor of the government. On June 25, 1993, petitioner Ferdinand "Bongbong" Marcos II filed the instant petition for certiorariand prohibition under Rule 65 of the Rules of Court, with prayer for temporary restraining order and/or writ of preliminary injunction.

It has been repeatedly observed, and not without merit, that the enforcement of tax laws and the collection of taxes, is of paramount importance for the sustenance of government. Taxes are the lifeblood of the government and should be collected without unnecessary hindrance. However, such collection should be made in accordance with law as any arbitrariness will negate the very reason for government itself. It is therefore necessary to reconcile the apparently conflicting interests of the authorities and the taxpayers so that the real purpose of taxation, which is the promotion of the common good, may be achieved. 3 Whether or not the proper avenues of assessment and collection of the said tax obligations were taken by the respondent Bureau is now the subject of the Court's inquiry. Petitioner posits that notices of levy, notices of sale, and subsequent sale of properties of the late President Marcos effected by the BIR are null and void for disregarding the established procedure for the enforcement of taxes due upon the estate of the deceased. The case of Domingo vs. Garlitos 4 is specifically cited to bolster the argument that "the ordinary procedure by which to settle claims of indebtedness against the estate of a deceased, person, as in an inheritance (estate) tax, is for the claimant to present a claim before the probate court so that said court may order the administrator to pay the amount therefor." This remedy is allegedly, exclusive, and cannot be effected through any other means. Petitioner goes further, submitting that the probate court is not precluded from denying a request by the government for the immediate payment of taxes, and should order the payment of the same only within the period fixed by the probate court for the payment of all the debts of the decedent. In this regard, petitioner cites the case of Collector of Internal Revenue vs. The Administratrix of the Estate of Echarri (67 Phil 502), where it was held that: The case of Pineda vs. Court of First Instance of Tayabas and Collector of Internal Revenue (52 Phil 803), relied upon by the petitioner-appellant is good authority on the proposition that the court having control over the administration proceedings has jurisdiction to entertain the claim presented by the government for taxes due and to order the administrator to pay the tax should it find that the assessment was proper, and that the tax was legal, due and collectible. And the rule laid down in that case must be understood in relation to the case of Collector of Customs vs. Haygood, supra., as to the procedure to be followed in a given case by the government to effectuate the collection of the tax. Categorically stated, where during the pendency of judicial administration over the estate of a deceased person a claim for taxes is presented by the government, the court has the authority to order payment by the administrator; but, in the same way that it has authority to order payment or satisfaction, it also has the negative authority to deny the same. While there are cases where courts are required to perform certain duties mandatory and ministerial in character, the function of the court in a case of the present character is not one of them; and here, the court cannot be an organism endowed with latitude of judgment in one direction, and converted into a mere mechanical contrivance in another direction. On the other hand, it is argued by the BIR, that the state's authority to collect internal revenue taxes is paramount. Thus, the pendency of probate proceedings over the estate of the deceased does not preclude the assessment and collection, through summary remedies, of estate taxes over the same. According to the respondent, claims for payment of estate and income taxes due and assessed after the death of the decedent need not be presented in the form of a claim against the estate. These can and should be paid immediately. The probate court is not the government agency to decide

whether an estate is liable for payment of estate of income taxes. Well-settled is the rule that the probate court is a court with special and limited jurisdiction. Concededly, the authority of the Regional Trial Court, sitting, albeit with limited jurisdiction, as a probate court over estate of deceased individual, is not a trifling thing. The court's jurisdiction, once invoked, and made effective, cannot be treated with indifference nor should it be ignored with impunity by the very parties invoking its authority. In testament to this, it has been held that it is within the jurisdiction of the probate court to approve the sale of properties of a deceased person by his prospective heirs before final adjudication; 5 to determine who are the heirs of the decedent; 6 the recognition of a natural child; 7 the status of a woman claiming to be the legal wife of the decedent; 8 the legality of disinheritance of an heir by the testator; 9 and to pass upon the validity of a waiver of hereditary rights. 10 The pivotal question the court is tasked to resolve refers to the authority of the Bureau of Internal Revenue to collect by the summary remedy of levying upon, and sale of real properties of the decedent, estate tax deficiencies, without the cognition and authority of the court sitting in probate over the supposed will of the deceased. The nature of the process of estate tax collection has been described as follows:
Strictly speaking, the assessment of an inheritance tax does not directly involve the administration of a decedent's estate, although it may be viewed as an incident to the complete settlement of an estate, and, under some statutes, it is made the duty of the probate court to make the amount of the inheritance tax a part of the final decree of distribution of the estate. It is not against the property of decedent, nor is it a claim against the estate as such, but it is against the interest or property right which the heir, legatee, devisee, etc., has in the property formerly held by decedent. Further, under some statutes, it has been held that it is not a suit or controversy between the parties, nor is it an adversary proceeding between the state and the person who owes the tax on the inheritance. However, under other statutes it has been held that the hearing and determination of the cash value of the assets and the determination of the tax are adversary proceedings. The proceeding has been held to be necessarily a proceeding in rem. 11

In the Philippine experience, the enforcement and collection of estate tax, is executive in character, as the legislature has seen it fit to ascribe this task to the Bureau of Internal Revenue. Section 3 of the National Internal Revenue Code attests to this: Sec. 3. Powers and duties of the Bureau. ² The powers and duties of the Bureau of Internal Revenue shall comprehend the assessment and collection of all national internal revenue taxes, fees, and charges, and the enforcement of all forfeitures, penalties, and fines connected therewith, including the execution of judgments in all cases decided in its favor by the Court of Tax Appeals and the ordinary courts. Said Bureau shall also give effect to and administer the supervisory and police power conferred to it by this Code or other laws. Thus, it was in Vera vs. Fernandez 12 that the court recognized the liberal treatment of claims for taxes charged against the estate of the decedent. Such taxes, we said, were exempted from the application of the statute of non-claims, and this is justified by the necessity of government funding, immortalized in the maxim that taxes are the lifeblood of the government. Vectigalia nervi sunt rei publicae ² taxes are the sinews of the state.

Taxes assessed against the estate of a deceased person, after administration is opened, need not be submitted to the committee on claims in the ordinary course of administration. In the exercise of its control over the administrator, the court may direct the payment of such taxes upon motion showing that the taxes have been assessed against the estate. Such liberal treatment of internal revenue taxes in the probate proceedings extends so far, even to allowing the enforcement of tax obligations against the heirs of the decedent, even after distribution of the estate's properties.
Claims for taxes, whether assessed before or after the death of the deceased, can be collected from the heirs even after the distribution of the properties of the decedent. They are exempted from the application of the statute of non-claims. The heirs shall be liable therefor, in proportion to their share in the inheritance. 13

Thus, the Government has two ways of collecting the taxes in question. One, by going after all the heirs and collecting from each one of them the amount of the tax proportionate to the inheritance received. Another remedy, pursuant to the lien created by Section 315 of the Tax Code upon all property and rights to property belong to the taxpayer for unpaid income tax, is by subjecting said property of the estate which is in the hands of an heir or transferee to the payment of the tax due the estate. (Commissioner of Internal Revenue vs. Pineda, 21 SCRA 105, September 15, 1967.) From the foregoing, it is discernible that the approval of the court, sitting in probate, or as a settlement tribunal over the deceased is not a mandatory requirement in the collection of estate taxes. It cannot therefore be argued that the Tax Bureau erred in proceeding with the levying and sale of the properties allegedly owned by the late President, on the ground that it was required to seek first the probate court's sanction. There is nothing in the Tax Code, and in the pertinent remedial laws that implies the necessity of the probate or estate settlement court's approval of the state's claim for estate taxes, before the same can be enforced and collected. On the contrary, under Section 87 of the NIRC, it is the probate or settlement court which is bidden not to authorize the executor or judicial administrator of the decedent's estate to deliver any distributive share to any party interested in the estate, unless it is shown a Certification by the Commissioner of Internal Revenue that the estate taxes have been paid. This provision disproves the petitioner's contention that it is the probate court which approves the assessment and collection of the estate tax. If there is any issue as to the validity of the BIR's decision to assess the estate taxes, this should have been pursued through the proper administrative and judicial avenues provided for by law. Section 229 of the NIRC tells us how: Sec. 229. Protesting of assessment. ² When the Commissioner of Internal Revenue or his duly authorized representative finds that proper taxes should be assessed, he shall first notify the taxpayer of his findings. Within a period to be prescribed by implementing regulations, the taxpayer shall be required to respond to said notice. If the taxpayer fails to respond, the Commissioner shall issue an assessment based on his findings.

Such assessment may be protested administratively by filing a request for reconsideration or reinvestigation in such form and manner as may be prescribed by implementing regulations within (30) days from receipt of the assessment; otherwise, the assessment shall become final and unappealable. If the protest is denied in whole or in part, the individual, association or corporation adversely affected by the decision on the protest may appeal to the Court of Tax Appeals within thirty (30) days from receipt of said decision; otherwise, the decision shall become final, executory and demandable. (As inserted by P.D. 1773) Apart from failing to file the required estate tax return within the time required for the filing of the same, petitioner, and the other heirs never questioned the assessments served upon them, allowing the same to lapse into finality, and prompting the BIR to collect the said taxes by levying upon the properties left by President Marcos. Petitioner submits, however, that "while the assessment of taxes may have been validly undertaken by the Government, collection thereof may have been done in violation of the law. Thus, the manner and method in which the latter is enforced may be questioned separately, and irrespective of the finality of the former, because the Government does not have the unbridled discretion to enforce collection without regard to the clear provision of law." 14 Petitioner specifically points out that applying Memorandum Circular No. 38-68, implementing Sections 318 and 324 of the old tax code (Republic Act 5203), the BIR's Notices of Levy on the Marcos properties, were issued beyond the allowed period, and are therefore null and void:
. . . the Notices of Levy on Real Property (Annexes O to NN of Annex C of this Petition) in satisfaction of said assessments were still issued by respondents well beyond the period mandated in Revenue Memorandum Circular No. 38-68. These Notices of Levy were issued only on 22 February 1993 and 20 May 1993 when at least seventeen (17) months had already lapsed from the last service of tax assessment on 12 September 1991. As no notices of distraint of personal property were first issued by respondents, the latter should have complied with Revenue Memorandum Circular No. 38-68 and issued these Notices of Levy not earlier than three (3) months nor later than six (6) months from 12 September 1991. In accordance with the Circular, respondents only had until 12 March 1992 (the last day of the sixth month) within which to issue these Notices of Levy. The Notices of Levy, having been issued beyond the period allowed by law, are thus void and of no effect. 15

We hold otherwise. The Notices of Levy upon real property were issued within the prescriptive period and in accordance with the provisions of the present Tax Code. The deficiency tax assessment, having already become final, executory, and demandable, the same can now be collected through the summary remedy of distraint or levy pursuant to Section 205 of the NIRC. The applicable provision in regard to the prescriptive period for the assessment and collection of tax deficiency in this instance is Article 223 of the NIRC, which pertinently provides: Sec. 223. Exceptions as to a period of limitation of assessment and collection of taxes. ² (a) In the case of a false or fraudulent return with intent to evade tax or of a failure to file a return, the tax may be assessed, or a proceeding in court for the collection of such tax may be begun without assessment, at any time within ten (10) years after the discovery of the falsity, fraud, or omission:Provided, That, in a fraud assessment which has become final and executory, the fact of fraud shall be judicially taken cognizance of in the civil or criminal action for the collection thereof.

xxx xxx xxx (c) Any internal revenue tax which has been assessed within the period of limitation above prescribed, may be collected by distraint or levy or by a proceeding in court within three years following the assessment of the tax. xxx xxx xxx The omission to file an estate tax return, and the subsequent failure to contest or appeal the assessment made by the BIR is fatal to the petitioner's cause, as under the above-cited provision, in case of failure to file a return, the tax may be assessed at any time within ten years after the omission, and any tax so assessed may be collected by levy upon real property within three years following the assessment of the tax. Since the estate tax assessment had become final and unappealable by the petitioner's default as regards protesting the validity of the said assessment, there is now no reason why the BIR cannot continue with the collection of the said tax. Any objection against the assessment should have been pursued following the avenue paved in Section 229 of the NIRC on protests on assessments of internal revenue taxes. Petitioner further argues that "the numerous pending court cases questioning the late president's ownership or interests in several properties (both real and personal) make the total value of his estate, and the consequent estate tax due, incapable of exact pecuniary determination at this time. Thus, respondents' assessment of the estate tax and their issuance of the Notices of Levy and sale are premature and oppressive." He points out the pendency of Sandiganbayan Civil Case Nos. 0001-0034 and 0141, which were filed by the government to question the ownership and interests of the late President in real and personal properties located within and outside the Philippines. Petitioner, however, omits to allege whether the properties levied upon by the BIR in the collection of estate taxes upon the decedent's estate were among those involved in the said cases pending in the Sandiganbayan. Indeed, the court is at a loss as to how these cases are relevant to the matter at issue. The mere fact that the decedent has pending cases involving ill-gotten wealth does not affect the enforcement of tax assessments over the properties indubitably included in his estate. Petitioner also expresses his reservation as to the propriety of the BIR's total assessment of P23,292,607,638.00, stating that this amount deviates from the findings of the Department of Justice's Panel of Prosecutors as per its resolution of 20 September 1991. Allegedly, this is clear evidence of the uncertainty on the part of the Government as to the total value of the estate of the late President. This is, to our mind, the petitioner's last ditch effort to assail the assessment of estate tax which had already become final and unappealable. It is not the Department of Justice which is the government agency tasked to determine the amount of taxes due upon the subject estate, but the Bureau of Internal Revenue, 16 whose determinations and assessments are presumed correct and made in good faith. 17 The taxpayer has the duty of proving otherwise. In the absence of proof of any irregularities in the performance of official duties, an assessment will not be disturbed. Even an assessment based on estimates is prima facie valid and lawful where it does not appear to have been arrived at arbitrarily or capriciously. The burden of proof is upon the complaining party to show clearly that the assessment is erroneous. Failure to present proof of error in the assessment will justify the judicial affirmance of said assessment. 18 In this instance, petitioner has not pointed out one single provision in the Memorandum of the Special Audit Team which gave rise to the questioned assessment, which bears a trace of falsity. Indeed, the petitioner's attack on the assessment bears mainly on the alleged improbable and

unconscionable amount of the taxes charged. But mere rhetoric cannot supply the basis for the charge of impropriety of the assessments made. Moreover, these objections to the assessments should have been raised, considering the ample remedies afforded the taxpayer by the Tax Code, with the Bureau of Internal Revenue and the Court of Tax Appeals, as described earlier, and cannot be raised now via Petition for Certiorari, under the pretext of grave abuse of discretion. The course of action taken by the petitioner reflects his disregard or even repugnance of the established institutions for governance in the scheme of a wellordered society. The subject tax assessments having become final, executory and enforceable, the same can no longer be contested by means of a disguised protest. In the main, Certiorari may not be used as a substitute for a lost appeal or remedy. 19 This judicial policy becomes more pronounced in view of the absence of sufficient attack against the actuations of government. On the matter of sufficiency of service of Notices of Assessment to the petitioner, we find the respondent appellate court's pronouncements sound and resilient to petitioner's attacks. Anent grounds 3(b) and (B) ² both alleging/claiming lack of notice ² We find, after considering the facts and circumstances, as well as evidences, that there was sufficient, constructive and/or actual notice of assessments, levy and sale, sent to herein petitioner Ferdinand "Bongbong" Marcos as well as to his mother Mrs. Imelda Marcos. Even if we are to rule out the notices of assessments personally given to the caretaker of Mrs. Marcos at the latter's last known address, on August 26, 1991 and September 12, 1991, as well as the notices of assessment personally given to the caretaker of petitioner also at his last known address on September 12, 1991 ² the subsequent notices given thereafter could no longer be ignored as they were sent at a time when petitioner was already here in the Philippines, and at a place where said notices would surely be called to petitioner's attention, and received by responsible persons of sufficient age and discretion. Thus, on October 20, 1992, formal assessment notices were served upon Mrs. Marcos c/o the petitioner, at his office, House of Representatives, Batasan Pambansa, Q.C. (Annexes "A", "A-1", "A-2", "A-3"; pp. 207-210, Comment/Memorandum of OSG). Moreover, a notice to taxpayer dated October 8, 1992 inviting Mrs. Marcos to a conference relative to her tax liabilities, was furnished the counsel of Mrs. Marcos ² Dean Antonio Coronel (Annex "B", p. 211, ibid). Thereafter, copies of Notices were also served upon Mrs. Imelda Marcos, the petitioner and their counsel "De Borja, Medialdea, Ata, Bello, Guevarra and Serapio Law Office", on April 7, 1993 and June 10, 1993. Despite all of these Notices, petitioner never lifted a finger to protest the assessments, (upon which the Levy and sale of properties were based), nor appealed the same to the Court of Tax Appeals.
There being sufficient service of Notices to herein petitioner (and his mother) and it appearing that petitioner continuously ignored said Notices despite several opportunities given him to file a protest and to thereafter appeal to the Court of Tax Appeals, ² the tax assessments subject of this case, upon which the levy and sale of properties were based, could no longer be contested (directly or indirectly) via this instant petition forcertiorari. 20

Petitioner argues that all the questioned Notices of Levy, however, must be nullified for having been issued without validly serving copies thereof to the petitioner. As a mandatory heir of the decedent,

petitioner avers that he has an interest in the subject estate, and notices of levy upon its properties should have been served upon him. We do not agree. In the case of notices of levy issued to satisfy the delinquent estate tax, the delinquent taxpayer is the Estate of the decedent, and not necessarily, and exclusively, the petitioner as heir of the deceased. In the same vein, in the matter of income tax delinquency of the late president and his spouse, petitioner is not the taxpayer liable. Thus, it follows that service of notices of levy in satisfaction of these tax delinquencies upon the petitioner is not required by law, as under Section 213 of the NIRC, which pertinently states: xxx xxx xxx . . . Levy shall be effected by writing upon said certificate a description of the property upon which levy is made. At the same time, written notice of the levy shall be mailed to or served upon the Register of Deeds of the province or city where the property is located and upon the delinquent taxpayer, or if he be absent from the Philippines, to his agent or the manager of the business in respect to which the liability arose, or if there be none, to the occupant of the property in question. xxx xxx xxx The foregoing notwithstanding, the record shows that notices of warrants of distraint and levy of sale were furnished the counsel of petitioner on April 7, 1993, and June 10, 1993, and the petitioner himself on April 12, 1993 at his office at the Batasang Pambansa. 21 We cannot therefore, countenance petitioner's insistence that he was denied due process. Where there was an opportunity to raise objections to government action, and such opportunity was disregarded, for no justifiable reason, the party claiming oppression then becomes the oppressor of the orderly functions of government. He who comes to court must come with clean hands. Otherwise, he not only taints his name, but ridicules the very structure of established authority. IN VIEW WHEREOF, the Court RESOLVED to DENY the present petition. The Decision of the Court of Appeals dated November 29, 1994 is hereby AFFIRMED in all respects. SO ORDERED. Regalado, Romero, Puno and Mendoza, JJ., concur. Republic of the Philippines SUPREME COURT Manila FIRST DIVISION G.R. No. L-29059 December 15, 1987 COMMISSIONER OF INTERNAL REVENUE, petitioner, vs. CEBU PORTLAND CEMENT COMPANY and COURT OF TAX APPEALS, respondents.

CRUZ, J.: By virtue of a decision of the Court of Tax Appeals rendered on June 21, 1961, as modified on appeal by the Supreme Court on February 27, 1965, the Commissioner of Internal Revenue was ordered to refund to the Cebu Portland Cement Company the amount of P 359,408.98, representing overpayments of ad valorem taxes on cement produced and sold by it after October 1957. 1 On March 28, 1968, following denial of motions for reconsideration filed by both the petitioner and the private respondent, the latter moved for a writ of execution to enforce the said judgment . 2 The motion was opposed by the petitioner on the ground that the private respondent had an outstanding sales tax liability to which the judgment debt had already been credited. In fact, it was stressed, there was still a balance owing on the sales taxes in the amount of P 4,789,279.85 plus 28% surcharge. 3 On April 22, 1968, the Court of Tax Appeals * granted the motion, holding that the alleged sales tax liability of the private respondent was still being questioned and therefore could not be set-off against the refund. 4 In his petition to review the said resolution, the Commissioner of Internal Revenue claims that the refund should be charged against the tax deficiency of the private respondent on the sales of cement under Section 186 of the Tax Code. His position is that cement is a manufactured and not a mineral product and therefore not exempt from sales taxes. He adds that enforcement of the said tax deficiency was properly effected through his power of distraint of personal property under Sections 316 and 318 5 of the said Code and, moreover, the collection of any national internal revenue tax may not be enjoined under Section 305, 6 subject only to the exception prescribed in Rep. Act No. 1125. 7 This is not applicable to the instant case. The petitioner also denies that the sales tax assessments have already prescribed because the prescriptive period should be counted from the filing of the sales tax returns, which had not yet been done by the private respondent. For its part, the private respondent disclaims liability for the sales taxes, on the ground that cement is not a manufactured product but a mineral product. 8 As such, it was exempted from sales taxes under Section 188 of the Tax Code after the effectivity of Rep. Act No. 1299 on June 16, 1955, in accordance with Cebu Portland Cement Co. v. Collector of Internal Revenue, 9 decided in 1968. Here Justice Eugenio Angeles declared that "before the effectivity of Rep. Act No. 1299, amending Section 246 of the National Internal Revenue Code, cement was taxable as a manufactured product under Section 186, in connection with Section 194(4) of the said Code," thereby implying that it was not considered a manufactured product afterwards. Also, the alleged sales tax deficiency could not as yet be enforced against it because the tax assessment was not yet final, the same being still under protest and still to be definitely resolved on the merits. Besides, the assessment had already prescribed, not having been made within the reglementary five-year period from the filing of the tax returns. 10 Our ruling is that the sales tax was properly imposed upon the private respondent for the reason that cement has always been considered a manufactured product and not a mineral product. This matter was extensively discussed and categorically resolved in Commissioner of Internal Revenue v. Republic Cement Corporation, 11 decided on August 10, 1983, where Justice Efren L. Plana, after an exhaustive review of the pertinent cases, declared for a unanimous Court: From all the foregoing cases, it is clear that cement qua cement was never considered as a mineral product within the meaning of Section 246 of the Tax Code, notwithstanding that at least 80% of its components are minerals, for the simple

reason that cement is the product of a manufacturingprocess and is no longer the mineral product contemplated in the Tax Code (i.e.; minerals subjected to simple treatments) for the purpose of imposing the ad valorem tax. What has apparently encouraged the herein respondents to maintain their present posture is the case of Cebu Portland Cement Co. v. Collector of Internal Revenue, L20563, Oct. 29, 1968 (28 SCRA 789) penned by Justice Eugenio Angeles. For some portions of that decision give the impression that Republic Act No. 1299, which amended Section 246, reclassified cement as a mineral product that was not subject to sales tax. ... xxx xxx xxx After a careful study of the foregoing, we conclude that reliance on the decision penned by Justice Angeles is misplaced. The said decision is no authority for the proposition that after the enactment of Republic Act No. 1299 in 1955 (defining mineral product as things with at least 80% mineral content), cement became a 'mineral product," as distinguished from a "manufactured product," and therefore ceased to be subject to sales tax. It was not necessary for the Court to so rule. It was enough for the Court to say in effect that even assuming Republic Act No. 1299 had reclassified cement was a mineral product, the reclassification could not be given retrospective application (so as to justify the refund of sales taxes paid before Republic Act 1299 was adopted) because laws operate prospectively only, unless the legislative intent to the contrary is manifest, which was not so in the case of Republic Act 1266. [The situation would have been different if the Court instead had ruled in favor of refund, in which case it would have been absolutely necessary (1) to make an unconditional ruling that Republic Act 1299 re-classified cement as a mineral product (not subject to sales tax), and (2) to declare the law retroactive, as a basis for granting refund of sales tax paid before Republic Act 1299.] In any event, we overrule the CEPOC decision of October 29, 1968 (G.R. No. L20563) insofar as its pronouncements or any implication therefrom conflict with the instant decision. The above views were reiterated in the resolution 12 denying reconsideration of the said decision, thus: The nature of cement as a "manufactured product" (rather than a "mineral product") is well-settled. The issue has repeatedly presented itself as a threshold question for determining the basis for computing the ad valorem mining tax to be paid by cement Companies. No pronouncement was made in these cases that as a "manufactured product" cement is subject to sales tax because this was not at issue. The decision sought to be reconsidered here referred to the legislative history of Republic Act No. 1299 which introduced a definition of the terms "mineral" and "mineral products" in Sec. 246 of the Tax Code. Given the legislative intent, the holding in the CEPOC case (G.R. No. L-20563) that cement was subject to sales tax prior to the effectivity f Republic Act No. 1299 cannot be construed to mean that, after the law took effect, cement ceased to be so subject to the tax. To erase any and all misconceptions that may have been spawned by reliance on the case of Cebu Portland Cement Co. v. Collector of Internal Revenue, L-20563, October 29, 1968 (28 SCRA 789) penned by Justice Eugenio Angeles, the Court has expressly

overruled it insofar as it may conflict with the decision of August 10, 1983, now subject of these motions for reconsideration. On the question of prescription, the private respondent claims that the five-year reglementary period for the assessment of its tax liability started from the time it filed its gross sales returns on June 30, 1962. Hence, the assessment for sales taxes made on January 16, 1968 and March 4, 1968, were already out of time. We disagree. This contention must fail for what CEPOC filed was not the sales returns required in Section 183(n) but the ad valorem tax returns required under Section 245 of the Tax Code. As Justice Irene R. Cortes emphasized in the aforestated resolution: In order to avail itself of the benefits of the five-year prescription period under Section 331 of the Tax Code, the taxpayer should have filed the required return for the tax involved, that is, a sales tax return. (Butuan Sawmill, Inc. v. CTA, et al., G.R. No. L21516, April 29, 1966, 16 SCRA 277). Thus CEPOC should have filed sales tax returns of its gross sales for the subject periods. Both parties admit that returns were made for the ad valorem mining tax. CEPOC argues that said returns contain the information necessary for the assessment of the sales tax. The Commissioner does not consider such returns as compliance with the requirement for the filing of tax returns so as to start the running of the five-year prescriptive period.
We agree with the Commissioner. It has been held in Butuan Sawmill Inc. v. CTA, supra, that the filing of an income tax return cannot be considered as substantial compliance with the requirement of filing sales tax returns, in the same way that an income tax return cannot be considered as a return for compensating tax for the purpose of computing the period of prescription under Sec. 331. (Citing Bisaya Land Transportation Co., Inc. v. Collector of Internal Revenue, G.R. Nos. L-12100 and L11812, May 29, 1959). There being no sales tax returns filed by CEPOC, the statute of stations in Sec. 331 did not begin to run against the government. The assessment made by the Commissioner in 1968 on CEPOC's cement sales during the period from July 1, 1959 to December 31, 1960 is not barred by the five-year prescriptive period. Absent a return or when the return is false or fraudulent, the applicable period is ten (10) days from the discovery of the fraud, falsity or omission. The question in this case is: When was CEPOC's omission to file tha return deemed discovered by the government, so as to start the running of said period? 13

The argument that the assessment cannot as yet be enforced because it is still being contested loses sight of the urgency of the need to collect taxes as "the lifeblood of the government." If the payment of taxes could be postponed by simply questioning their validity, the machinery of the state would grind to a halt and all government functions would be paralyzed. That is the reason why, save for the exception already noted, the Tax Code provides: Sec. 291. Injunction not available to restrain collection of tax. ² No court shall have authority to grant an injunction to restrain the collection of any national internal revenue tax, fee or charge imposed by this Code. It goes without saying that this injunction is available not only when the assessment is already being questioned in a court of justice but more so if, as in the instant case, the challenge to the assessment is still-and only-on the administrative level. There is all the more reason to apply the rule here because it appears that even after crediting of the refund against the tax deficiency, a balance of more than P 4 million is still due from the private respondent.

To require the petitioner to actually refund to the private respondent the amount of the judgment debt, which he will later have the right to distrain for payment of its sales tax liability is in our view an Idle ritual. We hold that the respondent Court of Tax Appeals erred in ordering such a charade. WHEREFORE, the petition is GRANTED. The resolution dated April 22, 1968, in CTA Case No. 786 is SET ASIDE, without any pronouncement as to costs. SO ORDERED. Teehankee, C.J., Narvasa, Paras and Gancayco, JJ., concur. Republic of the Philippines SUPREME COURT Manila EN BANC G.R. No. L-8840 February 8, 1957

THE COLLECTOR OF INTERNAL REVENUE, petitioner, vs. JOSE C. ZULUETA and THE COURT OF TAX APPEALS, respondents. Office of the Solicitor General Ambrosio Padilla, Solicitor Jose P. Alejandro and Special Attorney Librada del Rosario-Natividad for petitioner. Lorenzo F. Miracvite, and Jose C. Zulueta in his own behalf as respondent. FELIX, J.: This is a petition for certiorari filed by the Collector of Internal Revenue with this Court wherein he prays that the resolution of the Court of Tax Appeals, ordering him to desist from proceeding through the extra-judicial methods of distraint and levy with the collection of the alleged income tax liabilities of Jose C. Zulueta (Annex D), be declared null and void. The facts of the case may be stated as follows: On February 10, 1954, respondent Jose C. Zulueta (who had not filed his income tax returns for the years 1945 to 1948 and 1950 ² Annex N), received a letter from the Collector of Internal Revenue informing him that his income tax deficiency for the years 1945 to 1951, inclusive amounted to P550,527.50 (Annex A). Respondent Zulueta immediately sent a query as to how this amount was arrived at, asking for a particularized statement thereof and at the same time protesting against said assessment on the ground that he had religiously paid his annual income tax liabilities (Annex C). This communication was answered by the Acting Collector of Internal Revenue informing him that the bases of the assessment were embodied in working sheets in his Office which were made available to respondent Zulueta or his duly authorized representative and gave the latter 30 days from receipt of that letter for the purpose of verifying said assessment (Annex D). It appearing that respondent Jose C. Zulueta failed to submit a memorandum in support of his contention that the assessment on his income tax was erroneous, the Collector of Internal Revenue, on June 3, 1954, required said taxpayer to pay the taxes demanded of him amounting to P616,630.81 not later than June 30, 1954 (Annex F). An exchange of communications between them ensued wherein respondent was granted several extensions of time within which to file his said memorandum.

On November 10, 1954, the Collector of Internal Revenue sent to the City Fiscal of Manila papers pertinent to the possible prosecution of Jose C. Zulueta on account of the latter's failure to file his income tax returns in violation of section 45 of the National Internal Revenue Code, if such action was warranted by the circumstances of the case (Annex N). And on December 29, 1954, the City Treasurer of Manila placed under distraint and levy certain real properties of the respondent taxpayer described in the warrant (Annex O) to be sold at public auction on February 21, 1954 (this must be a clerical error as to the year, which should be in 1955 instead of 1954), to meet the amount of P550,326.50 representing deficiency income taxes for 1945 to 1951, plus the corresponding deficiency penalties, which sale was published in the January 24, 1955 issue of the Manila Times. Thereafter, respondent Jose C. Zulueta filed with the Court of Tax Appeals on January 17, 1955, a petition to review the deficiency income tax assessment made by the Collector of Internal Revenue and on January 26, 1955, filed an urgent petition to enjoin the Collector of Internal Revenue and the City Treasurer of Manila from proceeding with the contemplated sale of his properties, on the ground that the right to collect by summary proceedings his alleged income tax deficiency for 1945 to 1950 had already prescribed; that the extra-judicial methods of collection by distraint and levy contravenes the mandate of section 51 (d) of the Tax Code; that the assessment of his tax liability for 1951 is unwarranted because all his income for that year originated from war damage payments which are not taxable under Republic Act No. 227; that the execution of the warrant of distraint and levy would result in injustice and irreparable injury to him; and that the filing of a bond is not necessary as the interest of the State will not be prejudiced due to the existence of a tax lien in its favor as provided for by Section 315 of the Tax Code. After proper hearing, the respondent Court declared the proper order of distraint and levy against the properties of respondent Zulueta to insure the collection of alleged income tax deficiency for 1945, 1946, 1947, 1948 and 1950 null and void, but required said respondent to file a bond for P116,000 in favor of the Republic of the Philippines to guarantee the payment of his income tax and surcharges for the year 1951, before issuing the writ of injunction to restrain the herein petitioner from proceeding with the scheduled sale of respondent's properties (Annex B). After the bond in said amount was posted, the Court of Tax Appeals issued its order of February 18, 1955, enjoining the Collector of Internal Revenue and the City Treasurer of Manila from selling any real or personal property of Jose C. Zulueta at public auction pending the outcome of the appeal (Annex E). The records show that respondent Jose C. Zulueta was able to present evidence to prove that he had filed his income tax returns for the years 1949 and 1951 which he did on May 11, 1950, and may 15, 1952, respectively. However, he failed to present duplicate copies of his income tax returns for the years 1945, 1946, 1947, 1948 and 1950 although they claimed that they had been already settled. But this fact will not alter the situation for the legal provision that applies in this case is section 51 (d) of the National Internal Revenue Code which reads as follows: SEC. 51. ASSESSMENT AND PAYMENT OF INCOME TAX. ² xxx xxx xxx

(d) Refusal or neglect to make return; fraudulent return, etc. ² In cases of refusal or neglect to make a return and in cases of erroneous, false or fraudulent returns, the Collector of Internal Revenue shall, upon the discovery thereof, at any time within three years after said return is due, or has been made, make a return upon information obtained as provided for in this code or by existing law, or require the necessary corrections to be made, and the assessment made by the Collector of Internal Revenue thereon shall be paid by such person or corporation immediately upon notification of the amount of such assessment.

It will be noted that this section treats not only of cases where false or fraudulent returns are filed, but also where the taxpayer refuses or neglects to file the same. We agree with the lower court when it states that: . . . the computation of the three-year prescriptive period provided therein varies, depending on whether or not the taxpayer has filed its income tax return for a particular year. If he has filed one, the running of the prescriptive period of three years commences from the time the return shall have been filed. In those cases where there has been a neglect or refusal to file one, the period commences from the date when the return is due, which is March 1 of the succeeding year. As construed by this Tribunal in several decisions, the three-year prescriptive period provided for in the aforequoted section is meant to serve as a limitation on the right of the Government to collect income taxes by the summary methods of distraint and levy, although it could proceed to recover the taxes due by the institution of the corresponding civil action (Collector of Internal Revenue vs. Villegas, 56 Phil. 554; Collector of Internal Revenuevs. Haygood, 65 Phil. 520; Juan de la Viña vs. El Gobierno de las Filipinas, G.R. No. 42669, January 29, 1938; Collector of Internal Revenue vs. Jose Avelino et al., supra, p. 327 and Collector of Internal Revenue vs. Aurelio P. Reyes et al., supra, p. 822). The Collector of Internal Revenue in the instant case issued the warrant of distraint and levy against certain real properties of the respondent Jose C. Zulueta for the collection of deficiency income taxes for the years 1945, 1946, 1947, 1948 and 1950 only on December 29, 1954, or 3 years, 9 months and 28 days from March 1, 1951, when the return for the tax year 1950 should have been due. It is very patent therefore that the order of the Collector of Internal Revenue to effect collection of the alleged deficiency income taxes through summary administrative proceedings, having been issued well beyond the three-year period of limitation, was null and void. We disagree with the contention of petitioner that the act of the respondent Court in declaring the order of distraint and levy a nullity was done in excess of its jurisdiction, because said pronouncement was made in the lawful exercise of its power to pass upon all matters brought before it and sanctioned by section 7 of Republic Act No. 1125, which reads as follows: SEC. 7. Jurisdiction. ² The Court of Tax Appeals shall exercise exclusive jurisdiction to review by appeal, as herein provided ² (1) Decisions of the Collector of Internal Revenue in cases involving disputed assessments, refunds of internal revenue taxes, fees or other charges, penalties imposed in relation thereto, or other matters arising under the National Internal Revenue Code or other law or part of law administered by the Bureau of Internal Revenue. Petitioner likewise asserts that even assuming that the respondent Court of Tax Appeals had jurisdiction to order him to desist from collecting through summary administrative methods the taxes due from respondent Zulueta, yet the Court committed a grave abuse of discretion in its failure to require the filing of a bond or deposit the amount assessed for the tax years 1945, 1946, 1947, 1948 and 1950. This Court had occasion to pass upon this matter squarely in the case of Collector of Internal Revenue vs. Aurelio P. Reyes and the Court of Tax Appeals, (supra, p. 822) wherein it was held that: . . . At first blush it might be as contended by the Solicitor General, but a careful analysis of the second paragraph of said section 11 will lead Us to the conclusion that the requirement

of the bond as a condition precedent to the issuance of a writ of injunction applies only in cases where the processes by which the collection sought to be made by means thereof are carried out in consonance with law for such cases provided and not when said processes are obviously in violation of the law to the extreme that they have to be SUSPENDED for jeopardizing the interests of the taxpayer. Section 11 of Republic Act No. 1125 is therefore premised on the assumption that the collection by summary proceedings is by itself in accordance with existing laws; and then what is suspended is the act of collecting, whereas, in the case at bar, what the respondent Court suspended was the use of the method employed to verify the collection which was evidently illegal after the lapse of the three-year limitation period. The respondent Court issued the injunction in question on the basis of its findings that the means intended to be used by petitioner in the collection of the alleged deficiency taxes were in violation of law. It would certainly be an absurdity on the part of the Court of Tax Appeals to declare that the collection by the summary methods of distraint and levy was violative of the law, and then, on the same breath, require the petitioner to deposit or file a bond as a pre-requisite of the issuance of a writ of injunction. Let us suppose, for the sake of argument, that the Court a quo would have required the petitioner to post the bond in question and that the taxpayer would refuse or fail to furnish said bond, would the Court a quo be obliged to authorized or allow the Collector of Internal Revenue to proceed with the collection from the petitioner of the taxes due by a means it previously declared to be contrary to law? It is for this reason that the respondent Court in the case at bar required respondent Zulueta to post only a bond for P116,000 in favor of the Government to guarantee the collection of his income tax deficiency for the year 1951, before the writ of injunction was issued, and declined to order a similar requirement with respect to the income taxes for the years 1945, 1946, 1947, 1948 and 1950. Wherefore, the petition for certiorari filed by the Collector of Internal Revenue praying that the resolution of the respondent Court of Tax Appeals dated February 17, 1955, restraining the herein petitioner from selling the Manila properties of Jose C. Zulueta to satisfy his income tax liabilities for the years 1945, 1946, 1947, 1948 and 1950 be declared null and void, is hereby denied, without pronouncement as to costs. It is so ordered. Paras, C.J., Bengzon, Padilla, Montemayor, Reyes, A., Bautista Angelo, Labrador and Endencia, JJ., concur. Republic of the Philippines SUPREME COURT Manila EN BANC G.R. No. L-23534 May 16, 1967

JOSE A. ARCHES, petitioner-appellant, vs. ANACLETO I. BELLOSILLO and JAIME ARANETA, respondents-appellees. Jose A. Arches for petitioner-appellant. Office of the Solicitor General Arturo A. Alafriz, Solicitor A.B. Afurong and Atty. S.S. Soriano for respondents-appellees.

BENGZON, J.P., J.: Petitioner-appellant Jose Arches filed on February 27, 1954 his income tax return for 1953. Within five years thereafter, or on February 26, 1959, deficiency income tax and residence tax assessments were issued against him. Said assessments not having been disputed, the Republic represented by the Bureau of Internal Revenue Regional, Director, filed suit on December 29, 1960, in the municipal court of Roxas City, to recover from petitioner-appellant the sum of P4,441.25 as deficiency income tax and additional residence tax for 1953. Arches then moved to dismiss the complaint on the ground that it did not expressly show the approval of the Revenue Commissioner, as required by Section 308 of the Tax Code, and on the further ground of prescription of the action.
1äwp hï1. ñët

The municipal court denied the motion. Petitioner-appellant, his motion to reconsider having been denied also, resorted to the Court of First Instance of Capiz on a petition for certiorari and prohibition assailing the order denying his motion to dismiss. The trial court dismissed the petition. Hence, this appeal. The only question here is the correctness of dismissal of the petition by the Court of First Instance. The order was predicated upon the impropriety of the writ. We find no error committed by said court. The municipal court had jurisdiction over the parties and over the subject matter, the amount demanded being less than P5,000.00.1 The suit below instituted by the Republic, based on an uncontested assessment, was one merely for the recovery of a sum of money where the amount demanded constitutes the jurisdictional test.2 Petitioner-appellant would make much of the lack of approval of the Revenue Commissioner. First of all, in this case, such requisite is not jurisdictional, but one relating to capacity to sue or affecting the cause of action only.3So, in ruling on said question, whatever error ² if any ² the municipal court committed, was merely an error of judgment, not correctible by certiorari.4 Neither was there grave abuse of the discretion on the part of the municipal court in ruling that the express approval of the Revenue Commissioner himself was not necessary. The court relied upon Memorandum Order No. V-634 of the Revenue Commissioner, approved by the Finance Secretary of July 1, 1956, wherein the former's functions regarding the administration and enforcement of revenue laws and regulations ² powers broad enough to cover the approval of court actions as required in Section 308 of the Tax Code ² were expressly delegated to the Regional Directors. This regulation, the issuance of which was authorized by statute, has the force and effect of law.5 To rely upon it, hence, would not be tantamount to whimsical, capricious and arbitrary exercise of judgment. The verification by the Regional Director of the complaint constitutes sufficient approval thereof already. It states,inter alia, that said Director has caused the preparation of the complaint and that he has read the allegations thereof and they are true and correct to the best of his knowledge and belief. Pleadings are to be liberally construed.6 Assuming, therefore, in gratia argumenti, that the suit is being erroneously ² but not invalidly ² entertained, for lack of express approval of the Commissioner or the Regional Director, certiorari would still not lie. An order denying a motion to dismiss is interlocutory and the remedy of the unsuccessful movant is to await the judgment on the merits and then appeal therefrom.7 And, as the Court of First Instance rightly observed, there was no showing of a special reason or urgent need to stop the proceedings at such early stage in the municipal court.

Petitioner-appellant would also raise the question of prescription. Again, this is not jurisdictional. And, We have already ruled8 that the proper prescriptive period for bringing civil actions is five years from the date of the assessment, under Section 332 of the Tax Code. The three-year period urged by petitioner-appellant under Section 51 (d) refers only to the summary remedies of distraint and levy. Here, the action was commenced one year, ten months and three days after the assessments were made; hence, well within the period. Wherefore, the dismissal of appellant's petition for certiorari by the Court of First Instance is hereby affirmed. Costs against petitioner-appellant. So ordered. Concepcion, C.J., Reyes, J.B.L., Dizon, Regala, Makalintal, Zaldivar, Sanchez and Castro, JJ., concur. Republic of the Philippines SUPREME COURT Manila SECOND DIVISION

G.R. No. 130430 December 13, 1999 REPUBLIC OF THE PHILIPPINES, represented by the Commissioner of the Bureau of Internal Revenue (BIR), petitioner, vs. SALUD V. HIZON, respondent.

MENDOZA, J.: This is a petition for review of the decision 1 of the Regional Trial Court, Branch 44, San Fernando, Pampanga, dismissing the suit filed by the Bureau of Internal Revenue for collection of tax. The facts are as follows: On July 18, 1986, the BIR issued to respondent Salud V. Hizon a deficiency income tax assessment of P1,113,359.68 covering the fiscal year 1981-1982. Respondent not having contested the assessment, petitioner, on January 12, 1989, served warrants of distraint and levy to collect the tax deficiency. However, for reasons not known, it did not proceed to dispose of the attached properties. More than three years later, or on November 3, 1992, respondent wrote the BIR requesting a reconsideration of her tax deficiency assessment. The BIR, in a letter dated August 11, 1994, denied the request. On January 1, 1997, it filed a case with the Regional Trial Court, Branch 44, San Fernando, Pampanga to collect the tax deficiency. The complaint was signed by Norberto Salud, Chief of the Legal Division, BIR Region 4, and verified by Amancio Saga, the Bureau's Regional Director in Pampanga. Respondent moved to dismiss the case on two grounds: (1) that the complaint was not filed upon authority of the BIR Commissioner as required by §221 2 of the National Internal Revenue Code,

and (2) that the action had already prescribed. Over petitioner's objection, the trial court, on August 28, 1997, granted the motion and dismissed the complaint. Hence, this petition. Petitioner raises the following issues: 3 I. WHETHER OR NOT THE INSTITUTION OF THE CIVIL CASE FOR COLLECTION OF TAXES WAS WITHOUT THE APPROVAL OF THE COMMISSIONER IN VIOLATION OF SECTION 221 OF THE NATIONAL INTERNAL REVENUE CODE. II. WHETHER OR NOT THE ACTION FOR COLLECTION OF TAXES FILED AGAINST RESPONDENT HAD ALREADY BEEN BARRED BY THE STATUTE OF LIMITATIONS. First. In sustaining respondent's contention that petitioner's complaint was filed without the authority of the BIR Commissioner, the trial court stated: 4 There is no question that the National Internal Revenue Code explicitly provides that in the matter of filing cases in Court, civil or criminal, for the collection of taxes, etc., the approval of the commissioner must first be secured. . . . [A]n action will not prosper in the absence of the commissioner's approval. Thus, in the instant case, the absence of the approval of the commissioner in the institution of the action is fatal to the cause of the plaintiff . . . . The trial court arrived at this conclusion because the complaint filed by the BIR was not signed by then Commissioner Liwayway Chato. Sec. 221 of the NIRC provides: Form and mode of proceeding in actions arising under this Code. ² Civil and criminal actions and proceedings instituted in behalf of the Government under the authority of this Code or other law enforced by the Bureau of Internal Revenue shall be brought in the name of the Government of the Philippines and shall be conducted by the provincial or city fiscal, or the Solicitor General, or by the legal officers of the Bureau of Internal Revenue deputized by the Secretary of Justice, but no civil and criminal actions for the recovery of taxes or the enforcement of any fine, penalty or forfeiture under this Code shall begun without the approval of the Commissioner. (Emphasis supplied) To implement this provision Revenue Administrative Order No. 5-83 of the BIR provides in pertinent portions: The following civil and criminal cases are to be handled by Special Attorneys and Special Counsels assigned in the Legal Branches of Revenues Regions: xxx xxx xxx II. Civil Cases 1. Complaints for collection on cases falling within the jurisdiction of the Region . . . .

In all the abovementioned cases, the Regional Director is authorized to sign all pleadings filed in connection therewith which, otherwise, requires the signature of the Commissioner. xxx xxx xxx Revenue Administrative Order No. 10-95 specifically authorizes the Litigation and Prosecution Section of the Legal Division of regional district offices to institute the necessary civil and criminal actions for tax collection. As the complaint filed in this case was signed by the BIR's Chief of Legal Division for Region 4 and verified by the Regional Director, there was, therefore, compliance with the law. However, the lower court refused to recognize RAO No. 10-95 and, by implication, RAO No. 5-83. It held:
[M]emorand[a], circulars and orders emanating from bureaus and agencies whether in the purely public or quasi-public corporations are mere guidelines for the internal functioning of the said offices. They are not laws which courts can take judicial notice of. As such, they have no binding effect upon the courts for such memorand[a] and circulars are not the official acts of the legislative, executive and judicial departments of the Philippines. . . . 5

This is erroneous. The rule is that as long as administrative issuances relate solely to carrying into effect the provisions of the law, they are valid and have the force of law. 6 The governing statutory provision in this case is §4(d) of the NIRC which provides: Specific provisions to be contained in regulations. ² The regulations of the Bureau of Internal Revenue shall, among other things, contain provisions specifying, prescribing, or defining: xxx xxx xxx (d) The conditions to be observed by revenue officers, provincial fiscals and other officials respecting the institution and conduct of legal actions and proceedings. RAO Nos. 5-83 and 10-95 are in harmony with this statutory mandate. As amended by R.A. No. 8424, the NIRC is now even more categorical. Sec. 7 of the present Code authorizes the BIR Commissioner to delegate the powers vested in him under the pertinent provisions of the Code to any subordinate official with the rank equivalent to a division chief or higher, except the following: (a) The power to recommend the promulgation of rules and regulations by the Secretary of Finance; (b) The power to issue rulings of first impression or to reverse, revoke or modify any existing ruling of the Bureau;
(c) The power to compromise or abate under §204 (A) and (B) of this Code, any tax deficiency: Provided,however, that assessment issued by the Regional Offices involving basic deficiency taxes of five hundred thousand pesos (P500,000.00) or less, and minor criminal violations as may be determined by rules and regulations to be promulgated by

the Secretary of Finance, upon the recommendation of the Commissioner, discovered by regional and district officials, may be compromised by a regional evaluation board which shall be composed of the Regional Director as Chairman, the Assistant Regional Director, heads of the Legal, Assessment and Collection Divisions and the Revenue District Officer having jurisdiction over the taxpayer, as members; and

(d) The power to assign or reassign internal revenue officers to establishments where articles subject to excise tax are produced or kept. None of the exceptions relates to the Commissioner's power to approve the filing of tax collection cases. Second. With regard to the issue that the case filed by petitioner for the collection of respondent's tax deficiency is barred by prescription, §223(c) of the NIRC provides:
Any internal revenue tax which has been assessed within the period of limitation aboveprescribed may be collected by distraint or levy or by a proceeding in court within three years 7 following the assessment of the tax.

The running of the three-year prescriptive period is suspended 8 ² for the period during which the Commissioner is prohibited from making the assessment or beginning distraint or levy or a proceeding in court and for sixty days thereafter; when the taxpayer requests for a reinvestigation which is granted by the Commissioner; when the taxpayer cannot be located in the address given by him in the return filed upon which the tax is being assessed or collected; provided, that, if the taxpayer informs the Commissioner of any change in address, the running of the statute of limitations will not be suspended; when the warrant of distraint or levy is duly served upon the taxpayer, his authorized representative or a member of his household with sufficient discretion, and no property could be located; and when the taxpayer is out of the Philippines. Petitioner argues that, in accordance with this provision, respondent's request for reinvestigation of her tax deficiency assessment on November 3, 1992 effectively suspended the running of the period of prescription such that the government could still file a case for tax collection. 9 The contention has no merit. Sec. 229 10 of the Code mandates that a request for reconsideration must be made within 30 days from the taxpayer's receipt of the tax deficiency assessment, otherwise the assessment becomes final, unappealable and, therefore, demandable. 11 The notice of assessment for respondent's tax deficiency was issued by petitioner on July 18, 1986. On the other hand, respondent made her request for reconsideration thereof only on November 3, 1992, without stating when she received the notice of tax assessment. She explained that she was constrained to ask for a reconsideration in order to avoid the harassment of BIR collectors. 12 In all likelihood, she must have been referring to the distraint and levy of her properties by petitioner's agents which took place on January 12, 1989. Even assuming that she first learned of the deficiency assessment on this date, her request for reconsideration was nonetheless filed late since she made it more than 30 days thereafter. Hence, her request for reconsideration did not suspend the running of the prescriptive period provided under §223(c). Although the Commissioner acted on her request by eventually denying it on August 11, 1994, this is of no moment and does not detract from the fact that the assessment had long become demandable.

Nonetheless, it is contended that the running of the prescriptive period under §223(c) was suspended when the BIR timely served the warrants of distraint and levy on respondent on January 12, 1989. 13 Petitioner cites for this purpose our ruling in Advertising Associates Inc., v. Court of Appeals. 14 Because of the suspension, it is argued that the BIR could still avail of the other remedy under §223(c) of filing a case in court for collection of the tax deficiency, as the BIR in fact did on January 1, 1997. Petitioner's reliance on the Court's ruling in Advertising Associates Inc. v. Court of Appeals is misplaced. What the Court stated in that case and, indeed, in the earlier case of Palanca v. Commissioner of Internal Revenue, 15 is that the timely service of a warrant of distraint or levy suspends the running of the period to collect the tax deficiency in the sense that the disposition of the attached properties might well take time to accomplish, extending even after the lapse of the statutory period for collection. In those cases, the BIR did not file any collection case but merely relied on the summary remedy of distraint and levy to collect the tax deficiency. The importance of this fact was not lost on the Court. Thus, in Advertising Associates, it was held: 16 "It should be noted that the Commissioner did not institute any judicial proceeding to collect the tax. He relied on the warrants of distraint and levy to interrupt the running of the statute of limitations. Moreover, if, as petitioner in effect says, the prescriptive period was suspended twice, i.e., when the warrants of distraint and levy were served on respondent on January 12, 1989 and then when respondent made her request for reinvestigation of the tax deficiency assessment on November 3, 1992, the three-year prescriptive period must have commenced running again sometime after the service of the warrants of distraint and levy. Petitioner, however, does not state when or why this took place and, indeed, there appears to be no reason for such. It is noteworthy that petitioner raised this point before the lower court apparently as an alternative theory, which, however, is untenable. For the foregoing reasons, we hold that petitioner's contention that the action in this case had not prescribed when filed has no merit. Our holding, however, is without prejudice to the disposition of the properties covered by the warrants of distraint and levy which petitioner served on respondent, as such would be a mere continuation of the summary remedy it had timely begun. Although considerable time has passed since then, as held inAdvertising Associates Inc. v. Court of Appeals 17 and Palanca v. Commissioner of Internal Revenue, 18 the enforcement of tax collection through summary proceedings may be carried out beyond the statutory period considering that such remedy was seasonably availed of. WHEREFORE, the petition is DENIED. Bellosillo, Quisumbing, Buena and De Leon, Jr., JJ., concur. Republic of the Philippines SUPREME COURT Manila FIRST DIVISION G.R. No. L-39910 September 26, 1988 CECILIA TEODORO DAYRIT, TORIBIA TEODORO CASTANEDA, PRUDENCIO J. TEODORO, FRANCISCO J. TEODORO, AND JOSEFINA TEODORO TIONGSON, petitioners, vs. THE HONORABLE FERNANDO A. CRUZ, Presiding Judge, Branch XII, Court of First Instance

of Rizal, and MISAEL P. VERA, in his capacity as the Commissioner of Internal Revenue, respondents. Atienza, Tabora, Del Rosario & Castillo Law Offices and Tanada, Sanchez, Tanada & Tanada Law Offices for petitioners.

GANCAYCO, J.: The application of tax amnesty to the estate of the Teodoros is the issue in this case. Petitioners are the legitimate children and heirs of the deceased spouses Marta J. Teodoro who died intestate on July 1, 1965 and Don Toribio Teodoro who died testate on August 30, 1965. Thereafter, the heirs of the deceased filed separate estate and inheritance tax returns for the estates of the late spouses with the Bureau of Internal Revenue. * In the meantime, testate and intestate proceedings for the settlement of the decedents' estates were filed 1 by Cecilia Teodoro-Dayrit, one of the petitioners herein, in the then Court of First Instance of Caloocan City, ** Branch XII docketed as Special Proceedings No. C-113. 2 On August 14, 1968, said petitioner was appointed administratrix of the estate of Dona Marta and letters testamentary was issued in her favor as executrix of the estate of Don Toribio. On August 9,1972, the respondent Commissioner of Internal Revenue issued the following deficiency estate and inheritance tax assessments: 3
Estate of Doña Marta Estate Tax & penalties Inheritance Tax & interests P1,662,072.34 1,747,790.94 Estate of Don Toribio *** P1,542,293.01 1,518,458.72

The aforementioned notice of deficiency assessments was received by petitioner Dayrit on August 14, 1972. In a letter dated October 7, 1972, **** petitioners through counsel, asked for a reconsideration of the said assessments alleging that the same are contrary to law and not supported by sufficient evidence. 4 In the same letter, petitioners requested a period of thirty (30) days within which to submit their position paper in support of their claim. Meanwhile, on October 16, 1972, Presidential Decree (P.D) No. 23, entitled "Proclaiming Tax Amnesty Subject to Certain Conditions," was issued by then President Ferdinand E. Marcos, quoted hereunder as follows: xxx xxx xxx 1. In all cases of voluntary disclosure of previously untaxed income realized here or abroad by any taxpayer, natural or juridical, the collection of the income tax and penalties incident to nonpayment, as well as all criminal and civil liabilities under the National Internal Revenue Code, the Revised Penal Code, the Anti-Graft and Corrupt Practices Act or any other law applicable thereto, is hereby condoned and, in lieu

thereof, a tax of TEN PERCENTUM (10%) on such previously untaxed income is hereby imposed, subject to the following conditions: (a) Such previously untaxed income must have been earned or realized prior to 1972; (b) The taxpayer must file a notice and return with the Commissioner of Internal Revenue on or before March 31, 1972 showing such previously untaxed income; ... 2. The tax imposed under Paragraph 1 hereof, shall be paid within the following period: (a) If the amount does not exceed P10,000.00 the tax must be paid at the time of the filing of notice and return but not later than March 31, 1973; (b) If the amount exceeds P10,000.00 the tax maybe paid in two (2) installments, the first installment to be paid upon the filing of the notice and return but not later than March 31, 1973; and the second installment within three (3) months from the date of the filing of the return but not later than June 30, 1973. .... On November 24, 1972, P.D. No. 67, was issued amending paragraphs 1 and 3 of P.D. No. 23, to read as follows: xxx xxx xxx 1. In all cases of voluntary disclosure of previously untaxed income and/or wealth such as earnings, receipts, gifts bequests or any other acquisitions from any source whatsoever which are taxable under the National Internal Revenue Code, as amended realized here or abroad by any taxpayer, natural or juridical; the collection of all internal revenue taxes including the increments or penalties on account of nonpayment as well as all civil, criminal or administrative liabilities arising from or incident to such disclosures under the National Internal Revenue Code, the Revised Penal Code, the Anti-Graft and Corrupt Practices Act, the Revised Administrative Code, the Civil Service laws and regulations, laws and regulations on Immigration and Deportation, or any other applicable law or proclamation, are hereby condoned and, in lieu thereof, a tax of ten per centum (10%) on such previously untaxed income or wealth is hereby imposed, subject to the following conditions: a. Such previously untaxed income and/or wealth must have been earned or realized prior to 1972; b. The taxpayer must file a return with the Commissioner of Internal Revenue on or before March 31, 1973, showing such previously untaxed income and/or wealth; .... In a tax return dated March 31, 1973, petitioner Cecilia Teodoro-Dayrit declared an additional amount of P3,655,595.78 as part of the estates of the Teodoro spouses, for additional valuation over and above the amount declared in the previous return for estates and inheritance taxes of the said

late spouses. 5 The Bureau of Internal Revenue issued tax payment acceptance order Nos.112718586 and 1533011. 6 Pursuant to the aforesaid tax acceptance orders, the estates and heirs of the deceased spouses Teodoro paid the amounts of P5,000.00, P30,046.68 and P250,000.00 per official receipts Nos. 73201, 774037 and 964467 dated April 2, 1973, July 17, 1973 and October 31, 1973, respectively, 7 amounting to a total of P285,046.68. On March 14, 1974, respondent Commissioner of Internal Revenue filed a motion for Allowance of Claim against the estates of spouses Teodoro and for an order of payment of taxes in S.P. No. C113 with the then Court of First Instance of Rizal, Branch XII, praying that petitioner Dayrit be ordered to pay the Bureau of Internal Revenue the sum of P6,470,396.81 plus surcharges and interest 8 Petitioners filed two (2) separate oppositions alleging that the estate and inheritance taxes sought to be collected have already been settled in accordance with the provisions of P.D. No. 23, as amended by P.D. No. 67 and that at any rate, the assessments have not become final and executory. 9 In reply thereto, respondent Commissioner alleged that petitioners could not avail of the tax amnesty in view of the existence of a prior assessment. 10 Petitioners insisted that the tax amnesty could still be availed of invoking Section 4, BIR Revenue Regulation No. 8-72. 11 On July 10, 1974, respondent Judge issued an order approving the claim of respondent Commissioner and directing the payment of the estate and inheritance taxes. 12 Dissastisfied with the decision, petitioners filed a motion for reconsideration 13 but it was denied 14 in an order dated September 30, 1974.***** Hence, the present petition. Petitioners contend that respondent Judge acted without jurisdiction or in excess of jurisdiction or with grave abuse of discretion amounting to lack of jurisdiction in granting the respondent Commissioner's claim for estate and inheritance taxes against the estates of the Teodoro spouses on the ground that due to the pendency of their motion for reconsideration of the deficiency assessments issued by the Commissioner, said tax assessments are not yet final and executory. Petitioners stressed that the absence of a decision on the disputed assessments was a bar against collection of taxes. Finally, petitioners insist that their act of filing an estate and inheritance tax return of a previously untaxed wealth of the estates entitles said estates to tax amnesty under P.D. No. 23, as amended by P.D. 67 and hence, it is an error to grant respondent Commissioner's claim for collection of estate and inheritance taxes. On the other hand, respondent Commissioner contends that petitioners cannot avail of the tax amnesty in view of the prior existing assessments issued against the estates of the deceased spouses before the promulgation of P.D. No. 23. In support thereof, respondent cited Section 4 of Revenue Regulation No. 15-72, amending Section 4 of Regulation No. 8-12. Respondent Commissioner contends further that neither may petitioners' act of filing a return of a previously untaxed income or wealth in the amount of P3,655,595.98 entitled the estates to tax amnesty where petitioners failed to pay the 10% tax in full within the time frame required under P.D. No. 23, and that to allow petitioners to avail of the tax amnesty will render nugatory the provisions of P.D. No. 68. Moreover, said respondent argues that certiorari is not the proper remedy in that respondent Judge committed no grave abuse of discretion in allowing the claim for collection of taxes and that if at all, it was merely an error of judgment which can be corrected only on appeal, and in which case the reglementary period for the same has already prescribed. The main issue in this petition is whether an estate may avail of tax amnesty under Presidential Decree No. 23 where there is already an existing assessment made prior to the issuance of the said decree on the basis of the submitted estate and inheritance tax returns by merely filing separate

estate tax returns of an undeclared and untaxed income over and above the original amount of the estate declared. Anent petitioners' claim that the tax assessments against the estates of the Teodoro spouses are not yet final, the court finds the claim untenable. In petitioners' motion for reconsideration of the aforementioned assessments, petitioners requested then Commissioner Misael P. Vera for a period of thirty (30) days from October 7, 1972 within which to submit a position paper that would embody their grounds for reconsideration. However, no position paper was ever filed. 15 Such failure to file a position paper may be construed as abandonment of the petitioners' request for reconsideration. The court notes that it took the respondent Commissioner a period of more than one (1) year and five (5) months, from October 7, 1972 to March 14, 1974, before finally instituting the action for collection. Under the circumstances of the case, the act of the Commissioner in filing an action for allowance of the claim for estate and inheritance taxes, may be considered as an outright denial of petitioners' request for reconsideration. From the date of receipt of the copy of the Commissioner's letter for collection of estate and inheritance taxes against the estates of the late Teodoro spouses, petitioners must contest or dispute the same and, upon a denial thereof, the petitioners have a period of thirty (30) days within which to appeal the case to the Court of Tax Appeals. 16 This they failed to avail of . Tax assessments made by tax examiners are presumed correct and made in good faith. A taxpayer has to prove otherwise. 17 Failure of the petitioners to appeal to the Court of Tax Appeals in due time made the assessments in question, final, executory and demandable. 18 The petitioners' allegation that the Court of First Instance (CFI) lacks jurisdiction over the subject of the case is likewise untenable. The assessments having become final and executory, the CFI properly acquired jurisdiction. 19Neither is there merit in petitioners' claim that the exclusive jurisdiction of the Court of Tax Appeals (CTA) applies in the case. The aforesaid exclusive jurisdiction of the CTA arises only in cases of disputed tax assessments. 20As noted earlier, petitioners' letter dated October 7, 1972 asking for reconsideration of the questioned assessments cannot be considered as one disputing the assessments because petitioners failed to substantiate their claim that the deficiency assessments are contrary to law. Petitioners asked for a period of thirty (30) days within which to submit their position paper but they failed to submit the same nonetheless. Hence, petitioners' letter for a reconsideration of the assessments is nothing but a mere scrap of paper. Petitioners' contention that the absence of a decision on their request for reconsideration of the assessments is a bar to granting the claim for collection is likewise without merit. In Republic vs. Lim Tian Teng Son & Co., Inc., 21this Court had occasion to rule that a decision on a request for reinvestigation is not a condition precedent to the filing of an action for collection of taxes already assessed. This Court ruled that "nowhere in the Tax Code is the Collector of Internal Revenue required to rule first on a taxpayer's request for reconsideration before he can go to court for the purpose of collecting the tax assessed. On the contrary, Section 305 of the same Code withheld from all courts, except the Court of Tax Appeals under Republic Act No. 1125, 22 the authority to restrain the collection of any national internal revenue tax, fee or charge, thereby indicating the legislative policy to allow the Collector of Internal Revenue much latitude on the speedy and prompt collection of taxes." Petitioners argue, however, that the Commissioner of Internal Revenue must first rule on the taxpayer's protest against tax assessment so as not to deprive the taxpayer of the remedy of appeal and that it is only from the receipt of the decision that the right to appeal to the Court of Tax Appeals

should run, citing for the purpose San Juan vs. Velasquez 23 as well as Commissioner of Internal Revenue vs. Gonzales. 24 The aforementioned cases are both not in point. In San Juan, the taxpayer concerned, through his accountant, disputed the assessments of income tax and deficiency income tax by adducing the reasons and explanations why said assessments of income tax were not due and owing from the taxpayer. Thus, it was therein ruled that having disputed the assessments at the opportune time, the Commissioner of Internal Revenue cannot ignore the disputed assessments by immediate immediately bringing an action to collect. By the same token in Commissioner of Internal Revenue vs. Gonzales, the assessments of estate and inheritance taxes were disputed by the taxpayer by invoking prescription as a defense claiming that the assessments were made after the lapse of more than five (5) years. Payment of taxes being admittedly a burden, taxpayers should not be left without any recourse when they feel aggrieved due to the erroneous and burdensome assessments made by a Bureau of Internal Revenue agent or by the Commissioner. Said right is vested upon adversely affected taxpayers under Republic Act No. 1125. It cannot be rendered nugatory through the Commissioner's act of immediately filing an action for collection without ruling beforehand on the disputed assessments. 25 However, the remedy of an aggrieved taxpayer is not without any limitation. A taxpayer's right to contest assessments, particularly the right to appeal to the Court of Tax Appeals, may be waived or lost as in this case. 26 The requirement for the Commissioner to rule on disputed assessments before bringing an action for collection is applicable only in cases where the assessment was actually disputed, adducing reasons in support thereto. In the present case where the petitioners did not actually contest the assessments by stating the basis thereof, the respondent Commissioner need not rule on their request. Taxes are the lifeblood of the nation through which the government agencies continue to operate and with which the State effects its functions for the welfare of its constituents. We cannot tolerate taxpayers hampering expedient collection of taxes by their failure to act within a reasonable period. No government could exist if all litigants were permitted to delay the collection of its taxes. 27 Thus, this Court ruled earlier that a suit for the collection of internal revenue taxes, as in this case, where the assessment has already become final and executory, the action to collect is akin to an action to enforce the judgment. No inquiry can be made therein as to the merits of the original case or the justness of the judgment relied upon. 28 In view of the foregoing discussions, petitioners' allegation of grave abuse of discretion on the part of the respondent judge must perforce fall. Considering further that the court a quo properly acquired jurisdiction over the subject matter of the case, petitioners should have appealed the case. The order of the court a quo dated September 30,1974, was received by the petitioners on October 16, 1974. Petitioners should have appealed within a period of fifteen (15) days from receipt thereof but they failed to do so. ****** As petitioners failed to file a timely appeal from the order of the trial court, they can no longer avail of the remedy of a special civil action for certiorari in lieu of appeal. There is no error of jurisdiction committed by the trial court. 29 On the other hand with respect the petitioners' plea that the estate is at any rate entitled to tax amnesty, a reading of P.D. No. 23 30 reveals that in order to avail of tax amnesty, it is required, among others, that there should be a voluntary disclosure of a previously untaxed income. This was the pronouncement of this Court in Nepomuceno vs. Montecillo 31 with respect to P.D. 370 32 which was decreed as a complement of P.D. Nos. 23 and 157. In addition thereto, said income must have been earned or realized prior to 1972 and the tax return must be filed on or before March 31, 1973.

Considering that P.D. No. 23 was issued on October 16, 1972, the court rules that the said decree embraces only those income declared in pursuance thereof within the taxable year 1972. The time frame cannot be stretched to include declarations made prior to the issuance of the said decree or those made outside of the time frame as envisioned in the said decree. Thus, the estates of the Teodoro spouses which have been declared separately sometime in the 1960's are clearly outside the coverage of the tax amnesty provision. Petitioners argue, however, that even if a notice of deficiency assessment had already been issued, the estates may still avail of tax amnesty if the basis of such deficiency assessment is either the failure to file a return or the omission of items of taxable income for a return already filed or the under declaration of said return, citing P.D. No. 67 and Section 4 of BIR Revenue Regulation No. 872. There is no merit in this contention. Even if P.D. No. 67, as an amendment to P.D. 23, enlarges the coverage of tax amnesty to include wealth such as earnings, receipts, gifts, bequests or any other acquisitions from any source whatsoever, said decree reiterates the need of voluntary disclosure on the part of the taxpayer filing the return in order to avail of the tax amnesty. The only noticeable departure from P.D. No. 23 is the extension of the date for the filing of the return from March 31, 1972 to March 31, 1973. Thus, this Court finds that the same policy observed in the issuance of P.D. No. 23, governs P.D. No. 67. In addition thereto, it gives the tax evaders who failed to avail of the provisions of P.D. No. 23 a chance to reform themselves. An examination of both decrees does not show that taxpayers availing of the tax amnesty in accordance with P.D. No. 67, are entitled to blanket coverage of declarations made prior to the issuance of said decrees. Petitioners argue that the estates of their parents declared for estate tax valuation sometime in the 1960's can avail of the tax amnesty when petitioners declared an additional amount of the estates over and above that which was previously declared. A reading of P.D. No. 67 reveals that tax amnesty is extendible only to those declarations made pursuant to said decree. Thus, if at all, it is only the estates in the amount of P3,655,595.78 declared pursuant to P.D. No. 67 that is covered, upon payment of 10% of the said amount within the period prescribed under P.D. No. 23, which was up to June 30, 1973. Considering that there has been partial compliance with the said requirement by the payment of P285,046.68, petitioner may claim the benefit of amnesty for said declared amount upon payment of the balance of 10% thereof required to be paid. WHEREFORE, with the above modification of the questioned order of July 10, 1974, said order is hereby affirmed in all other respect. No pronouncement as to costs. SO ORDERED. Cruz, Griño-Aquino and Medialdea, JJ., concur. Narvasa, J., took no part.

Republic of the Philippines SUPREME COURT Manila EN BANC

G.R. No. L-22356

July 21, 1967

REPUBLIC OF THE PHILIPPINES, plaintiff-appellant, vs. PEDRO B. PATANAO, defendant-appellee. Office of the Solicitor General Arturo A. Alafriz, Solicitor A. B. Afurong and L. O. Gal-lang for plaintiffappellant. Tranquilino O. Calo, Jr. for defendant-appellee. ANGELES, J.: This is an appeal from an order of the Court of First Instance of Agusan in civil case No. 925, dismissing plaintiff's complaint so far as concerns the collection of deficiency income taxes for the years 1951, 1953 and 1954 and additional residence taxes for 1951 and 1952, and requiring the defendant to file his answer with respect to deficiency income tax for 1955 and residence taxes for 1953-1955. In the complaint filed by the Republic of the Philippines, through the Solicitor General, against Pedro B. Patanao, it is alleged that defendant was the holder of an ordinary timber license with concession at Esperanza, Agusan, and as such was engaged in the business of producing logs and lumber for sale during the years 1951-1955; that defendant failed to file income tax returns for 1953 and 1954, and although he filed income tax returns for 1951, 1952 and 1955, the same were false and fraudulent because he did not report substantial income earned by him from his business; that in an examination conducted by the Bureau of Internal Revenue on defendant's income and expenses for 1951-1955, it was ascertained that the sum of P79,892.75, representing deficiency; income taxes and additional residence taxes for the aforesaid years, is due from defendant; that on February 14, 1958, plaintiff, through the Deputy Commissioner of Internal Revenue, sent a letter of demand with enclosed income tax assessment to the defendant requiring him to pay the said amount; that notwithstanding repeated demands the defendant refused, failed and neglected to pay said taxes; and that the assessment for the payment of the taxes in question has become final, executory and demandable, because it was not contested before the Court of Tax Appeals in accordance with the provisions of section 11 of Republic Act No. 1125. Defendant moved to dismiss the complaint on two grounds, namely: (1) that the action is barred by prior judgment, defendant having been acquitted in criminal cases Nos. 2089 and 2090 of the same court, which were prosecutions for failure to file income tax returns and for non-payment of income taxes; and (2) that the action has prescribed. After considering the motion to dismiss, the opposition thereto and the rejoinder to the opposition, the lower court entered the order appealed from, holding that the only cause of action left to the plaintiff in its complaint is the collection of the income tax due for the taxable year 1955 and the residence tax (Class B) for 1953, 1954 and 1955. A motion to reconsider said order was denied, whereupon plaintiff interposed the instant appeal, which was brought directly to this Court, the questions involved being purely legal. The conclusion of the trial court, that the present action is barred by prior judgment, is anchored on the following rationale: There is no question that the defendant herein has been accused in Criminal Cases Nos. 2089 and 2090 of this Court for not filing his income tax returns and for non-payment of income taxes for the years 1953 and 1954. In both cases, he was acquitted. The rule in this

jurisdiction is that the accused once acquitted is exempt from both criminal and civil responsibility because when a criminal action is instituted, civil action arising from the same offense is impliedly instituted unless the offended party expressly waives the civil action or reserves the right to file it separately. In the criminal cases abovementioned wherein the defendant was completely exonerated, there was no waiver or reservation to file a separate civil case so that the failure to obtain conviction on a charge of non-payment of income taxes is fatal to any civil action to collect the payment of said taxes.
1 äwphï 1.ñë t

Plaintiff-appellant assails the ruling as erroneous. Defendant-appellee on his part urges that it should be maintained. In applying the principle underlying the civil liability of an offender under the Penal Code to a case involving the collection of taxes, the court a quo fell into error. The two cases are circumscribed by factual premises which are diametrically opposed to each either, and are founded on entirely different philosophies. Under the Penal Code the civil liability is incurred by reason of the offender's criminal act. Stated differently, the criminal liability gives birth to the civil obligation such that generally, if one is not criminally liable under the Penal Code, he cannot become civilly liable thereunder. The situation under the income tax law is the exact opposite. Civil liability to pay taxes arises from the fact, for instance, that one has engaged himself in business, and not because of any criminal act committed by him. The criminal liability arises upon failure of the debtor to satisfy his civil obligation. The incongruity of the factual premises and foundation principles of the two cases is one of the reasons for not imposing civil indemnity on the criminal infractor of the income tax law. Another reason, of course, is found in the fact that while section 73 of the National Internal Revenue Code has provided the imposition of the penalty of imprisonment or fine, or both, for refusal or neglect to pay income tax or to make a return thereof, it failed to provide the collection of said tax in criminal proceedings. The only civil remedies provided, for the collection of income tax, in Chapters I and II, Title IX of the Code and section 316 thereof, are distraint of goods, chattels, etc. or by judicial action, which remedies are generally exclusive in the absence of a contrary intent from the legislator. (People vs. Arnault, G.R. No. L-4288, November 20, 1952; People vs. Tierra, G.R. Nos. L-1717717180, December 28, 1964) Considering that the Government cannot seek satisfaction of the taxpayer's civil liability in a criminal proceeding under the tax law or, otherwise stated, since the said civil liability is not deemed included in the criminal action, acquittal of the taxpayer in the criminal proceeding does not necessarily entail exoneration from his liability to pay the taxes. It is error to hold, as the lower court has held, that the judgment in the criminal cases Nos. 2089 and 2090 bars the action in the present case. The acquittal in the said criminal cases cannot operate to discharge defendant appellee from the duty of paying the taxes which the law requires to be paid, since that duty is imposed by statute prior to and independently of any attempts by the taxpayer to evade payment. Said obligation is not a consequence of the felonious acts charged in the criminal proceeding, nor is it a mere civil liability arising from crime that could be wiped out by the judicial declaration of non-existence of the criminal acts charged. (Castro vs. The Collector of Internal Revenue, G.R. No. L-12174, April 20, 1962). Regarding prescription of action, the lower court held that the cause of action on the deficiency income tax and residence tax for 1951 is barred because appellee's income tax return for 1951 was assessed by the Bureau of Internal Revenue only on February 14, 1958, or beyond the five year period of limitation for assessment as provided in section 331 of the National Internal Revenue Code. Appellant contends that the applicable law is section 332 (a) of the same Code under which a proceeding in court for the collection of the tax may be commenced without assessment at any time within 10 years from the discovery of the falsity, fraud or omission. The complaint filed on December 7, 1962, alleges that the fraud in the appellee's income tax return for 1951, was discovered on February 14, 1958. By filing a motion to dismiss, appellee hypothetically admitted this allegation as all the other averments in the complaint were so admitted. Hence, section

332 (a) and not section 331 of the National Internal Revenue Code should determine whether or not the cause of action of deficiency income tax and residence tax for 1951 has prescribed. Applying the provision of section 332 (a), the appellant's action instituted in court on December 7, 1962 has not prescribed. Wherefore, the order appealed from is hereby set aside. Let the records of this case be remanded to the court of origin for further proceedings. No pronouncement as to costs. Reyes, J.B.L., Makalintal, Bengzon, J.P., Zaldivar, Sanchez, Castro and Fernando, JJ., concur. Concepcion, C.J. and Dizon, J., are on leave. Republic of the Philippines SUPREME COURT Manila THIRD DIVISION

G.R. No. 128315 June 29, 1999 COMMISSIONER OF INTERNAL REVENUE, petitioner, vs. PASCOR REALTY AND DEVELOPMENT CORPORATION, ROGELIO A. DIO and VIRGINIA S. DIO, respondents.

PANGANIBAN, J.: An assessment contains not only a computation of tax liabilities, but also a demand for payment within a prescribed period. It also signals the time when penalties and protests begin to accrue against the taxpayer. To enable the taxpayer to determine his remedies thereon, due process requires that it must be served on and received by the taxpayer. Accordingly, an affidavit, which was executed by revenue officers stating the tax liabilities of a taxpayer and attached to a criminal complaint for tax evasion, cannot be deemed an assessment that can be questioned before the Court of Tax Appeals. Statement of the Case Before this Court is a Petition for Review on Certiorari under Rule 45 of the Rules of Court praying for the nullification of the October 30, 1996 Decision 1 of the Court of Appeals 2 in CA-GR SP No. 40853, which effectively affirmed the January 25, 1996 Resolution 3 of the Court of Tax Appeals 4 CTA Case No. 5271. The CTA disposed as follows: WHEREFORE, finding [the herein petitioner's] "Motion to Dismiss" as UNMERITORIOUS, the same is hereby DENIED. [The CIR] is hereby given a period of thirty (30) days from receipt hereof to file her answer.

Petitioner also seeks to nullify the February 13, 1997 Resolution 5 of the Court of Appeals denying reconsideration. The Facts As found by the Court of Appeals, the undisputed facts of the case are as follows: It appears that by virtue of Letter of Authority No. 001198, then BIR Commissioner Jose U. Ong authorized Revenue Officers Thomas T. Que, Sonia T. Estorco and Emmanuel M. Savellano to examine the books of accounts and other accounting records of Pascor Realty and Development Corporation. (PRDC) for the years ending 1986, 1987 and 1988. The said examination resulted in a recommendation for the issuance of an assessment in the amounts of P7,498,434.65 and P3,015,236.35 for the years 1986 and 1987, respectively. On March 1, 1995, the Commissioner of Internal Revenue filed a criminal complaint before the Department of Justice against the PRDC, its President Rogelio A. Dio, and its Treasurer Virginia S. Dio, alleging evasion of taxes in the total amount of P10,513,671 .00. Private respondents PRDC, et.al. filed an Urgent Request for Reconsideration/Reinvestigation disputing the tax assessment and tax liability. On March 23, 1995, private respondents received a subpoena from the DOJ in connection with the criminal complaint filed by the Commissioner of Internal Revenue (BIR) against them.
1âwphi1.nêt

In a letter dated May 17, 1995, the CIR denied the urgent request for reconsideration/reinvestigation of the private respondents on the ground that no formal assessment of the has as yet been issued by the Commissioner. Private respondents then elevated the Decision of the CIR dated May 17, 1995 to the Court of Tax Appeals on a petition for review docketed as CTA Case No. 5271 on July 21, 1995. On September 6, 1995, the CIR filed a Motion to Dismiss the petition on the ground that the CTA has no jurisdiction over the subject matter of the petition, as there was no formal assessment issued against the petitioners. The CTA denied the said motion to dismiss in a Resolution dated January 25, 1996 and ordered the CIR to file an answer within thirty (30) days from receipt of said resolution. The CIR received the resolution on January 31, 1996 but did not file an answer nor did she move to reconsider the resolution. Instead, the CIR filed this petition on June 7, 1996, alleging as grounds that: Respondent Court of Tax Appeals acted with grave abuse of discretion and without jurisdiction in considering the affidavit/report of the revenue officer and the indorsement of said report to the secretary of justice as assessment which may be appealed to the Court of Tax Appeals; Respondent Court Tax Appeals acted with grave abuse of discretion in considering the denial by petitioner of private respondents' Motion for Reconsideration as [a] final decision which may be appealed to the Court of Tax Appeals.

In denying the motion to dismiss filed by the CIR, the Court of Tax Appeals stated: We agree with petitioners' contentions, that the criminal complaint for tax evasion is the assessment issued, and that the letter denial of May 17, 1995 is the decision properly appealable to [u]s. Respondent's ground of denial, therefore, that there was no formal assessment issued, is untenable. It is the Court's honest belief, that the criminal case for tax evasion is already anassessment. The complaint, more particularly, the Joint Affidavit of Revenue Examiners Lagmay and Savellano attached thereto, contains the details of the assessment like the kind and amount of tax due, and the period covered: Petitioners are right, in claiming that the provisions of Republic Act No. 1125, relating to exclusive appellate jurisdiction of this Court, do not, make any mention of "formal assessment." The law merely states, that this Court has exclusive appellate jurisdiction over decisions of the Commissioner of Internal Revenue on disputed assessments, and other matters arising under the National Internal Revenue Code, other law or part administered by the Bureau of Internal Revenue Code. As far as this Court is concerned, the amount and kind of tax due, and the period covered, are sufficient details needed for an "assessment." These details are more than complete, compared to the following definitions of the term as quoted hereunder. Thus: Assessment is laying a tax. Johnson City v. Clinchfield R. Co., 43 S.W. (2d) 386, 387, 163 Tenn. 332. (Words and Phrases, Permanent Edition, Vol. 4, p. 446). The word assessment when used in connection with taxation, may have more than one meaning. The ultimate purpose of an assessment to such a connection is to ascertain the amount that each taxpayer is to pay. More commonly, the word "assessment" means the official valuation of a taxpayer's property for purpose of taxation. State v. New York, N.H. and H.R. Co. 22 A. 765, 768, 60 Conn. 326, 325. (Ibid. p. 445)
From the above, it can be gleaned that an assessment simply states how much tax is due from a taxpayer. Thus, based on these definitions, the details of the tax as given in the Joint Affidavit of respondent's examiners, which was attached to the tax evasion complaint, more than suffice to qualify as an assessment. Therefore, this assessment having been disputed by petitioners, and there being a denial of their letter disputing such assessment, this Court unquestionably acquired jurisdiction over the instant petition for review. 6

As earlier observed, the Court of Appeals sustained the CTA and dismissed the petition. Hence, this recourse to this Court. 7 Ruling of the Court of Appeals The Court of Appeals held that the tax court committed no grave abuse of discretion in ruling that the Criminal Complaint for tax evasion filed by the Commissioner of Internal Revenue with the Department of Justice constituted an "assessment" of the tax due, and that the said assessment

could be the subject of a protest. By definition, an assessment is simply the statement of the details and the amount of tax due from a taxpayer. Based on this definition, the details of the tax contained in the BIR examiners' Joint Affidavit, 8 which was attached to the criminal Complaint, constituted an assessment. Since the assailed Order of the CTA was merely interlocutory and devoid of grave abuse of discretion, a petition for certiorari did not lie. Issues Petitioners submit for the consideration of this Court following issues: (1) Whether or not the criminal complaint for tax evasion can be construed as an assessment. (2) Whether or not an assessment is necessary before criminal charges for tax evasion may be instituted.
(3) Whether or not the CTA can take cognizance of the case in the absence of an assessment. 9

In the main, the Court will resolve whether the revenue officers' Affidavit-Report, which was attached to criminal revenue Complaint filed the Department of Justice, constituted an assessment that could be questioned before the Court of Tax Appeals. The Court's Ruling The petition is meritorious. Main Issue: Assessment Petitioner argues that the filing of the criminal complaint with the Department of Justice cannot in any way be construed as a formal assessment of private respondents' tax liabilities. This position is based on Section 205 of the National Internal Revenue Code 10 (NIRC), which provides that remedies for the collection of deficient taxes may be by either civil or criminal action. Likewise, petitioner cites Section 223(a) of the same Code, which states that in case of failure to file a return, the tax may be assessed or a proceeding in court may be begun without assessment. Respondents, on the other hand, maintain that an assessment is not an action or proceeding for the collection of taxes, but merely a notice that the amount stated therein is due as tax and that the taxpayer is required to pay the same. Thus, qualifying as an assessment was the BIR examiners' Joint Affidavit, which contained the details of the supposed taxes due from respondent for taxable years ending 1987 and 1988, and which was attached to the tax evasion Complaint filed with the DOJ. Consequently, the denial by the BIR of private respondents' request for reinvestigation of the disputed assessment is properly appealable to the CTA. We agree with petitioner. Neither the NIRC nor the regulations governing the protest of assessments 11 provide a specific definition or form of an assessment. However, the NIRC defines the specific functions and effects of an assessment. To consider the affidavit attached to the Complaint as a proper assessment is to subvert the nature of an assessment and to set a bad precedent that will prejudice innocent taxpayers.

True, as pointed out by the private respondents, an assessment informs the taxpayer that he or she has tax liabilities. But not all documents coming from the BIR containing a computation of the tax liability can be deemed assessments. To start with, an assessment must be sent to and received by a taxpayer, and must demand payment of the taxes described therein within a specific period. Thus, the NIRC imposes a 25 percent penalty, in addition to the tax due, in case the taxpayer fails to pay deficiency tax within the time prescribed for its payment in the notice of assessment. Likewise, an interest of 20 percent per annum, or such higher rates as may be prescribed by rules and regulations, is to be collected form the date prescribed for its payment until the full payment. 12 The issuance of an assessment is vital in determining, the period of limitation regarding its proper issuance and the period within which to protest it. Section 203 13 of the NIRC provides that internal revenue taxes must be assessed within three years from the last day within which to file the return. Section 222, 14 on the other hand, specifies a period of ten years in case a fraudulent return with intent to evade was submitted or in case of failure to file a return. Also, Section 228 15 of the same law states that said assessment may be protested only within thirty days from receipt thereof. Necessarily, the taxpayer must be certain that a specific document constitutes an assessment. Otherwise, confusion would arise regarding the period within which to make an assessment or to protest the same, or whether interest and penalty may accrue thereon. It should also be stressed that the said document is a notice duly sent to the taxpayer. Indeed, an assessment is deemed made only when the collector of internal revenue releases, mails or sends such notice to the taxpayer. 16 In the present case, the revenue officers' Affidavit merely contained a computation of respondents' tax liability. It did not state a demand or a period for payment. Worse, it was addressed to the justice secretary, not to the taxpayers. Respondents maintain that an assessment, in relation to taxation, is simply understood' to mean:
A notice to the effect that the amount therein stated is due as tax and a demand for payment thereof. 17 Fixes the liability of the taxpayer and ascertains the facts and furnishes the data for the proper presentation of tax rolls. 18

Even these definitions fail to advance private respondents' case. That the BIR examiners' Joint Affidavit attached to the Criminal Complaint contained some details of the tax liabilities of private respondents does not ipso factomake it an assessment. The purpose of the Joint Affidavit was merely to support and substantiate the Criminal Complaint for tax evasion. Clearly, it was not meant to be a notice of the tax due and a demand to the private respondents for payment thereof. The fact that the Complaint itself was specifically directed and sent to the Department of Justice and not to private respondents shows that the intent of the commissioner was to file a criminal complaint for tax evasion, not to issue an assessment. Although the revenue officers recommended the issuance of an assessment, the commissioner opted instead to file a criminal case for tax evasion. What private respondents received was a notice from the DOJ that a criminal case for tax evasion had been filed against them, not a notice that the Bureau of Internal Revenue had made an assessment.

In addition, what private respondents sent to the commissioner was a motion for a reconsideration of the tax evasion charges filed, not of an assessment, as shown thus: This is to request for reconsideration of the tax evasion charges against my client, PASCOR Realty and Development Corporation and for the same to be referred to the Appellate Division in order to give my client the opportunity of a fair and objective hearing. 19 Additional Issues: Assessment Not Necessary Before Filing of Criminal Complaint Private respondents maintain that the filing of a criminal complaint must be preceded by an assessment. This is incorrect, because Section 222 of the NIRC specifically states that in cases where a false or fraudulent return is submitted or in cases of failure to file a return such as this case, proceedings in court may be commenced without an assessment. Furthermore, Section 205 of the same Code clearly mandates that the civil and criminal aspects of the case may be pursued simultaneously. In Ungab v. Cusi, 20 petitioner therein sought the dismissal of the criminal Complaints for being premature, since his protest to the CTA had not yet been resolved. The Court held that such protests could not stop or suspend the criminal action which was independent of the resolution of the protest in the CTA. This was because the commissioner of internal revenue had, in such tax evasion cases, discretion on whether to issue an assessment or to file a criminal case against the taxpayer or to do both. Private respondents insist that Section 222 should be read in relation to Section 255 of the NLRC, 21 which penalizes failure to file a return. They add that a tax assessment should precede a criminal indictment. We disagree. To reiterate, said Section 222 states that an assessment is not necessary before a criminal charge can be filed. This is the general rule. Private respondents failed to show that they are entitled to an exception. Moreover, the criminal charge need only be supported by a prima facie showing of failure to file a required return. This fact need not be proven by an assessment. The issuance of an assessment must be distinguished from the filing of a complaint. Before an assessment is issued, there is, by practice, a pre-assessment notice sent to the taxpayer. The taxpayer is then given a chance to submit position papers and documents to prove that the assessment is unwarranted. If the commissioner is unsatisfied, an assessment signed by him or her is then sent to the taxpayer informing the latter specifically and clearly that an assessment has been made against him or her. In contrast, the criminal charge need not go through all these. The criminal charge is filed directly with the DOJ. Thereafter, the taxpayer is notified that a criminal case had been filed against him, not that the commissioner has issued an assessment. It must be stressed that a criminal complaint is instituted not to demand payment, but to penalize the taxpayer for violation of the Tax Code. WHEREFORE, the petition is hereby GRANTED. The assailed Decision is REVERSED and SET ASIDE. CTA Case No. 5271 is likewise DISMISSED. No costs. SO ORDERED.

Vitug, Purisima and Gonzaga-Reyes, JJ., concur. Romero, J., abroad on official business. Republic of the Philippines SUPREME COURT Manila SECOND DIVISION G.R. No. L-41919-24 May 30, 1980 QUIRICO P. UNGAB, petitioner, vs. HON. VICENTE N. CUSI, JR., in his capacity as Judge of the Court of First Instance, Branch 1, 16TH Judicial District, Davao City, THE COMMISSIONER OF INTERNAL REVENUE, and JESUS N. ACEBES, in his capacity as State Prosecutor, respondents.

CONCEPCION JR., J: Petition for certiorari and prohibition with preliminary injunction and restraining order to annul and set aside the informations filed in Criminal Case Nos. 1960, 1961, 1962, 1963, 1964, and 1965 of the Court of First Instance of Davao, all entitled: "People of the Philippines, plaintiff, versus Quirico Ungab, accused;" and to restrain the respondent Judge from further proceeding with the hearing and trial of the said cases. It is not disputed that sometime in July, 1974, BIR Examiner Ben Garcia examined the income tax returns filed by the herein petitioner, Quirico P. Ungab, for the calendar year ending December 31, 1973. In the course of his examination, he discovered that the petitioner failed to report his income derived from sales of banana saplings. As a result, the BIR District Revenue Officer at Davao City sent a "Notice of Taxpayer" to the petitioner informing him that there is due from him (petitioner) the amount of P104,980.81, representing income, business tax and forest charges for the year 1973 and inviting petitioner to an informal conference where the petitioner, duly assisted by counsel, may present his objections to the findings of the BIR Examiner. 1 Upon receipt of the notice, the petitioner wrote the BIR District Revenue Officer protesting the assessment, claiming that he was only a dealer or agent on commission basis in the banana sapling business and that his income, as reported in his income tax returns for the said year, was accurately stated. BIR Examiner Ben Garcia, however, was fully convinced that the petitioner had filed a fraudulent income tax return so that he submitted a "Fraud Referral Report," to the Tax Fraud Unit of the Bureau of Internal Revenue. After examining the records of the case, the Special Investigation Division of the Bureau of Internal Revenue found sufficient proof that the herein petitioner is guilty of tax evasion for the taxable year 1973 and recommended his prosecution:
t êñ. £îhqw â£

(1) For having filed a false or fraudulent income tax return for 1973 with intent to evade his just taxes due the government under Section 45 in relation to Section 72 of the National Internal Revenue Code;

(2) For failure to pay a fixed annual tax of P50.00 a year in 1973 and 1974, or a total of unpaid fixed taxes of P100.00 plus penalties of 175.00 or a total of P175.00, in accordance with Section 183 of the National Internal Revenue Code;
(3) For failure to pay the 7% percentage tax, as a producer of banana poles or saplings, on the total sales of P129,580.35 to the Davao Fruit Corporation, depriving thereby the government of its due revenue in the amount of P15,872.59, inclusive of surcharge. 2

In a second indorsement to the Chief of the Prosecution Division, dated December 12, 1974, the Commissioner of Internal Revenue approved the prosecution of the petitioner. 3 Thereafter, State Prosecutor Jesus Acebes who had been designated to assist all Provincial and City Fiscals throughout the Philippines in the investigation and prosecution, if the evidence warrants, of all violations of the National Internal Revenue Code, as amended, and other related laws, in Administrative Order No. 116 dated December 5, 1974, and to whom the case was assigned, conducted a preliminary investigation of the case, and finding probable cause, filed six (6) informations against the petitioner with the Court of First Instance of Davao City, to wit:
têñ.£î hqwâ £

(1) Criminal Case No. 1960 ² Violation of Sec. 45, in relation to Sec. 72 of the National Internal-Revenue Code, for filing a fraudulent income tax return for the calendar year ending December 31, 1973; 4 (2) Criminal Case No. 1961 ² Violation of Sec. 182 (a), in relation to Secs. 178, 186, and 208 of the National Internal Revenue Code, for engaging in business as producer of saplings, from January, 1973 to December, 1973, without first paying the annual fixed or privilege tax thereof; 5 (3) Criminal Case No. 1962 ² Violation of Sec. 183 (a), in relation to Secs. 186 and 209 of the National Internal Revenue Code, for failure to render a true and complete return on the gross quarterly sales, receipts and earnings in his business as producer of banana saplings and to pay the percentage tax due thereon, for the quarter ending December 31, 1973; 6 (4) Criminal Case No. 1963 ² Violation of Sec. 183 (a), in relation to Secs. 186 and 209 of the National Internal Revenue Code, for failure to render a true and complete return on the gross quarterly sales receipts and earnings in his business as producer of saplings, and to pay the percentage tax due thereon, for the quarter ending on March 31, 1973; 7 (5) Criminal Case No. 1964 ² Violation of Sec. 183 (a), in relation to Secs. 186 and 209 of the National Internal Revenue Code, for failure to render a true and complete return on the gross quarterly sales, receipts and earnings in his business as producer of banana saplings for the quarter ending on June 30, 1973, and to pay the percentage tax due thereon; 8 (6) Criminal Case No. 1965 ² Violation of Sec. 183 (a), in relation to Secs. 186 and 209 of the National Internal Revenue Code, for failure to render a true and complete return on the gross quarterly sales, receipts and earnings as producer of banana saplings, for the quarter ending on September 30, 1973, and to pay the percentage tax due thereon. 9

On September 16, 1975, the petitioner filed a motion to quash the informations upon the grounds that: (1) the informations are null and void for want of authority on the part of the State Prosecutor to initiate and prosecute the said cases; and (2) the trial court has no jurisdiction to take cognizance of the above-entitled cases in view of his pending protest against the assessment made by the BIR

Examiner. 10 However, the trial court denied the motion on October 22, 1975. 11 Whereupon, the petitioner filed the instant recourse. As prayed for, a temporary restraining order was issued by the Court, ordering the respondent Judge from further proceeding with the trial and hearing of Criminal Case Nos. 1960, 1961, 1962, 1963, 1964, and 1965 of the Court of First Instance of Davao, all entitled: "People of the Philippines, plaintiff, versus Quirico Ungab, accused." The petitioner seeks the annulment of the informations filed against him on the ground that the respondent State Prosecutor is allegedly without authority to do so. The petitioner argues that while the respondent State Prosecutor may initiate the investigation of and prosecute crimes and violations of penal laws when duly authorized, certain requisites, enumerated by this Court in its decision in the case of Estrella vs. Orendain, 12should be observed before such authority may be exercised; otherwise, the provisions of the Charter of Davao City on the functions and powers of the City Fiscal will be meaningless because according to said charter he has charge of the prosecution of all crimes committed within his jurisdiction; and since "appropriate circumstances are not extant to warrant the intervention of the State Prosecution to initiate the investigation, sign the informations and prosecute these cases, said informations are null and void." The ruling adverted to by the petitioner reads, as follows:
têñ.£îhqw â£

In view of all the foregoing considerations, it is the ruling of this Court that under Sections 1679 and 1686 of the Revised Administrative Code, in any instance where a provincial or city fiscal fails, refuses or is unable, for any reason, to investigate or prosecute a case and, in the opinion of the Secretary of Justice it is advisable in the public interest to take a different course of action, the Secretary of Justice may either appoint as acting provincial or city fiscal to handle the investigation or prosecution exclusively and only of such case, any practicing attorney or some competent officer of the Department of Justice or office of any city or provincial fiscal, with complete authority to act therein in all respects as if he were the provincial or city fiscal himself, or appoint any lawyer in the government service, temporarily to assist such city of provincial fiscal in the discharge of his duties, with the same complete authority to act independently of and for such city or provincial fiscal provided that no such appointment may be made without first hearing the fiscal concerned and never after the corresponding information has already been filed with the court by the corresponding city or provincial fiscal without the conformity of the latter, except when it can be patently shown to the court having cognizance of the case that said fiscal is intent on prejudicing the interests of justice. The same sphere of authority is true with the prosecutor directed and authorized under Section 3 of Republic Act 3783, as amended and/or inserted by Republic Act 5184. The observation in Salcedo vs. Liwag, supra, regarding the nature of the power of the Secretary of Justice over fiscals as being purely over administrative matters only was not really necessary, as indicated in the above relation of the facts and discussion of the legal issues of said case, for the resolution thereof. In any event, to any extent that the opinion therein may be inconsistent herewith the same is hereby modified. The contention is without merit. Contrary to the petitioner's claim, the rule therein established had not been violated. The respondent State Prosecutor, although believing that he can proceed independently of the City Fiscal in the investigation and prosecution of these cases, first sought permission from the City Fiscal of Davao City before he started the preliminary investigation of these cases, and the City Fiscal, after being shown Administrative Order No. 116, dated December 5, 1974, designating the said State Prosecutor to assist all Provincial and City fiscals throughout the Philippines in the investigation and prosecution of all violations of the National Internal Revenue Code, as amended, and other related laws, graciously allowed the respondent State Prosecutor to conduct the investigation of said cases, and in fact, said investigation was conducted in the office of the City Fiscal. 13

The petitioner also claims that the filing of the informations was precipitate and premature since the Commissioner of Internal Revenue has not yet resolved his protests against the assessment of the Revenue District Officer; and that he was denied recourse to the Court of Tax Appeals. The contention is without merit. What is involved here is not the collection of taxes where the assessment of the Commissioner of Internal Revenue may be reviewed by the Court of Tax Appeals, but a criminal prosecution for violations of the National Internal Revenue Code which is within the cognizance of courts of first instance. While there can be no civil action to enforce collection before the assessment procedures provided in the Code have been followed, there is no requirement for the precise computation and assessment of the tax before there can be a criminal prosecution under the Code.
t êñ.£ îhqw â£

The contention is made, and is here rejected, that an assessment of the deficiency tax due is necessary before the taxpayer can be prosecuted criminally for the charges preferred. The crime is complete when the violator has, as in this case, knowingly and willfully filed fraudulent returns with intent to evade and defeat a part or all of the tax. 14 An assessment of a deficiency is not necessary to a criminal prosecution for willful attempt to defeat and evade the income tax. A crime is complete when the violator has knowingly and willfuly filed a fraudulent return with intent to evade and defeat the tax. The perpetration of the crime is grounded upon knowledge on the part of the taxpayer that he has made an inaccurate return, and the government's failure to discover the error and promptly to assess has no connections with the commission of the crime. 15

Besides, it has been ruled that a petition for reconsideration of an assessment may affect the suspension of the prescriptive period for the collection of taxes, but not the prescriptive period of a criminal action for violation of law.16 Obviously, the protest of the petitioner against the assessment of the District Revenue Officer cannot stop his prosecution for violation of the National Internal Revenue Code. Accordingly, the respondent Judge did not abuse his discretion in denying the motion to quash filed by the petitioner. WHEREFORE, the petition should be, as it is hereby dismissed. The temporary restraining order heretofore issued is hereby set aside. With costs against the petitioner. SO ORDERED. Barredo (Chairman), Aquino, Abad Santos and De Castro, JJ., concur. Republic of the Philippines SUPREME COURT Manila FIRST DIVISION
1äwph ï1.ñ ët

G.R. No. 119322 June 4, 1996 COMMISSIONER ON INTERNAL REVENUE, SENIOR STATE PROSECUTOR AURORA S. LAGMAN, SENIOR STATE PROSECUTOR BERNELITO R. FERNANDEZ, SENIOR STATE PROSECUTOR HENRICK P. GINGOYON, ROGELIO F. VISTA, STATE PROSECUTOR ALFREDO AGCAOILI, PROSECUTING ATTORNEY EMMANUEL VELASCO, CITY

PROSECUTOR CANDIDO V. RIVERA, AND ASSISTANT CITY PROSECUTOR LEOPOLDO E. BARAQUIA, petitioners, vs. THE HONORABLE COURT OF APPEALS, THE HONORABLE TIRSO D'C VELASCO, PRESIDING JUDGE, REGIONAL TRIAL COURT OF QUEZON CITY, BRANCH 88, FORTUNE TOBACCO CORPORATION, LUCIO TAN, HARRY C. TAN, CARMEN KAO TAN, FLORENCIO SANTOS, SALVADOR MISON, CHUNG POE KEE, ROJAS CHUA, MARIANO TANENGLIAN, JUANITA LEE AND ANTONIO P. ABAYA, respondents. DAGUPAN COMBINED COMMODITIES, INC., TOWNSMAN COMMERCIALS, INC., LANDMARK SALES AND MARKETING INC., CRIMSON CROCKER DISTRIBUTORS, INC., MOUNT MATUTUM MARKETING CORP., FIRST UNION TRADING CORP., CARLSBURG AND SONS, INC., OMAR ALI DISTRIBUTORS, INC., ORIEL AND COMPANY, NEMESIO TAN, QUINTIN CALLEJA, YOLANDA MANALILI, CARLOS CHAN, ROMEO TAN, VICENTE CO, WILLIAM YU, LETICIA LIM, GLORIA LOPEZ, ROBERT TANTAMCO, FELIPE LOY, ROLANDO CHUA, HONORINA TAN, WILLIE TANTAMCO, HENRY WEECHEE, JESUS LIM, TEODORO TAN, ANTONIO APOSTOL, DOMINGO TENG, CANDELARIO LI, ERLINDA CRUZ, CARLOS TUMPALAN, LARRY JOHN SY, ERNESTO ONG, WILFREDO MACROHON, ANTONIO TIU, ROSARIO LESTER, WILFREDO ONG, BONIFACIO CHUA, GO CHING CHUAN, HENRY CHUA, LOPE LIM GUAN, EMILIO TAN, FELIPE TAN SEH CHUAN, ANDRES CO, FELIPE KEE, HENRY GO CO, NARCISO GO, ADOLFO LIM, CO SHU, DANIEL YAO CABIGUN, GABRIELLE. QUINTELA, NELSON TE, EMILLIO GO, EDWIN LEE, CESAR LEDESMA, JR., JAO CHEP SENG, ARNULFO TAN, BENJAMIN T. HONG, PHILIP JAO, JOSE P. YU, AND DAVID R. CORTES, respondents-intervenors.

KAPUNAN, J.:p The pivotal issue in this petition for review is whether or not respondent Court of Appeals in its decision 1 in CA-G.R. SP No. 33599 correctly ruled that the Regional Trial Court of Quezon City (Branch 88) in Civil Case No. Q-94- 18790 did not commit grave abuse of discretion amounting to lack of jurisdiction in issuing four (4) orders directing the issuance of writs of preliminary injunction restraining petitioner prosecutors from continuing with the preliminary injunction of I.S. Nos. 93-508 and 93-584 in the Department of Justice and I.S. No. 93-17942 in the Office of the City Prosecutors of Quezon City wherein private respondents were respondents and denying petitioners' Motion to Dismiss said Civil Case No. 94-18790. 2 In resolving the issue raised in the petition, the Court may be guided by its definition of what constitutes grave abuse of discretion. By grave abuse of discretion is meant such capricious and whimsical exercise of judgment as is equivalent to lack of jurisdiction. The abuse of discretion must be patent and gross as to amount to an evasion of positive duty or a virtual refusal to perform a duty enjoined by law, or to act at all in contemplation of law as where the power is exercised in an arbitrary and despotic manner by reason of passion and hostility. 3 On June 1, 1993, the President issued a Memorandum creating a Task Force to investigate the tax liabilities of manufacturers engaged in tax evasion scheme, such as selling products through dummy marketing corporations to avoid payment of correct internal revenue tax, to collect from them any tax liabilities discovered from such investigation, and to file the necessary criminal actions against those who may have violated the tax code. The task force was composed of the Commissioner of Internal Revenue as Chairman, a representative of the Department of Justice and a representative of the Executive Secretary.

On July 1, 1993, the Commissioner of Internal Revenue issued a Revenue Memorandum Circular No. 37-93 reclassifying best selling cigarettes bearing the brands "Hope," "More," and "Champion" as cigarettes of foreign brands subject to a higher rate of tax. On August 3, 1993, respondent Fortune Tobacco Corporation (Fortune) questioned the validity of the reclassification of said brands of cigarettes as violative of its right to due process and equal protection of law. Parenthetically, on September 8, 1993, the Court of Tax Appeals by resolution ruled that the reclassification made by the Commissioner "is of doubtful legality" and enjoined its enforcement. In a letter of August 13, 1993 which was received by Fortune on August 24, 1993, the Commissioner assessed against Fortune the total amount of P7,685,942,221.66 representing deficiency income, ad valorem and value-added tax for the year 1992 with the request that the said amount be paid within thirty (30) days upon receipt thereof. 4 Fortune on September 17, 1993 moved for reconsideration of the assessments. On September 7, 1993, the Commissioner of Internal Revenue filed a complaint with the Department of Justice against respondent Fortune, its corporate officers, nine (9) other corporations and their respective corporate officers for alleged fraudulent tax evasion for supposed non-payment by Fortune of the correct amount of income tax, ad valorem tax and value-added tax for the year 1992. The complaint alleged, among others, that: In the said income tax return, the taxpayer declared a net taxable income of P183,613,408.00 and an income tax due of P64,264,693.00. Based mainly on documentary evidence submitted by the taxpayer itself, these declarations are false and fraudulent because the correct taxable income of the corporation for the said year is P1,282,959,399.25. This underdeclaration which resulted in the evasion of the amount of P723,773,759.79 as deficiency income tax for the year 1992 is a violation of Section 45 of the Tax Code, penalized under Section 253 in relation to Sections 252(b) and (d) and 253 thereof, thus: . . . xxx xxx xxx Fortune Tobacco Corporation, through its Vice-President for Finance, Roxas Chua, likewise filed value-added tax returns for the 1st, 2nd, 3rd and 4th quarters of 1992 with the Rev. District Office of Marikina, Metro Manila, declaring therein gross taxable sales, as follows: 1st Qtr. P 2,924,418,055.00 2nd Qtr. 2,980,335,235.00 3rd Qtr. 2,839,519,325.00 4th Qtr. 2,992,386,005.00 However, contrary to what have been reported in the said value- added tax returns, and based on documentary evidence obtained from the taxpayer, the total actual taxable sales of the corporation for the year 1992 amounted to P16,158,575,035.00

instead of P11,929,322,334.52 as declared by the corporation in the said VAT returns. These fraudulent underdeclarations which resulted in the evasion of value-added taxes in the aggregate amount of P1,169,688,645.63 for the entire year 1992 are violations of Section 110 in relation to Section 100 of the Tax Code, which are likewise penalized under the aforequoted Section 253, in relation to Section 252, thereof. Sections 110 and 100 provide: xxx xxx xxx Furthermore, based on the corporation's VAT returns, the corporation reported its taxable sales for 1992 in the amount of P11,736,658,580. This declaration is likewise false and fraudulent because, based on the daily manufacturer's sworn statements submitted to the BIR by the taxpayer, its total taxable sales during the year 1992 is P16,686,372,295.00. As a result thereof, the corporation was able to evade the payment of ad valorem taxes in the aggregate amount of P5,792,479,816.24 in violation of Section 127 in relation to Section 142, as amended by R.A. 6956, penalized under the aforequoted Section 253, in relation to Section 252, all of the Tax Code. Sections 127 and 142, as amended by R.A. 6956, are quoted as follows: . .. The complaint docketed as I.S. No. 93-508, was referred to the Department of Justice Task Force on revenue cases which found sufficient basis to further investigate the allegations that Fortune, through fraudulent means, evaded payment of income tax, ad valorem tax, and value-added tax for the year 1992 thus, depriving the government of revenues in the amount of Seven and One-half (P7.5) Billion Pesos. The fraudulent scheme allegedly adopted by private respondents consisted of making fictitious and simulated sales of Fortune's cigarette products to non-existing individuals and to entities incorporated and existing only for the purpose of such fictitious sales by declaring registered wholesale prices with the BIR lower than Fortune's actual wholesale prices which are required for determination of Fortune's correct income, ad valorem, and value-added tax liabilities. The "ghosts wholesale buyers" then ostensibly sold the products to customers and other wholesalers/retailers at higher wholesale prices determined by Fortune. The tax returns and manufacturer's sworn statements filed by Fortune would then declare the fictitious sales it made to the conduit corporators and non-existing individual buyers as its gross sales. 5 On September 8, 1993, the Department of Justice Task Force issued a subpoena directing private respondents to submit their counter-affidavits not later than September 20, 1993. 6 Instead of filing their counter-affidavits, the private respondents on October 15, 1993 filed a Verified Motion to Dismiss; Alternatively Motion to Suspend, 7 based principally on the following grounds: 1. The complaint of petitioner Commissioner follows a pattern of prosecution against private respondents in violation of their right to due process and equal protection of the law. 2. Petitioner Commissioner and the Court of Tax Appeals have still to determine Fortune's tax liability for 1992 in question; without any tax liability, there can be no tax evasion.

3. Exclusive jurisdiction to determine tax liability is vested in the Court of Tax Appeals; therefore, the DOJ is without jurisdiction to conduct preliminary investigation. 4. The complaint of petitioner Commissioner is not supported by any evidence to serve as adequate basis for the issuance of subpoena to private respondents and to put them to their defense. At the scheduled preliminary investigation on October 15, 1993, private respondents were asked by the panel of prosecutors to inform it of the aspects of the Verified Motion to Dismiss which counsel for private respondents did so briefly. Counsel for the Commissioner of Internal Revenue asked for fifteen (15) days within which to file a reply in writing to private respondents' Verified Motion to Dismiss. Thereupon, the panel of prosecutors declared a recess. Upon reconvening, the panel of prosecutors denied the motion to dismiss and treated the same as private respondents' counteraffidavits. 8 On October 20, 1993, private respondents filed a motion for reconsideration of the order of October 15, 1993. 9On October 21, 1993, private respondents filed a motion to require the submission by the Bureau of Internal Revenue of certain documents in further support of their Verified Motion to Dismiss. Among the documents sought to be produced are the "Daily Manufacturer's Sworn Statements" which according to petitioner Commissioner in her complaint were submitted by Fortune to the BIR and which were the basis of her conclusion that Fortune's tax declarations were false and fraudulent. Fortune claimed that without the "Daily Manufacturer's Sworn Statements," there is no evidence to support the complaint, hence, warranting its outright dismissal. On October 26, 1993, private respondents moved for the inhibition of the State prosecutors assigned to the case for alleged lack of impartiality. 10 Private respondents also sought the production of the "Daily Manufacturer's Sworn Statements" submitted by certain cigarette companies similarly situated as Fortune but were not proceeded against, thus, private respondents charged that Fortune and its officers were being singled out for criminal prosecution which is discriminatory and in violation of the equal protection clause of the Constitution. On December 20, 1993, the panel of prosecutors issued an Omnibus Order 11 denying private respondents' motion for reconsideration, motion for suspension of investigation, motion to inhibit the State Prosecutors, and motion to require submission by the BIR of certain documents to further support private respondents' motion to dismiss. On January 4, 1994, private respondents filed a petition for certiorari and prohibition with prayer for preliminary injunction with the Regional Trial Court, Branch 88, Quezon City, docketed as Q-9418790, praying that the complaint of the Commissioner of Internal Revenue and the orders of the prosecutors in I.S. No. 93-508 be dismissed or set aside, alternatively, the proceedings on the preliminary investigation be suspended pending final determination by the Commissioner of Fortune's motion for reconsideration/ reinvestigation of the August 13, 1993 assessment of the taxes due. 12 On January 17, 1994, petitioners filed a motion to dismiss the petition 13 on the grounds that (a) the trial court is bereft of jurisdiction to enjoin a criminal prosecution under preliminary investigation; (b) a criminal prosecution for tax fraud can proceed independently of criminal or administrative action; (c) there is no prejudicial question to justify suspension of the preliminary investigation; (d) private respondents' rights to due process was not violated; and (e) selective prosecution is not a valid defense in this jurisdiction.

On January 19, 1994, at the hearing of the incident for the issuance of a writ of preliminary injunction in the petition, private respondents offered in evidence their verified petition for certiorari and prohibition and its annexes. Petitioners responded by praying that their motion to dismiss the petition for certiorari and prohibition be considered as their opposition to private respondents' application for the issuance of a writ of preliminary injunction. On January 25, 1994, the trial court issued an order granting the prayer for the issuance of a preliminary injunction. 14 The trial court rationalized its order in this wise: a) It is private respondents' claim that the ad valorem tax for the year 1992 was levied, assessed and collected by the BIR under Section 142(c) of the Tax Code on the basis of the "manufacturer's registered wholesale price" duly approved by the BIR. Fortune's taxable sales for 1992 was in the amount of P11,736,658,580.00. b) On the other hand, it is petitioners' contention that Fortune's declaration was false and fraudulent because, based on its daily manufacturer's sworn statements submitted to the BIR, its taxable sales in 1992 were P16,686,372,295.00, as a result of which, Fortune was able to evade the payment of ad valorem tax in the aggregate amount of P5,792,479,816.24. c) At the hearing for preliminary investigation, the "Daily Manufacturer's Sworn Statements" which, according to petitioners, were submitted to the BIR by private respondents and made the basis of petitioner Commissioner's complaint that the total taxable sales of Fortune in 1992 amounted to P16,686,372, 295.00 were not produced as part of the evidence for petitioners. In fact, private respondents had filed a motion to require petitioner Commissioner to submit the aforesaid daily manufacturer's sworn statements before the DOJ panel of prosecutors to show that Fortune's actual taxable sales totaled P16,686,373,295.00, but the motion was denied. d) There is nothing on record in the preliminary investigation before the panel of investigators which supports the allegation that Fortune made a fraudulent declaration of its 1992 taxable sales. e) Since, as alleged by private respondents, the ad valorem tax for the year 1992 should be based on the "manufacturer's registered wholesale price" while, as claimed by petitioners, the ad valorem taxes should be based on the wholesale price at which the manufacturer sold the cigarettes, which is a legal issue as admitted by a BIR lawyer during the hearing for preliminary injunction, the correct interpretation of the law involved, which is Section 142(c) of the Tax Code, constitutes a prejudicial question which must first be resolved before criminal proceedings for tax evasion may be pursued. In other words, the BIR must first make a final determination, which it has not, of Fortune's tax liability relative to its 1992 ad valorem, value-added and income taxes before the taxpayer can be made liable for tax evasion. f) There was a precipitate issuance by the panel of prosecutors of subpoenas to private respondents, on the very day following the filing of the complaint with the DOJ consisting of about 600 pages, and the precipitate denial by the panel of prosecutors, after a recess of about twenty (20) minutes, of private respondents' motion to dismiss, consisting of one hundred and thirty five (135) pages.

g) Private respondents had been especially targeted by the government for prosecution. Prior to the filing of the complaint in I.S. No. 93-508, petitioner Commissioner issued Revenue Memorandum Circular No. 37-93 reclassifying Fortune's best selling cigarettes, namely "Hope," "More," and "Champion" as cigarettes bearing a foreign brand, thereby imposing upon them a higher rate of tax that would price them out of the market. h) While in petitioner Commissioner's letter of August 13, 1993, she gave Fortune a period of thirty (30) days from receipt thereof within which to pay the alleged tax deficiency assessments, she filed the criminal complaint for tax evasion before the period lapsed. i) Based on the foregoing, the criminal complaint against private respondents was filed prematurely and in violation of their constitutional right to equal protection of the laws. On January 26, 1994, private respondents filed with the trial court a Motion to Admit Supplemental Petition and sought the issuance of a writ of preliminary injunction to enjoin the State Prosecutors from continuing with the preliminary investigation filed by them against private respondents with the Quezon City Prosecutor's Office, docketed as I.S. 93-17942, for alleged fraudulent tax evasion, committed by private respondents for the taxable year 1990. Private respondents averred in their motion that no supporting documents or copies of the complaint were attached to the subpoena in I.S. 93-17942; that the subpoena violates private respondents' constitutional right to due process, equal protection and presumption of innocence; that I.S. 93-17942 is substantially the same as I.S. 93-508; that no tax assessment has been issued by the Commission of Internal Revenue and considering that taxes paid have not been challenged, no tax liability exists; and that since Assistant City Prosecutor Baraquia was a former classmate of Presidential Legal Counsel Antonio T. Carpio, the former cannot conduct the preliminary investigation in an impartial manner. On January 28, 1994, private respondents filed with the trial court a second supplemental petition, 15 also seeking to stay the preliminary investigation in I.S. 93-584, which was the third complaint filed against private respondents with the DOJ for alleged fraudulent tax evasion for the taxable year 1991. On January 31, 1994, the lower court admitted the two (2) supplemental petitions and issued a temporary restraining order in I.S. 93-17942 and I.S. 93-584. 16 Also, on the same day, petitioners filed an Urgent Motion for Immediate Resolution of petitioners' motion to dismiss. On February 7, 1994, the trial court issued an order denying petitioners' motion to dismiss private respondents' petition seeking to stay preliminary investigation in I.S. 93-508, ruling that the issue of whether Sec. 127(b) of the National Tax Revenue Code should be the basis of private respondents' tax liability as contended by the Bureau of Internal Revenue, or whether it is Section 142(c) of the same Code that applies, as argued by herein private respondents, should first be settled before any complaint for fraudulent tax evasion can be initiated. 17 On February 14, 1994, the trial court issued an order granting private respondents' petition for a supplemental writ of preliminary injunction, likewise enjoining the preliminary investigation of the two (2) other complaints filed with the Quezon City Prosecutor's Office and the DOJ for fraudulent tax evasion, I.S. 93-17942 and I.S. 93-584, for alleged tax evasion for the taxable years 1990 and 1991 respectively. 18 In granting the supplemental writ, the trial court stated that the two other complaints are the same as in I.S. 93-508, except that the former refer to the taxable years 1990 and 1991.

On March 7, 1994, petitioners filed a petition for certiorari and prohibition with prayer for preliminary injunction before this Court. However, the petition was referred to the Court of Appeals for disposition by virtue of its original concurrent jurisdiction over the petition. On December 19, 1994, the Court of Appeals in CA-G.R No. SP-33599 rendered a decision denying the petition. The Court of Appeals ruled that the trial court committed no grave abuse of discretion in ordering the issuance of writs of preliminary injunction and in denying petitioners' motion to dismiss. In upholding the reasons and conclusions given by the trial court in its orders for the issuance of the questioned writs, the Court of Appeals said in part: In making such conclusion the respondent Court must have understood from herein petitioner Commissioner's letter-complaint of 14 pages (pp. 477-490, rollo of this case) and the joint affidavit of eight revenue officers of 17 pages attached thereto (pp. 491-507, supra) and its annexes (pp. 508-1077, supra), that the charge against herein respondents is for tax evasion for non-payment by herein respondent Fortune of the correct amounts of income tax, ad valorem tax and value added tax, not necessarily "fraudulent tax evasion." Hence, the need for previous assessment of the correct amount by herein petitioner Commissioner before herein respondents may be charged criminally.Certiorari will not be issued to cure errors in proceedings or correct erroneous conclusions of law or fact. As long as a Court acts within its jurisdictions, any alleged error committed in the exercise of its jurisdiction, will amount to nothing more than errors of judgment which are reviewable by timely appeal and not by a special civil action of certiorari (Santos, Jr. vs. Court of Appeals, 152 SCRA 378; Gold City Integrated Port Services, Inc. vs. Intermediate Appellate Court, 171 SCRA 579). The questioned orders issued after hearing (Annexes A, B, C and D, petition) being but interlocutory, review thereof by this Court is inappropriate until final judgment is rendered, absent a showing of grave abuse of discretion on the part of the issuing court (See Van Dorn vs. Romillo, 139 SCRA 139, 141; Newsweek, Inc. vs. IAC, 171, 177; Mendoza vs. Court of Appeals, 201 SCRA 343, 352). The factual and legal issues involved in the main case still before the respondent Court are best resolved after trial. Petitioners, therefore, instead of resorting to this petition for certiorari and prohibition should have filed an answer to the petition as ordained in Section 4, Rule 16, in connection with Rule 11 of the Revised Rules of Court, interposing as defense or defenses the objection or objections raised in their motion to dismiss, then proceed to trial in order that thereafter the case may be decided on the merits by the respondent Court. In case of an adverse decision, they may appeal therefrom by which the entire record of the case would be elevated for review (See Mendoza vs. Court of Appeals, supra). Therefore, certiorari and prohibition resorted to by herein petitioners will not lie in view of the remedy open to them. Thus, the resulting delay in the final disposition of the case before the respondent Court would not have been incurred. Grave abuse of discretion as a ground for issuance of writs of certiorari and prohibition implies capricious and whimsical exercise of judgment as is equivalent to lack of jurisdiction, or where the power is exercised in an arbitrary or despotic manner by reason of passion, prejudice, or personal hostility, amounting to an evasion of positive duty or to a virtual refusal to perform the duty enjoined, or to act at all in contemplation of law (Confederation of Citizens Labor Union vs. NLRC, 60 SCRA 84; Bustamante vs. Commission on Audit, 216 SCRA 134). For such writs to lie, there must be capricious, arbitrary and whimsical exercise of power, the very antithesis of the judicial prerogative in accordance with centuries of both civil law and

common law traditions (Young vs. Sulit, 162 SCRA 659, 664; FCC vs. IAC, 166 SCRA 155; Purefoods Corp. vs. NLRC, 171 SCRA 45). Certiorari and prohibition are remedies narrow in scope and inflexible in character. They are not general utility tools in the legal workshop (Vda. de Guia vs. Veloso, 158 SCRA 340, 344). Their function is but limited to correction of defects of jurisdiction solely, not to be used for any other purpose (Garcia vs. Ranada, 166 SCRA 9), such as to cure errors in. proceedings or to correct erroneous conclusions of law or fact (Gold City Integrated Ports Services vs. IAC, 171 SCRA 579). Due regard for the foregoing teachings enunciated in the decisions cited can not bring about a decision other than what has been reached herein. Needless to say, the case before the respondent court involving those against herein respondents for alleged non-payment of the correct amounts due as income tax, ad valorem tax and value added tax for the years 1990, 1991 and 1992 (Civil Case No. Q-94-18790) is not ended by this decision. The respondent Court is still to try the case and decide it on the merits. All that is decided here is but the validity of the orders of the respondent Court granting herein respondents' application for preliminary injunction and denying herein petitioners' motion to dismiss. If upon the facts established after trial and the applicable law, dissolution of the writ of preliminary injunction allowed to be issued by the respondent Court is called for and a judgment favorable to herein petitioners is demanded, the respondent Court is duty bound to render judgment accordingly.
WHEREFORE, the instant petition for certiorari and prohibition with application for issuance of restraining order and writ of preliminary injunction is DISMISSED. Costs de oficio. 19

Their motion for reconsideration having been denied by respondent appellate court on February 23, 1995, petitioners filed the present petition for review based on the following grounds: THE RESPONDENT COURTS COMMITTED GRAVE ABUSE OF DISCRETION AMOUNTING TO LACK OR EXCESS OF JURISDICTION IN HOLDING THAT: I. THERE IS A PREJUDICIAL AND/OR LEGAL QUESTION TO JUSTIFY THE SUSPENSION OF THE PRELIMINARY INVESTIGATION. II. PRIVATE RESPONDENTS' RIGHTS TO DUE PROCESS, EQUAL PROTECTION AND PRESUMPTION OF INNOCENCE WERE VIOLATED; ON THE CONTRARY, THE STATE ITSELF WAS DEPRIVED OF DUE PROCESS. III. THE ADMISSION OF PRIVATE RESPONDENTS' SUPPLEMENTAL PETITIONS WERE PROPER. IV. THERE WAS SELECTIVE PROSECUTION. V. THE FACTUAL ALLEGATIONS IN THE PETITION ARE HYPOTHETICALLY ADMITTED IN A MOTION TO DISMISS BASED ON JURISDICTIONAL GROUNDS.
VI. THE ISSUANCE OF THE WRITS OF INJUNCTION IS NOT A DECISION ON THE MERITS OF THE PETITION BEFORE THE LOWER COURT. 20

The petition is bereft of merit.

In essence, the complaints in I.S. Nos. 93-508, 93-584 and 93-17942 charged private respondents with fraudulent tax evasion or wilfully attempting to evade or defeat payment of income tax, ad valorem tax and value-added tax for the year 1992, as well as for the years 1990-1991. The pertinent provisions of law involved are Sections 127(b) and 142(c) of the National Internal Revenue Code which state: Sec. 127. . . . (b) Determination of gross selling price of goods subject to ad valorem tax. -- Unless otherwise provided, the price, excluding the value-added tax, at which the goods are sold at wholesale in the place of production or through their sales agents to the public shall constitute the gross selling price. If the manufacturer also sells or allows such goods to be sold at wholesale price in another establishment of which he is the owner or in the profits at which he has an interest, the wholesale price in such establishment shall constitute the gross selling price. Should such price be less than the costs of manufacture plus expenses incurred until the goods are finally sold, then a proportionate margin of profit, not less than 10% of such manufacturing costs and expenses, shall be added to constitute the gross selling price. Sec. 142. . . . (c) Cigarettes packed in twenties. -- There shall be levied, assessed and collected on cigarettes packed in twenties an ad valorem tax at the rates prescribed below based on the manufacturer's registered wholesale price. xxx xxx xxx Private respondents contend that per Fortune's VAT returns, correct taxable sales for 1992 was in the amount of P11,736,658,580.00 which was the "manufacturer's registered wholesale price" in accordance with Section 142(c) of the Tax Code and paid the amount of P4,805,254,523 as ad valorem tax. On the other hand, petitioners allege, as specifically worded in the complaint in I.S. No. 93-508, that "based on the daily manufacturer's sworn statements submitted to the BIR by the Taxpayer (Fortune's) total taxable sales during the year 1992 is P16,686,372,295.00," as result of which Fortune "was able to evade the payment of ad valorem taxes in the aggregate amount of P5,792,479,816.24 . . ." Petitioners now argue that Section 127(b) lays down the rule that in determining the gross selling price of goods subject to ad valorem tax, it is the price, excluding the value-added tax, at which the goods are sold at wholesale price in the place of production or through their sales agents to the public. The registered wholesale price shall then be used for computing the ad valorem tax which is imposable upon removal of the taxable goods from the place of production. However, petitioners claim that Fortune used the "manufacturer's registered wholesale price" in selling the goods to alleged fictitious individuals and dummy corporations for the purpose of evading the payment of the correct ad valorem tax. There can be no question that under Section 127(b), the ad valorem tax should be based on the correct price excluding the value-added tax, at which goods are sold at wholesale in the place of production. It is significant to note that among the goods subject to ad valorem tax, the law -specifically Section 142(c) -- requires that the corresponding tax on cigarettes shall be levied,

assessed and collected at the rates based on the "manufacturer's registered wholesale price." Why does the wholesale price need to be registered and what is the purpose of the registration? The reason is self-evident, which is to ensure the payment of the correct taxes by the manufacturers of cigarettes through close supervision, monitoring and checking of the business operations of the cigarette companies. As pointed out by private respondents, no industry is as intensely supervised by the BIR and also by the National Tobacco Administration (NTA). Thus, the purchase and use of raw materials are subject to prior authorization and approval by the NTA. Importations of bobbins or cigarette paper, the manufacture, sale, and utilization of the same, are subject to BIR supervision and approval. 21 Moreover, as pointed to by private respondents, for purposes of closer supervision by the BIR over the production of cigarettes, Revenue Enforcement Officers are detailed on a 24-hour basis in the premises of the manufacturer to secure production and removal of finished products. Composite Mobile Teams conduct counter-security on the business operations as well as the performance of the Revenue Enforcement Officers detailed thereat. Every transfer of any raw material is not allowed unless, in addition to the required permits, accompanied by Revenue Enforcement Officer. For the purpose of determining the "Manufacturer's Registered Wholesale Price" a cigarette manufacturer is required to file a Manufacturer's Declaration (BIR Form No. 31.03) for each brand of cigarette manufactured, stating: a) Materials, b) Labor; c) Overhead; d) Tax Burden and the Wholesale Price by Case. The data submitted therewith is verified by the Revenue Officers and approved by the Commission of Internal Revenue. Any change in the manufacturer's registered wholesale price of any brand cannot be effected without submitting the corresponding Sworn Manufacturer's Declaration and verified by the Revenue Officer and approved by the Commissioner on Internal Revenue. 22 The amount of ad valorem tax payments together with the Payment Order and Confirmation Receipt Nos. must be indicated in the sales and delivery invoices and together with the Manufacturer's Sworn Declarations on (a) the quantity of raw materials used during the day's operations; (b) the total quantity produced according to brand; and (c) the corresponding quantity removed during the day, the corresponding wholesale price thereof, and the VAT paid thereon must be presented to the corresponding BIR representative for authentication before removal. Thus, as observed by the trial court in its order of January 25, 1994 granting private respondents' prayer for the issuance of a writ of preliminary injunction, Fortune's registered wholesale price (was) duly approved by the BIR, which fact is not disputed by petitioners. 23 Now, if every step in the production of cigarettes was closely monitored and supervised by the BIR personnel specifically assigned to Fortune's premises, and considering that the Manufacturer's Sworn Declarations on the data required to be submitted by the manufacturer were scrutinized and verified by the BIR and, further, since the manufacturer's wholesale price was duly approved by the BIR, then it is presumed that such registered wholesale price is the same as, or approximates "the price, excluding the value-added tax, at which the goods are sold at wholesale in the place production," otherwise, the BIR would not have approved the registered wholesale price of the goods for purposes of imposing the ad valorem tax due. In such case, and in the absence of contrary evidence, it was precipitate and premature to conclude that private respondents made fraudulent returns or wilfully attempted to evade payment of taxes due. "Wilful" means "premeditated; malicious; done with intent, or with bad motive or purpose, or with indifference to the natural consequence . . ." 24 "Fraud" in its general sense, "is deemed to comprise anything calculated to deceive, including all acts, omissions, and concealment involving a breach of legal or equitable duty, trust or confidence justly reposed, resulting in the damage to another, or by which an undue and unconscionable advantage taken of another. 25 Fraud cannot be presumed. If there was fraud or wilful attempt to evade payment of ad valorem taxes by private respondents through the manipulation of the registered wholesale price of the cigarettes, it must have been with the connivance or cooperation of certain BIR officials and

employees who supervised and monitored Fortune's production activities to see to it that the correct taxes were paid. But there is no allegation, much less evidence, of BIR personnel's malfeasance. In the very least, there is the presumption that the BIR personnel performed their duties in the regular course in ensuing the correct taxes were paid by Fortune. 26 It is the opinion of both the trial court and respondent Court of Appeals, that before Fortune and the other private respondents could be prosecuted for tax evasion under Sections 253 and 255 of the Tax Code, the fact that the deficiency income, ad valorem and value-added taxes were due from Fortune for the year 1992 should first be established. Fortune received form the Commissioner of Internal Revenue the deficiency assessment notices in the total amount of P7,685,942,221.06 on August 24, 1993. However, under Section 229 of the Tax Code, the taxpayer has the right to move for reconsideration of the assessment issued by the Commissioner of Internal Revenue within thirty (30) days from receipt of the assessment; and if the motion for reconsideration is denied, it may appeal to the Court of Appeals within thirty (30) days from receipt of the Commissioner's decision. Here, Fortune received the Commissioner's assessment notice dated August 13, 1993 on August 24, 1993 asking for the payment of the deficiency taxes. Within thirty (30) days from receipt thereof, Fortune moved for reconsideration. The Commissioner has not resolved the request for reconsideration up to the present. We share with the view of both the trial court and court of Appeals that before the tax liabilities of Fortune are first finally determined, it cannot be correctly asserted that private respondents have wilfully attempted to evade or defeat the taxes sought to be collected from Fortune. In plain words, before one is prosecuted for wilful attempt to evade or defeat any tax under Sections 253 and 255 of the Tax code, the fact that a tax is due must first be proved. Suppose the Commissioner eventually resolves Fortune's motion for reconsideration of the assessments by pronouncing that the taxpayer is not liable for any deficiency assessment, then, the criminal complaints filed against private respondents will have no leg to stand on. In view of the foregoing reasons, we cannot subscribe to the petitioners' thesis citing Ungad v. Cusi, 27 that the lack of a final determination of Fortune's exact or correct tax liability is not a bar to criminal prosecution, and that while a precise computation and assessment is required for a civil action to collect tax deficiencies, the Tax Code does not require such computation and assessment prior to criminal prosecution. Reading Ungad carefully, the pronouncement therein that deficiency assessment is not necessary prior to prosecution is pointedly and deliberately qualified by the Court with following statement quoted from Guzik v. U.S.:28 "The crime is complete when the violator has knowingly and wilfully filed a fraudulent return with intent to evade and defeat apart or all of the tax." In plain words, for criminal prosecution to proceed before assessment, there must be a prima facie showing of a wilful attempt to evade taxes. There was a wilful attempt to evade tax in Ungadbecause of the taxpayer's failure to declare in his income tax return "his income derived from banana sapplings." In the mind of the trial court and the Court of Appeals, Fortune's situation is quite apart factually since the registered wholesale price of the goods, approved by the BIR, is presumed to be the actual wholesale price, therefore, not fraudulent and unless and until the BIR has made a final determination of what is supposed to be the correct taxes, the taxpayer should not be placed in the crucible of criminal prosecution. Herein lies a whale of difference between Ungad and the case at bar. This brings us to the erroneous disquisition that private respondents' recourse to the trial court by way of special civil action of certiorari and prohibition was improper because: a) the proceedings before the state prosecutors (preliminary injunction) were far from terminated -- private respondents

were merely subpoenaed and asked to submit counter affidavits, matters that they should have appealed to the Secretary of Justice; b) it is only after the submission of private respondents' counter affidavits that the prosecutors will determine whether or not there is enough evidence to file in court criminal charges for fraudulent tax evasion against private respondents; and c) the proper procedure is to allow the prosecutors to conduct and finish the preliminary investigation and to render a resolution, after which the aggrieved party can appeal the resolution to the Secretary of Justice. We disagree. As a general rule, criminal prosecutions cannot be enjoined. However, there are recognized exceptions which, as summarized in Brocka v. Enrile 29 are: a. To afford adequate protection to the constitutional rights of the accused (Hernandez vs. Albano, et al., L-19272, January 25, 1967, 19 SCRA 95); b. When necessary for the orderly administration of justice or to avoid oppression or multiplicity of actions (Dimayuga, et al. vs. Fernandez, 43 Phil. 304; Hernandez vs. Albano, supra; Fortun vs. Labang, et al., L-38383, May 27, 1981, 104 SCRA 607); c. When there is a prejudicial question which is sub judice (De Leon vs. Mabanag, 70 Phil 202); d. When the acts of the officer are without or in excess of authority (Planas vs. Gil, 67 Phil 62); e. Where the prosecution is under an invalid law, ordinance or regulation (Young vs. Rafferty, 33 Phil. 556; Yu Cong Eng vs. Trinidad, 47 Phil. 385, 389); f. When double jeopardy is clearly apparent (Sangalang vs. People and Alvendia, 109 Phil. 1140); g. Where the court had no jurisdiction over the offense (Lopez vs. City Judge, L25795, October 29, 1966, 18 SCRA 616); h. Where it is a case of persecution rather than prosecution (Rustia vs. Ocampo, CAG.R. No. 4760, March 25, 1960); i. Where the charges are manifestly false and motivated by the lust for vengeance (Recto vs. Castelo, 18 L.J. [1953], cited in Rano vs. Alvenia, CA-G.R. No. 30720-R, October 8, 1962; Cf. Guingona, et al. vs. City Fiscal, L-60033, April 4, 1984, 128 SCRA 577); and j. When there is clearly no prima facie case against the accused and a motion to quash on that ground has been denied (Salonga vs. Pane, et al., L-59524, February 18, 1985, 134 SCRA 438). In issuing the questioned orders granting the issuance of a writ of preliminary injunction, the trial court believed that said orders were warranted to afford private respondents adequate protection of their constitutional rights, particularly in reference to presumption of innocence, due process and equal protection of the laws. The trial court also found merit in private respondents' contention that preliminary injunction should be issued to avoid oppression and because the acts of the state

prosecutors were without or in excess of authority and for the reason that there was a prejudicial question. Contrary to petitioners' submission, preliminary investigation may be enjoined where exceptional circumstances so warrant. In Hernandez v. Albano 30 and Fortun v. Labang, 31 injunction was issued to enjoin a preliminary investigation. In the case at bar, private respondents filed a motion to dismiss the complaint against them before the prosecution and alternatively, to suspend the preliminary investigation on the grounds cited hereinbefore, one of which is that the complaint of the Commissioner is not supported by any evidence to serve as adequate basis for the issuance of the subpoena to them and put them to their defense. Indeed, the purpose of a preliminary injunction is to secure the innocent against hasty, malicious and oppressive prosecution and to protect him from an open and public accusation of crime, from the trouble, expense and anxiety of a public trial and also to protect the state from useless and expensive trials. 32 Thus, the pertinent provisions of Rule 112 of the Rules of Court state: Sec. 3. Procedure. -- Except as provided for in Section 7 hereof, no complaint or information for an offense cognizable by the Regional Trial Court shall be filed without a preliminary investigation having been first conducted in the following manner: (a) The complaint shall state the known address of the respondent and be accompanied by affidavits of the complainant and his witnesses as well as other supporting documents, in such number of copies as there are respondents, plus two (2) copies for the official file. The said affidavits shall be sworn to before any fiscal, state prosecutor or government official authorized to administer oath, or, in their absence or unavailability, a notary public, who must certify that he personally examined the affiants and that he is satisfied that they voluntarily executed and understood their affidavits. (b) Within ten (10) days after the filing of the complaint, the investigating officer shall either dismiss the same if he finds no ground to continue with the inquiry, or issue a subpoena to the respondent, attaching thereto a copy of the complaint, affidavits and other supporting documents. Within ten (10) days from receipt thereof, the respondent shall submit counter-affidavits and other supporting documents. He shall have the right to examine all other evidence submitted by the complainant. (c) Such counter-affidavits and other supporting evidence submitted by the respondent shall also be sworn to and certified as prescribed in paragraph (a) hereof and copies thereof shall be furnished by him to the complainant. (d) If the respondent cannot be subpoenaed, or if subpoenaed, does not submit counter-affidavits within the ten (10) day period, the investigating officer shall base his resolution on the evidence presented by the complainant. (e) If the investigating officer believes that there are matters to be clarified, he may set a hearing to propound clarificatory questions to the parties or their witnesses, during which the parties shall be afforded an opportunity to be present but without the right to examine or cross-examine. If the parties so desire, they may submit questions to the investigating officer which the latter may propound to the parties or witnesses concerned.

(f) Thereafter, the investigation shall be deemed concluded, and the investigating officer shall resolve the case within ten (10) days therefrom. Upon the evidence thus adduced, the investigating officer shall determine whether or not there is sufficient ground to hold the respondent for trial. As found by the Court of Appeals, there was obvious haste by which the subpoena was issued to private respondents, just the day after the complaint was filed, hence, without the investigating prosecutors being afforded material time to examine and study the voluminous documents appended to the complaint for them to determine if preliminary investigation should be conducted. The Court of Appeals further added that the precipitate haste in the issuance of the subpoena justified private respondents' misgivings regarding the objectivity and neutrality of the prosecutors in the conduct of the preliminary investigation and so, the appellate court concluded, the grant of preliminary investigation by the trial court to afford adequate protection to private respondents' constitutional rights and to avoid oppression does not constitute grave abuse of discretion amounting to lack of jurisdiction. The complaint filed by the Commissioner on Internal Revenue states itself that the primary evidence establishing the falsity of the declared taxable sales in 1992 in the amount of P11,736,658,580.00 were the "daily Manufacturer's Sworn Statements" submitted by the taxpayer which would show that the total taxable sales in 1992 are in the amount of P16,686,372,295.00. However, the Commissioner did not present the "Daily Manufacturer's Sworn Statements" supposedly submitted to the BIR by the taxpayer, prompting private respondents to move for their production in order to verify the basis of petitioners' computation. Still, the Commissioner failed to produce the declarations. In Borja v. Moreno, 33 it was held that the act of the investigator in proceeding with the hearing without first acting on respondents' motion to dismiss is a manifest disregard of the requirement of due process. Implicit in the opinion of the trial court and the Court of Appeals is that, if upon the examination of the complaint, it was clear that there was no ground to continue, with the inquiry, the investigating prosecutor was duty bound to dismiss the case. On this point, the trial court stressed that the prosecutor conducting the preliminary investigation should have allowed the production of the "Daily Manufacturer's Sworn Statements" submitted by Fortune without which there was no valid basis for the allegation that private respondents wilfully attempted to evade payment of the correct taxes. The prosecutors should also have produced the "Daily Manufacturer's Sworn Statements" by other cigarette companies, as sought by private respondents, to show that these companies which had paid the ad valorem taxes on the same basis and in the same manner as Fortune were not similarly criminally charged. But the investigating prosecutors denied private respondents' motion, thus, indicating that only Fortune was singled out for prosecution. The trial court and the Court of Appeals maintained that at that stage of the preliminary investigation, where the complaint and the accompanying affidavits and supporting documents did not show any violation of the Tax Code providing penal sanctions, the prosecutors should have dismissed the complaint outright because of total lack of evidence, instead of requiring private respondents to submit their counter affidavits under Section 3(b) of Rule 112. We believe that the trial court in issuing its questioned orders, which are interlocutory in nature, committed no grave abuse of discretion amounting to lack of jurisdiction. There are factual and legal bases for the assailed orders. On the other hand, the burden is upon the petitioners to demonstrate that the questioned orders constitute a whimsical and capricious exercise of judgment, which they have not. For certiorari will not be issued to cure errors in proceedings or correct erroneous conclusions of law or fact. As long as a court acts within its jurisdiction, any alleged errors committed in the exercise of its jurisdiction will amount to nothing more than errors of judgment which are reviewable by timely appeal and not by a special civil action of certiorari. 34 Consequently, the Regional Trial Court acted correctly and judiciously, and as demanded by the facts and the law, in issuing the orders granting the writs of preliminary injunction, in denying petitioners' motion to dismiss and in admitting the supplemental petitions. What petitioners should have done was to file

an answer to the petition filed in the trial court, proceed to the hearing and appeal the decision of the court if adverse to them. WHEREFORE, the instant petition is hereby DISMISSED. SO ORDERED. Hermosisima, Jr., J., concurs.

Republic of the Philippines SUPREME COURT Manila FIRST DIVISION G.R. No. 120935 May 21, 2009

LUCAS G. ADAMSON, THERESE JUNE D. ADAMSON, and SARA S. DE LOS REYES, in their capacities as President, Treasurer and Secretary of Adamson Management Corporation, Petitioners, vs. COURT OF APPEALS and LIWAYWAY VINZONS-CHATO, in her capacity as Commissioner of the Bureau of Internal Revenue, Respondents. x - - - - - - - - - - - - - - - - - - - - - - -x G.R. No. 124557 May 21, 2009

INTERNAL REVENUE, Petitioner, vs. COMMISSIONER OF COURT OF APPEALS, COURT OF TAX APPEALS, ADAMSON MANAGEMENT CORPORATION, LUCAS G. ADAMSON, THERESE JUNE D. ADAMSON, and SARA S. DE LOS REYES,Respondents. DECISION PUNO, C.J.: Before the Court are the consolidated cases of G.R. No. 120935 and G.R. No. 124557. G.R. No. 120935 involves a petition for review on certiorari filed by petitioners LUCAS G. ADAMSON, THERESE JUNE D. ADAMSON, and SARA S. DE LOS REYES (private respondents), in their respective capacities as president, treasurer and secretary of Adamson Management Corporation (AMC) against then Commissioner of Internal Revenue Liwayway Vinzons-Chato (COMMISSIONER), under Rule 45 of the Revised Rules of Court. They seek to review and reverse the Decision promulgated on March 21, 1995 and Resolution issued on July 6, 1995 of the Court of

Appeals in CA-G.R. SP No. 35488 (Liwayway Vinzons-Chato, et al. v. Hon. Judge Erna FalloranAliposa, et al.). G.R. No. 124557 is a petition for review on certiorari filed by the Commissioner, assailing the Decision dated March 29, 1996 of the Court of Appeals in CA-G.R. SP No. 35520, titled Commissioner of Internal Revenue v. Court of Tax Appeals, Adamson Management Corporation, Lucas G. Adamson, Therese June D. Adamson and Sara S. de los Reyes. In the said Decision, the Court of Appeals upheld the Resolution promulgated on September 19, 1994 by the Court of Tax Appeals (CTA) in C.T.A. Case No. 5075 (Adamson Management Corporation, Lucas G. Adamson, Therese Adamson and Sara de los Reyes v. Commissioner of Internal Revenue). The facts, as culled from the findings of the appellate court, follow: On June 20, 1990, Lucas Adamson and AMC sold 131,897 common shares of stock in Adamson and Adamson, Inc. (AAI) to APAC Holding Limited (APAC). The shares were valued at P7,789,995.00.1 On June 22, 1990,P159,363.21 was paid as capital gains tax for the transaction. On October 12, 1990, AMC sold to APAC Philippines, Inc. another 229,870 common shares of stock in AAI forP17,718,360.00. AMC paid the capital gains tax of P352,242.96. On October 15, 1993, the Commissioner issued a "Notice of Taxpayer" to AMC, Lucas G. Adamson, Therese June D. Adamson and Sara S. de los Reyes, informing them of deficiencies on their payment of capital gains tax and Value Added Tax (VAT). The notice contained a schedule for preliminary conference. The events preceding G.R. No. 120935 are the following: On October 22, 1993, the Commissioner filed with the Department of Justice (DOJ) her Affidavit of Complaint2against AMC, Lucas G. Adamson, Therese June D. Adamson and Sara S. de los Reyes for violation of Sections 45 (a) and (d)3 , and 1104 , in relation to Section 1005 , as penalized under Section 255,6 and for violation of Section 2537 , in relation to Section 252 (b) and (d) of the National Internal Revenue Code (NIRC).8 AMC, Lucas G. Adamson, Therese June D. Adamson and Sara S. de los Reyes filed with the DOJ a motion to suspend proceedings on the ground of prejudicial question, pendency of a civil case with the Supreme Court, and pendency of their letter-request for re-investigation with the Commissioner. After the preliminary investigation, State Prosecutor Alfredo P. Agcaoili found probable cause. The Motion for Reconsideration against the findings of probable cause was denied by the prosecutor. On April 29, 1994, Lucas G. Adamson, Therese June D. Adamson and Sara S. de los Reyes were charged before the Regional Trial Court (RTC) of Makati, Branch 150 in Criminal Case Nos. 94-1842 to 94-1846. They filed a Motion to Dismiss or Suspend the Proceedings. They invoked the grounds that there was yet no final assessment of their tax liability, and there were still pending relevant Supreme Court and CTA cases. Initially, the trial court denied the motion. A Motion for Reconsideration was however filed, this time assailing the trial court¶s lack of jurisdiction over the nature of the subject cases. On August 8, 1994, the trial court granted the Motion. It ruled that the complaints for tax evasion filed by the Commissioner should be regarded as a decision of the Commissioner regarding the tax liabilities of Lucas G. Adamson, Therese June D. Adamson and Sara S. de los Reyes, and appealable to the CTA. It further held that the said cases cannot proceed independently of the assessment case pending before the CTA, which has jurisdiction to determine the civil and criminal tax liability of the respondents therein.

On October 10, 1994, the Commissioner filed a Petition for Review with the Court of Appeals assailing the trial court¶s dismissal of the criminal cases. She averred that it was not a condition prerequisite that a formal assessment should first be given to the private respondents before she may file the aforesaid criminal complaints against them. She argued that the criminal complaints for tax evasion may proceed independently from the assessment cases pending before the CTA. On March 21, 1995, the Court of Appeals reversed the trial court¶s decision and reinstated the criminal complaints. The appellate court held that, in a criminal prosecution for tax evasion, assessment of tax deficiency is not required because the offense of tax evasion is complete or consummated when the offender has knowingly and willfully filed a fraudulent return with intent to evade the tax.9 It ruled that private respondents filed false and fraudulent returns with intent to evade taxes, and acting thereupon, petitioner filed an Affidavit of Complaint with the Department of Justice, without an accompanying assessment of the tax deficiency of private respondents, in order to commence criminal action against the latter for tax evasion.10 Private respondents filed a Motion for Reconsideration, but the trial court denied the motion on July 6, 1995. Thus, they filed the petition in G.R. No. 120935, raising the following issues: 1. WHETHER OR NOT THE RESPONDENT HONORABLE COURT OF APPEALS ERRED IN APPLYING THE DOCTRINE IN UNGAB V. CUSI (Nos. L-41919-24, May 30, 1980, 97 SCRA 877) TO THE CASE AT BAR. 2. WHETHER OR NOT AN ASSESSMENT IS REQUIRED UNDER THE SECOND CATEGORY OF THE OFFENSE IN SECTION 253 OF THE NIRC. 3. WHETHER OR NOT THERE WAS A VALID ASSESSMENT MADE BY THE COMMISSIONER IN THE CASE AT BAR. 4. WHETHER OR NOT THE FILING OF A CRIMINAL COMPLAINT SERVES AS AN IMPLIED ASSESSMENT ON THE TAX LIABILITY OF THE TAXPAYER. 5. WHETHER OR NOT THE FILING OF THE CRIMINAL INFORMATION FOR TAX EVASION IN THE TRIAL COURT IS PREMATURE BECAUSE THERE IS YET NO BASIS FOR THE CRIMINAL CHARGE OF WILLFULL INTENT TO EVADE THE PAYMENT OF A TAX. 6. WHETHER OR NOT THE DOCTRINES LAID DOWN IN THE CASES OF YABES V. FLOJO (No. L-46954, July 20, 1982, 115 SCRA 286) AND CIR V. UNION SHIPPING CORP. (G.R. No. 66160, May 21, 1990, 185 SCRA 547) ARE APPLICABLE TO THE CASE AT BAR. 7. WHETHER OR NOT THE COURT OF TAX APPEALS HAS JURISDICTION OVER THE DISPUTE ON WHAT CONSTITUTES THE PROPER TAXES DUE FROM THE TAXPAYER. In parallel circumstances, the following events preceded G.R. No. 124557: On December 1, 1993, AMC, Lucas G. Adamson, Therese June D. Adamson and Sara S. de los Reyes filed a letter request for re-investigation with the Commissioner of the "Examiner¶s Findings" earlier issued by the Bureau of Internal Revenue (BIR), which pointed out the tax deficiencies. On March 15, 1994 before the Commissioner could act on their letter-request, AMC, Lucas G. Adamson, Therese June D. Adamson and Sara S. de los Reyes filed a Petition for Review with the CTA. They assailed the Commissioner¶s finding of tax evasion against them. The Commissioner

moved to dismiss the petition, on the ground that it was premature, as she had not yet issued a formal assessment of the tax liability of therein petitioners. On September 19, 1994, the CTA denied the Motion to Dismiss. It considered the criminal complaint filed by the Commissioner with the DOJ as an implied formal assessment, and the filing of the criminal informations with the RTC as a denial of petitioners¶ protest regarding the tax deficiency. The Commissioner repaired to the Court of Appeals on the ground that the CTA acted with grave abuse of discretion. She contended that, with regard to the protest provided under Section 229 of the NIRC, there must first be a formal assessment issued by the Commissioner, and it must be in accord with Section 6 of Revenue Regulation No. 12-85. She maintained that she had not yet issued a formal assessment of tax liability, and the tax deficiency amounts mentioned in her criminal complaint with the DOJ were given only to show the difference between the tax returns filed and the audit findings of the revenue examiner. The Court of Appeals sustained the CTA¶s denial of the Commissioner¶s Motion to Dismiss. Thus, the Commissioner filed the petition for review under G.R. No. 124557, raising the following issues: 1. WHETHER OR NOT THE INSTANT PETITION SHOULD BE DISMISSED FOR FAILURE TO COMPLY WITH THE MANDATORY REQUIREMENT OF A CERTIFICATION UNDER OATH AGAINST FORUM SHOPPING; 2. WHETHER OR NOT THE CRIMINAL CASE FOR TAX EVASION IN THE CASE AT BAR CAN PROCEED WITHOUT AN ASSESSMENT; 3. WHETHER OR NOT THE COMPLAINT FILED WITH THE DEPARTMENT OF JUSTICE CAN BE CONSTRUED AS AN IMPLIED ASSESSMENT; and 4. WHETHER OR NOT THE COURT OF TAX APPEALS HAS JURISDICTION TO ACT ON PRIVATE RESPONDENTS¶ PETITION FOR REVIEW FILED WITH THE SAID COURT. The issues in G.R. No. 124557 and G.R. No. 120935 can be compressed into three: 1. WHETHER THE COMMISSIONER HAS ALREADY RENDERED AN ASSESSMENT (FORMAL OR OTHERWISE) OF THE TAX LIABILITY OF AMC, LUCAS G. ADAMSON, THERESE JUNE D. ADAMSON AND SARA S. DE LOS REYES; 2. WHETHER THERE IS BASIS FOR THE CRIMINAL CASES FOR TAX EVASION TO PROCEED AGAINST AMC, LUCAS G. ADAMSON, THERESE JUNE D. ADAMSON AND SARA S. DE LOS REYES; and 3. WHETHER THE COURT OF TAX APPEALS HAS JURISDICTION TO TAKE COGNIZANCE OF BOTH THE CIVIL AND THE CRIMINAL ASPECTS OF THE TAX LIABILITY OF AMC, LUCAS G. ADAMSON, THERESE JUNE D. ADAMSON AND SARA S. DE LOS REYES. The case of CIR v. Pascor Realty, et al.11 is relevant. In this case, then BIR Commissioner Jose U. Ong authorized revenue officers to examine the books of accounts and other accounting records of Pascor Realty and Development Corporation (PRDC) for 1986, 1987 and 1988. This resulted in a recommendation for the issuance of an assessment in the amounts of P7,498,434.65 and P3,015,236.35 for the years 1986 and 1987, respectively.

On March 1, 1995, the Commissioner filed a criminal complaint before the DOJ against PRDC, its President Rogelio A. Dio, and its Treasurer Virginia S. Dio, alleging evasion of taxes in the total amount of P10,513,671.00. Private respondents filed an Urgent Request for Reconsideration/Reinvestigation disputing the tax assessment and tax liability. The Commissioner denied the urgent request for reconsideration/reinvestigation because she had not yet issued a formal assessment. Private respondents then elevated the Decision of the Commissioner to the CTA on a petition for review. The Commissioner filed a Motion to Dismiss the petition on the ground that the CTA has no jurisdiction over the subject matter of the petition, as there was yet no formal assessment issued against the petitioners. The CTA denied the said motion to dismiss and ordered the Commissioner to file an answer within thirty (30) days. The Commissioner did not file an answer nor did she move to reconsider the resolution. Instead, the Commissioner filed a petition for review of the CTA decision with the Court of Appeals. The Court of Appeals upheld the CTA order. However, this Court reversed the Court of Appeals decision and the CTA order, and ordered the dismissal of the petition. We held: An assessment contains not only a computation of tax liabilities, but also a demand for payment within a prescribed period. It also signals the time when penalties and interests begin to accrue against the taxpayer. To enable the taxpayer to determine his remedies thereon, due process requires that it must be served on and received by the taxpayer. Accordingly, an affidavit, which was executed by revenue officers stating the tax liabilities of a taxpayer and attached to a criminal complaint for tax evasion, cannot be deemed an assessment that can be questioned before the Court of Tax Appeals. Neither the NIRC nor the revenue regulations governing the protest of assessments12 provide a specific definition or form of an assessment. However, the NIRC defines the specific functions and effects of an assessment. To consider the affidavit attached to the Complaint as a proper assessment is to subvert the nature of an assessment and to set a bad precedent that will prejudice innocent taxpayers. True, as pointed out by the private respondents, an assessment informs the taxpayer that he or she has tax liabilities. But not all documents coming from the BIR containing a computation of the tax liability can be deemed assessments. To start with, an assessment must be sent to and received by a taxpayer, and must demand payment of the taxes described therein within a specific period. Thus, the NIRC imposes a 25 percent penalty, in addition to the tax due, in case the taxpayer fails to pay the deficiency tax within the time prescribed for its payment in the notice of assessment. Likewise, an interest of 20 percent per annum, or such higher rate as may be prescribed by rules and regulations, is to be collected from the date prescribed for its payment until the full payment.13 The issuance of an assessment is vital in determining the period of limitation regarding its proper issuance and the period within which to protest it. Section 20314 of the NIRC provides that internal revenue taxes must be assessed within three years from the last day within which to file the return. Section 222,15 on the other hand, specifies a period of ten years in case a fraudulent return with intent to evade was submitted or in case of failure to file a return. Also, Section 22816 of the same law states that said assessment may be protested only within thirty days from receipt thereof. Necessarily, the taxpayer must be certain that a specific document constitutes an assessment. Otherwise, confusion would arise regarding the period within which to make an assessment or to protest the same, or whether interest and penalty may accrue thereon.

It should also be stressed that the said document is a notice duly sent to the taxpayer. Indeed, an assessment is deemed made only when the collector of internal revenue releases, mails or sends such notice to the taxpayer.17 In the present case, the revenue officers¶ Affidavit merely contained a computation of respondents¶ tax liability. It did not state a demand or a period for payment. Worse, it was addressed to the justice secretary, not to the taxpayers.
lawphil.ne t

Respondents maintain that an assessment, in relation to taxation, is simply understood to mean: "A notice to the effect that the amount therein stated is due as tax and a demand for payment thereof."18 "Fixes the liability of the taxpayer and ascertains the facts and furnishes the data for the proper presentation of tax rolls."19 Even these definitions fail to advance private respondents¶ case. That the BIR examiners¶ Joint Affidavit attached to the Criminal Complaint contained some details of the tax liabilities of private respondents does not ipso factomake it an assessment. The purpose of the Joint Affidavit was merely to support and substantiate the Criminal Complaint for tax evasion. Clearly, it was not meant to be a notice of the tax due and a demand to the private respondents for payment thereof. The fact that the Complaint itself was specifically directed and sent to the Department of Justice and not to private respondents shows that the intent of the commissioner was to file a criminal complaint for tax evasion, not to issue an assessment. Although the revenue officers recommended the issuance of an assessment, the commissioner opted instead to file a criminal case for tax evasion. What private respondents received was a notice from the DOJ that a criminal case for tax evasion had been filed against them, not a notice that the Bureau of Internal Revenue had made an assessment. Private respondents maintain that the filing of a criminal complaint must be preceded by an assessment. This is incorrect, because Section 222 of the NIRC specifically states that in cases where a false or fraudulent return is submitted or in cases of failure to file a return such as this case, proceedings in court may be commenced without an assessment. Furthermore, Section 205 of the same Code clearly mandates that the civil and criminal aspects of the case may be pursued simultaneously. In Ungab v. Cusi,20 petitioner therein sought the dismissal of the criminal Complaints for being premature, since his protest to the CTA had not yet been resolved. The Court held that such protests could not stop or suspend the criminal action which was independent of the resolution of the protest in the CTA. This was because the commissioner of internal revenue had, in such tax evasion cases, discretion on whether to issue an assessment or to file a criminal case against the taxpayer or to do both. Private respondents insist that Section 222 should be read in relation to Section 255 of the NIRC,21 which penalizes failure to file a return. They add that a tax assessment should precede a criminal indictment. We disagree. To reiterate, said Section 222 states that an assessment is not necessary before a criminal charge can be filed. This is the general rule. Private respondents failed to show that they are entitled to an exception. Moreover, the criminal charge need only be supported by a prima facie showing of failure to file a required return. This fact need not be proven by an assessment. The issuance of an assessment must be distinguished from the filing of a complaint. Before an assessment is issued, there is, by practice, a pre-assessment notice sent to the taxpayer. The

taxpayer is then given a chance to submit position papers and documents to prove that the assessment is unwarranted. If the commissioner is unsatisfied, an assessment signed by him or her is then sent to the taxpayer informing the latter specifically and clearly that an assessment has been made against him or her. In contrast, the criminal charge need not go through all these. The criminal charge is filed directly with the DOJ. Thereafter, the taxpayer is notified that a criminal case had been filed against him, not that the commissioner has issued an assessment. It must be stressed that a criminal complaint is instituted not to demand payment, but to penalize the taxpayer for violation of the Tax Code. In the cases at bar, the Commissioner denied that she issued a formal assessment of the tax liability of AMC, Lucas G. Adamson, Therese June D. Adamson and Sara S. de los Reyes. She admits though that she wrote the recommendation letter22 addressed to the Secretary of the DOJ recommending the filing of criminal complaints against AMC and the aforecited persons for fraudulent returns and tax evasion. The first issue is whether the Commissioner¶s recommendation letter can be considered as a formal assessment of private respondents¶ tax liability. In the context in which it is used in the NIRC, an assessment is a written notice and demand made by the BIR on the taxpayer for the settlement of a due tax liability that is there definitely set and fixed. A written communication containing a computation by a revenue officer of the tax liability of a taxpayer and giving him an opportunity to contest or disprove the BIR examiner¶s findings is not an assessment since it is yet indefinite.23 We rule that the recommendation letter of the Commissioner cannot be considered a formal assessment. Even a cursory perusal of the said letter would reveal three key points: 1. It was not addressed to the taxpayers. 2. There was no demand made on the taxpayers to pay the tax liability, nor a period for payment set therein. 3. The letter was never mailed or sent to the taxpayers by the Commissioner. In fine, the said recommendation letter served merely as the prima facie basis for filing criminal informations that the taxpayers had violated Section 45 (a) and (d), and 110, in relation to Section 100, as penalized under Section 255, and for violation of Section 253, in relation to Section 252 9(b) and (d) of the Tax Code.24 The next issue is whether the filing of the criminal complaints against the private respondents by the DOJ is premature for lack of a formal assessment. Section 269 of the NIRC (now Section 222 of the Tax Reform Act of 1997) provides: Sec. 269. Exceptions as to period of limitation of assessment and collection of taxes.-(a) In the case of a false or fraudulent return with intent to evade tax or of failure to file a return, the tax may be assessed, or a proceeding in court after the collection of such tax may be begun without assessment, at any time within ten years after the discovery of the falsity, fraud or omission: Provided, That in a fraud assessment which has become final and executory, the fact of fraud shall be judicially taken cognizance of in the civil or criminal action for collection thereof«

The law is clear. When fraudulent tax returns are involved as in the cases at bar, a proceeding in court after the collection of such tax may be begun without assessment. Here, the private respondents had already filed the capital gains tax return and the VAT returns, and paid the taxes they have declared due therefrom. Upon investigation of the examiners of the BIR, there was a preliminary finding of gross discrepancy in the computation of the capital gains taxes due from the sale of two lots of AAI shares, first to APAC and then to APAC Philippines, Limited. The examiners also found that the VAT had not been paid for VAT-liable sale of services for the third and fourth quarters of 1990. Arguably, the gross disparity in the taxes due and the amounts actually declared by the private respondents constitutes badges of fraud. Thus, the applicability of Ungab v. Cusi25 is evident to the cases at bar. In this seminal case, this Court ruled that there was no need for precise computation and formal assessment in order for criminal complaints to be filed against him. It quoted Merten¶s Law of Federal Income Taxation, Vol. 10, Sec. 55A.05, p. 21, thus: An assessment of a deficiency is not necessary to a criminal prosecution for willful attempt to defeat and evade the income tax. A crime is complete when the violator has knowingly and willfully filed a fraudulent return, with intent to evade and defeat the tax. The perpetration of the crime is grounded upon knowledge on the part of the taxpayer that he has made an inaccurate return, and the government¶s failure to discover the error and promptly to assess has no connections with the commission of the crime. This hoary principle still underlies Section 269 and related provisions of the present Tax Code. We now go to the issue of whether the CTA has no jurisdiction to take cognizance of both the criminal and civil cases here at bar.
1avvp hi1

Under Republic Act No. 1125 (An Act Creating the Court of Tax Appeals) as amended, the rulings of the Commissioner are appealable to the CTA, thus: SEC. 7. Jurisdiction. ± The Court of Tax Appeals shall exercise exclusive appellate jurisdiction to review by appeal, as herein provided (1) Decisions of the Commissioner of Internal Revenue in cases involving disputed assessments, refunds of internal revenue taxes, fees or other charges, penalties imposed in relation thereto, or other matters arising under the National Internal Revenue Code or other laws or part of law administered by the Bureau of Internal Revenue; Republic Act No. 8424, titled "An Act Amending the National Internal Revenue Code, As Amended, And For Other Purposes," later expanded the jurisdiction of the Commissioner and, correspondingly, that of the CTA, thus: SEC. 4. Power of the Commissioner to Interpret Tax Laws and to Decide Tax Cases. ± The power to interpret the provisions of this Code and other tax laws shall be under the exclusive and original jurisdiction of the Commissioner, subject to review by the Secretary of Finance. The power to decide disputed assessments, refunds of internal revenue taxes, fees or other charges, penalties imposed in relation thereto, or other matters arising under this Code or other laws or portions thereof administered by the Bureau of Internal Revenue is vested in the Commissioner, subject to the exclusive appellate jurisdiction of the Court of Tax Appeals.

The latest statute dealing with the jurisdiction of the CTA is Republic Act No. 9282.26 It provides: SEC. 7. Section 7 of the same Act is hereby amended to read as follows: Sec. 7. Jurisdiction. ² The CTA shall exercise: (a) Exclusive appellate jurisdiction to review by appeal, as herein provided: (1) Decisions of the Commissioner of Internal Revenue in cases involving disputed assessments, refunds of internal revenue taxes, fees or other charges, penalties in relation thereto, or other matters arising under the National Internal Revenue or other laws administered by the Bureau of Internal Revenue; (2) Inaction by the Commissioner of Internal Revenue in cases involving disputed assessments, refunds of internal revenue taxes, fees or other charges, penalties in relation thereto, or other matters arising under the National Internal Revenue Code or other laws administered by the Bureau of Internal Revenue, where the National Internal Revenue Code provides a specific period of action, in which case the inaction shall be deemed a denial; (3) Decisions, orders or resolutions of the Regional Trial Courts in local tax cases originally decided or resolved by them in the exercise of their original or appellate jurisdiction; xxx (b) Jurisdiction over cases involving criminal offenses as herein provided: (1) Exclusive original jurisdiction over all criminal offenses arising from violations of the National Internal Revenue Code or Tariff and Customs Code and other laws administered by the Bureau of Internal Revenue or the Bureau of Customs: Provided, however, That offenses or felonies mentioned in this paragraph where the principal amount of taxes and fees, exclusive of charges and penalties, claimed is less than One million pesos (P1,000,000.00) or where there is no specified amount claimed shall be tried by the regular courts and the jurisdiction of the CTA shall be appellate. Any provision of law or the Rules of Court to the contrary notwithstanding, the criminal action and the corresponding civil action for the recovery of civil liability for taxes and penalties shall at all times be simultaneously instituted with, and jointly determined in the same proceeding by the CTA, the filing of the criminal action being deemed to necessarily carry with it the filing of the civil action, and no right to reserve the filling of such civil action separately from the criminal action will be recognized. (2) Exclusive appellate jurisdiction in criminal offenses: (a) Over appeals from the judgments, resolutions or orders of the Regional Trial Courts in tax cases originally decided by them, in their respected territorial jurisdiction. (b) Over petitions for review of the judgments, resolutions or orders of the Regional Trial Courts in the exercise of their appellate jurisdiction over tax cases originally

decided by the Metropolitan Trial Courts, Municipal Trial Courts and Municipal Circuit Trial Courts in their respective jurisdiction. (c) Jurisdiction over tax collection cases as herein provided: (1) Exclusive original jurisdiction in tax collection cases involving final and executory assessments for taxes, fees, charges and penalties: Provided, however, That collection cases where the principal amount of taxes and fees, exclusive of charges and penalties, claimed is less than One million pesos (P1,000,000.00) shall be tried by the proper Municipal Trial Court, Metropolitan Trial Court and Regional Trial Court. (2) Exclusive appellate jurisdiction in tax collection cases: (a) Over appeals from the judgments, resolutions or orders of the Regional Trial Courts in tax collection cases originally decided by them, in their respective territorial jurisdiction. (b) Over petitions for review of the judgments, resolutions or orders of the Regional Trial Courts in the exercise of their appellate jurisdiction over tax collection cases originally decided by the Metropolitan Trial Courts, Municipal Trial Courts and Municipal Circuit Trial Courts, in their respective jurisdiction. These laws have expanded the jurisdiction of the CTA. However, they did not change the jurisdiction of the CTA to entertain an appeal only from a final decision or assessment of the Commissioner, or in cases where the Commissioner has not acted within the period prescribed by the NIRC. In the cases at bar, the Commissioner has not issued an assessment of the tax liability of private respondents. Finally, we hold that contrary to private respondents¶ stance, the doctrines laid down in CIR v. Union Shipping Co. and Yabes v. Flojo are not applicable to the cases at bar. In these earlier cases, the Commissioner already rendered an assessment of the tax liabilities of the delinquent taxpayers, for which reason the Court ruled that the filing of the civil suit for collection of the taxes due was a final denial of the taxpayers¶ request for reconsideration of the tax assessment. IN VIEW WHEREOF, premises considered, judgment is rendered: 1. In G.R. No. 120935, AFFIRMING the CA decision dated March 21, 1995, which set aside the Regional Trial Court¶s Order dated August 8, 1994, and REINSTATING Criminal Case Nos. 94-1842 to 94-1846 for further proceedings before the trial court; and 2. In G.R. No. 124557, REVERSING and SETTING ASIDE the Decision of the Court of Appeals dated March 29, 1996, and ORDERING the dismissal of C.T.A. Case No. 5075. No costs. SO ORDERED. REYNATO S. PUNO Chief Justice

WE CONCUR: ANTONIO T. CARPIO Associate Justice RENATO C. CORONA Associate Justice TERESITA J. LEONARDO-DE CASTRO Associate Justice

LUCAS P. BERSAMIN Associate Justice CERTIFICATION Pursuant to Section 13, Article VIII of the Constitution, I certify that the conclusions in the above decision had been reached in consultation before the case was assigned to the writer of the opinion of the Court¶s Division. REYNATO S. PUNO Chief Justice Republic of the Philippines SUPREME COURT Manila SECOND DIVISION

G.R. No. 74965 November 9, 1994 COMMISSIONER OF INTERNAL REVENUE, petitioner, vs. NATIONAL LABOR RELATIONS COMMISSION, DEPUTY CITY SHERIFF CARMELO V. CACHERO, MARITIME COMPANY OF THE PHILIPPINES, DOMINGO C. NIANGAR, DANIEL C. SABINO, FERNANDO S. TULIAO and TULMAR TRADING CORPORATION, respondents. Reynaldo L. Libanan for respondent deputy sheriff. Joaquin G. Chung, Jr. Law Office for respondent Tulmar Trading Corp. Eliodoro C. Cruz & Arsenio P. Dizon for Maritime Co. of the Philippines.

MENDOZA, J.: This is a petition for certiorari to set aside the resolution dated April 4, 1986 1 of the National Labor Relations Commission in NLRC Case No. NCR-12-4233-84 (Domingo C. Niangar v. Maritime Company of the Philippines), affirming the denial by the Labor Arbiter 2 of petitioner's motion to annul the sheriff's sale of four barges or, in the alternative, to order him to remit the proceeds of his sale to

the Bureau of the Internal Revenue for the satisfaction of the tax liabilities of private respondent Maritime Company of the Philippines. The facts are as follows: On January 12, 1984 the Commissioner of the Internal Revenue sent two letters 3 of demand to the respondent Maritime Company of the Philippines for deficiency common carrier's tax, fixed tax, 6% Commercial Broker's tax, documentary stamp tax, income tax and withholding taxes in the total amount of P17,284,882.45. The assessment became final and executory as private respondent did not contest it. But as private respondent did not pay its tax liability either, the Commissioner of Internal Revenue issued warrants of distraint of personal property and levy of real property of private respondent. Copies of the warrants, both dated January 23, 1985, were served on January 28, 1985 on Yoly T. Petrache, private respondent's accountant. 4 On April 16, 1985 a "Receipt for Goods, Articles, and Things Seized 5 under Authority of the National Internal Revenue Code" was executed, covering, among other things, six barges identified as MCP1,2,3,4,5 and 6. This receipt is required by § 303 (now § 206) of the NIRC as proof of the constructive distraint of property. It is an undertaking by the taxpayer or person in possession of the property covered that he will preserve the property and deliver it upon order of the court or the Internal Revenue Commissioner. The receipt was prepared by the BIR for the signature of a representative of respondent Maritime Company of the Philippines, but it was not in fact signed. Petitioner later explained that the individuals who had possession of the barges had refused to sign the receipt. This circumstance has given rise to the question in this case as it appears that four of the barges placed under constructive distraint were levied upon execution by respondent deputy sheriff of Manila on July 20, 1985 to satisfy a judgment for unpaid wages and other benefits of employees of respondent Maritime Company of the Philippines. More specifically, the question in this case is the validity of the warrant of distraint served by the Revenue Seizure Officer against the writ of execution subsequently levied upon the same property by the deputy sheriff of Manila to satisfy the claims of employees in NLRC Case No. NCR-12-4233-84 (Domingo C. Niangar, et al. v. Maritime Company of the Philippines) for P490,749.21. The four barges were sold by respondent deputy sheriff at a public auction on August 12, 1985. The highest bidder, Daniel C. Sabino, subsequently sold them to private respondents Fernando S. Tuliao and Tulmar Trading Corporation. On September 4, 1985, petitioner asked the Labor Arbiter to annul the sale and to enjoin the sheriff from disposing of the proceeds of the sale or, in the alternative, to remit them to the Bureau of Internal Revenue so that the amount could be applied to the payment of private respondent Maritime Company's tax liabilities. In an order dated September 30, 1985, Labor Arbiter Ceferina Diosana denied the motion on the ground that petitioner Commissioner of Internal Revenue failed to show that the barges which were levied upon in execution and sold at public auction had been validly placed under constructive distraint. 6 The Labor Arbiter likewise rejected petitioner's contention that the government's claim for taxes was preferred under Art. 2247, in relation to Art. 2241(1) of the Civil Code, on the ground that under this provisions only taxes and fees which are due on specific movables enjoy preference, whereas the taxes claimed by petitioner were not due on the four barges in question.

The order was appealed to the NLRC, which in resolution dated April 4, 1986, affirmed the denial of the Internal Revenue Commissioner's motion. Hence this petition for certiorari. For reasons to be presently stated, the petition is granted. The National Internal Revenue Code provides for the collection of delinquent taxes by any of the following remedies: (a) distraint of personal property or levy of real property of the delinquent taxpayer and (b) civil or criminal action. With respect to the four barges in question, petitioner resorted to constructive distraint pursuant to § 303 (now § 206) of the NLRC. This provisions states: Constructive distraint of the property of a taxpayer. ² To safeguard the interest of the Government, the Commissioner of Internal Revenue may place under constructive distraint the property of a delinquent taxpayer or any taxpayer who, in his opinion, is retiring from any business subject to tax, or intends to leave the Philippines, or remove his property therefrom, or hide or conceal his property, or perform any act tending to obstruct the proceedings, for collecting the tax due or which may be due from him. The constructive distraint of personal property shall be effected by requiring the taxpayer or any person having possession or control of such property to sign a receipt covering the property distrained and obligate himself to preserve the same intact and unaltered and not to dispose of the same in any manner whatever without the express authority of the Commissioner of Internal Revenue. In case the taxpayer or the person having the possession and control of the property sought to be placed under constructive distraint refuses or fails to sign the receipt herein referred to, the revenue officer effecting the constructive distraint shall proceed to prepare a list of such property and in the presence of two witnesses leave a copy thereof in the premises where the property distrained is located, after which the said property shall be deemed to have been placed under constructive distraint.. Although the warrant of distraint in this case had been issued earlier (January 23,1985) than the levy on execution in the labor case on July 20, 1985, the Labor Arbiter nevertheless held that there was no valid distraint of personal property on the ground that the receipt of property distrained had not been signed by the taxpayer as required above. In her order, which the NLRC affirmed in toto, the Labor Arbiter said: It is claimed by the Commissioner of the Internal Revenue that on January 23, 1984, he issued a warrant of distraint of personal property on respondent to satisfy the collection of the deficiency taxes in the aggregate sum of P17,284,882.45 and a copy of said warrant was served upon Maritime Company on January 28, 1985 and pursuant to the warrant, the Commissioner, through Revenue Seizure Agent Roland L. Bombay, issued on April 16, 1985, to Maritime Company a receipt for goods, articles and things seized pursuant to authority granted to him under the National Internal Revenue Code. Such personal properties seized includes, among others, "Six (6) units of barges MCI-6 . . . " However, his own receipts for goods attached to his motions does not show that it was received by Maritime; neither does it show any signature of any of Maritime's Officers.

Apart from the foregoing, in his affidavit of 11 September 1985, Sheriff Cachero stated that before he sold the subject four barges at public auction, he conducted an investigation on the ownership of the said four barges. In brief, he found out that the said four barges were purchased by respondent through Makati Leasing and that the whole purchase price has been paid by respondent. In fact, the corresponding deed of sale has already been signed. He did not find any lien or encumbrance on any of the said four barges. Thus it cannot be true that the Commissioner effected a valid warrant of distraint of personal property on the four barges in question. 7

However, this case arose out of the same facts involved in Republic v. Enriquez, 8 in which we sustained the validity of the distraint of the six barges, which included the four involved in this case, against the levy on execution made by another deputy sheriff of Manila in another case filed against Maritime Company. Two barges (MCP-1 and MCP-4) were the subject of a levy in the case. There we found that the "Receipt for Goods, Articles and Things Seized under Authority of the National Internal Revenue Code" covering the six barges had been duly executed, with the Headquarters, First Coast Guard District, Farola Compound Binondo, Manila acknowledging receipt of several barges, vehicles and two (2) bodegas of spare parts belonging to Maritime Company of the Philippines. Apparently, what had been attached to the petitioner's motion filed by the government with the Labor Arbiter in this case was a copy, not the original one showing the rubber stamp of the Coast Guard and duly signed by its representative. A xerox copy of this signed receipt was submitted in the prior case. 9 This could be due to the fact that, except for Solicitor Erlinda B. Masakayan, the government lawyers who prepared the petition in the prior case were different from those who filed the present petition. They admitted that the receipt of property distrained had not been signed by the taxpayer or person in possession of the taxpayer's property allegedly because they had refused to do so. What apparently they did not know is that the receipt had been acknowledged by the Coast Guard which obviously had the barges in its possession. In addition to the receipt duly acknowledged by the Coast Guard, the record of the prior case also shows that on October 4, 1985, the Commissioner of the Internal Revenue issued a "Notice of Seizure of Personal Property" stating that the goods and chattels listed on its reverse side, among which were the four barges (MCP-2, MCP-3, MCP-5, and MCP-6), had been distrained by the Commissioner of Internal Revenue. 10 The "Notice of Seizure of Personal Property," a copy of which was received by Atty. Redentor R. Melo in behalf of Maritime Company of the Philippines, together with the receipt of the Coast Guard, belies the claim of respondent deputy sheriff that when he levied upon the four barges there was no indication that the barges had previously been placed under distraint by the Commissioner of Internal Revenue. Accordingly, what we said in the prior case 11 in upholding the validity of distraint of two of the six barges (MCP Nos. 1 and 4), fully applies in this case: It is settled that the claim of the government predicated on a tax lien is superior to the claim of a private litigant predicated on a judgment. The tax lien attaches not only from the service of the warrant of distraint of personal property but from the time the tax became due and payable. Besides, the distraint on the subject properties of the Maritime Company of the Philippines as well as the notice of their seizure were made by petitioner, through the Commissioner of the Internal Revenue, long before the writ of the execution was issued by the Regional Trial Court of Manila, Branch 31. There is no question then that at the time the writ of execution was issued, the two (2) barges, MPC-1 and MCP-4, were no longer properties of the Maritime Company of

the Philippines. The power of the court in execution of judgments extends only to properties unquestionably belonging to the judgment debtor. Execution sales affect the rights of the judgment debtor only, and the purchaser in an auction sale acquires only such right as the judgment debtor had at the time of sale. It is also well-settled that the sheriff is not authorized to attach or levy on property not belonging to the judgment debtor. Nor is there any merit in the contention of the NLRC that taxes are absolutely preferred claims only with respect to movable or immovable properties on which they are due and that since the taxes sought to be collected in this case are not due on the barges in question the government's claim cannot prevail over the claims of employees of the Maritime Company of the Philippines which, pursuant to Art. 110 of the Labor Code, "enjoy first preference." In Republic v. Peralta 12 this Court rejected a similar contention. Through Mr. Justice Feliciano we held: . . . [T]he claim of the Bureau of Internal Revenue for unpaid tobacco inspection fees constitutes a claim for unpaid internal revenue taxes which gives rise to a tax lien upon all the properties and assets, movable or immovable, of the insolvent as taxpayer. Clearly, under Articles 2241 No. 1, 2242 No. 1, and 2246-2249 of the Civil Code, this tax claim must be given preference over any other claim of any other creditor, in respect of any and all properties of the insolvent. xxx xxx xxx Article 110 of the Labor Code does not purport to create a lien in favor of workers or employees for unpaid wages either upon all of the properties or upon any particular property owned by their employer. Claims for unpaid wages do not therefore fall at all within the category of specially preferred claims established under Articles 2241 and 2242 of the Civil Code, except to the extent that such claims for unpaid wages are already covered by Article 2241, number 6: "claims for laborer's wages, on the goods manufactured or the work done," or by Article 2242, number 3: "claims of laborers and other workers engaged in the construction, reconstruction or repair of buildings, canals and other works, upon said buildings, canals or other works." To the extent that claims for unpaid wages fall outside the scope of Article 2241, number 6 and 2242, number 3, they would come with the ambit of the category of ordinary preferred credits under Article 2244. Applying Article 2241, number 6 to the instant case, the claims of the Unions for separation pay of their members constitute liens attaching to the processed leaf tobacco, cigars and cigarettes and other products produced or manufactured by the Insolvent, but not to other assets owned by the Insolvent. And even in respect of such tobacco and tobacco products produced by the Insolvent, the claims of the Unions may be given effect only after the Bureau of Internal Revenue's claim for unpaid tobacco inspection fees shall have been satisfied out of the products so manufactured by the Insolvent. Article 2242, number 3, also creates a lien or encumbrance upon a building or other real property of the Insolvent in favor of workmen who constructed or repaired such building or other real property. Article 2242, number 3, does not however appear relevant in the instant case, since the members of the Unions to whom separation pay is due rendered services to the Insolvent not (so far as the record of this case

would show) in the construction or repair of buildings or other real property, but rather, in the regular course of the manufacturing operations of the Insolvent. The Unions' claims do not therefore constitute a lien or encumbrance upon any immovable property owned by the insolvent, but rather, as already indicated, upon the Insolvent's existing inventory (if any) of processed tobacco and tobacco products. In addition, we have held 13 that Art. 110 of the Labor Code applies only in case of bankruptcy or judicial liquidation of the employer. This is clear from the text of the law. Art. 110. Worker preference in case of bankruptcy. ² In the event of bankruptcy or liquidation of an employer's business, his workers shall enjoy first preference as regards wages due them for services rendered during the period prior to the bankruptcy or liquidation, any provision of law to the contrary notwithstanding. Unpaid wages shall be paid in full before other creditors may establish any claims to a share in the assets of the employer. This case does not involve the liquidation of the employer's business. WHEREFORE, the petition for certiorari is GRANTED and the resolution dated April 4, 1986 of respondent NLRC in NLRC Case No. NCR-12-4233-84 is SET ASIDE insofar as it denies the government's claim for taxes, and respondent deputy sheriff Carmelo V. Cachero or his successor is ORDERED to remit the proceeds of the auction sale to the Bureau of Internal Revenue to be applied as part payment of respondent Maritime Company's tax liabilities. SO ORDERED. Narvasa, C.J., Regalado and Puno, JJ., concur.

Republic of the Philippines SUPREME COURT Manila EN BANC G.R. No. L-16014 October 4, 1921

BANK OF THE PHILIPPINE ISLANDS, plaintiff-appellant, vs. WENCESLAO TRINIDAD, Collector of Internal Revenue, defendant-appellee. Yeager and Armstrong for appellant. No appearance for appellee.

JOHNSON, J.: There is a practically no dispute about the facts in this case. They are as follows:

On the 13th day of July, 1916, the defendant Collector of Internal Revenue, through his duly authorized agent at Zamboanga, seized and distrained certain personal property, consisting of machinery for sawing lumber which is particularly enumerated and described in paragraph 3 of the complaint, and advertised the same for sale, to realize the sum of P2,159.79, alleged to be due to the Government of the Philippine Islands from Pujalte and Co., as forestry charges. The defendant claimed that said personality belonged to the said company, was used in the business on which the taxes were due, and was liable to seizure to cover said taxes. On the other hand, the plaintiff claimed to be the owner of said property, and demanded its release. The demand being denied, the plaintiff paid to the defendant the said sum of P2,159.79 under protest to prevent the sale of said property, and immediately brought the present action in the Court of First Instance of Zamboanga to recover the said sum of P2,159.78 together with interest and costs. The lower court, after due trial, dismissed the plaintiff's complaint and absolved the defendant from all liability thereunder. From that judgment the plaintiff appealed to this court. The property in question formerly belonged to the Taba Saw Mill Co., a copartnership formed by Pujalte and Co. and one Ramon Murga. In April, 1914, Ramon Murga sold all his rights, title, and interest in and to the said copartnership to Pujalte and Co., which thereby became the sole owner of the concern. It appears from plaintiff's Exhibit AA, which was admitted in evidence without objection on the part of the defendant, that on the 26th day of September, 1912, the said Taba Saw Mill Co. conveyed to the plaintiff bank, by way of chattel mortgage, the property here in question together with other personalities, as security for the payment to said bank of two certain promissory notes for the sum of P180,000. Said chattel mortgage was duly registered in the office of the register of deeds of Zamboanga on the 26th day of December, 1912. On that date the property in question was free from all tax liens; at least, the plaintiff mortgagee had no notice thereof. On the 13th day of July, 1916, when the amount here in question was found to be due to the Government from Pujalte and Co. as forestry charges, and when the property in question was seized by the defendant, the said chattel mortgage was still subsisting. It is admitted that at the time of its seizure the said property was being used in the sawmill of Pujalte and Co. Upon the foregoing facts the lower court absolved the defendant from all liability under the plaintiff's complaint, for the following reasons: 1. That the party who was liable to pay the taxes for which the property in question was distrained was not the plaintiff but Pujalte and Co.; and that the plaintiff having "voluntarily and spontaneously" paid the debt of the latter, had no cause of action against the defendant collector, and could only recover the sum so paid by it from Pujalte and Co., under article 1158 of the Civil Code (p. 15, B. of E.); that the plaintiff should have proceeded under section 141 of Act No. 2339 (now sec. 1580 of Act No. 2711), and not under section 140 of the said Act (sec. 1579 of Act No. 2711). 2. That "even supposing for a moment" that the plaintiff had a right of action against the defendant to recover the sum paid by it to the latter, yet this action must fail because the property in question, having been used by Pujalte and Co. in its business of cutting and sawing lumber, was liable to seizure and distraint under section 149 of Act No. 2339. We are of the opinion that neither of the foregoing reasons is sound, and that the judgment of the lower court should be revoked. First. There is absolutely no basis for the finding of the trial court that "the plaintiff bank had voluntarily and spontaneously paid the debt of a third party, that is, that of the firm of Pujalte and

Co." (p. 15, B. of E.). Paragraph 7 of the plaintiff's complaint alleges: "That thereupon, involuntarily and under due protest in writing, the plaintiff bank made payment of the required sum of P2,159.79 in order to secure the release of its seized property." These allegations were specially admitted by the defendant (par. 5, stipulation, Plaintiff's Exhibit G). Section 140 of the Internal Revenue Law (Act No. 2339 provides as follows: SEC. 140. Recovery of tax paid under protest. ² When the validity of any tax in questioned, or amount disputed, or other question raised as to liability therefor, the person against whom or against whose property the same is sought to be enforced shall pay the tax under instant protest, or upon protest within ten days, and shall thereupon request the decision of the Collector of Internal Revenue. If the decision of the Collector of Internal Revenue is adverse, or if no decision is made by him within six months from the date when his decision was requested, the taxpayer may proceed, at any time within two years after the payment of the tax, to bring an action against the Collector of Internal Revenue for the recovery of the sum alleged to have been illegally collected, the process to be served upon him, upon the provincial treasurer, or upon the officer collecting the tax. Section 141 of the same Act provides: SEC. 141. Action to contest forfeiture of chatted. ² In case of the seizure of personal property under claim of forfeiture the owner, desiring to contest the validity of the forfeiture, may at any time before sale or destruction of the property bring an action against the person seizing the property or having possession thereof to recover the same, and upon giving proper bond may enjoin the sale; or after the sale and within six months he may bring an action to recover the net proceeds realized at the sale. The lower court was of the opinion that the plaintiff should have proceeded under the latter section above quoted and not under the former. It cannot be maintained that the personal property here in question was seized by the defendant "under claim of forfeiture;" nor could it have been legally seized under claim of forfeiture. It was seized to enforce an alleged tax lien, under section 149 of Act No. 2339 (sec. 1588, Act No. 2711), which was quoted by the lower court in its decision (p. 19 B. of E.) and which in no way provides for the forfeiture of the property on which such a lien attaches. Forfeiture is "the divestiture of property without compensation, in consequence of an offense. The effect of such forfeiture is to transfer the title to the specific thing from the owner to the sovereign power." (12 R. C. L., 124.) There is a great difference between a seizure under forfeiture and a seizure to enforce a tax lien. In the former all the proceeds derived from the sale of the thing forfeited are turned over to the Collector of Internal Revenue (sec. 148, Act No. 2339) in the latter the residue of such proceeds over and above what is required to pay the tax sought to be realized, including expenses, is returned to the owner of the property (second paragraph, sec. 152, Act No. 2339). Clearly, the remedy applicable to the present case is that provided for in section 140, above quoted, and which the plaintiff invoked. (See Hongkong and Shanghai Banking Corporation vs. Rafferty, 39 Phil., 145, 147.) Second. At the time of the seizure of the property here in question, the plaintiff held a valid and subsisting chattel mortgage on the same, duly registered in the registry of deeds. "A chattel mortgage is a conditional sale of personal property as security for the payment of a debt, or the performance of some other obligation specified therein, the condition being that the sale shall be void upon the seller paying the purchaser a sum of money or doing some other act named." (Sec. 3, Act No. 1508.) "Therefore, so long as the mortgage exists, the dominionwith respect to the mortgaged personal property rests with the creditor-pledgee from the time of the inscription of the mortgage in the registry, and the furniture ceases to be the property of the debtor for the reason

that it has become the property of the creditor, in like manner as the domination of a thing sold is transferred to the purchaser and ceases to belong to the vendor from the moment of the delivery thereof, as a result of the sale." (Meyers vs. Thein, 15 Phil., 303, 303-309; see also Bachrach vs. Mantel, 25 Phil., 410; In re Du Tec Chuan, 34 Phil., 488, 490.)
1aw ph!l. net

The chattel mortgage in question was registered in the registry of deeds on the 26th day of December, 1912. The forest charges sought to be collected by the defendant were found to be due from Pujalte and Co. on the 13th day of July, 1916, and on that date the property covered by said chattel mortgage was seized by the defendant to enforce the payment of said forest charges. It is clear from these facts and from the legal provisions and jurisprudence above quoted that the plaintiff-mortgagee, and not Pujalte and Co., the mortgagor, was, and had been for more than three years, the legal owner of the property in question at the time the same was seized by the defendant. And even granting, without deciding, that the forest charges are a tax on business or occupationwithin the meaning of section 149 of Act No. 2339 (sec. 1588, Act No. 2711), yet we are of the opinion and so decide that the mere fact that said property was used in the business of Pujalte and Co. could not and did not make such property liable for the payment of taxes due from said company, said property belonging as it did to an innocent third party. "The property used in the business or occupation," referred to in said section 149, can only mean property belonging to the owner of the business or occupation. Any other construction would be unwarranted and unjust. For the foregoing reasons the judgment appealed from is hereby revoked, and it is hereby ordered and decreed that a judgment be entered in favor of the plaintiff and against the defendant, ordering the latter to refund to the former the sum of P2,159.79, with interest thereon at the legal rate from the 13th day of July, 1916, until paid, and without any finding as to costs. So ordered. Araullo, Avanceña and Villamor, JJ., concur.

Republic of the Philippines SUPREME COURT Manila EN BANC G.R. No. L-44372 November 3, 1938

BENITO GARCIA, plaintiff-appellee, vs. THE COLLECTOR OF INTERNAL REVENUE, defendant-appellant. Solicitor-General Hilado for appellant. Apolonio Suntay for appellee.

CONCEPCION, J.:

The Collector of Internal Revenue, defendant herein, required Benito Garcia to pay a specific tax of P204.08 after the latter had been sentenced in a criminal case to pay a fine for having taken six hundred and sixteen liters of alcohol from the distillery of Jose B. Suntay for the purpose of removing the same to a distant store without having previously paid the corresponding specific tax therefor. Appelle paid the tax under protest, filing afterwards a complaint to recover its amount. The court decided the case in favor of plaintiff, and the Collector of Internal Revenue appealed from the decision to this court. Appellant, in his brief, assigned the following as errors committed by the lower court: The lower court erred in holding that the Government had made a claim against Benito Garcia for the amount of P204.08 as specific tax, in criminal case No. 5922 of the Court of First Instance of Bulacan, and that the court, in its decision, declined to award it to the Government. The lower court erred in holding that the manufacturer of alcohol ordinarily pays the tax and that, as the manufacturer of the alcohol in question was Jose B. Suntay, and Benito Garcia was a mere employee, the latter cannot be made to pay the tax in question.
lawp hi1.n et

The lower court erred in ordering the defendant to pay the plaintiff the amount of P204.08, plus costs. The lower court erred in denying the motion for new trial filed by the defendant. In the decision appealed from the court has proceeded upon the assumption that in the criminal case filed against plaintiff herein, the Government had sought payment from him of the amount of P204.08 as specific tax; but that the court in its decision refused to impose the same for the alleged reason that, as the alcohol in question had been confiscated an as the value of the same was probably greater than the amount of the tax, the Government already has had an opportunity to recover it. In truth, however, the payment of the tax was not sought in the criminal case above referred to because the object of the information was the imposition upon the offender of the corresponding penalty for violation of section 2727 of the Revised Administrative Code. The tax should have been recovered by the Collector of Internal Revenue independently of the criminal action instituted by the People of the Philippines against the accused Benito Garcia. Therefore, the fact that in the judgment rendered in said case no pronouncement whatsoever as regard said tax had been made, was no bar to the Government's recovering it afterwards, a s the Collector of Internal Revenue, appellant herein, has done in his own name. Furthermore, the confiscation in the criminal case was an accessory penalty imposed by article 25 of the Revised Penal Code, which is entirely different from the payment of the tax. Another ground of the appealed decision, according t the reasoning of the court, is that the payment of the tax is in reality made by the consumer, although the distiller has to pay it first, charging the same later in the price of the sale. In the present case, says the court, the plaintiff Garcia never had the opportunity to sell the alcohol and consequently would never be reimbursed for the amount of the corresponding specific tax. All this loses its apparent merit by the single consideration that one who violates the law must suffer all the consequences the law is confiscation. According to section 1479 of the Revised Administrative Code, the tax should be paid immediately before the removal of the article from the place of production. The law does not say that the tax may be paid immediately before the sale.

The second error committed by the court consists in holding that the distiller of alcohol ordinarily is the one who pays the tax and inasmuch as Jose B. Suntay was the distiller of the alcohol in question, while Benito Garcia was mere employee, the latter could not be compelled to pay tax referred to. This is an inaccurate interpretation of the law. Section 1479 aforecited of the Revised Administrative Code provides that the specific taxes on domestic products shall be paid by the manufacturer, producer, owner or person having possession of the same. It is a fact that the six hundred and sixteen liters of alcohol were found in the possession of plaintiff when he transferred them from the factory to a distant store and there is neither allegation nor evidence that plaintiff had taken the alcohol from the distillery to remove the same to the store by order of his principal, Jose B. Suntay. In order to avoid dispute and to determine easily the person who should pay the specific tax, section 1479 of the Revised Administrative Code has farsightedly provided that the manufacturer, producer, owner or person having possession of the article shall pay the tax. The judgment appealed from is reversed without a special pronouncement as the costs. So ordered. Avanceña, C.J., Villa-Real, Abad Santos, Imperial, Diaz and Laurel, JJ., concur. Republic of the Philippines SUPREME COURT Manila EN BANC G.R. No. L-18804 May 27, 1965

COMMISSIONER OF INTERNAL REVENUE, petitioner, vs. WESTERN PACIFIC CORPORATION, respondent. Office of the Solicitor General for petitioner. R. Melo and A. S. Velasquez for respondent. PAREDES, J.: On March 2, 1959, the respondent Western Pacific Corporation, was assessed for P3,731.00, as deficiency income tax for the year 1953. This assessment was brought about by the disallowance of P8,265.82, listed in respondent's return for 1953, as expense items, and P10,387.50, as written off "bad debts." The assessment was received by respondent on the same date (March 2, 1959). On March 5, 1959, the Commissioner of Internal Revenue wrote the respondent corporation a letter of demand for the payment of the amount, including therein a breakdown of said assessment. Under date of June 29, 1959, respondent corporation, thru Ruifino Melo & Company, Consulting and Examining Auditors, requested for non-assessment, claiming that there has been prescription in making the assessment, that the expense items and bad debts were allowable deduction. The letter was accompanied by a Resolution of the corporation, dated February 2, 1954, where it was resolved to write off the debts of the people appearing in another annex. The Commissioner on July 30, 1959 replied to the request, denying the same, and demanding the payment of the amount due within thirty (30) days from receipt of said demand. On September 19, 1958, respondent corporation requested that it be permitted until September 25, 1959, to submit formal objections to the assessment. The formal objections appearing in the letter of September 22, 1959, were identical to those of the June 29, 1959 communication, reason for which the Commissioner did not give any

favorable action. The last letter of the Commissioner, dated October 28, 1959, among others, requested payment of the assessment within ten (10) days from receipt thereof. On December 18, 1959, respondent Western Pacific Corporation, presented with the Court of Tax Appeals a petition for Review of assessment made by the Commissioner, on three (3) counts, to wit: (1) whether or not the making of the assessment had prescribed; (2) whether expenses incurred in securing IGC Licenses are capital expenditures, and, as such, not deductible from the income; and (3) whether the bad debts written off should likewise be deducted. When the issues were joined, by the filing of the Answer, and after hearing, the CTA rendered judgment absolving the Western Pacific Corporation from the assessment. It, however, ruled out prescription, stating that March 2, 1959, was the last day of the five (5) year period within which to make the assessment. On this point, the CTA ruled: However, we do not agree with petitioner that the assessment in question was issued beyond the 5-year statutory limitation. February 28, 1959 fell on a Saturday. Pursuant to Republic Act No. 1880, as, implemented by Executive Order No. 25, effective July 1, 1959, all bureaus and offices of the government, except schools, court, hospitals and health clinics, hold office only five days a week or from Monday to Friday. Saturday and Sunday, are constituted public holidays or days of exemption from labor or work as far as government offices, including that of respondent Commissioner, are concerned. The offices and bureaus concerned are officially closed on those days. So that on February 28, 1959 and March 1, 1959, which were Saturday and Sunday, respectively, the office of respondent was officially closed. And where the last day for doing an act required by law falls on a holiday, the act may be done on the next succeeding business day. (Section 31, Revised Administrative Code.) Similarly, in computing any period of time prescribed by statute, the day of the act after which the designated period of time begins to run is not included. But the last day of the period so computed is to be included, unless it is a Sunday or a legal holiday, in which event the time shall run until the end of the next day which is neither a Sunday or a holiday (Section 1, Rule 28, Rules of Court). Consequently, since February 28, 1959 was a Saturday and the next day, March 1, 1959, a Sunday, respondent had until the next succeeding business day, March 2, 1959, Monday, within which to issue the deficiency assessment. The assessment in question having been issued on March 2, 1959, it was, therefore, seasonably made. We concur in the above findings and conclusions, convinced as We are, that they are actually and legally correct.. The above ruling notwithstanding, the Commissioner of Internal Revenue appealed against the judgment which absolved respondent Western Pacific Corporation from liability, alleging that the CTA erred:. (1) In taking cognizance of the case, notwithstanding lack of jurisdiction; and (2) Granting it had jurisdiction, in considering the expense items and the written off bad debts as deductible.
1äwphï 1.ñë t

Without going into the merits of the decision absolving the respondent corporation of tax liability, We find that the assessment made by the Commissioner should be maintained, for the simple reason that when the petition for review was brought to the CTA by the respondent corporation, the said Court no longer had jurisdiction to entertain the same. The assessment had long become final. A petition for review should be presented, within the reglementary period, as provided for in Section 11, Republic Act No. 1125, which is "thirty (30) days from receipt of the assessment." The thirty (30) day period is jurisdictional (Pangasinan Transportation Co. vs. Blaquera, L-13101, April 29, 1960). It will be noted that the assessment was received by the respondent corporation on March 2, 1959. It was only on June 29, 1959, when said corporation formally assailed the assessment, on the grounds of prescription in making the assessment and the impropriety of the disallowance of the listed deductions. From March 3 to June 29, 1959, manifestly more than thirty (30) days had lapsed and the assessment became final, executory and demandable (Ventanilla vs. Bd. of Tax Appeals, et al., L-7384, Dec. 19, 1955). Of course, in the interim, a number of communications were exchanged between the parties, the latest of which was dated October 28, 1959. Even if this date is considered as the commencement of the thirty (30) day period, still the petition for review with the CTA was out of time, because it was only on December 18, 1959, that said petition was presented. Failure to comply with the thirty-day statutory period would bar appeal and deprive the CTA of its jurisdiction to entertain and determine the correctness of the assessment (Gibbs & Gibbs vs. Coll. of Int. Rev. & CTA, L-13453, Feb. 29, 1960). IN VIEW OF THE FOREGOING, the decision of the CTA is hereby set aside for having been rendered without jurisdiction, the assessment in question having been already final, executory and demandable before the petition for review was presented; and another entered, ordering respondent Western Pacific Corporation to pay the assessment made by the Collector of Internal Revenue, and the further amount of 5% surcharge and 1% monthly interest on the amount assessed, from April 1, 1959 until date of full payment. Costs against the respondent corporation. Bengzon, C.J., Concepcion, Reyes, J.B.L., Barrera, Dizon, Regala, Makalintal, Bengzon, J.P., and Zaldivar, JJ., concur. Republic of the Philippines SUPREME COURT Manila EN BANC G.R. No. L-19727 May 20, 1965

THE COMMISSIONER OF INTERNAL REVENUE, petitioner, vs. PHOENIX ASSURANCE CO., LTD., respondent. ----------------------------G.R. No. L-19903 May 20, 1965

PHOENIX ASSURANCE, CO., LTD., petitioner, vs. COMMISSIONER OF INTERNAL REVENUE, respondent.

Office of the Solicitor General for petitioner-respondent Commissioner of Internal Revenue. Sycip, Salazar, Luna & Associates and A. S. Monzon, B. V. Abela & J. M. Castillo for respondentpetitioner Phoenix Assurance Co., Ltd. BENGZON, J.P., J.: From a judgment of the Court of Tax Appeals in C.T.A. Cases Nos. 305 and 543, consolidated and jointly heard therein, these two appeals were taken. Since they involve the same facts and interrelated issues, the appeals are herein decided together. Phoenix Assurance Co., Ltd., a foreign insurance corporation organized under the laws of Great Britain, is licensed to do business in the Philippines with head office in London. Through its head office, it entered in London into worldwide reinsurance treaties with various foreign insurance companies. It agree to cede a portion of premiums received on original insurances underwritten by its head office, subsidiaries, and branch offices throughout the world, in consideration for assumption by the foreign insurance companies of an equivalent portion of the liability from such original insurances.
1äw phï1 .ñët

Pursuant to such reinsurance treaties, Phoenix Assurance Co., Ltd., ceded portions of the premiums it earned from its underwriting business in the Philippines, as follows: Year Amount Ceded 1952 1953 1954 P316,526.75 P246,082.04 P203,384.69

upon which the Commissioner of Internal Revenue, by letter of May 6, 1958, assessed the following withholding tax: Year 1952 1953 1954 Total Withholding Tax P 75,966.42 59,059.68 48,812.32 P183,838.42 =============

On April 1, 1951, Phoenix Assurance Co., Ltd. filed its Philippine income tax return for 1950, claiming therein, among others, a deduction of P37,147.04 as net addition to marine insurance reserve equivalent to 40% of the gross marine insurance premiums received during the year. The Commissioner of Internal Revenue disallowed P11,772.57 of such claim for deduction and subsequently assessed against Phoenix Assurance Co., Ltd. the sum of P1,884.00 as deficiency income tax. The disallowance resulted from the fixing by the Commissioner of the net addition to the marine insurance reserve at 100% of the marine insurance premiums received during the last three months of the year. The Commissioner assumed that "ninety and third, days are approximately the

length of time required before shipments reach their destination or before claims are received by the insurance companies." On April 1, 1953, Phoenix Assurance Co., Ltd. filed its Philippine income tax return for 1952, declaring therein a deduction from gross income of P35,912.25 as part of the head office expenses incurred for its Philippine business, computed at 5% on its gross Philippine income. On August 30, 1955 it amended its income tax return for 1952 by excluding from its gross income the amount of P316,526.75 representing reinsurance premiums ceded to foreign reinsurers and further eliminating deductions corresponding to the coded premiums. The amended return showed an income tax due in the amount of P2,502.00. The Commissioner of Internal Revenue disallowed P15,826.35 of the claimed deduction for head office expenses and assessed a deficiency tax of P5,667.00 on July 24, 1958. On April 30, 1954, Phoenix Assurance Co., Ltd. filed its Philippine income tax return for 1953 and claimed therein a deduction from gross income of P33,070.88 as head office expenses allocable to its Philippine business, equivalent to 5%, of its gross Philippine income. On August 30, 1955 it amended its 1953 income tax return to exclude from its gross income the amount of P246,082.04 representing reinsurance premiums ceded to foreign reinsurers. At the same time, it requested the refund of P23,409.00 as overpaid income tax for 1953. To avoid the prescriptive period provided for in Section 306 of the Tax Code, it filed a petition for review on April 11, 1956 in the Court of Tax Appeals praying for such refund. After verification of the amended income tax return the Commissioner of Internal Revenue disallowed P12,304.10 of the deduction representing head office expenses allocable to Philippine business thereby reducing the refundable amount to P20,180.00. On April 29, 1955, Phoenix Assurance Co., Ltd. filed its Philippine income tax return for 1954 claiming therein, among others, a deduction from gross income of P99,624.75 as head office expenses allocable to its Philippine business, computed at 5% of its gross Philippine income. It also excluded from its gross income the amount of P203,384.69 representing reinsurance premiums ceded to foreign reinsurers not doing business in the Philippines. On August 1, 1958 the Bureau of Internal Revenue released the following assessment for deficiency income tax for the years 1952 and 1954 against Phoenix Assurance Co., Ltd.: 1952 Net income per audited return Unallowable deduction & additional income: Overclaimed Head Office expenses: Amount claimed . . . . . . . . . . . . P 35,912.25 Amount allowed . . . . . . . . . . . . Net income per investigation Tax due thereon 20,085.90 P 15,826.35 P 28,337.96 P 5,667.00 =========== P 12,511.61

1954 Net income per audited Unallowable deduction & additional income: Overclaimed Head Office expenses: Amount claimed . . . . . . . . . . . . Amount allowed . . . . . . . . . . . . Net income per investigation Tax due thereon Less: amount already assessed DEFICIENCY TAX DUE P29,624.73 19,455.50 10,16.23 P170,489.41 P 39,737.00 36,890.00 P 2,847.00 =========== P160,320.21

The above assessment resulted from the disallowance of a portion of the deduction claimed by Phoenix Assurance Co., Ltd. as head office expenses allocable to its business in the Philippines fixed by the Commissioner at 5% of the net Philippine income instead of 5% of the gross Philippine income as claimed in the returns. Phoenix Assurance Co., Ltd. protested against the aforesaid assessments for withholding tax and deficiency income tax. However, the Commissioner of Internal Revenue denied such protest. Subsequently, Phoenix Assurance Co., Ltd. appealed to the Court of Tax Appeals. In a decision dated February 14, 1962, the Court of Tax Appeals allowed in full the decision claimed by Phoenix Assurance Co., Ltd. for 1950 as net addition to marine insurance reserve; determined the allowable head office expenses allocable to Philippine business to be 5% of the net income in the Philippines; declared the right of the Commissioner of Internal Revenue to assess deficiency income tax for 1952 to have prescribed; absolved Phoenix Assurance Co., Ltd. from payment of the statutory penalties for non-filing of withholding tax return; and, rendered the following judgment: WHEREFORE, petitioner Phoenix Assurance Company, Ltd. is hereby ordered to pay the Commissioner of Internal Revenue the respective amounts of P75,966.42, P59,059.68 and P48,812.32, as withholding tax for the years 1952, 1953 and 1954, and P2,847.00 as income tax for 1954, or the total sum of P186,685.42 within thirty (30) days from the date this decision becomes final. Upon the other hand, the respondent Commissioner is ordered to refund to petitioner the sum of P20,180.00 as overpaid income tax for 1953, which sum is to be deducted from the total sum of P186,685.42 due as taxes. If any amount of the tax is not paid within the time prescribed above, there shall be collected a surcharge of 5% of the tax unpaid, plus interest at the rate of 1% a month from the date of delinquency to the date of payment, provided that the maximum amount that may be collected as interest shall not exceed the amount corresponding to a period of three (3) years. Without pronouncement as to costs.

Phoenix Assurance Co., Ltd. and the Commissioner of Internal Revenue have appealed to this Court raising the following issues: (1) Whether or not reinsurance premiums ceded to foreign reinsurers not doing business in the Philippines pursuant to reinsurance contracts executed abroad are subject to withholding tax; (2) Whether or not the right of the Commissioner of Internal Revenue to assess deficiency income tax for the year 1952 against Phoenix Assurance Co., Ltd., has prescribed; (3) Whether or not the deduction of claimed by the Phoenix Assurance Co., Ltd.as net addition to reserve for the year 1950 is excessive; (4) Whether or not the deductions claimed by Phoenix Assurance Co., Ltd. for head office expenses allocable to Philippine business for the years 1952, 1953 and 1954 are excessive. The question of whether or not reinsurance premiums ceded to foreign reinsurers not doing business in the Philippines pursuant to contracts executed abroad are income from sources within the Philippines subject to withholding tax under Sections 53 and 54 of the Tax Code has already been resolved in the affirmative in British Traders' Insurance Co., Ltd.v. Commisioner of Internal Revenue, L-20501, April 30, 1965. 1 We come to the issue of prescription. Phoenix Assurance Co., Ltd. filed its income tax return for 1952 on April 1, 1953 showing a loss of P199,583.93. It amended said return on August 30, 1955 reporting a tax liability of P2,502.00. On July 24, 1958, after examination of the amended return, the Commissioner of Internal Revenue assessed deficiency income tax in the sum of P5,667.00. The Court of Tax Appeals found the right of the Commissioner of Internal Revenue barred by prescription, the same having been exercised more than five years from the date the original return was filed. On the other hand, the Commissioner of Internal Revenue insists that his right to issue the assessment has not prescribed inasmuch as the same was availed of before the 5-year period provided for in Section 331 of the Tax Code expired, counting the running of the period from August 30, 1955, the date when the amended return was filed. Section 331 of the Tax Code, which limits the right of the Commissioner of Internal Revenue to assess income tax within five years from the Filipino of the income tax return, states: SEC. 331. Period of limitation upon assessment and collection. ² Except as provided in the succeeding section internal revenue taxes shall be assessed within five years after the return was filed, and no proceeding in court without assessment for the collection of such taxes shall be begun after the expiration of such period. For the purposes of this section, a return filed before the last day prescribed by law for the filing thereof shall be considered as filed on such last day: Provided, That this limitation shall not apply to cases already investigated prior to the approval of this Code. The question is: Should the running of the prescriptive period commence from the filing of the original or amended return? The Court of Tax Appears that the original return was a complete return containing "information on various items of income and deduction from which respondent may intelligently compute and determine the tax liability of petitioner, hence, the prescriptive period should be counted from the filing of said original return. On the other hand, the Commissioner of Internal Revenue maintains that: "... the deficiency income tax in question could not possibly be determined, or assessed, on the basis of the original return filed on April 1, 1953, for considering that the declared loss amounted to P199,583.93, the mere disallowance of part of the head office expenses could not probably result in said loss being completely wiped out and Phoenix being liable to

deficiency tax. Not until the amended return was filed on August 30, 1955 could the Commissioner assess the deficiency income tax in question." Accordingly, he would wish to press for the counting of the prescriptive period from the filing of the amended return. To our mind, the Commissioner's view should be sustained. The changes and alterations embodied in the amended income tax return consisted of the exclusion of reinsurance premiums received from domestic insurance companies by Phoenix Assurance Co., Ltd.'s London head office, reinsurance premiums ceded to foreign reinsurers not doing business in the Philippines and various items of deduction attributable to such excluded reinsurance premiums thereby substantially modifying the original return. Furthermore, although the deduction for head office expenses allocable to Philippine business, whose disallowance gave rise to the deficiency tax, was claimed also in the original return, the Commissioner could not have possibly determined a deficiency tax thereunder because Phoenix Assurance Co., Ltd. declared a loss of P199,583.93 therein which would have more than offset such disallowance of P15,826.35. Considering that the deficiency assessment was based on the amended return which, as aforestated, is substantially different from the original return, the period of limitation of the right to issue the same should be counted from the filing of the amended income tax return. From August 30, 1955, when the amended return was filed, to July 24, 1958, when the deficiency assessment was issued, less than five years elapsed. The right of the Commissioner to assess the deficiency tax on such amended return has not prescribed. To strengthen our opinion, we believe that to hold otherwise, we would be paving the way for taxpayers to evade the payment of taxes by simply reporting in their original return heavy losses and amending the same more than five years later when the Commissioner of Internal Revenue has lost his authority to assess the proper tax thereunder. The object of the Tax Code is to impose taxes for the needs of the Government, not to enhance tax avoidance to its prejudice. We next consider Phoenix Assurance Co., Ltd.'s claim for deduction of P37,147.04 for 1950 representing net addition to reserve computed at 40% of the marine insurance premiums received during the year. Treating said said deduction to be excessive, the Commissioner of Internal Revenue reduced the same to P25,374.47 which is equivalent to 100% of all marine insurance premiums received during the last months of the year. Paragraph (a) of Section 32 of the Tax Code states: SEC. 32. Special provisions regarding income and deductions of insurance companies, whether domestic or foreign. ² (a) Special deductions allowed to insurance companies. ² In the case of insurance companies, except domestic life insurance companies and foreign life insurance companies doing business in the Philippines, the net additions, if any, required by law to be made within the year to reserve funds and the sums other than dividends paid within the year on policy and annuity contracts may be deducted from their gross income: Provided, however, That the released reserve be treated as income for the year of release. Section 186 of the Insurance Law requires the setting up of reserves for liability on marine insurance: SEC. 186. ... Provided, That for marine risks the insuring company shall be required to charge as the liability for reinsurance fifty per centum of the premiums written in the policies upon yearly risks, and the full premiums written in the policies upon all other marine risks not terminated (Emphasis supplied.)

The reserve required for marine insurance is determined on two bases: 50% of premiums under policies on yearly risks and 100% of premiums under policies of marine risks not terminated during the year. Section 32 (a) of the Tax Code quoted above allows the full amount of such reserve to be deducted from gross income. It may be noteworthy to observe that the formulas for determining the marine reserve employed by Phoenix Assurance Co., Ltd. and the Commissioner of Internal Revenue ² 40% of premiums received during the year and 100% of premiums received during the last three months of the year, respectively ² do not comply with Section 186. Said determination runs short of the requirement. For purposes of the Insurance Law, this Court therefore cannot countenance the same. The reserve called for in Section 186 is a safeguard to the general public and should be strictly followed not only because it is an express provision but also as a matter of public policy. However, for income tax purposes a taxpayer is free to deduct from its gross income a lesser amount, or not to claim any deduction at all. What is prohibited by the income tax law is to claim a deduction beyond the amount authorized therein. Phoenix Assurance Co., Ltd.'s claim for deduction of P37,147.04 being less than the amount required in Section 186 of the Insurance Law, the same cannot be and is not excessive, and should therefore be fully allowed. * We come now to the controversy on the taxpayer's claim for deduction on head office expenses incurred during 1952, 1953, and 1954 allocable to its Philippine business computed at 5% of its gross income in the Philippines The Commissioner of Internal Revenue redetermined such deduction at 5% on Phoenix Assurance Co., Ltd's net income thereby partially disallowing the latter's claim. The parties are agreed as to the percentage ² 5% ² but differ as to the basis of computation. Phoenix Assurance Co. Lt. insists that the 5% head office expenses be determined from the gross income, while the Commissioner wants the computation to be made on the net income. What, therefore, needs to be resolved is: Should the 5% be computed on the gross or net income? The record shows that the gross income of Phoenix Assurance Co., Ltd. consists of income from its Philippine business as well as reinsurance premiums received for its head office in London and reinsurance premiums ceded to foreign reinsurance. Since the items of income not belonging to its Philippine business are not taxable to its Philippine branch, they should be excluded in determining the head office expenses allowable to said Philippine branch. This conclusion finds support in paragraph 2, subsection (a), Section 30 of the Tax Code, quoted hereunder: (2) Expenses allowable to non-resident alien individuals and foreign corporations. In the case of a non-resident alien individual or a foreign corporation, the expenses deductible are the, necessary expenses paid or incurred in carrying on any business or trade conducted within the Philippines exclusively. (Emphasis supplied.) Consequently, the deficiency assessments for 1952, 1953 and 1954, resulting from partial disallowance of deduction representing head office expenses, are sustained. Finally, the Commissioner of Internal Revenue assails the dispositive portion of the Tax Court's decision limiting the maximum amount of interest collectible for deliquency of an amount corresponding to a period of three years. He contends that since such limitation was incorporated into Section 51 of the Tax Code by Republic Act 2343 which took effect only on June 20, 1959, it must not be applied retroactively on withholding tax for the years 1952, 1953 and 1954.

The imposition of interest on unpaid taxes is one of the statutory penalties for tax delinquency, from the payments of which the Court of Tax Appeals absolved the Phoenix Assurance Co., Ltd. on the equitable ground that the latter's failure to pay the withholding tax was due to the Commissioner's opinion that no withholding tax was due. Consequently, the taxpayer could be held liable for the payment of statutory penalties only upon its failure to comply with the Tax Court's judgment rendered on February 14. 1962, after Republic Act 2343 took effect. This part of the ruling of the lower court ought not to be disturbed. WHEREFORE, the decision appealed from is modified, Phoenix Assurance Co., Ltd. is hereby ordered to pay the Commissioner, of Internal Revenue the amount of P75,966.42, P59,059.68 and P48,812.32 as withholding tax for the years 1952, 1953 and 1954, respectively, and the sums of P5,667.00 and P2,847.00 as income tax for 1952 and 1954 or a total of P192,352.42. The Commissioner of Internal Revenue is ordered to refund to Phoenix Assurance Co., Ltd. the amount of P20,180.00 as overpaid income tax for 1953, which should be deducted from the amount of P192,352.42. If the amount of P192,352.42 or a portion thereof is not paid within thirty (30) days from the date this judgment becomes final, there should be collected a surcharge and interest as provided for in Section 51(c) (2) of the Tax Code. No costs. It is so ordered. Bengzon, C.J., Bautista Angelo, Concepcion, Reyes, J.B.L., Barrera, Paredes, Dizon, Regala, Makalintal and Zaldivar, JJ., concur. Footnotes
1

See also Alexander Howden & Co., Ltd. v. Commissioner of Internal Revenue, L-19392, April 14, 1965; Philippine Guaranty Co., Inc. v. Commissioner of Internal Revenue, L-22074, April 30, 1965.
*

See Maryland Casualty Co. v. U.S., 251 U.S. 342, 64 L. Ed. 297; State Farm Mutual Automotive Insurance Company v. Duel, 324 U.S. 154, 89 L. Ed. 812; Insurance Company of North America v. McCoach D.C. Pa 218 F. 905; City of Newark v. State Board of Equalization of Taxes, 79343, 81 N.J.L. 416; interpreting charges for liability on insurance contracts as reserves. Republic of the Philippines SUPREME COURT Manila EN BANC G.R. No. L-20601 February 28, 1966

BUTUAN SAWMILL, INC., petitioner, vs. HON. COURT OF TAX APPEALS, ET AL., respondents. David G. Nitafan for the petitioner. Office of the Solicitor General for the respondents. REYES, J.B.L., J.:

Appeal from a decision of the Court of Tax Appeals, in its CTA Case No. 965, ordering petitioner herein, Butuan Sawmill, Inc., to pay respondent Commissioner of Internal Revenue the sum of P36,107.74 as deficiency sales tax and surcharge due on its sales of logs to buyers in Japan from January 31, 1951 to June 8, 1953. The facts, as found and stated by the lower court in its decision, are in full accord with the evidences presented therein; hence, we quote them hereunder: . . . that during the period from January 31, 1951 to June 8, 1953, it sold logs to Japanese firms at prices FOB Vessel Magallanes, Agusan (in some cases FOB Vessel, Nasipit, also in Agusan); that the FOB prices included costs of loading, wharfage stevedoring and other costs in the Philippines; that the quality, quantity and measurement specifications of the logs were certified by the Bureau of Forestry; that the freight was paid by the Japanese buyers; and the payments of the logs were effected by means of irrevocable letters of credit in favor of petitioner and payable through the Philippine National Bank or any other bank named by it. Upon investigation by the Bureau of Internal Revenue, it was ascertained that no sales tax return was filed by the petitioner and neither did it pay the corresponding tax on the sales. On the basis of agent Antonio Mole's report dated September 17, 1957, respondent, on August 27, 1958, determined against petitioner the sum of P40,004.01 representing sales tax, surcharge and compromise penalty on its sales [tax, surcharge and compromise penalty on its sales] of logs from January 1951 to June 1953 pursuant to Sections 183, 186 and 209 of the National Internal Revenue Code (Exhibit "E", p. 14, CTA rec. & p. 14, BIR rec.). And in consequence of a reinvestigation, respondent, on November 6, 1958, amended the amount of the previous assessment to P38,917.74 (Exh. "F", p. 52, BIR rec.). Subsequent requests for reconsideration of the amended assessment having been denied (Exh. "G", p. 55, BIR rec.; Exh. "H", pp. 75-76, BIR rec.: Exh. "I", pp. 79-80, BIR rec.; Exh. "J", p. 81, BIR rec.), petitioner filed the instant petition for review on November 7, 1960. On the bases of the above-quoted findings and circumstances, the lower court upheld the legality and correctness of the amended assessment of the sales tax and surcharge, ruling that the sales in question, in the light of our previous decisions1, were domestic or "local" sales, and, therefore, subject to sales tax under the provision of section 186 of the Tax Code, as amended by Republic Acts Nos. 558 and 594; and that the assessment thereof was made well within the ten-year period prescribed by Section 332(a) of the same Code, since petitioner herein omitted to file its sales tax returns for the years 1951, 1952 and 1953, and this omission was discovered only on September 17, 1957. The imposition of the compromise penalty was, however, eliminated therefrom for want of agreement between the taxpayer and the Collector (now Commissioner) of Internal Revenue. A motion to reconsider said decision having been denied, petitioner herein interposed the present appeal before this Court. The issues presented in this appeal are: whether or not petitioner herein is liable to pay the 5% sales tax as then prescribed by Section 186 of the Tax Code on its sales of logs to the Japanese buyers; and whether or not the assessment thereof was made within the prescriptive period provided by law therefor.
1äwphï 1.ñë t

On the first issue, petitioner herein insists that the circumstances enumerated in the above finding, which this Court had, in previous decisions (Cf. footnote [1]), considered as determinative of the place of transfer of ownership of the logs sold, for purposes of taxation, are not in themselves evidentiary indications to show that the parties intended the title of the logs to pass to the Japanese buyers in Japan. Thus, it points out that the "FOB" feature of the sales contract was made only to fix

its price and not to fix the place of delivery; that the requirement of certification of quality, quantity, and measurement specifications of the logs by local authorities was done to comply with local laws, rules, and regulations, and was not a part of the sales arrangement; that the payment of freight by the Japanese buyers is not an uncommon feature of "FOB" shipments; and that the payment of prices by means of irrevocable letters of credit is but a common established business practice to secure payment of the price to the seller. It also insists that, even assuming that the "FOB" feature of the disputed sales determines thesitus of transfer of ownership, the same is merely a prima facie presumption which yields to contrary proof such as that the logs were made deliverable to the "order of the shipper" and the logs were shipped at the risk of the shipper, which circumstances, if considered, would negate the above implications. Hence, petitioner herein contends that the disputed sales were consummated in Japan, and, therefore, not subject to the taxing jurisdiction of our Government. The above contentions of petitioner are devoid of merit. In a decided case with practically identical set of facts obtaining in the case at bar, this Court declared: . . . it is admitted that the agreed price was "F.O.B. Agusan", thus indicating, although prima facie, that the parties intended the title to pass to the buyer upon delivery of the logs in Agusan; on board the vessels that took the goods to Japan. Moreover, said prima facie proof was bolstered up by the following circumstances, namely: 1. Irrevocable letters of credit were opened by the Japanese buyers in favor of the petitioners. 2. Payment of freight charges of every shipment by the Japanese buyers. 3. The Japanese buyers chartered the ships that carried the logs they purchased from the Philippines to Japan. 4. The Japanese buyers insured the shipment of logs and collected the insurance coverage in case of loss in transit. 5. The petitioner collected the purchase price of every shipment of logs by surrendering the covering letter of credit, bill of lading, which was indorsed in blank, tally sheet, invoice and export entry, to the corresponding bank in Manila of the Japanese agent bank with whom the Japanese buyers opened letters of credit. 6. In case of natural defects in logs shipped to the buyers discovered in Japan, instead of returning such defective logs, accepted them, but were granted a corresponding credit based on the contract price. 7. The logs purchased by the Japanese buyers were measured by a representative of the Director of Forestry and such measurement was final, thereby making the Government of the Philippines a sort of agent of the Japanese buyers. Upon the foregoing facts and authority of Bislig (Bay) Lumber Co., Inc. vs. Collector of Internal Revenue, G.R. No. L-13186 (January 28, 1961), Misamis Lumber Co., Inc. vs. Collector of Internal Revenue (56 Off. Gaz. 517) andWestern Mindanao Lumber Development Co., Inc. vs. Court of Tax Appeals, et al. (G.R. No. L-11710, June 30, 1958), it is clear that said export sales had been consummated in the Philippines and were, accordingly, subject to sales tax therein." (Taligaman Lumber Co., Inc. vs. Collector of Internal Revenue, G.R. No. L-15716, March 31, 1962).

With respect to petitioner's contention that there are proofs to rebut the prima facie finding and circumstances that the disputed sales were consummated here in the Philippines, we find that the allegation is not borne out by the law or the evidence. That the specification in the bill of lading to the effect that the goods are deliverable to the order of the seller or his agent does not necessarily negate the passing of title to the goods upon delivery to the carrier is clear from the second part of paragraph 2 of Article 1503 of the Civil Code of the Philippines (which appellant's counsel improperly omits from his citation): Where goods are shipped, and by the bill of lading the goods are deliverable to the seller or his agent, or to the order of the seller or of his agent, the seller thereby reserves the ownership in the goods. But, if except for the form of the bill of lading, the ownership would have passed to the buyer on shipment of the goods, the sellers's property in the goods shall be deemed to be only for the purpose of securing performance by the buyer of his obligations under the contract. Moreover, it has been "a settled rule that in petitions to review decisions of the Court of Tax Appeals, only questions of law may be raised and may be passed upon by this Court" (Gutierrez vs. Court of Tax Appeals & Collector of Internal Revenue vs. Gutierrez, G.R. Nos. L-7938 & L-9771, May 21, 1957, cited in Sanchez vs. Commissioner of Customs, G.R. No. L-8556, September 30, 1957); and it having been found that there is no proof to substantiate the foregoing contention of petitioner, the same should also be ruled as devoid of merit. On the second issue, petitioner avers that the filing of its income tax returns, wherein the proceeds of the disputed sales were declared, is substantial compliance with the requirement of filing a sales tax return, and, if there should be deemed a return filed, Section 331, and not Section 332(a), of the Tax Code providing for a five-year prescriptive period within which to make an assessment and collection of the tax in question from the time the return was deemed filed, should be applied to the case at bar. Since petitioner filed its income tax returns for the years 1951, 1952 and 1953, and the assessment was made in 1957 only, it further contends that the assessment of the sales tax corresponding to the years 1951 and 1952 has already prescribed for having been made outside the five-year period prescribed in Section 331 of the Tax Code and should, therefore, be deducted from the assessment of the deficiency sales tax made by respondent. The above contention has already been raised and rejected as not meritorious in a previous case decided by this Court. Thus, we held that an income tax return cannot be considered as a return for compensating tax for purposes of computing the period of prescription under Section 331 of the Tax Code, and that the taxpayer must file a return for the particular tax required by law in order to avail himself of the benefits of Section 331 of the Tax Code; otherwise, if he does not file a return, an assessment may be made within the time stated in Section 332(a) of the same Code (Bisaya Land Transportation Co., Inc. vs. Collector of Internal Revenue & Collector of Internal Revenue vs. Bisaya Land Transportation Co., Inc., G.R. Nos. L-12100 & L-11812, May 29, 1959). The principle enunciated in this last cited case is applicable by analogy to the case at bar. It being undisputed that petitioner failed to file a return for the disputed sales corresponding to the years 1951, 1952 and 1953, and this omission was discovered only on September 17, 1957, and that under Section 332(a) of the Tax Code assessment thereof may be made within ten (10) years from and after the discovery of the omission to file the return, it is evident that the lower court correctly held that the assessment and collection of the sales tax in question has not yet prescribed. Wherefore, the decision appealed from should be, as it is hereby affirmed, with costs against petitioner.

Bengzon, C.J., Bautista Angelo, Concepcion, Barrera, Dizon, Regala, Makalintal, Bengzon, J.P., Zaldivar and Sanchez, JJ., concur. Republic of the Philippines SUPREME COURT Manila EN BANC G.R. No. L-10370 January 31, 1958

THE COLLECTOR OF INTERNAL REVENUE, petitioner, vs. MATIAS H. AZNAR and THE COURT OF TAX APPEALS, respondents. Office of the Solicitor General Ambrosio Padilla, Solicitor Felicisimo R. Rosete, and Special Attorney Librada del Rosario-Natividad for petitioner. Primitivo N. Sato and Jose P. Enad for respondent Aznar. FELIX, J.: This is a petition filed by the Collector of Internal Revenue to review by certiorari the resolution of the Court of Tax Appeals dated February 8, 1956, in C.T.A. Case No. 109 enjoining him from enforcing collection of the alleged income tax liability of Matias H. Aznar through summary administrative method. The facts of the case are as follows: In a letter dated November 28, 1952, the Collector of Internal Revenue, through the office of the City Treasurer of Cebu, demanded from Matias H. Aznar the payment of P732,032.66 allegedly representing the latter's income tax deficiencies for the tax years 1945 to 1951. It appears on record that the Collector of Internal Revenue also instructed the City Treasurer of Cebu to place the properties of said taxpayer under constructive distraint to guarantee the satisfaction of the taxes thus assessed (Exh. 9), and this instruction was supposedly complied with by the latter official in virtue of a warrant of distraint and levy dated February 17, 1953, and served on taxpayer Aznar on February 20, 1953 (Exh. 11). An exchange of communications between the Internal Revenue Office and the taxpayer ensued as a result of which a reinvestigation of the income tax assessment of the latter was made and the same was finally reduced from P380,999.70. Aznar was correspondingly informed of this correction in a communication dated February 16, 1955, specifically stating that this later figure superseded the previous one sent by the Bureau of Internal Revenue. Upon receipt of the corrected assessment, the taxpayer filed with the Court of Tax Appeals a petition to review the same and subsequently an urgent petition was also filed to restrain therein respondent Collector of Internal Revenue from proceeding with the collection of the alleged tax deficiencies by means of the summary methods of distraint and levy (Annex B of petition), on the ground that the right of the respondent Collector to effect the collection of the taxes demanded of said taxpayer by extra judicial methods had already prescribed; that the employment of these means would cause petitioner injustice and irreparable injury; and that the petition was not merely intended to delay the payment of the taxes because petitioner stood on an even chance of winning the case if given a day in court (Annex B of the petition). The Collector of Internal Revenue set up an opposition against the grant of this petition and consequently, hearing on the matter was duly conducted by the lower Court. After the parties had filed their respective memoranda and rested their case, the lower Court, in a resolution dated February 8, 1956, issued an injunction prayed for enjoining the Collector of Internal Revenue from proceeding with the collection of the taxes by means of the summary

methods of distraint and levy after finding that the warrant issued by the Treasurer of the City of Cebu dated February 17, 1953, placing the properties of Matias H. Aznar under constructive distraint and levy and which was supposedly received by said taxpayer on February 20, 1953, was not actually served on petitioner Aznar; that the warrant of garnishment served on the Philippine National Bank in Manila on August 14, 1953, was null and void in view of the respondent's failure to furnish the taxpayer with a copy of the same and that at the placing of the properties of the taxpayer under constructive distraint and levy on April 28, 1955, was made beyond the 3-year prescriptive period as provided by Section 51-(d) of the National Internal Revenue Code. From this resolution, the Collector of Internal Revenue brought the matter to this Court in a petition to review by certiorari contending that collection of taxes cannot be restrained by injunction; and that even if the court a quo could have lawfully issued the same, said tribunal acted with grave abuse of discretion when it did not require the taxpayer to file a bond as exhorted by Section 11 of Republic Act No. 1125. There appears no record as to when Matias H. Aznar filed his income tax returns for the years 1945 and 1946, but it is not controverted that his tax returns for 1947 was filed on March 1, 1948; for 1948 on February 28, 1949; for 1949 on March 1, 1950; for 1950, on March 1, 1951; and for 1951, on March 1, 1952. There is likewise no dispute that Matias H. Aznar's alleged income tax deficiencies were assessed at P723,132.66 on November 28, 1952, which was reduced to P380,999.70 on March 17, 1955. During the hearing had in the court below, the Collector of Internal Revenue, trying to establish the fact that the properties of the taxpayer were already placed under constructive distraint and levy as of February 20, 1953, offered in evidence Exhibit 11, purportedly a duplicate of the warrant dated February 17, 1953, and allegedly received by Aznar on February 20 of the same year. Aznar, however disputed the authenticity of Exhibit 11 maintaining that it was never served on him. It appears on record of a warrant of garnishment to distrain the deposits Mr. and Mrs,. Aznar had with the Philippine National Bank was also served on said banking institution on August 14, 1953, but the taxpayer once again interposed an objection to the use of this measure on the ground that he was not notified thereof pursuant to the provision of Section 319 of the Tax Code and asserted that the issuance of said warrant was null and void. There is likewise no controversy that the City Treasurer of Cebu levied upon certain real properties belonging to the taxpayer on May 6, 1955, but Aznar took exception to the employment of this administrative method to effect collection of the taxes allegedly due by him, it having been issued 3 years, 2 months and 5 days after he ahd filed his income tax returns for the year 1951, and therefore beyond the 3-year prescriptive period required by Section 51-d of the Tax Code. Actually, the questions at issue in the instant case are: whether the Collector of Internal Revenue could enforce collection of the alleged deficiency income taxes of Matias H. Aznar through the summary methods of distraint and levy and, consequently, whether the Court of Tax Appeals erred in issuing the injunction restraining said official from employing the same; and granting the Court of Tax Appeals could issue an injunction, whether said tribunal erred in not requiring the taxpayer to make a deposit. We agree with petitioner that Section 305 of the National Internal Revenue Code precludes the use of injunction to restrain the collection of taxes, but as this Court has already pronounced, in view of the existence of the provisions of Section 11 of Republic Act No. 1125 allowing the Tax Court to issue said writ of injunction subject certain limitations, the former (Sec. 305) must be deemed to have modified by the later enactment-Republic Act No. 1125 (Collector of Internal Revenue vs. Avelino, 100 Phil., 327, 53 Off. Gaz., 645). And we have since then and even before, adhered to the doctrine that the collection of income taxes, after the lapse of three years from the date the income tax return said to be false, fraudulent or erroneous had been filed, may no longer be effected by means of administrative methods but only through judicial proceedings (Collectors of Internal Revenue vs. Villegas, 56 Phil. 554; Collector of Internal Revenue vs. Haygood, 65 Phil. 520; Juan de

la Vifla vs. El Gobierno de las Islas Filipinas, G.R. No. 42669, Jan. 29, 1938; Philippine Sugar Estate Development Co., Inc. vs. Posadas, 68 Phil. 216; Collector vs. Avelino, supra; Collector vs. A.P. Reyes, 100 Phil., 822; Collector vs. Zulueta, 100 Phil., 872; 53 Off. Gaz., [9] 6532; Sambrano vs. Court of Tax Appeals, 101 Phil., 1, 53 Off. Gaz., [15] 4839. In the light of the aforementioned ruling, were We to consider as valid andin order the disputed warrant dated February 17, 1953, placing the propertiesof the taxpayer under constructive destraint and levy, the collection of thetaxes for 1949, 1950 and 1951 by extra-judicial methods would proper and the resolution of the Court of Tax Appeals as far as it concerns this later period would be erroneous, although summary administrative means would nolonger be the proper recourse for the collection of taxes corresponding to1948 and the years previous to that as 3 years, 11 months and 22 days has already elapsed from the time the income tax return for that year was filed. The respondent Court of Tax Appeals, however, made a finding that while itmay be true that Exhibit 11 could have been prepared at the time referred to,probably through omission, oversight or negligence, same was not executed and thus actually the properties of the taxpayer were only placed underconstructive distraint and levy on May 6, 1955. As Republic Act No 1125 creating the Court of Tax Appeals keep silence as to matters left open to Usfor review or the issues that We may take conizance of, and as courts have to construe statutes as they are found and not to amend or change them under the guise of construction (82 C.J.S. 530), this Court in passing upon petitionsto review by certiorari decisions or rulings of the Court of Tax Appeals may review, revise, reverse, amend or modify not only the legalissues involved therein but also the findings of fact upon which said decision or ruling is based. Notwithstanding the foregoing, it may be stated that any party adverselyaffected by any rulign, order or decision of the Court of Tax Appeals has by law two ways of elevating his case to the Supreme Court, i.e., first, by filing in the Court a quo a notice of appeal and with this Court a petition for review within 30 days from the date he receives notice of said ruling, order or decision adverse to him (Sec. 18, Rep. Act 1125), and second, by causing such ruling, order or decision of the Court of Tax Appeals likewise reviewed by Us upon a writ of certiorari in proper cases (Sec. 19, R.A. No. 1125). Premised on these provisions, it may be alleged that when a case is taken up to this Court by petition for review, We could go over the evidence on record and pass upon the questions of fact; but that in cases of review upon petition for a writ of certiorari, this Court could only pass upon issues involving questions of law. In answer to these possible arguments We may say that when the interest of justice so demands, We may interchangeably consider petitions for review as petitions for a writ of certiorari and vice-versa, and if We have the power to consider the evidence to determine the facts in the cases of review, We find no plausible reason for depriving this Court of such power in petitions for certiorari specially if We consider that in the latter cases the petitioner oftenly charges the respondent Court with the commission of grave abuse of discretion the determination of which usually depends on the facts and circumstances of the points in controversy. Moreover, in the case at bar, We find that on March 1, 1956, respondent Collector of Internal Revenue filed with the Court of Tax Appeals a notice of appeal from the resolution of said Court that is now subject of this recourse (p. 466, CTA records) and no matter how inappropriatemay be the wording of the petition filed in this instance, it could not conceal that respondent's intention was to appeal the matter to this Court, as otherwise he would not have filed said notice of appeal which is required in petitions for review (Sec. 18, R.A. No. 1125) and not in petitions forcertiorari (Sec. 19, id.). It is also to be noted that in the instant case of the Solicitor General has not filed any motion for the reconsideration of said resolution, a requisite that is necessary in petitions for certiorari. Having all this in mind, We are inclined to consider the question of facts invoved in the present controversy, and in going over the evidence presented We find contrary to the conclusion arrived at by the court a quo, that there are proofs supporting petitioner's contention that Exhibit 11 was properly executed. The respondent Court refused to give credence to the employees of the City

Treasurer's Office who claimed to have served the converted warrant on the taxpayer on February 20, 1953, by reason of certain inconcistencies in their declarations during the extensive cross examination conducted by counsel for respondent Aznar. We must remember, however, that considering the time that had lasped when the incident took place and the date they were questioned under oath as regard their affidavits recounting the event, it is but natural for the human brain not to pick up certain details of an event that transpired sometime ago and thus expect minor inconsistencies in the testimonies of several witnesses. On substantial points²as to who and how the warrant in question was served, the person receiving the same, and other facts surrounding said service, the witnesses are in unison in their declarations. It is true that exhibit 11 is merely a duplicate copy of the warrant and that the original thereof could nowhere be found. But the personel of the office of the City Treasurer of Cebu admitted it was lost and for this reason thier affidavits recounting said service were executed (Exhs. 13, 14, 15, 16 and 17²p. 68-79, CTA Records). Moreover we find in the records a decisive factor that props up the contention of petitioner Collector of Internal Revenue, for We must not lose sight of the fact that Exhibit 11 contains a list of the properties of Matias H. Aznar which were levied upon by the City Treasurer of Cebu and which the Treasurer in turn placed in the possession of said owner for safe keeping, as acknowledged by the latter in said exhibit, and such properties could not have been mentioned in the document if said properties had not been taken from and returned to taxpayer Aznar who has not denied that same were his. We cannot simply disregard this form of evidence not only because affidavits are admissable to prove the service of a summons, notice or other papers in an action or special proceeding (See Wigmore on Evidence, Vol. 6, 3rd ed., p. 42-49,) but also because respondent taxpayer was given opportunity to cross-examine the affiants before the Municipal Court of Cebu. The lower Court also placed much stress on the supposition that the Collector of Internal Revenue should not have sent the communications dated March 15 and March 28, 1955, inquiring as to what action the City Treasurer of Cebu had taken on the tax case of Matias H. Aznar if the latter had really sent the original of Exhibit 11 to respondent Collector. A close scrutiny of the letters referred to reveals that they were in connection with the correctedassessment sent by the Internal Revenue Office to Matias H. Anzar. datedFebruary 16, 1955 (p. 311, BIR records), of which the Treasurer's Office was also duly notified (p. 315, BIR records) and not in connection with the assessment of November 28, 1952. The issuance of another warrant by theTreasurer's Office on April 28, 1955 and which the was admittedly received byAnzar on May 6, of the same year was likewise taken by the Tax Court to contradict the existence of Exhibit 11. It is correct that a mistake wascommitted by said office in stating therein that the income tax deficiencies of Matiaz H. Anzar amounted to P723,032.07 because this figure as correctedshould properly be P380,999.70, but it must be noted that this second warrantcovers 2 buildings belonging to the taxpayer found in the province of Leytewhich were not included among those listed in the first warrant, Exhibit 11.As explained, this warrant was issued because the properties covered by thefirst writ would not be sufficient to satisfy the amount demanded by the Government. Thus, piecing the evidence together, it is clear to our mind thatthe warrant placing the properties of Matias H. Anzar under constructive distraint and levy was served on the latter on February 20, 1953, which was2 years, 11 months and 10 days after the taxpayer had filed his income tax return of the tax year 1949; 1 year 11 months and 19 days after he had filedhis returns for 1950; and 11 months and 19 days after he did so far for the year 1951. Section 11 of Republic Act No. 1125 contains the following: SEC 11. WHO MAY APPEAL; EFFECT OF APPEAL.² xxx xxx xxx.

No appeal taken to the Court of Appeals from the decision of the Collector of Internal Revenue or the Collector of Customs shall suspend the payment,levy, distraint and/or sale of

any property of the taxpayer for the satisfaction of his tax liability as provided by existing law; Provided, however, That when in the opinion of the Court the collection by the Bureauof Internal Revenue or the Commissioner of Customs may jeopardize the interest of the Government and/or the taxpayer the Court at any stage of theproceeding may suspend the said collection and require the taxpayer eitherto deposit the amount claimed or to file a surety bond for not more than double the amount with the Court. It may be seen that the Court is allowed by law to suspend the collectionof taxes subject to certain limitations. Teh second question posed herein is whether the Court of Tax Appeals couldissue an injunction to suspend such collection without requiring the taxpayerto make a deposit or file a bond? This Court, resolving the same question in similae case, held that the requirement of a bond before a writ of injunctioncould be issued by the Tax Court applies only to cases where the means soughtbe employed for the enforcement of the collection of the tax are by themselves legal and not where same were declared null and void, as where thesummary methods of distraint and levy would be utilized in the collection ofdeficiency income taxes, after the 3-year prescriptive period as provided bySection 51-d of the Internal Revenue Code has already elapsed (Collector ofInternal Revenue vs. A.P. Reyes,supra; Sambrano vs. CTA, supra). The court,in upholding this theory, explains: SECTION 11 of Republic Act No. 1125 is therefore promised on the assumptionthat the collection by summary proceedings is by itself in accordance with existing law; and then what is suspended in the act of collecting, whereas, in the case at bar what the respondent Court suspended was the use of methodemployed to verify the collection which was evidently illegal after the lapse of the three-year limitation period. The respondent Court issued the injunction in question on the basis of its findings that the means of intended to be used by the petitioner in the collection of the alleged deficiency taxes were in violation of law. It certainly would be an absurdityon the part of the Court of Tax Appeals to declare that the collection bysummary methods of distraint and levyt was violative of the law, and then,on the same breath, require the petitioner to deposit or file a bond as aprerequisite for the issuance of a writ of injunction. Let us suppose, for the sake of argument, that the Court a quo would have required the petitioner to post the bond in question that the taxpayer would refuse or fall to furnish said bond, would the Court, a quo be obliged to authorize or allow the Collector to proceed with the collection from the petitioner of the taxesdue by a means it previously declared to the contrary to law? (Collector vs.Reyes, supra). As the Collector of Internal Revenue, thought the Office of the City Treasurer of Cebu, placed the properties of the taxpayer under distraint andlevy only on February 20, 1953, to secure the payment of alleged income taxdeficiencies for the tax years 1945 to 1951, and as with respect to the taxesdemanded for the yeatr 1945, 1946, 1947 and 1948, the said warrant was issuedbeyond the 3year period of limitations as prescribed by Section 51-d of the Tax Code, and the following the ruling adopted by this Court as regards theissuance by the Tax Court of writs of injunction, the respondent Court didnot err in enjoining the Collector from using summary administrative methodswithout requiring the taxpayer to post a bond or make a deposit as far as thetax years 1945, 1946, 1946 and 1948 are concerned. As regards 1949 to 1951,the answer is all too obvious, though We must have in mind that the cuort a quo acted on the erroneuos assumption that the period for said summary adminstrative methods had already lapsed and that the effect of its ruling isa fait accompli. Wherefore, the resolution of the Court of Tax Apeals dated February 8, 1956,is set aside and this case is hereby remanded to the lower Court for furtherproceedings so that it may determine the income tax liabilities of MatiasH. Aznar that have not prescribed under the terms and period fixed by Sections 331 and 332 of the National Internal Revenue. Pending the disposition of this case in the

lower Court, respondent Matias H. Aznar isordered to deposit with said court the amount demanded from him for the years1949 to 1951 or furnis a surety bond for not more than double said amount.Costs are taxed against respondent Matias H. Aznar. It is so ordered. Bengzon, Padilla, Montemayor, Bautista Angelo, Reyes, J.B.L., and Endencia, JJ., concur. Paras, C.J., and Reyes, A.J., concur, in the result. Republic of the Philippines SUPREME COURT Manila EN BANC G.R. No. L-17438 April 30, 1964

REPUBLIC OF THE PHILIPPINES, plaintiff-appellant, vs. RITA LIM DE YU, defendant-appellee. Office of the Solicitor General for plaintiff-appellant. Ignacio M. Orendain for defendant-appellee. MAKALINTAL, J.: Appellee Rita Lim de Yu filed her yearly income tax returns from 1948 through 1953. The Bureau of Internal Revenue assessed the taxes due on each return, and appellee paid them accordingly. On July 17, 1956 the Bureau issued to appellee deficiency income tax assessments for the years 1945 to 1953 in the total amount of P22,450.50. She protested the assessments and requested a reinvestigation. On August 30, 1956 she signed a "waiver" of the statute of limitations under the Tax Code as condition to the reinvestigation requested. Thereafter, or on July 18, 1958, the Bureau issued to her income tax assessment notices for the years 1948 to 1953 totalling P35,379.63. This last assessment, like the one issued in 1956, covered not only the basic deficiency income taxes, but also 50% thereof as surcharge. Upon appellee's failure to pay, an action for collection was filed against her in the Court of First Instance of Cotabato on May 11, 1959. After trial the suit was dismissed, and the Government appealed to the Court of Appeals, which forwarded the case to this Court, the issues involved being purely legal. Appellant claims that the lower court erred (1) in ruling that the deficiency income taxes due from appellee for the years 1948, 1949 and 1956 were not assessed on time; and (2) in dismissing the case on the ground that the right of appellant to collect the deficiency income tax assessment had already prescribed. Sections 331 and 332 of the Tax Code provide: SEC. 331. Period of limitation upon assessment and collection. ² Except as provided in the succeeding section, internal revenue taxes shall be assessed within five years after the return was filed, and no proceeding in court without assessment for the collection of such taxes shall be begun after the expiration of such period. For the purposes of this section a return filed before the last day prescribed by law for the filing thereof shall be considered as filed on such last day: Provided, That this limitation shall not apply to cases already investigated prior to the approval of this Code.

SEC. 332. Exceptions as to period of limitation of assessment and collection of taxes. ² (a) In the case of a false or fraudulent return with intent to evade tax or of a failure to file a return, the tax may be assessed, or a proceeding in court for the collection of such tax may be begun without assessment, at any time within ten years after the discovery of the falsity fraud, or omission. (b) Where before the expiration of the time prescribed in the preceding section for the assessment of the tax, both the Commissioner of Internal Revenue and the taxpayer have consented in writing to its assessment after such time, the tax may be assessed at any time prior to the expiration of the period agreed upon. The period so agreed upon may be extended by subsequent agreements in writing made before the expiration of the period previously agreed upon. (c) Where the assessment of any internal revenue, tax has been made within the period of limitation above prescribed such tax may be collected by distraint or levy or by a proceeding in court, but only if begun (1) within five years after the assessment of the tax, or (2) prior to the expiration of any period for collection agreed upon in writing by the Commissioner of Internal Revenue and the taxpayer before the expiration of such five-year period. The period so agreed upon may be extended by subsequent agreements in writing made before the expiration of the period previously agreed upon. The first issue raised by appellant is whether or not the returns filed by appellee for the years 1948 to 1953 are false and fraudulent. Appellant maintains they are because the yearly net incomes reported in her returns are much less than as computed by the Bureau, and consequently, under par. (a), Section 332 of the Tax Code, it has ten years from the date of the discovery of the fraud or falsity, i.e., May 25, 1955, within which to assess the taxes or file a suit for collection without assessment. And since, it is further contended, appellee can no longer question correctness of the assessment in view of her failure to the Court of Tax Appeals to review the same, she should be ordered to pay the amounts being collected. But while fraud is alleged in the complaint, the same has not been established. It is one thing to say that the correctness of the last assessment made by appellant, July 18, 1958, may no longer be challenged on the technical ground just stated and quite another thing to say that appellee committed a deliberate fraud in declaring small incomes for the years in which she filed her returns. Indeed the Bureau itself appears none too sure as to the amounts of her net incomes for those years. On three different occasions it arrived at three highly different computation. First, it accepted appellee's yearly statements of income from 1945 to 1953 and assessed her a total tax P2,732,37, which she paid. Then in 1956 the Bureau came up with a different set of figures for the same period, considerably higher than those stated in the returns, and using such figures as basis assessed her deficiency taxes in the total amount of P22,450.50. Finally, in 1958 the Bureau made another computation of appellee's net incomes for years 1948 to 1953, and assessed her deficiency taxes in the sum of P35,379.63. Note that the disparity between the 1956 and the 1958 assessments is really much greater than what appears at first glance, as the latter do not include the taxes corresponding to the years 1945, 1946 and 1947. Attention may likewise be drawn to the fact that in paragraph 3 of the complaint appellant seeks to collect appellee the sum of P28.53, plus a surcharge of 50%, unpaid tax for the year 1948, notwithstanding the fact admitted in the stipulation, that appellee filed her return that year and duly paid the said amount. Fraud not having been proven, the period of limitation for assessment or collection was five years from the filing of the return, according to Section 331 of the tax code. The right to assess or collect the income taxes for the years 1948 to 1950 had already prescribed, therefore, when the Bureau of Internal Revenue issued the deficiency income tax assessments on July 17, 1956.

The tax years 1948 to 1950 cannot be deemed included in the "waiver of the statute of limitations under the National Internal Revenue Code" executed by appellee on August 30, 1956. The five-year period for assessment, counted from the date the return is filed, may be extended upon agreement of the Commissioner and the taxpayer, but such agreement must be made before, not after, the expiration of the original period (Section 332 [b], Tax Code). The clear import of the provision is that it does not authorize extension once prescription has attached. The waiver validly covers only the tax years 1951 and 1952, with respect to which the five-year period had not yet elapsed when the said waiver was executed. With respect to the tax year 1953, as to which the return was filed by appellee on March 1, 1954, the waiver was not necessary for the effectivity of the assessment made on July 18, 1958, since such assessment was well within the original five-year period provided by law. After the assessment on July 18, 1958, appellant had five years within which to file suit for collection pursuant to Section 332 (c) of the tax code. Appellee's theory that collection could be made only up to the end of the period of extension stated in the waiver, namely, December 31, 1958, is without merit. Assessment and collection are two different processes. An assessment is not an action or proceeding for the collection of taxes. It is merely a notice to the effect that the amount therein stated is due as tax and a demand for the payment thereof. It is a step preliminary, but essential to warrant distraint, if still feasible, and, also, to establish a cause for "judicial action" as the phrase is, used in section 316 of the Tax Code ... (Alhambra Cigar and Cigarette Manufacturing Company v. The Collector of Internal Revenue, L-12026, May 29, 1959). Section 331 gives the Government five years from filing of the return (which is not false or fraudulent) within which to assess the tax due. Paragraph (b) of Section 332 allows the extension of this period by means of a written agreement between the taxpayer and the Commissioner of Internal Revenue. On the other hand, paragraph (c) of the same section is concerned with the collection of the tax after assessment, regardless of whether the assessment was made during the original fiveyear period or within an agreed period of extension. Collection then may be effected within five years after assessment or within the "period for collection agreed upon in writing by the Commissioner of Internal Revenue and the taxpayer before the expiration of such five-year period." Thus, although under the waiver appellee consented to the "assessment and collection" if made not later than December 31, 1958, such ,expiration date must be deemed to refer only to the extension of the assessment period. Insofar as collection is concerned, the period does not apply, for otherwise the effect of the waiver would be to shorten, not extend, the legal period for that purpose. Appellant therefore had five years from 1958 within which to file his action, which was actually filed in 1959. WHEREFORE, the appealed decision is modified by ordering appellee to pay appellant the sum of P26,182.00 as deficiency income taxes for the years 1951, 1952 and 1953, plus 5% surcharge and 1% monthly interest thereon from July 31, 1958 until payment of the full obligation, with costs. Padilla, Bautista Angelo, Labrador, Concepcion, Reyes, J.B.L., Barrera, Paredes and Dizon, JJ., concur. Bengzon, C.J., took no part. Republic of the Philippines SUPREME COURT Manila EN BANC

G.R. No. L-22492

September 5, 1967

BASILAN ESTATES, INC., petitioner, vs. THE COMMISSIONER OF INTERNAL REVENUE and THE COURT OF TAX APPEALS, respondents. Felix A. Gulfin and Antonio S. Alano for petitioner. Office of the Solicitor General for respondents.

BENGZON, J.P., J.: A Philippine corporation engaged in the coconut industry, Basilan Estates, Inc., with principal offices in Basilan City, filed on March 24, 1954 its income tax returns for 1953 and paid an income tax of P8,028. On February 26, 1959, the Commissioner of Internal Revenue, per examiners' report of February 19, 1959, assessed Basilan Estates, Inc., a deficiency income tax of P3,912 for 1953 and P86,876.85 as 25% surtax on unreasonably accumulated profits as of 1953 pursuant to Section 25 of the Tax Code. On non-payment of the assessed amount, a warrant of distraint and levy was issued but the same was not executed because Basilan Estates, Inc. succeeded in getting the Deputy Commissioner of Internal Revenue to order the Director of the district in Zamboanga City to hold execution and maintain constructive embargo instead. Because of its refusal to waive the period of prescription, the corporation's request for reinvestigation was not given due course, and on December 2, 1960, notice was served the corporation that the warrant of distraint and levy would be executed. On December 20, 1960, Basilan Estates, Inc. filed before the Court of Tax Appeals a petition for review of the Commissioner's assessment, alleging prescription of the period for assessment and collection; error in disallowing claimed depreciations, travelling and miscellaneous expenses; and error in finding the existence of unreasonably accumulated profits and the imposition of 25% surtax thereon. On October 31, 1963, the Court of Tax Appeals found that there was no prescription and affirmed the deficiency assessment in toto. On February 21, 1964, the case was appealed to Us by the taxpayer, upon the following issues: 1. Has the Commissioner's right to collect deficiency income tax prescribed? 2. Was the disallowance of items claimed as deductible proper? 3. Have there been unreasonably accumulated profits? If so, should the 25% surtax be imposed on the balance of the entire surplus from 1947-1953, or only for 1953? 4. Is the petitioner exempt from the penalty tax under Republic Act 1823 amending Section 25 of the Tax Code? PRESCRIPTION There is no dispute that the assessment of the deficiency tax was made on February 26, 1959; but the petitioner claims that it never received notice of such assessment or if it did, it received

the notice beyond the five-year prescriptive period. To show prescription, the annotation on the notice (Exhibit 10, No. 52, ACR, p. 54-A of the BIR records) "No accompanying letter 11/25/" is advanced as indicative of the fact that receipt of the notice was after March 24, 1959, the last date of the five-year period within which to assess deficiency tax, since the original returns were filed on March 24, 1954. Although the evidence is not clear on this point, We cannot accept this interpretation of the petitioner, considering the presence of circumstances that lead Us to presume regularity in the performance of official functions. The notice of assessment shows the assessment to have been made on February 26, 1959, well within the five-year period. On the right side of the notice is also stamped "Feb. 26, 1959" ² denoting the date of release, according to Bureau of Internal Revenue practice. The Commissioner himself in his letter (Exh. H, p. 84 of BIR records) answering petitioner's request to lift, the warrant of distraint and levy, asserts that notice had been sent to petitioner. In the letter of the Regional Director forwarding the case to the Chief of the Investigation Division which the latter received on March 10, 1959 (p. 71 of the BIR records), notice of assessment was said to have been sent to petitioner. Subsequently, the Chief of the Investigation Division indorsed on March 18, 1959 (p. 24 of the BIR records) the case to the Chief of the Law Division. There it was alleged that notice was already sent to petitioner on February 26, 1959. These circumstances pointing to official performance of duty must necessarily prevail over petitioner's contrary interpretation. Besides, even granting that notice had been received by the petitioner late, as alleged, under Section 331 of the Tax Code requiring five years within which to assessdeficiency taxes, the assessment is deemed made when notice to this effect is released, mailed or sent by the Collector to the taxpayer and it is not required that the notice be received by the taxpayer within the aforementioned five-year period.1 ASSESSMENT The questioned assessment is as follows: Net Income per return Add: Over-claimed depreciation Mis. expenses disallowed Officer's travelling expenses disallowed Net Income per Investigation 20% tax on P59,702.96 Less: Tax already assessed Deficiency income tax Add: Additional tax of 25% on P347,507.01 Tax Due & Collectible P40,142.90 P10,500.49 6,759.17 2,300.40 19,560.06 P59,702.96 11,940.00 8,028.00 P3,912.00 86,876.75 P90,788.75 =========

The Commissioner disallowed: Over-claimed depreciation Miscellaneous expenses Officer's travelling expenses P10,500.49 6,759.17 2,300.40

DEDUCTIONS A. Depreciation. ² Basilan Estates, Inc. claimed deductions for the depreciation of its assets up to 1949 on the basis of their acquisition cost. As of January 1, 1950 it changed the depreciable value of said assets by increasing it to conform with the increase in cost for their replacement. Accordingly, from 1950 to 1953 it deducted from gross income the value of depreciation computed on the reappraised value. In 1953, the year involved in this case, taxpayer claimed the following depreciation deduction: Reappraised assets New assets consisting of hospital building and equipment Total depreciation P47,342.53 3,910.45 P51,252.98 Upon investigation and examination of taxpayer's books and papers, the Commissioner of Internal Revenue found that the reappraised assets depreciated in 1953 were the same ones upon which depreciation was claimed in 1952. And for the year 1952, the Commissioner had already determined, with taxpayer's concurrence, the depreciation allowable on said assets to be P36,842.04, computed on their acquisition cost at rates fixed by the taxpayer. Hence, the Commissioner pegged the deductible depreciation for 1953 on the same old assets at P36,842.04 and disallowed the excess thereof in the amount of P10,500.49. The question for resolution therefore is whether depreciation shall be determined on the acquisition cost or on the reappraised value of the assets. Depreciation is the gradual diminution in the useful value of tangible property resulting from wear and tear and normal obsolescense. The term is also applied to amortization of the value of intangible assets, the use of which in the trade or business is definitely limited in duration.2 Depreciation commences with the acquisition of the property and its owner is not bound to see his property gradually waste, without making provision out of earnings for its replacement. It is entitled to see that from earnings the value of the property invested is kept unimpaired, so that at the end of any given term of years, the original investment remains as it was in the beginning. It is not only the right of a company to make such a provision, but it is its duty to its bond and stockholders, and, in the case of a public service corporation, at least, its plain duty to the public.3 Accordingly, the law permits the taxpayer to recover gradually his capital investment in wasting assets free from income tax.4 Precisely, Section 30 (f) (1) which states: (1)In general. ² A reasonable allowance for deterioration of property arising out of its use or employment in the business or trade, or out of its not being used: Provided, That when the allowance authorized under this subsection shall equal the capital invested by the taxpayer . . . no further allowance shall be made. . . . allows a deduction from gross income for depreciation but limits the recovery to the capital invested in the asset being depreciated. The income tax law does not authorize the depreciation of an asset beyond its acquisition cost. Hence, a deduction over and above such cost cannot be claimed and allowed. The reason is that deductions from gross income are privileges,5 not matters of right.6 They are not created by implication but upon clear expression in the law.7

Moreover, the recovery, free of income tax, of an amount more than the invested capital in an asset will transgress the underlying purpose of a depreciation allowance. For then what the taxpayer would recover will be, not only the acquisition cost, but also some profit. Recovery in due time thru depreciation of investment made is the philosophy behind depreciation allowance; the idea of profit on the investment made has never been the underlying reason for the allowance of a deduction for depreciation. Accordingly, the claim for depreciation beyond P36,842.04 or in the amount of P10,500.49 has no justification in the law. The determination, therefore, of the Commissioner of Internal Revenue disallowing said amount, affirmed by the Court of Tax Appeals, is sustained. B. Expenses. ² The next item involves disallowed expenses incurred in 1953, broken as follows: Miscellaneous expenses Officer's travelling expenses Total P6,759.17 2,300.40 P9,059.57

These were disallowed on the ground that the nature of these expenses could not be satisfactorily explained nor could the same be supported by appropriate papers. Felix Gulfin, petitioner's accountant, explained the P6,759.17 was actual expenses credited to the account of the president of the corporation incurred in the interest of the corporation during the president's trip to Manila (pp. 33-34 of TSN of Dec. 5, 1962); he stated that the P2,300.40 was the president's travelling expenses to and from Manila as to the vouchers and receipts of these, he said the same were made but got burned during the Basilan fire on March 30, 1962 (p. 40 of same TSN). Petitioner further argues that when it sent its records to Manila in February, 1959, the papers in support of these miscellaneous and travelling expenses were not included for the reason that by February 9, 1959, when the Bureau of Internal Revenue decided to investigate, petitioner had no more obligation to keep the same since five years had lapsed from the time these expenses were incurred (p. 41 of same TSN). On this ground, the petitioner may be sustained, for under Section 337 of the Tax Code, receipts and papers supporting such expenses need be kept by the taxpayer for a period of five years from the last entry. At the time of the investigation, said five years had lapsed. Taxpayer's stand on this issue is therefore sustained. UNREASONABLY ACCUMULATED PROFITS Section 25 of the Tax Code which imposes a surtax on profits unreasonably accumulated, provides: Sec. 25. Additional tax on corporations improperly accumulating profits or surplus ² (a) Imposition of tax. ² If any corporation, except banks, insurance companies, or personal holding companies, whether domestic or foreign, is formed or availed of for the purpose of preventing the imposition of the tax upon its shareholders or members or the shareholders or members of another corporation, through the medium of permitting its gains and profits to accumulate instead of being divided or distributed, there is levied and assessed against such corporation, for each taxable year, a tax equal to twenty-five per centum of the undistributed portion of its accumulated profits or surplus which shall be in addition to the tax imposed by section twenty-four, and shall be computed, collected and paid in the same manner and subject to the same provisions of law, including penalties, as that tax.
1 awph îl.nèt

The Commissioner found that in violation of the abovequoted section, petitioner had unreasonably accumulated profits as of 1953 in the amount of P347,507.01, based on the following circumstances (Examiner's Report pp. 62-68 of BIR records): 1. Strong financial position of the petitioner as of December 31, 1953. Assets were P388,617.00 while the liabilities amounted to only P61,117.31 or a ratio of 6:1. 2. As of 1953, the corporation had considerable capital adequate to meet the reasonable needs of the business amounting to P327,499.69 (assets less liabilities). 3. The P200,000 reserved for electrification of drier and mechanization and the P50,000 reserved for malaria control were reverted to its surplus in 1953. 4. Withdrawal by shareholders, of large sums of money as personal loans. 5. Investment of undistributed earnings in assets having no proximate connection with the business ² as hospital building and equipment worth P59,794.72. 6. In 1953, with an increase of surplus amounting to P677,232.01, the capital stock was increased to P500,000 although there was no need for such increase. Petitioner tried to show that in considering the surplus, the examiner did not take into account the possible expenses for cultivation, labor, fertilitation, drainage, irrigation, repair, etc. (pp. 235-237 of TSN of Dec. 7, 1962). As aptly answered by the examiner himself, however, they were already included as part of the working capital (pp. 237-238 of TSN of Dec. 7, 1962). In the unreasonable accumulation of P347,507.01 are included P200,000 for electrification of driers and mechanization and P50,000 for malaria control which were reserved way back in 1948 (p. 67 of the BIR records) but reverted to the general fund only in 1953. If there were any plans for these amounts to be used in further expansion through projects, it did not appear in the records as was properly indicated in 1948 when such amounts were reserved. Thus, while in 1948 it was already clear that the money was intended to go to future projects, in 1953 upon reversion to the general fund, no such intention was shown. Such reversion therefore gave occasion for the Government to consider the same for tax purposes. The P250,000 reverted to the general fund was sought to be explained as later used elsewhere: "part of it in the Hilano Industries, Inc. in building the factory site and buildings to house technical men . . . part of it was spent in the facilities for the waterworks system and for industrialization of the coconut industry" (p. 117 of TSN of Dec. 6, 1962). This is not sufficient explanation. Persuasive jurisprudence on the matter such as those in the United States from where our tax law was derived,8has it that: "In order to determine whether profits were accumulated for the reasonable needs of the business or to avoid the surtax upon shareholders, the controlling intention of the taxpayer is that which is manifested at the time of the accumulation, not subsequently declared intentions which are merely the products of after-thought."9 The reversion here was made because the reserved amount was not enough for the projects intended, without any intent to channel the same to some particular future projects in mind. Petitioner argues that since it has P560,717.44 as its expenses for the year 1953, a surplus of P347,507.01 is not unreasonably accumulated. As rightly contended by the Government, there is no need to have such a large amount at the beginning of the following year because during the year, current assets are converted into cash and with the income realized from the business as the year goes, these expenses may well be taken care of (pp. 238 of TSN of Dec. 7, 1962). Thus, it is erroneous to say that the taxpayer is entitled to retain enough liquid net assets in amounts approximately equal to current operating needs for the year to cover "cost of goods sold and

operating expenses" for "it excludes proper consideration of funds generated by the collection of notes receivable as trade accounts during the course of the year."10 In fact, just because the fatal accumulations are less than 70% of the annual operating expenses of the year, it does not mean that the accumulations are reasonable as a matter of law."11 Petitioner tried to show that investments were made with Basilan Coconut Producers Cooperative Association and Basilan Hospital (pp. 103-105 of TSN of Dec. 6, 1962) totalling P59,794.72 as of December 31, 1953. This shows all the more the unreasonable accumulation. As of December 31, 1953 already P59,794.72 was spent ² yet as of that date there was still a surplus of P347,507.01. Petitioner questions why the examiner covered the period from 1948-1953 when the taxable year on review was 1953. The surplus of P347,507.01 was taken by the examiner from the balance sheet of petitioner for 1953. To check the figure arrived at, the examiner traced the accumulation process from 1947 until 1953, and petitioner's figure stood out to be correct. There was no error in the process applied, for previous accumulations should be considered in determining unreasonable accumulations for the year concerned. "In determining whether accumulations of earnings or profits in a particular year are within the reasonable needs of a corporation, it is neccessary to take into account prior accumulations, since accumulations prior to the year involved may have been sufficient to cover the business needs and additional accumulations during the year involved would not reasonably be necessary."12 Another factor that stands out to show unreasonable accumulation is the fact that large amounts were withdrawn by or advanced to the stockholders. For the year 1953 alone these totalled P197,229.26. Yet the surplus of P347,507.01 was left as of December 31, 1953. We find unacceptable petitioner's explanation that these were advances made in furtherance of the business purposes of the petitioner. As correctly held by the Court of Tax Appeals, while certain expenses of the corporation were credited against these amounts, the unspent balance was retained by the stockholders without refunding them to petitioner at the end of each year. These advances were in fact indirect loans to the stockholders indicating the unreasonable accumulation of surplus beyond the needs of the business. ALLEGED EXEMPTION Petitioner wishes to avail of the exempting proviso in Sec. 25 of the Internal Revenue Code as amended by R.A. 1823, approved June 22, 1957, whereby accumulated profits or surplus if invested in any dollar-producing or dollar-earning industry or in the purchase of bonds issued by the Central Bank, may not be subject to the 25% surtax. We have but to point out that the unreasonable accumulation was in 1953. The exemption was by virtue of Republic Act 1823 which amended Sec. 25 only on June 22, 1957 ² more than three years after the period covered by the assessment. In resume, Basilan Estates, Inc. is liable for the payment of deficiency income tax and surtax for the year 1953 in the amount of P88,977.42, computed as follows: Net Income per return Add: Over-claimed depreciation Net income per finding 20% tax on P50,643.39 P40,142.90 10,500.49 P50,643.39 P10,128.67

Less:

Tax already assessed

8,028.00 P2,100.67 86,876.75 P88,977.42 ===========

Deficiency income tax Add: 25% surtax on P347,507.01 Total tax due and collectible

WHEREFORE, the judgment appealed from is modified to the extent that petitioner is allowed its deductions for travelling and miscellaneous expenses, but affirmed insofar as the petitioner is liable for P2,100.67 as deficiency income tax for 1953 and P86,876.75 as 25% surtax on the unreasonably accumulated profit of P347,507.01. No costs. So ordered. Concepcion, C.J., Reyes, J.B.L., Dizon, Makalintal, Zaldivar, Sanchez, Castro, Angeles and Fernando, JJ., concur. Republic of the Philippines SUPREME COURT Manila EN BANC G.R. No. L-23534 May 16, 1967

JOSE A. ARCHES, petitioner-appellant, vs. ANACLETO I. BELLOSILLO and JAIME ARANETA, respondents-appellees. Jose A. Arches for petitioner-appellant. Office of the Solicitor General Arturo A. Alafriz, Solicitor A.B. Afurong and Atty. S.S. Soriano for respondents-appellees. BENGZON, J.P., J.: Petitioner-appellant Jose Arches filed on February 27, 1954 his income tax return for 1953. Within five years thereafter, or on February 26, 1959, deficiency income tax and residence tax assessments were issued against him. Said assessments not having been disputed, the Republic represented by the Bureau of Internal Revenue Regional, Director, filed suit on December 29, 1960, in the municipal court of Roxas City, to recover from petitioner-appellant the sum of P4,441.25 as deficiency income tax and additional residence tax for 1953. Arches then moved to dismiss the complaint on the ground that it did not expressly show the approval of the Revenue Commissioner, as required by Section 308 of the Tax Code, and on the further ground of prescription of the action.
1äwp hï1. ñët

The municipal court denied the motion. Petitioner-appellant, his motion to reconsider having been denied also, resorted to the Court of First Instance of Capiz on a petition for certiorari and prohibition assailing the order denying his motion to dismiss. The trial court dismissed the petition. Hence, this appeal.

The only question here is the correctness of dismissal of the petition by the Court of First Instance. The order was predicated upon the impropriety of the writ. We find no error committed by said court. The municipal court had jurisdiction over the parties and over the subject matter, the amount demanded being less than P5,000.00.1 The suit below instituted by the Republic, based on an uncontested assessment, was one merely for the recovery of a sum of money where the amount demanded constitutes the jurisdictional test.2 Petitioner-appellant would make much of the lack of approval of the Revenue Commissioner. First of all, in this case, such requisite is not jurisdictional, but one relating to capacity to sue or affecting the cause of action only.3So, in ruling on said question, whatever error ² if any ² the municipal court committed, was merely an error of judgment, not correctible by certiorari.4 Neither was there grave abuse of the discretion on the part of the municipal court in ruling that the express approval of the Revenue Commissioner himself was not necessary. The court relied upon Memorandum Order No. V-634 of the Revenue Commissioner, approved by the Finance Secretary of July 1, 1956, wherein the former's functions regarding the administration and enforcement of revenue laws and regulations ² powers broad enough to cover the approval of court actions as required in Section 308 of the Tax Code ² were expressly delegated to the Regional Directors. This regulation, the issuance of which was authorized by statute, has the force and effect of law.5 To rely upon it, hence, would not be tantamount to whimsical, capricious and arbitrary exercise of judgment. The verification by the Regional Director of the complaint constitutes sufficient approval thereof already. It states,inter alia, that said Director has caused the preparation of the complaint and that he has read the allegations thereof and they are true and correct to the best of his knowledge and belief. Pleadings are to be liberally construed.6 Assuming, therefore, in gratia argumenti, that the suit is being erroneously ² but not invalidly ² entertained, for lack of express approval of the Commissioner or the Regional Director, certiorari would still not lie. An order denying a motion to dismiss is interlocutory and the remedy of the unsuccessful movant is to await the judgment on the merits and then appeal therefrom.7 And, as the Court of First Instance rightly observed, there was no showing of a special reason or urgent need to stop the proceedings at such early stage in the municipal court. Petitioner-appellant would also raise the question of prescription. Again, this is not jurisdictional. And, We have already ruled8 that the proper prescriptive period for bringing civil actions is five years from the date of the assessment, under Section 332 of the Tax Code. The three-year period urged by petitioner-appellant under Section 51 (d) refers only to the summary remedies of distraint and levy. Here, the action was commenced one year, ten months and three days after the assessments were made; hence, well within the period. Wherefore, the dismissal of appellant's petition for certiorari by the Court of First Instance is hereby affirmed. Costs against petitioner-appellant. So ordered. Concepcion, C.J., Reyes, J.B.L., Dizon, Regala, Makalintal, Zaldivar, Sanchez and Castro, JJ., concur. Footnotes
1

Sec. 88, Judiciary Act, as amended by Sec. 10, Rep. Act No. 2613.

2

Ventanilla vs. B.T.A., L-7384, Dec. 19, 1955; Republic vs. Gamboa, L- 16504, Oct. 27, 1961.
3

In this it differs from petitions for adoption or guardianship, where notice to or consent of specified persons is jurisdictional, since the latter are specified proceedings, in rem, whereas the suit for collection in question is a simple money claim.
4

Gala vs. Cui, 25 Phil. 522; Villa Rey Transit vs. Hon. Bello, L-18957, April 23, 1963; J.R.S. Business Corp. vs. Imperial Insurance, L-19891, July 31, 1964.
5

Art. 7, Civil Code; Re Huttman, 70 Fed. 669; Re Weeks, 82 Fed. 729; Gratiot vs. U.S., 4 How. 80; U.S. vs. Elliason, 16 Pet. 291.
6

Rule 6, Sec. 15, Rev. Rules of Court.

7

Harrison Foundry & Machinery vs. Harrison Foundry Worker's Assn., L-18432, June 20, 1963; Bautista vs. de la Cruz, L-21107, Dec. 24, 1963; 3 Moran, 1953 ed., pp. 152-153.
8

Republic vs. Ledesma, L-18759, Feb. 28, 1967.

ATLAS CONSOLIDATED MINING

[G.R. No. 31230-32. February 14, 2000] COMMISSIONER OF INTERNAL REVENUE vs. CMS LOGGING, INC., et al. THIRD DIVISION Gentlemen: Quoted hereunder, for your information, is a resolution of this Court dated FEB 14 2000. G.R. No. 31230-32 (Commissioner of Internal Revenue vs. CMS Logging, Inc. and Court of Tax Appeals.) Assailed in this petition for review is the decision of the Court of Tax Appeals (CTA) promulgated on August 30, 1969 in CTA Cases Nos. 1569, 1674 and 1804 entitled "CMS Logging, Inc. vs. Commissioner of Internal Revenue." CMS Logging, Inc., a domestic corporation, is a duly licensed forest concessionaire operating in Baganga, Davao. Pursuant to Section 5 of republic Act (RA) No. 1435, CMS Logging filed several claims for tax refund for 25% of the specific taxes on fuel and lubricants used by it in its logging operation, enumerated as follows:

1. On June 21, 1962, the sum of P4,894.63, covering the period from July 1 to December 31, 1961; 2. On November 5, 1962, the sums of P6,294.26 covering the period from January 1, to June 30, 1962; 3. On August 6, 1963, the sum of P5,998.46, covering the period from July 1 to December 31, 1962. 4. On January 20, 1964, the sum of P4,918.00, covering the period from January 1, to June 30, 1963; 5. On September 25, 1964, the sum of P4,932.96, covering the period from July 1 to December 31, 1963; 6. On May 31, 1965, the sum P5,859.84, covering the period from January 1 to July 31, 1964; 7. On June 16, 1965, the sum of P5,707.01, covering the period from August 1, 1964 to February 23, 1965; 8. On September 27, 1966, the sum of P6,135.24 covering the period from March 1 to August 31, 1965. The first five claims and the seventh were denied by the Commissioner of Internal Revenue in six separate letters addressed to private respondent. However, no letters of denial were received by private as to the sixth and eight claims. Private respondent¶s motion for reconsideration of the denials of the first five claims were similarly rejected. Private respondent filed three separate cases with the CTA, which were docketed as CTA Cases Nos. 1569, 1674 and 1804.1 CTA Cases Nos. 1569, 1674 and 1804 were filed on February 8, 1965, August 5, 1965 and October 6, 1966, respectively. CTA Case No. 1569 involved the first five claims, the sixth and seventh claims constituted CTA Case No. 1674 and the eighth claim was dealt with CTA Case No. 1804.2 CTA Decision, 1-2;Rollo, 8283.

After a joint hearing, the Court of Tax Appeals rendered the assailed decision, the dispositive portion of which provides ± IN VIEW OF THE FOREGOING, the petitions for review filed by petitioner in CTA Case No. 1569 is dismissed with regard to the 1st, 2nd, 3rd, 5th, and part of the 4th causes of action. Respondent is hereby ordered to refund or grant a tax credit to

petitioner [in] the sums of P3,893.44, corresponding to the period from February 8 to June 30, 1963 in CTA Case No. 1569; P11,566.85 in CTA Case No. 1674; and P6,135.25 in CTA Case No. 1804, representing 25% of the specific taxes paid on manufactured oils and other fuels. Without pronouncement as to costs. SO ORDERED. Both parties appealed to this Court from CTA¶s decision.3 The appeal of private respondent was docketed as G.R. No. 31140-42. However, in our Resolution dated February 21, 1996, we dismissed private respondent¶s petition for lack of interest to prosecute, which dismissal became final and executory on June 5, 1996. Petitioner, however, in compliance with our December 15, 1999 Resolution, manifested on January 7, 2000 that he is still interested in continuing his appeal since there have been no supervening events which would render the present case moot and academic.4 Rollo, 180-181. We are therefore resolving the petition filed by the Commissioner of Internal Revenue. The parties agree that the sole issue in this case is whether the 25% specific tax exemption granted by Section 5 of RA 1435 on manufactured oils and other fuels used by miners and forest concessionaires in their operation is limited to a period of five (5) years from the effectivity of RA 1435 on June 14, 1956.5 Petition, 4; Rollo, 9; Answer,
1 Rollo, 100.

Republic Act No. 1435, entitled "An Act To Provide Means For Increasing The Highway Special Fund," was approved and took effect on June 14, 1956. Its full text is reproduced herein: Sec. 1. Section one hundred and forty-two of the National Internal Revenue Code, as amended, is further amended to read as follows: "Sec. 42. Specific Tax on manufactured oils and other fuels. ± On refined and manufactured mineral oils and motor fuels, there shall be collected the following taxes: "(a) Kerosene or petroleum, per liter of volume capacity, two and one-half centavos; "(b) Lubricating oils, per liter of volume capacity, seven centavos;

"(c) Naptha, gasoline, and all other similar products of distillation, per liter of volume capacity, eight centavos; and "(d) On denatured alcohol to be used for motive power, per liter of volume capacity, one centavo: Provided, That if the denatured alcohol is mixed with gasoline, the specific tax on which has already been paid, the purpose of this subsection, the removal of denatured alcohol of not less than one hundred eighty degrees proof (ninety percentum absolute alcohol) shall be deemed to have been removed for motive power, unless shown to the contrary. "Whenever any of the oils mentioned above are, during the five years from June eighteen, nineteen hundred and fifty-two, used in agriculture and aviation, fifty percentum of the specific tax paid thereon shall be refunded by the Collector of Internal Revenue upon the submission of the following: "(1) A sworn affidavit of the producer and two disinterested persons proving that the said oils were actually used in agriculture, or in lieu thereof. "(2) Should the producer belong to any producer¶s association or federation, duly registered with the Securities and Exchange Commission, the affidavit of the president of the association or federation, attesting to the fact that the oils were actually used in agriculture. "(3) In the case of aviation oils, a sworn certificate satisfactory to the Collector proving that the said oils were actually used in aviation: Provided, that no such refunds shall be granted in respect to the

oils used in aviation by citizens and corporations of foreign countries which do not grant equivalent refunds or exemptions in respect to similar oils used in aviation by citizen and corporation of the Philippines." Sec. 2. Section one hundred and forty-five of the National Internal Revenue Code, as amended, is further amended to read as follows: "Sec. 145. Specific Tax on Diesel fuel oil. - On fuel oil commercially known as diesel fuel oil, and on all similar fuel oils, having more or less the same generating power, there shall be collected, per metric ton, one peso." Sec. 3. The proceed of the increased taxes accruing to the Highway Special Fund, as a result of the amendment of sections one hundred and forty-two and one hundred and forty-five of the National Internal revenue Code as above provided, shall be set aside exclusively for amortizing loans or bonds that may have been authorized for the construction, reconstruction or improvement for highways including bridges as well as for liquidating toll bridges constructed from revolving funds authorized under Act Numbered Thirty-five hundred, as amended, whenever such liquidation is recommended by the Secretary of Public Works and Communication and approved by the President. Sec. 4. Municipal boards or councils may, notwithstanding the provisions of sections one hundred and forty-two and one hundred and forty-five of the National Internal Revenue Code, as hereinabove amended, levy an additional tax of not exceeding twenty-five per cent of the rates fixed in said sections, on manufactured oils sold or distributed within the limits of the city or the municipality: Provided, That municipal taxes heretofore levied by cities through city ordinances on gasoline, airplane fuel, lubricating oil and other fuels, are hereby ratified and declared valid. The method of collecting said additional tax shall be prescribe by the municipal board or council concerned. Sec. 5. The proceeds of the additional tax on manufactured oils shall accrue to the road and bridge funds of the political

subdivision for whose benefit the tax is collected. Provided, however, That whenever any oils mentioned above are used by miners or forest concessionaires in their operations, twenty-five per centum of the specific tax paid thereon shall be refunded by the Collector of Internal Revenue upon submission of proof of actual use of oils and under similar conditions enumerated in subparagraphs one and two of section one hereof, amending section one hundred forty-two of the Internal Revenue Code: Provided, further, That no new road shall be constructed unless the routes or location thereof shall have been approved by the Commissioner of Public Highways after a determination that such road can be made part of an integral and articulated route in the Philippine Highway System, as required in section twenty-six of the Philippine Highway Act of 1953. Sec. 6. This Act shall take effect upon its approval. Approved, June 14, 1956. It is petitioner¶s contention that the 25% tax refund of specific tax paid on oils used by miners or forest concessionaires granted by Section 5 RA 1435 should be limited to a period of five years to be counted from June 14, 1956, the date of effectivity of RA 1435, as is the case in the partial tax refund of specific tax paid on oils used in agriculture and aviation granted by Section 142 of the National Internal Revenue Code, as amended by Section 1 of RA 1435. Petitioner maintains that the Court of Tax Appeals erred in ordering him to grant tax refunds to private respondents corresponding to the period from February 8, 1963 to August 31, 1965 since said period is obviously beyond the five-year period.6 Petition, 4-5; Rollo, 9-10. The petition must fail. This issue has already been resolved in Insular Lumber Co. vs. Court of Tax Appeals,7 104 SCRA 710 (1981). which is squarely applicable to the case at bench. In Insular Lumber, a licensed forest concessionaire filed a claim with the Commissioner of Internal revenue for refund of P19,921.37 representing 25% of the specific tax paid on manufactured oil and fuel used in its operations pursuant to Section 5 of RA 1435. The Commissioner denied the company¶s claim for refund on the ground that the privileged of partial tax refund granted by Section 5 of RA 1435 to those using oil in the operation of forest and mining concessions is limited to a period of five (5) years from June 14, 1956, the date of effectivity of said law. Thus, oil used in such concessions after June 14, 1961 are subject to the full tax prescribe in Section 142 of the National Internal Revenue Code (NIRC). In passing upon this issue, the Court held that ±

Based on the aforequoted provisions, it is very apparent that the partial refund of specific tax paid for oils in agriculture and aviation is limited to five years while there is no time limit for the partial refund of specific tax paid for oils used by miners and forest concessionaires. We find no basis in applying the limitation of the operative period provided for oils used in agriculture and aviation to the provisions on the refund to miners and forest concessionaires. It should be noted that Section 5 makes reference to subparagraphs 1 and 2 of Section 1 only for the purpose of prescribing the procedure for refund. This express reference cannot be expanded in scope to include the limitation to include the limitation of the period of refund. If the limitations of the period of refund of specific taxes on oils used in aviation and agriculture is intended to cover similar taxes paid on oil use by miners and forest concessionaires, there would have been no need of dealing with oil used in mining and forest concessions separately and Section 5 should very well have been included in Section 1 of Republic Act No. 1435, notwithstanding the different rate of exemption.8 Id., 718719.

In fact, in a recent case we declared that mining and logging companies were entitled to the refund privilege granted by RA 1435 on specific taxes paid up to 1985, after which the Highway Special Fund was abolished.9 Commissioner of Internal Revenue vs.
Rio Tuba Nickel Mining Corporation, 207 SCRA 549 (1992).

The rationale for extending a tax privilege to lumber and mining companies was explained in Commissioner of Internal Revenue vs. Rio Tuba Nickel Mining Corporation.10 202 SCRA 137 (1991). Commissioner of Internal Revenue vs. Rio Tuba Nickel Mining Corporation, 207 SCRA 549 (1992). Citing the congressional deliberations on RA 1435, we explained that it would be unfair to subject miners and forest concessionaires to the increased rates and in effect make them subsidize the construction of highways from which they did not directly benefit since these companies seldom used the national highways because they have roads and compounds of their own.11 Id., citing Congressional
record, 3rd Congress, 3rd Regular Session, 7, 1967, Vol. III, No. 67, pp. 2093-2107.

WHEREFORE, the petition is DENIED. The challenged decision of the Court of Tax Appeals in CTA Cases Nos. 1569, 1674, and 1804 is AFFIRMED. SO ORDERED. Very truly yours,

(Sgd.) JULIETA Y. CARREON Clerk of Court
Republic of the Philippines SUPREME COURT Manila THIRD DIVISION

G.R. No. 115712 February 25, 1999 COMMISSIONER OF INTERNAL REVENUE, petitioner, vs. COURT OF APPEALS, COURT OF TAX APPEALS and CARNATION PHILIPPINES, INC. (now merged with Nestle Phils, Inc.), respondent.

PURISIMA, J.: Before the Court is an appeal from the decision of the Court of Appeals 1 dated May 31, 1994, which affirmed in toto the decision of the Court of Tax Appeals 2 dated January 26, 1993, the dispositive portion of which reads:
WHEREFORE, the Court, finds the assessments for allegedly deficient income and sales taxes for petitioner's fiscal year ending September 30, 1981 covered by Demand Letter NO. FAS-1B-81-87 and Assessment Notices Nos. FAS-1-81-87-005824, FAS-4-81-87005825 and FAS-4-81-87-005826 (all dated July 29, 1987) in the total amount of P19,535,183.44 to be NULL AND VOID for having been issued beyond the five-year prescriptive period provided by law. 3

The undisputed facts of the case as recited in the Decision (Annex "A") of the Court of Appeals, are: 4
On January 15, 1982, Carnation Phils. Inc. (Carnation), filed its Corporation Annual Income Tax Return for taxable year ending September 30, 1981; and its Manufacturers/Producers Percentage Tax Return for the quarter ending September 30, 1981. 5

On October 13, 1986, March 16, 1987 and May 18, 1987, Carnation, through its Senior Vice President Jaime O. Lardizabal, signed three separate "waivers of the Statute of Limitations Under the National Internal Revenue Code" wherein it: . . . waives the running of the prescriptive period provided for in sections 318 and 319 and other related provisions of the National Internal Revenue Code and consents to the assessment and collection of the

taxes which may be found due after reinvestigation and reconsideration at anytime before or after the lapse of the period of limitations fixed by said sections 318 and 319 and other relevant provisions of the National Internal Revenue Code, but not after (13 April 1987 for the earlier-executed waiver, or June 14, 1987 for the later waiver, or July 30, 1987 for the subsequent waiver, as the case may be). However, the taxpayer (petitioner herein) does not waive any prescription already accrued in its favor. The waivers were not signed by the BIR Commissioner or any of his agents. On August 5, 1987, Carnation received BIR's letter of demand dated July 29, 1987 asking the said corporation to pay P1,442,586.56 as deficiency income tax, P14,152,683.85 as deficiency sales tax and P3,939,913.03 as deficiency sales tax on undeclared sales, all for the year 1981. This demand letter was accompanied by assessment Notices Nos. FAS-4-81-87-005824, FAS-4-81-87-005825 and FAS-481-87-005826. In a basic protest dated August 17, 1987, Carnation disputed the assessments and requested a reconsideration and reinvestigation thereof. On September 30, 1987, Carnation filed a supplemental protest. These protests were denied by the BIR Commissioner in a letter dated March 15, 1988. Whereupon, Carnation appealed to the CTA. On January 26, 1993, the CTA issued the questioned order, the dispositive portion of which reads: WHEREFORE, the Court finds the assessments for allegedly deficient income and sales taxes for petitioner's fiscal year ending September 30, 1981 covered by Demand Letter No. FAS-1B-81-87 and assessment Notices No. FAS-1-81-87-005824, FAS4-81-87-005825, and FAS-4-81-87-005826 (all dated July 29, 1987) in the total amount of P19,535,183.44 to be NULL AND VOID for having been issued beyond the five-year prescriptive period provided by law. The pivot of inquiry here is whether or not the three (3) waivers signed by the private respondent are valid and binding 6 as to toll the running of the prescriptive period for assessment and not bar the Government from issuing subject deficiency tax assessments. Sec. 318 (now Section 203) of the National Internal Revenue Code, the law then applicable reads:
Sec 318. Period of Limitations upon assessment and collection. ² Except as provided in the succeeding section, internal revenue taxes shall be assessed within five years after

the return was filed, and no proceeding in court without assessment for the collection of such taxes shall be begun after the expiration of such period. For the purpose of this section, a return filed before the last day prescribed by law for the filing thereof shall be considered as filed on such last day: Provided, That this limitation shall not apply to cases already investigated prior to the approval of this Code. 7 (emphasis ours)

The decision of the Court of Appeals affirming what the Court of Tax Appeals decided, established that subject assessments of July 29, 1987 were issued outside the statutory prescriptive period. Carnation filed its annual income tax and percentage tax returns for the fiscal year ending September 30, 1981 on January 15, 1982 8 and November 20, 1981, 9 respectively. In accordance with the above-quoted provision of law, private respondent's 1981 income and sales taxes could have been validly assessed only until January 14, 1987 and November 19, 1986, respectively. 10 However, Carnation's income and sales taxes were assessed only on July 29, 1987, beyond the five-year prescriptive period. 11 Petitioner BIR Commissioner contends that the waivers signed by Carnation were valid although not signed by the BIR Commissioner because (a) when the BIR agents/examiners extended the period to audit and investigate Carnation's tax returns, the BIR gave its implied consent to such waivers; (b) the signature of the Commissioner is a mere formality and the lack of it does not vitiate binding effect of the waivers; and (c) that a waiver is not a contract but a unilateral act of renouncing ones right to avail of the defense of prescription and remains binding in accordance with the terms and conditions set forth in the waiver. 12 Petitioner's submission is inaccurate. The same tax code is clear on the matter, to wit: Sec. 319. Exceptions as to period of limitation of assessment and collection of taxes. ²(a) . . . (b) Where before the expiration of the time prescribed in the preceding section for the assessment of the tax, both the Commissioner of Internal Revenue and the taxpayer have consented in writing to its assessment after such time, the tax may be assessed at anytime prior to the expiration of the period agreed upon. The period so agreed upon may be extended by subsequent agreement in writing made before the expiration of the period previously agreed upon. The Court of Appeals itself also passed upon the validity of the waivers executed by Carnation, observing thus: We cannot go along with the petitioner's theory. Section 319 of the Tax code earlier quoted is clear and explicit that the waiver of the five-year prescriptive period must be in writing and signed by both the BIR Commissioner and the taxpayer. Here, the three waivers signed by Carnation do not bear the written consent of the BIR Commissioner as required by law. We agree with the CTA in holding "these "waivers" to be invalid and without any binding effect on petitioner (Carnation) for the reason that there was no consent by the respondent (Commissioner of Internal Revenue)."
The ruling of the Supreme Court in Collector of Internal Revenue vs. Solano 13 is in point, thus:

. . . The only agreement that could have suspended the running of the prescriptive period the collection of the tax in question is, as correctly pointed out by the Court of Tax Appeals, a written agreement between Solano and the Collector, entered into before the expiration of the of the five-year prescriptive period, extending the limitation prescribed by law.
For sure, no such written agreement concerning the said three waivers exists between the petitioner and private respondent Carnation. 14

Verily, we discern no basis for overruling the aforesaid conclusions arrived at by the Court of Appeals. In fact, there is every reason to leave undisturbed the said conclusions, having in mind the precept that all doubts as to the correctness of such conclusions will be resolved in favor of the Court of Appeals. 15 Besides being a reiteration of the holding of the Court of Tax Appeals, such decision should be accorded respect. Thus, the Court held in Philippine Refining Co. vs. Court of Appeals, 16 that the Court of Tax Appeals is a highly specialized body specifically created for the purpose of reviewing tax cases. As a matter of principle, this Court will not set aside the conclusion reached by an agency such as the Court of Tax Appeals which is, by the very nature of its function, dedicated exclusively to the study and consideration of tax problems and has necessarily developed an expertise on the subject, unless there has been an abuse or improvident exercise of authority. 17 This point becomes more evident in the case under consideration where the findings and conclusions of bath the Court of Tax Appeals and the Court of Appeals appear untainted by any abuse of authority, much less grave abuse of discretion. Indeed, we find the decision of the latter affirming that of the former free from any palpable error. 18 What is more, the waivers in question reveal that they are in no wise unequivocal, and therefore necessitates for its binding effect the concurrence of the Commissioner of Internal Revenue. In fact, in his reply dated April 18, 1995, the Solicitor General, representing the Commissioner of Internal Revenue, admitted that subject waivers executed by Carnation were "for end in consideration of the approval by the Commissioner of Internal Revenue of its request for reinvestigation and/or reconsideration of its internal revenue case involving tax assessments for the fiscal year ended September 30, 1981 which were all pending at the time". On this basis neither implied consent can be presumed nor can it be contended that the waiver required under Sec. 319 of the Tax Code is one which is unilateral nor can it be said that concurrence to such an agreements a mere formality because it is the very signatures of both the Commissioner of Internal Revenue and the taxpayer which give birth to such a valid agreement. WHEREFORE, the decision of the Court of Appeals is hereby AFFIRMED. No pronouncement as to costs. SO ORDERED. Romero, Panganiban and Gonzaga-Reyes, JJ., concur. Vitug, J, abroad on official business. Republic of the Philippines SUPREME COURT Manila THIRD DIVISION

G.R. No. 104171 February 24, 1999 COMMISSIONER OF INTERNAL REVENUE, petitioner, vs. B.F. GOODRICH PHILS., INC. (now SIME DARBY INTERNATIONAL TIRE CO., INC.) and THE COURT OF APPEALS, respondents.

PANGANIBAN, J.: Notwithstanding the expiration of the five-year prescriptive period, may the Bureau of Internal Revenue (BIR) still assess a taxpayer even after the latter has already paid the tax due, on the ground that the previous assessment was insufficient or based on a "false" return? The Case This is the main question raised before us in this Petition for Review on Certiorari assailing the Decision 1 dated February 14, 1992, promulgated by the Court of Appeals 2 in CA-GR SP No. 25100. The assailed Decision reversed the Court of Tax Appeals (CTA) 3 which upheld the BIR commissioner's assessments made beyond the five-year statute of limitations. The Facts The facts undisputed. 4 Private Respondent BF Goodrich Phils., Inc. (now Sime Darby International Tire Co, Inc.), was an American-owned and controlled corporation previous to July 3, 1974. As a condition for approving the manufacture by private respondent of tires and other rubber products, the Central Bank of the Philippines required that it should develop a rubber plantation. In compliance with this requirement, private respondent purchased from the Philippine government in 1961, under the Public Land Act and the Parity Amendment to the 1935 Constitution, certain parcels of land located in Tumajubong, Basilan, and there developed a rubber plantation. More than a decade later, on August 2, 1973, the justice secretary rendered an opinion stating that, upon the expiration of the Parity Amendment on July 3, 1974, the ownership rights of Americans over public agricultural lands, including the right to dispose or sell their real estate, would be lost. On the basis of this Opinion, private respondent sold to Siltown Realty Philippines, Inc. on January 21, 1974, its Basilan landholding for P500,000 payable in installments. In accord with the terms of the sale, Siltown Realty Philippines, Inc. leased the said parcels of land to private respondent for a period of 25 years, with an extension of another 25 years at the latter's option. Based on the BIR's Letter of Authority No. 10115 dated April 14, 1975, the books and accounts of private respondent were examined for the purpose of determining its tax liability for taxable year 1974. The examination resulted in the April 23, 1975 assessment of private respondent for deficiency income tax in the amount of P6,005.35, which it duly paid. Subsequently, the BIR also issued Letters of Authority Nos. 074420 RR and 074421 RR and Memorandum Authority Reference No. 749157 for the purpose of examining Siltown's business, income and tax liabilities. On the basis of this examination, the BIR commissioner issued against private respondent on October 10, 1980, an assessment for deficiency in donor's tax in the amount

of P1,020,850, in relation to the previously mentioned sale of its Basilan landholdings to Siltown. Apparently, the BIR deemed the consideration for the sale insufficient, and the difference between the fair market value and the actual purchase price a taxable donation. In a letter dated November 24, 1980, private respondent contested this assessment. On April 9, 1981, it received another assessment dated March 16, 1981, which increased to P 1,092,949 the amount demanded for the alleged deficiency donor's tax, surcharge, interest and compromise penalty. Private respondent appealed the correctness and the legality of these last two assessments to the CTA. After trial in due course, the CTA rendered its Decision dated March 29, 1991, the dispositive portion of which reads as follows: WHEREFORE, the decision of the Commissioner of Internal Revenue assessing petitioner deficiency gift tax is MODIFIED land petitioner is ordered to pay the amount of P1,311,179.01 plus 10% surcharge and 20% annual interest from March 16, 1981 until fully paid provided that the maximum amount that may be collected as interest on delinquency shall in no case exceed an amount corresponding to a period of three years pursuant to Section 130(b)(l) and (c) of the 1977 Tax Code, as amended by P.D. No. 1705, which took effect on August 1, 1980.
SO ORDERED. 5

Undaunted, private respondent elevated the matter to the Court of Appeals, which reversed the CTA, as follows:
What is involved here is not a first assessment; nor is it one within the 5-year period stated in Section 331 above. Since what is involved in this case is a multiple assessment beyond the five-year period, the assessment must be based on the grounds provided in Section 337, and not on Section 15 of the 1974 Tax Code. Section 337 utilizes the very specific terms "fraud, irregularity, and mistake". "Falsity does not appear to be included in this enumeration. Falsity suffices for an assessment, which is a first assessment made within the five-year period. When it is a subsequent assessment made beyond the fiveyear period, then, it may be validly justified only by "fraud, irregularity and mistake" on the part of the taxpayer. 6

Hence, this Petition for Review under Rule 45 of the Rules of Court. 7 The Issues Before us, petitioner raises the following issues: I Whether or not petitioner's right to assess herein deficiency donor's tax has indeed prescribed as ruled by public respondent Court of Appeals II Whether or not the herein deficiency donor's tax assessment for 1974 is valid and in accordance with law

Prescription is the crucial issue in the resolution of this case. The Court's Ruling The petition has no merit. Main Issue: Prescription The petitioner contends that the Court of Appeals erred in reversing the CTA on the issue of prescription, because its ruling was based on factual findings that should have been left undisturbed on appeal, in the absence of any showing that it had been tainted with gross error or grave abuse of discretion. 8 The Court is not persuaded. True, the factual findings of the CTA are generally not disturbed on appeal when supported by substantial evidence and in the absence of gross error or grave abuse of discretion. However, the CTA's application of the law to the facts of this controversy is an altogether different matter, for it involves a legal question. There is a question of law when the issue is the application of the law to a given set of facts. On the other hand, a question of fact involves the truth or falsehood of alleged facts. 9 In the present case, the Court of Appeals ruled not on the truth or falsity of the facts found by the CTA, but on the latter's application of the law on prescription. Sec. 331 of the National Internal Revenue Code provides: Sec. 331. Period of limitation upon assessment and collection. ² Except as provided in the succeeding section, internal-revenue taxes shall be assessed within five years after the return was filed, and no proceeding in court without assessment for the collection of such taxes shall be begun after expiration of such period. For the purposes of this section, a return filed before the last day prescribed by law for the filing thereof shall be considered as filed on such last day: Provided, That this limitation shall not apply to cases already investigated prior to the approval of this Code. Applying this provision of law to the facts at hand, it is clear that the October 16, 1980 and the March 1981 assessments were issued by the BIR beyond the five-year statute of limitations. The Court has thoroughly studied the records of this case and found no basis to disregard the five-year period of prescription. As succinctly pronounced by the Court of Appeals:
The subsequent assessment made by the respondent Commissioner on October 40, 1980, modified by that of March 16, 1981, violates the law. Involved in this petition is the income of the petitioner for the year 1974, the returns for which were required to be filed on or before April 15 of the succeeding year. The returns for the year 1974 were duly filed by the petitioner, and assessment of taxes due for such year ² including that on the transfer of properties on June 21, 1974 ² was made on April 13, 1975 and acknowledged by Letter of Confirmation No. 101155 terminating the examination on this subject. The subsequent assessment of October 10, 1980 modified, by that of March 16, 1981, was made beyond the period expressly set in Section 331 of the National Internal Revenue Code . . . . 10

Petitioner relies on the CTA ruling, the salient portion of which reads: Falsity is what we have here, and for that matter, we hasten to add that the second assessment (March 16, 1981) of the Commissioner was well-advised having been

made in contemplation of his power under Section 15 of the 1974 Code (now Section 16, of NIRC) to assess the proper tax on thebest evidence obtainable "when there is reason to believe that a report of a taxpayer is false, incomplete or erroneous. More, when there is falsity with intent to evade tax as in this case, the ordinary period of limitation upon assessment and collection does not apply so that contrary to the averment of petitioner, the right to assess respondent has not prescribed.
What is the considered falsity? The transfer through sale of the parcels of land in Tumajubong, Lamitan, Basilan in favor of Siltown Realty for the sum of P500,000.00 only whereas said lands had been sworn to under Presidential Decree No. 76 (Dec. 6, 1972) as having a value of P2,683,467 (P2,475,467 + P207,700) (seeDeclaration of Real Property form, p. 28, and p. 15, no. 5, BIR Record). 11

For the purpose of safeguarding taxpayers from any unreasonable examination, investigation or assessment, our tax law provides a statute of limitations in the collection of taxes. Thus, the law on prescription, being a remedial measure, should be liberally construed in order to afford such protection. 12 As a corollary, the exceptions to the law on prescription should perforce be strictly construed. Sec. 15 of the NIRC, on the other hand, provides that "[w]hen a report required by law as a basis for the assessment of any national internal revenue tax shall not be forthcoming within the time fixed by law or regulation, or when there is reason to believe that any such report is false, incomplete, or erroneous, the Commissioner of Internal Revenue shall assess the proper tax on the best evidence obtainable." Clearly, Section 15 does not provide an exception to the statute of limitations on the issuance of an assessment, by allowing the initial assessment to be made on the basis of the best evidence available. Having made its initial assessment in the manner prescribed, the commissioner could not have been authorized to issue, beyond the five-year prescriptive period, the second and the third assessments under consideration before us. Nor is petitioner's claim of falsity sufficient to take the questioned assessments out of the ambit of the statute of limitations. The relevant part of then Section 332 of the NIRC, which enumerates the exceptions to the period of prescription, provides: Sec. 332. Exceptions as to period of limitation of assessment and collection of taxes. ² (a) In the case of a false or fraudulent return with intent to evade a tax or of a failure to file a return, the tax may be assessed, or a proceeding in court for the collection of such tax may be begun without assessment, at any time within ten years after the discovery of the falsity, fraud, or omission: . . . . Petitioner insists that private respondent committed "falsity" when it sold the property for a price lesser than its declared fair market value. This fact alone did not constitute a false return which contains wrong information due to mistake, carelessness or ignorance. 13 It is possible that real property may be sold for less than adequate consideration for a bona fide business purpose; in such event, the sale remains an "arm's length" transaction. In the present case, the private respondent was compelled to sell the property even at a price less than its market value, because it would have lost all ownership rights over it upon the expiration of the parity amendment. In other words, private respondent was attempting to minimize its losses. At the same time, it was able to lease the property for 25 years, renewable for another 25. This can be regarded as another consideration on the price. Furthermore, the fact that private respondent sold its real property for a price less than its declared fair market value did not by itself justify a finding of false return. Indeed, private respondent declared the sale in its 1974 return submitted to the BIR. 14 Within the five-year prescriptive period, the BIR

could have issued the questioned assessment, because the declared fair market value of said property was of public record. This it did not do, however, during all those five years. Moreover, the BIR failed to prove that respondent's 1974 return had been filed fraudulently. Equally significant was its failure to prove respondent's intent to evade the payment of the correct amount of tax. Ineludibly, the BIR failed to show that private respondent's 1974 return was filed fraudulently with intent to evade the payment of the correct amount of tax. 15 Moreover, even though a donor's tax, which is defined as "a tax on the privilege of transmitting one's property or property rights to another or others without adequate and full valuable consideration," 16 is different from capital gains tax, a tax on the gain from the sale of the taxpayer's property forming part of capital assets, 17 the tax return filed by private respondent to report its income for the year 1974 was sufficient compliance with the legal requirement to file a return. In other words, the fact that the sale transaction may have partly resulted in a donation does not change the fact that private respondent already reported its income for 1974 by filing an income tax return. Since the BIR failed to demonstrate clearly that private respondent had filed a fraudulent return with the intent to evade tax, or that it had failed to file a return at all, the period for assessments has obviously prescribed. Such instances of negligence or oversight on the part of the BIR cannot prejudice taxpayers, considering that the prescriptive period was precisely intended to give them peace of mind. Based on the foregoing, a discussion of the validity and legality of the assailed assessments has become moot and unnecessary. WHEREFORE, the Petition for Review is DENIED and the assailed Decision of the Court of Appeals is AFFIRMED. No costs. SO ORDERED. Romero, Purisima and Gonzaga-Reyes, JJ., concur. Vitug, J., on official leave. Republic of the Philippines SUPREME COURT Manila EN BANC G.R. No. L-11527 November 25, 1958

THE COLLECTOR OF INTERNAL REVENUE, petitioner, vs. SUYOC CONSOLIDATED MINING COMPANY, ET AL., respondents. Office of the Solicitor General Ambrosio Padilla and Solicitor Sumilang V. Bernardo for petitioner. Ohnick, Velilla and Balongkita for respondents. BAUTISTA ANGELO, J.:

Suyoc Consolidated Mining Company, a mining corporation operating before the war, was unable to file in 1942 its income tax return for the year 1941 due to the last war. After liberation, Congress enacted Commonwealth Act No. 722 which extended the filing of tax returns for 1941 up to December 31, 1945. Its records having been lost or destroyed, the company requested the Collector of Internal Revenue to grant it an extension of time to file its return, which was granted until February 15, 1946, and the company was authorized to file its return for 1941 on the basis of the best evidence obtainable. The company filed three income tax returns for the calendar year ending December 31, 1941. On February 12, 1946, it filed a tentative return as it had not yet completely reconstructed its records. On November 28, 1946, it filed a second final return on the basis of the records it has been able to reconstruct at that time. On February 6, 1947, it filed its third amended final return on the basis of the available records which to that date it had been able to reconstruct. On the basis of the second final return filed by the company on November 28, 1946, the Collector assessed against it the sum of P28,289.96 as income tax for 1941, plus P1,414.50 as 5 per cent surcharge and P3,894.80 as 1 per cent monthly interest from March 1, 1946 to February 28, 1947, or a total of P33,099.26. The assessment was made on February 11, 1947. On February 21, 1947, the company asked for an extension of at least one year from February 28, 1947 within which to pay the amount assessed, reserving its right to question the correctness of the assessment. The Collector granted an extension of only three months from March 20, 1947. The company failed to pay the tax within the period granted to it and so the Collector sent to it a letter on November 28, 1950 demanding payment of the tax due as assessed, plus surcharge and interest up to December 31, 1950. On April 6, 1951, the company asked for a reconsideration and reinvestigation of the assessment, which was granted, the case being assigned to another examiner, but the Collector made another assessment against the company in the sum of P33,829.66. This new assessment was made on March 7, 1952. On April 18, 1952, the Collector revised this last assessment and required the company to pay the sum of P28,289.96 as income tax, P1,414.50 as surcharge, P20,934.57 as interest up to April 30, 1952 and P40 as compromise. After several other negotiations conducted at the request of respondent, including an appeal to the Conference Staff created to act on such matters in the Bureau of Internal Revenue, the assessment was finally reduced by the Collector to P24,438.96, without surcharge and interest, and of this new assessment the company was notified on July 28, 1955. Within the reglementary period, the company filed with the Court of Tax Appeals a petition for review of this assessment made on July 26, 1955 on the main ground that the right of the Government to collect the tax has already prescribed. After the case was heard, the court rendered its decision upholding this defense and, accordingly, it set aside the ruling of the Collector of Internal Revenue. The Collector interposed the present petition for review. Under the law, an internal revenue tax shall be assessed within five years after the return is filed by the taxpayer and no proceeding in court for its collection shall be begun after the expiration of such period (Section 331, National Internal Revenue Code). The law also provides that where an assessment of internal revenue tax is made within the above period, such tax may be collected by distraint or levy or by a proceeding in court but only if the same is begun (1) within five years after assessment or (2) within the period that may be agreed upon in writing between the Collector and the taxpayer before the expiration of the 5-year period [Section 332 (c), Idem.]. It appears that the first assessment made against respondent based on its second final return filed on November 28, 1946 was made on February 11, 1947. Upon receipt of this assessment respondent requested for at least one year within which to pay the amount assessed although it

reserved its right to question the correctness of the assessment before actual payment. Petitioner granted an extension of only three months. When it failed to pay the tax within the period extended, petitioner sent respondent a letter on November 28, 1950 demanding payment of the tax as assessed, and upon receipt of the letter respondent asked for a reinvestigation and reconsideration of the assessment. When this request was denied, respondent again requested for a reconsideration on April 25, 1952, which was denied on May 6, 1953, which denial was appealed to the Conference Staff. The appeal was heard by the Conference Staff from September 2, 1953 to July 16, 1955, and as a result of these various negotiations, the assessment was finally reduced on July 26, 1955. This is the ruling which is now being questioned after a protracted negotiation on the ground that the collection of the tax has already prescribed. It is obvious from the foregoing that petitioner refrained from collecting the tax by distraint or levy or by proceeding in court within the 5-year period from the filing of the second amended final return due to the several requests of respondent for extension to which petitioner yielded to give it every opportunity to prove its claim regarding the correctness of the assessment. Because of such requests, several reinvestigations were made and a hearing was even held by the Conference Staff organized in the collection office to consider claims of such nature which, as the record shows, lasted for several months. After inducing petitioner to delay collection as he in fact did, it is most unfair for respondent to now take advantage of such desistance to elude his deficiency income, tax liability to the prejudice of the Government invoking the technical ground of prescription. While we may agree with the Court of Tax Appeals that a mere request for reexamination or reinvestigation may not have the effect of suspending the running of the period of limitation for in such case there is need of a written agreement to extend the period between the Collector and the taxpayer, there are cases however where a taxpayer may be prevented from setting up the defense of prescription even if he has not previously waived it in writing as when by his repeated requests or positive acts the Government has been, for good reasons, persuaded to postpone collection to make him feel that the demand was not unreasonable or that no harassment or injustice is meant by the Government. And when such situation comes to pass there are authorities that hold, based on weighty reasons, that such an attitude or behavior should not be countenanced if only to protect the interest of the Government. This case has no precedent in this jurisdiction for it is the first time that such has risen, but there are several precedents that may be invoked in American jurisprudence. As Mr. Justice Cardozo has said: "The applicable principle is fundamental and unquestioned. 'He who prevents a thing from being done may not avail himself of the nonperformance which he has himself occasioned, for the law says to him in effect "this is your own act, and therefore you are not damnified." ' "(R. H. Stearns Co. vs. U.S., 78 L. ed., 647). Or, as was aptly said, "The tax could have been collected, but the government withheld action at the specific request of the plaintiff. The plaintiff is now estopped and should not be permitted to raise the defense of the Statute of Limitations." [Newport Co. vs. U.S., (DC-WIS), 34 F. Supp. 588]. The following authorities cited in the brief of the Solicitor General are in point: The petitioner makes the point that by the Revenue Act of May 29, 1928 (chap. 852, 45 Stat. at L. 791, 875, sec. 609, U.S.C. title 26, sec. 2609), a credit against a liability in respect of any taxable year shall be "void" if it has been made against a liability barred by limitation. The aim of that provision, as we view it, was to invalidate such a credit if made by the Commissioner of his own motion without the taxpayer's approval or with approval failing short of inducement or request. Cf. Stange vs. United States, 282 U. S. 270, 75 L. ed. 335, 51 S. Ct. 145, supra; Revenue Act of 1928, sec. 506 (b) (c), chap. 852, 45 Stat. at L. 791, 870, 871, U.S.C. title 26, see. 1062a. If nothing more than this appeared, there was to be no

exercise in invitum of governmental power. But the aim of the statute suggests a restraint upon its meaning. To know whether liability has been barred by limitation it will not do to refer to the flight of time alone. The limitation may have been postponed by force of a simple waiver, which must then be made in adherence to the statutory forms, or so we now assume. It may have been postponed by deliberate persuasion to withhold official action. We think it an unreasonable construction that would view the prohibition of the statute as over-riding the doctrine of estoppel (Randon vs. Tobey, 11 How. 493, 519, 13 L. ed. 784, 795) and invalidating a credit made at the taxpayer's request. Here at the time of the request, the liability was still alive, unaffected as yet by any statutory bar. The request in its fair meaning reached forward into the future and prayed for the postponement of collection till the audits for later years had been completed in the usual course. This having been done, the suspended collection might be effected by credit or by distraint or by other methods prescribed by law. Congress surely did not mean that a credit was to be void if made by the Government in response to such prayer. The applicable principle is fundamental and unquestioned. "He who prevents a thing from being done may not avail himself of the nonperformance which he has himself occasioned, for the law says to him in effect "this is your own act, and therefore you are not damnified," ' " Dolan vs. Rogers, 149 N. Y. 489, 491, 44 N.E. 167, and Imperator Realty Co. vs. Tull, 228 N. Y. 447, 457, 127 N.E. 263, quoting West vs. Blakeway, 2 Mann. & G. 729, 751, 133 Eng. Reprint, 940, 949. Sometimes the resulting disability has been characterized as an estoppel, sometimes as a waiver. The label counts for little. Enough for present purposes that the disability has its roots in a principle more nearly ultimate than either waiver or estoppel, the principle that no one shall be permitted to found any claim upon his own inequity or take advantage of his own wrong. Imperator Realty Co. vs. Tull, 228 N.Y. 447, 127 N.E. 263, supra. A suit may not be built on an omission induced by him who sues. Swain vs. Seamens, 9 Wall. 254, 274, 19 L. ed. 554, 560; United States vs. Peck, 102 U.S. 64, 26 L. ed. 46; Thomson vs. Poor, 147 N.Y. 402, 42 N.E. 13; New Zealand Shipping Co. vs. Societe des Ateliers (1919) A. C. 1, 6-H. L.; 2 Williston, Contr. sec. 689. (R. H. Stearns Co. vs. U.S., supra;Emphasis supplied.) . . . It is admitted that these assessments were timely made in August 1923. Upon the making of the assessment the Commissioner sought to make collection, which likewise was at a time when the statute had not ran on collection, but the authorized representative of the Lattimores strenuously objected to the collection and urged the Commissioner to withhold collection, pending adjustment of the controversy between them and the Commissioner. The Commissioner yielded to their request and postponed collection until August 19, 1926, which was after the statute had run on collection. In the meantime, further claims for refund and protests were filed, conferences were held and consideration was given to the settlement of the controversy, and the matter was not finally disposed of until 1926, when the statute had run on collection. The procedure carried out was that requested by plaintiffs, and they cannot now be heard to say that the collection was not timely. R. H. Stearns Company vs. United States, 291 U.S. 54, 54 S. Ct. 325, 78 L. Ed. 647. (Lattimore vs. U.S., 12 F. Supp. 895, 91.) Wherefore, the decision appealed from is reversed. The decision of the Collector of Internal Revenue rendered on July 26, 1955 is hereby affirmed. No costs. Paras, C. J., Bengzon, Labrador, Concepcion, Reyes, J. B. L. and Endencia, JJ., concur.

Separate Opinions MONTEMAYOR, J., dissenting: As stated in the majority opinion, the respondent Suyoc Consolidated Mining Company was unable to file in 1942 its income tax return for the year 1941, because of the last war. Acting upon an extension granted by Commonwealth Act 722 and by the Collector of Internal Revenue, it finally filed the first income tax return (tentative) on February 12, 1946. For purposes of reference I am listing below in chronological order, the dates which are material and relevant for purposes of computation of the period of prescription. February 12, 1946 November 28, 1946 February 6, 1947 February 11, 1947 Respondent filed its "tentative return". Respondent filed its "final return". Respondent filed its amended final return". Notice of 1st assessment (Based on the final return sent to the respondent) (Amount of assessment ² P33,099.26). Receipt of respondent said assessment. Respondent asked for extension of time (one year) to pay the assessment, but reserving right to question its validity. He was given only three months from March 20, 1957. Petitioner demanded payment of tax assessed. Respondent asked for reconsideration and reinvestigation of the assessment. Notice of 2nd assessment (Based on the amended final return) was sent to respondent. (Amount ² P33,289.96). Petitioner revised the assessment made on March 7, 1952 (Now it is P50,697.03) Petitioner reduced the assessment of April 18, 1952 after various negotiations. (Now it is P24,438.96)

February 14, 1947 February 21, 1947

November 28, 1950 April 6, 1951 March 7, 1952 April 18, 1952 July 26, 1955

It will be noticed that petitioner Collector made his first assessment based on the final return submitted by Suyoc on November 28, 1946, on February 11, 1947. The assessment was in the amount of P33,099.26. Suyoc asked for an extension of time of one year within which to make payment, at the same time reserving its right to question the validity of the assessment, but it was granted only three months from March 20, 1947, that is to say, up to June 20, 1947. After said deadline, the Collector should immediately have demanded payment or resorted to the administrative remedy of distraint and levy, but strange to say, the Collector did not act and allowed more than three years to pass (from June 20, 1947 to November 28, 1950). It was only on November 28, 1950 that the Collector demanded payment on the basis of his assessment. On April

6, 1951, Suyoc asked for reconsideration and reinvestigation. After about a year, that is, on March 7, 1952, the Collector made a second assessment of P33,829.66, which was larger than his first assessment by about P800. Then on April 18, 1952, the Collector made a revised third assessment of P28,289.96 as income tax, P1,414.50 as surcharge, P20,934.57 as interest up to April 30, 1952, and P40.00 as compromise, which all added up to the staggering amount of P50,679.03, far different from and much larger than the first and second assessment by almost P17,000. After several negotiations, including appeal to the conference staff created to act on such matters in the Bureau of Internal Revenue, the assessment was finally reduced on July 26, 1955 to only P24,438.96, without surcharge, without interest and without any amount as compromise. It is this last assessment which Suyoc appealed to the Court of Tax Appeals. For purposes of reference, I am reproducing the pertinent sections of the National Internal Revenue Code: SEC. 331. Period of limitation upon assessment and collection. ² Except as provided in the succeeding section, internal revenue taxes shall be assessed within five years after the return was filed, and no proceeding in court without assessment for the collection of such taxes shall be begun after the expiration of such period. For the purposes of this section a return filed before the last day prescribed by law for the filing thereof shall be considered as filed on such last day; Provided, that this limitation shall not apply to cases already investigated prior to the approval of this Code. SEC. 332. Exceptions as to period of limitation of assessment and collection of taxes. ² . . . . (c) Where the assessment of any internal revenue tax has been made within the period of limitation above prescribed such tax may be collected by distraint or levy or by a proceeding in court, but only if begun (1) within five years after the assessment of the tax, or (2) prior to the expiration of any period for collection agreed upon in writing by the Collector of Internal Revenue and the taxpayer before the expiration of such five-year period. The period so agreed upon may be extended by subsequent agreements in writing made before the expiration of the period previously agreed upon. SEC. 333. Suspension of running of statute. ² The running of the statute of limitations provided in section three hundred thirty-one or three hundred thirty-two on the making of "assessments and the beginning of distraint or levy or a proceeding in court for collection, in respect of any deficiency, shall be suspended for the period during which the Collector of Internal Revenue is prohibited from making the assessment or beginning distraint or levy or a proceeding in court, and for sixty days thereafter. To me, the best argument against the contention of the Collector, and the ruling contained in the majority opinion that the right of the Collector to collect the tax assessed by it has not prescribed, and that the petitions or petitions filed by Suyoc for investigation and revision of the assessment extended the period of prescription, is the well written and reasoned decision (Resolution) of the Court of Tax Appeals, through Judge Roman M. Umali to which I agree. I am reproducing with approval the pertinent portions of said decision: Petitioner filed the instant petition for review on the grounds that certain losses were improperly disallowed by respondent as deductions from its gross income, and that the right of the Government to collect the tax, if any is due, has prescribed. When this case was called for hearing counsel for petitioner asked that the question of prescription be first resolved before hearing the case on the question involving the correctness of the assessment. The

sole issue raised at this time for resolution of this Court is, therefore, confined to the question of prescription. Upon the evidence submitted and admitted by the parties, it appears that the last and final assessment made by respondent covering the income tax due from petitioner for the year 1941 was made on July 26, 1955, more than five years from the date the "amended return" was filed on November 28, 1946, or from the date the amended final return' was filed on February 6, 1947. The right of respondent to assess the tax has, therefore, prescribed pursuant to Section 331 of the National Internal Revenue which requires that the assessment be made within five years from the date the return was filed. Even granting that the first assessment made on February 11, 1947, is the one to be considered in determining whether or not the assessment was made within the statutory period it follows that it must have to be considered also as the starting point from which the period within which the right to collect should be computed. Accordingly, on the theory that the assessment in this case was made within five years from the date the return was filed, the right of the Government to collect the tax assessed has prescribed, respondent having failed at any time from February 14, 1947 up to the time the instant petition for review was filed on September 19, 1955, a period of more than 8 years, to institute appropriate proceedings, judicially or otherwise, for the collection of the tax. (See Sec. 332 [c], National Internal Revenue Code.) From whatever angle the case is viewed, we find that the right of the Government to collect the income tax assessed against petitioner for the year 1941 has prescribed. But it is insisted that the requests of petitioner for reconsideration of the assessment, and while the same were pending consideration by respondent, had the effect of suspending the running of the statute of limitations. The statute of limitations upon assessment and collection of national internal revenue taxes provided in Sections 331 and 332 of the Revenue Code may be suspended only "for the period during which the Collector of Internal Revenue is prohibited from making the assessment or beginning destraint or levy or a proceeding in court, and sixty day thereafter." (Sec. 333, Revenue Code.) Nowhere does the law recognize that a simple request for reconsideration of an assessment, unaccompanied by any positive indication that the taxpayer is waiving his right to assert the defense of prescription, has the effect of suspending the running of the statute of limitations. That a request for re-examination or reconsideration of an assessment does not suspend the running of the statute of limitations seems to be the prevailing opinion in the Bureau of Internal Revenue. This may he inferred from the fact that General Circular No. V-182 dated January 17, 1955 had to be promulgated. Paragraph 6 of said circular provides: 6. Within thirty (30) days from the receipt of the deficiency tax assessment notice, the taxpayer may request reinvestigation or re-examination of the assessment, subject to the following requirements prescribed in paragraph 3 of Department Order No. 213: "(a) The taxpayer shall put the specific grounds of his protest in writing and under oath, accompanied by such additional documents and evidence supporting his protest; (b) He shall pay one-half (1/2) of the total assessment and file a bond to guarantee the payment of the balance together with the penalties that shall have accrued at the time of final payment; and

(c) He shall sign a statement that he is waiving the periods of prescription involved in the assessment and collection of the deficiency tax in question." (Emphasis supplied.) If a simple request for reinvestigation or re-examination of an assessment suspends the running of the statute of limitations, as alleged by respondent, there is no necessity for the requirement that a taxpayer must sign a statement that he is waiving the periods of prescription' as a condition for the granting of the request for reinvestigation or reexamination. General Circular No. V-182 obviously in line with Section 332 (c) of the Revenue Code which provides that the waiver of the taxpayer must be contained in an agreement in writing extending the five year period of limitation upon the right of the respondent to collect internal revenue taxes. FOR THE FOREGOING CONSIDERATIONS We are of the opinion that the right of the Government to collect from petitioner the sum of P24,438.96 as income tax for the year 1941 has prescribed. Accordingly, the decision appealed from is hereby set aside, without pronouncement as to costs. I fully agree with the Court of Tax Appeals that whether we consider February 11, 1947 or July 26, 1955, as the date of the assessment, the right of the Collector, either to make collection within five years from February 11, 1947 or to make assessment within five years from February 6, 1947, has prescribed. I do not believe that a mere petition for revision or reinvestigation can be regarded as an agreement of the taxpayer to extend the period of prescription. The very law clearly so states. Section 333 says that the running of the statute of limitations provided in Sections 331 and 332 shall be suspended only when the Collector is prohibited from making the assessment or beginning the distraint. No such prohibition or inability to make assessment or begin the distraint is claimed for the Collector. And Section 332 (c) says that the period for collection may be extended only by express agreement in writing by the taxpayer and the Collector. Evidently, nothing short of such express written agreement to extend will suspend the running of the period. It will be observed that Suyoc made only one petition for extension, that is, for one year within which to pay the assessment, but reserving its right to question the validity thereof. It was given only three months. Thereafter, it never asked for any other extension. True, it asked for revision and reconsideration of the different assessments made by the Collector, but this in no way can be regarded as an express agreement to extend the period; and the Collector was well aware of the fact that a mere petition to amend, modify, revise or revive the assessment or reinvestigate the case cannot extend the period of prescription, as evidenced by the very General Circular No. V-182, promulgated for the guidance of the Bureau of Internal Revenue. Said circular among other things provides that in order that there be an extension of the period of prescription and presumably, for the protection of the Government, the taxpayer must sign a statement that he is waiving the period of prescription involved in the collection of the tax. The trouble with the actuations of the Collector in this case is that he would appear to have unduly delayed definite and affirmative action on the assessment and collection as shown by the wide gaps ² first, a period of more than three years from February 14, 1947, when Suyoc received notice of the first assessment (extended by the Collector to June 20, 1947) to November 28, 1950, when the Collector demanded payment; then another period of about two years from November 28, 1950 to March 7, 1952 when he made the second assessment. Not only was there undue delay on the part of the Collector, but his actuations would seem to have been characterized by indecision and uncertainty. First, he made an assessment in the amount of P33,099.26. Then he increased this to P33,829.66. Then on April 18, 1952, he again increased this assessment to P50,678.03, until on July 26, 1955, this sum of over P50,000 was reduced to

P24,438.96, without surcharge, without interest and without any amount as compromise. Why all this difference or differences in the amounts of the assessment? One could well imagine and understand that a first assessment more or less hastily prepared may be revised within a reasonable time, say a few months or even a year, either increasing it or decreasing it. But when the Collector over a period of more than eight years kept changing his assessment, increasing the same by substantial amounts and then decreasing the same substantially, and at the same time utterly forgetting the period of prescription set by the law and also forgetting to protect the interest of the Government by requiring the taxpayer to agree expressly and in writing to extend the period of such prescription; and equally important, forgetting and failing up to the present time to institute proceedings, administrative by distraint and levy or judicial by court action, to collect, the Government has no one to blame but itself and its officials, certainly not the taxpayer who did nothing but ask for revision of the assessment to obtain a correct figure while it finally got but too late, after a wait of over eight years. The majority opinion places much reliance on the case of R. H. Stearns Company vs. U.S., 291 U.S., 54, and makes extensive quotation therefrom. After reading said case, I agree with counsel for Suyoc that it not applicable, for the reason that in that case, the taxpayer signed two waivers of the period of limitation; that although the second waiver was not signed by the Commissioner, nevertheless, the taxpayer on several ocassions had requested him to withhold collection. Naturally, the United States Supreme Court was constrained to hold that when the taxpayer not only signed waivers but had deliberately asked and persuaded the Commissioner to postpone collection, he cannot invoke the benefit of prescription to the running of which he has contributed. Our law expressly and clearly provides that in order to suspend the period of prescription or to extend it, the taxpayer and the Collector must sign an agreement to that effect. Nothing short of this will effect said extension or suspension of the period of limitation. Mere petitions for revision or reinvestigation by the taxpayer cannot suspend the running of the period of prescription. The taxpayer may make as many requests for revision or examination as he wishes, but the Collector need not act upon them to the prejudice of the Government; and even if he does act upon said petitions, he should always keep an eye on the running of the period, on the dead line, so that for the protection of the Government, he could enforce collection before it is too late. Prescription in the assessment and in the collection of taxes is provided by the Legislature for the benefit of both the Government and taxpayer; for the Government for the purpose of expediting the collection of taxes, so that the agency charged with the assessment and collection may not tarry too long or indefinitely to the prejudice of the interests of the Government which needs said taxes to run it; and for the taxpayer so that within a reasonable time after filing his return, he may know the amount of the assessment which he is required to pay, whether or not such assessment is well founded and reasonable so that he may either pay the amount of the assessment or contest its validity in court, either by filing an action for the refund, if already paid, under the old law, or appeal the disputed assessment to the Court of Tax Appeals under the present law creating the Tax Court. It would surely be prejudicial to the interest of the taxpayer for the Government collecting agency to unduly delay the assessment and the collection because by the time that the collecting agency finally gets around to making the assessment or making the collection, the taxpayer may then have lost his papers and books to support his claim and contest that of the Government, and what is more, the tax is in the meantime accumulating interest which the taxpayer eventually has to pay. In connection with this extension of the period of prescription or limitation for the Government to collect taxes, it will be noticed from Section 332(c) of the Internal Revenue Code that even If the taxpayer and the Collector agree to extend the period of limitation, said period has to be specific or fixed, and if said period of extension is to be further extended, another agreement has to be made again specifying the period of said further extension. From all this, it is evident that to extend the period of limitation or prescription, an express agreement in writing to that effect, signed by the

Collector and the taxpayer is necessary. Naturally, a mere petition by the taxpayer for revision or reexamination of the assessment cannot and will not automatically extend the period of limitation. However, under the theory espoused by the majority, let the taxpayer just ask, not for an extension of the time to pay or the Government to collect, but for a mere re-examination or revision of the assessment, and lo, and behold, all the carefully prepared provisions of the tax law about prescription and statutory limitation are laid aside, and the collecting agency of the Government may then postpone and delay the collection indefinitely, until such time as it is good and ready to resume proceedings from where it left off, and if the taxpayer complains of the delay or invokes prescription, he is instantly met with and silenced by the done of estoppel. I believe that is not what the law and the Legislature contemplated. To me, this matter of the extension of the period of limitation is quite clear, but assuming for a moment that there were any doubt about it, then we have the time honored and well settled rule of statutory construction that tax laws should be interpreted liberally in favor of the taxpayer and strictly against the Government, except in the matter of tax exemptions, in which case the rule is reversed. In the case of Manila Railroad Co. vs. Collector of Customs, 52 Phil. 952, this Tribunal said: . . . . It is the general rule in the interpretation of statutes levying taxes or duties not to extend their provisions beyond the clear import of the language used. In every case of doubt, such statutes are construed most strongly against the Government and in favor of the citizen, because burdens are not to be imposed, nor presumed to be imposed, beyond what the statutes expressly and clearly import. (U. S. vs. Wigglesworth [1842], 2 Story, 369; Froehlich & Kuttner vs. Collector of Customs [1911], 19 Phil., 461.) Years ago, the Supreme Court of the United States, through Chief Justice Marshall, in the case of McCulloch vs. The State of Maryland, 4 Law Ed. 579, said that the power to tax is the power to destroy. Evidently, to moderate this awesome and dangerous taxing power of the Legislature, and in order to temper the rigor of tax laws, this sound and salutary rule of liberal construction of tax laws in favor of the taxpayer has been evolved and laid down. For the foregoing reasons, I dissent. Padilla, J., concurs. Republic of the Philippines SUPREME COURT Manila FIRST DIVISION G.R. No. 162852 December 16, 2004

PHILIPPINE JOURNALISTS, INC., petitioner, vs. COMMISSIONER OF INTERNAL REVENUE, respondent.

DECISION

YNARES-SANTIAGO, J.: This is a petition for review filed by Philippine Journalists, Incorporated (PJI) assailing the Decision1 of the Court of Appeals dated August 5, 2003,2 which ordered petitioner to pay the assessed tax liability of P111,291,214.46 and the Resolution3 dated March 31, 2004 which denied the Motion for Reconsideration. The case arose from the Annual Income Tax Return filed by petitioner for the calendar year ended December 31, 1994 which presented a net income of P30,877,387.00 and the tax due of P10,807,086.00. After deducting tax credits for the year, petitioner paid the amount of P10,247,384.00. On August 10, 1995, Revenue District Office No. 33 of the Bureau of Internal Revenue (BIR) issued Letter of Authority No. 871204 for Revenue Officer Federico de Vera, Jr. and Group Supervisor Vivencio Gapasin to examine petitioner¶s books of account and other accounting records for internal revenue taxes for the period January 1, 1994 to December 31, 1994. From the examination, the petitioner was told that there were deficiency taxes, inclusive of surcharges, interest and compromise penalty in the following amounts: Value Added Tax Income Tax Withholding Tax Total P 229,527.90 125,002,892.95 2,748,012.35 P 127,980,433.20

In a letter dated August 29, 1997, Revenue District Officer Jaime Concepcion invited petitioner to send a representative to an informal conference on September 15, 1997 for an opportunity to object and present documentary evidence relative to the proposed assessment. On September 22, 1997, petitioner¶s Comptroller, Lorenza Tolentino, executed a "Waiver of the Statute of Limitation Under the National Internal Revenue Code (NIRC)".5 The document "waive[d] the running of the prescriptive period provided by Sections 223 and 224 and other relevant provisions of the NIRC and consent[ed] to the assessment and collection of taxes which may be found due after the examination at any time after the lapse of the period of limitations fixed by said Sections 223 and 224 and other relevant provisions of the NIRC, until the completion of the investigation".6 On July 2, 1998, Revenue Officer De Vera submitted his audit report recommending the issuance of an assessment and finding that petitioner had deficiency taxes in the total amount of P136,952,408.97. On October 5, 1998, the Assessment Division of the BIR issued Pre-Assessment Notices which informed petitioner of the results of the investigation. Thus, BIR Revenue Region No. 6, Assessment Division/Billing Section, issued Assessment/Demand No. 33-1-000757-947 on December 9, 1998 stating the following deficiency taxes, inclusive of interest and compromise penalty: Income Tax P108,743,694.88

Value Added Tax Expanded Withholding Tax Total

184,299.20 2,363,220.38 P111,291,214.46

On March 16, 1999, a Preliminary Collection Letter was sent by Deputy Commissioner Romeo S. Panganiban to the petitioner to pay the assessment within ten (10) days from receipt of the letter. On November 10, 1999, a Final Notice Before Seizure8 was issued by the same deputy commissioner giving the petitioner ten (10) days from receipt to pay. Petitioner received a copy of the final notice on November 24, 1999. By letters dated November 26, 1999, petitioner asked to be clarified how the tax liability of P111,291,214.46 was reached and requested an extension of thirty (30) days from receipt of the clarification within which to reply.9 The BIR received a follow-up letter from the petitioner asserting that its (PJI) records do not show receipt of Tax Assessment/Demand No. 33-1-000757-94.10 Petitioner also contested that the assessment had no factual and legal basis. On March 28, 2000, a Warrant of Distraint and/or Levy No. 33-06-04611 signed by Deputy Commissioner Romeo Panganiban for the BIR was received by the petitioner. Petitioner filed a Petition for Review12 with the Court of Tax Appeals (CTA) which was amended on May 12, 2000. Petitioner complains: (a) that no assessment or demand was received from the BIR; (b) that the warrant of distraint and/or levy was without factual and legal bases as its issuance was premature; (c) that the assessment, having been made beyond the 3-year prescriptive period, is null and void; (d) that the issuance of the warrant without being given the opportunity to dispute the same violates its right to due process; and (e) that the grave prejudice that will be sustained if the warrant is enforced is enough basis for the issuance of the writ of preliminary injunction. On May 14, 2002, the CTA rendered its decision,13 to wit: As to whether or not the assessment notices were received by the petitioner, this Court rules in the affirmative. To disprove petitioner¶s allegation of non-receipt of the aforesaid assessment notices, respondent presented a certification issued by the Post Master of the Central Post Office, Manila to the effect that Registered Letter No. 76134 sent by the BIR, Region No. 6, Manila on December 15, 1998 addressed to Phil. Journalists, Inc. at Journal Bldg., Railroad St., Manila was duly delivered to and received by a certain Alfonso Sanchez, Jr. (Authorized Representative) on January 8, 1999. Respondent also showed proof that in claiming Registered Letter No. 76134, Mr. Sanchez presented three identification cards, one of which is his company ID with herein petitioner. « However, as to whether or not the Waiver of the Statute of Limitations is valid and binding on the petitioner is another question. Since the subject assessments were issued beyond the three-year prescriptive period, it becomes imperative on our part to rule first on the validity of the waiver allegedly executed on September 22, 1997, for if this court finds the same to be ineffective, then the assessments must necessarily fail.

« After carefully examining the questioned Waiver of the Statute of Limitations, this Court considers the same to be without any binding effect on the petitioner for the following reasons: The waiver is an unlimited waiver. It does not contain a definite expiration date. Under RMO No. 20-90, the phrase indicating the expiry date of the period agreed upon to assess/collect the tax after the regular three-year period of prescription should be filled up« « Secondly, the waiver failed to state the date of acceptance by the Bureau which under the aforequoted RMO should likewise be indicated« « Finally, petitioner was not furnished a copy of the waiver. It is to be noted that under RMO No. 20-90, the waiver must be executed in three (3) copies, the second copy of which is for the taxpayer. It is likewise required that the fact of receipt by the taxpayer of his/her file copy be indicated in the original copy. Again, respondent failed to comply. It bears stressing that RMO No. 20-90 is directed to all concerned internal revenue officers. The said RMO even provides that the procedures found therein should be strictly followed, under pain of being administratively dealt with should non-compliance result to prescription of the right to assess/collect« Thus, finding the waiver executed by the petitioner on September 22, 1997 to be suffering from legal infirmities, rendering the same invalid and ineffective, the Court finds Assessment/Demand No. 33-1-000757-94 issued on December 5, 1998 to be time-barred. Consequently, the Warrant of Distraint and/or Levy issued pursuant thereto is considered null and void. WHEREFORE, in view of all the foregoing, the instant Petition for Review is hereby GRANTED. Accordingly, the deficiency income, value-added and expanded withholding tax assessments issued by the respondent against the petitioner on December 9, 1998, in the total amount of P111,291,214.46 for the year 1994 are hereby declared CANCELLED, WITHDRAWN and WITH NO FORCE AND EFFECT. Likewise, Warrant of Distraint and/or Levy No. 33-06-046 is hereby declared NULL and VOID. SO ORDERED.14 After the motion for reconsideration of the Commissioner of Internal Revenue was denied by the CTA in a Resolution dated August 2, 2002, an appeal was filed with the Court of Appeals on August 12, 2002. In its decision dated August 5, 2003, the Court of Appeals disagreed with the ruling of the CTA, to wit: « The petition for review filed on 26 April 2000 with CTA was neither timely filed nor the proper remedy. Only decisions of the BIR, denying the request for reconsideration or

reinvestigation may be appealed to the CTA. Mere assessment notices which have become final after the lapse of the thirty (30)-day reglementary period are not appealable. Thus, the CTA should not have entertained the petition at all. « « [T]he CTA found the waiver executed by Phil. Journalists to be invalid for the following reasons: (1) it does not indicate a definite expiration date; (2) it does not state the date of acceptance by the BIR; and (3) Phil. Journalist, the taxpayer, was not furnished a copy of the waiver. These grounds are merely formal in nature. The date of acceptance by the BIR does not categorically appear in the document but it states at the bottom page that the BIR "accepted and agreed to:"«, followed by the signature of the BIR¶s authorized representative. Although the date of acceptance was not stated, the document was dated 22 September 1997. This date could reasonably be understood as the same date of acceptance by the BIR since a different date was not otherwise indicated. As to the allegation that Phil. Journalists was not furnished a copy of the waiver, this requirement appears ridiculous. Phil. Journalists, through its comptroller, Lorenza Tolentino, signed the waiver. Why would it need a copy of the document it knowingly executed when the reason why copies are furnished to a party is to notify it of the existence of a document, event or proceeding? « As regards the need for a definite expiration date, this is the biggest flaw of the decision. The period of prescription for the assessment of taxes may be extended provided that the extension be made in writing and that it be made prior to the expiration of the period of prescription. These are the requirements for a valid extension of the prescriptive period. To these requirements provided by law, the memorandum order adds that the length of the extension be specified by indicating its expiration date. This requirement could be reasonably construed from the rule on extension of the prescriptive period. But this requirement does not apply in the instant case because what we have here is not an extension of the prescriptive period but a waiver thereof. These are two (2) very different things. What Phil. Journalists executed was a renunciation of its right to invoke the defense of prescription. This is a valid waiver. When one waives the prescriptive period, it is no longer necessary to indicate the length of the extension of the prescriptive period since the person waiving may no longer use this defense. WHEREFORE, the 02 August 2002 resolution and 14 May 2002 decision of the CTA are hereby SET ASIDE. Respondent Phil. Journalists is ordered [to] pay its assessed tax liability of P111,291,214.46. SO ORDERED.15 Petitioner¶s Motion for Reconsideration was denied in a Resolution dated March 31, 2004. Hence, this appeal on the following assignment of errors: I. The Honorable Court of Appeals committed grave error in ruling that it is outside the jurisdiction of the Court of Tax Appeals to entertain the Petition for Review filed by the herein Petitioner at the CTA despite the fact that such case inevitably rests upon the validity of the issuance by the BIR of warrants of distraint and levy contrary to the provisions of Section 7(1) of Republic Act No. 1125. II.

The Honorable Court of Appeals gravely erred when it ruled that failure to comply with the provisions of Revenue Memorandum Order (RMO) No. 20-90 is merely a formal defect that does not invalidate the waiver of the statute of limitations without stating the legal justification for such conclusion. Such ruling totally disregarded the mandatory requirements of Section 222(b) of the Tax Code and its implementing regulation, RMO No. 20-90 which are substantive in nature. The RMO provides that violation thereof subjects the erring officer to administrative sanction. This directive shows that the RMO is not merely cover forms. III. The Honorable Court of Appeals gravely erred when it ruled that the assessment notices became final and unappealable. The assessment issued is void and legally non-existent because the BIR has no power to issue an assessment beyond the three-year prescriptive period where there is no valid and binding waiver of the statute of limitation. IV. The Honorable Court of Appeals gravely erred when it held that the assessment in question has became final and executory due to the failure of the Petitioner to protest the same. Respondent had no power to issue an assessment beyond the three year period under the mandatory provisions of Section 203 of the NIRC. Such assessment should be held void and non-existent, otherwise, Section 203, an expression of a public policy, would be rendered useless and nugatory. Besides, such right to assess cannot be validly granted after three years since it would arise from a violation of the mandatory provisions of Section 203 and would go against the vested right of the Petitioner to claim prescription of assessment. V. The Honorable Court of Appeals committed grave error when it HELD valid a defective waiver by considering the latter a waiver of the right to invoke the defense of prescription rather than an extension of the three year period of prescription (to make an assessment) as provided under Section 222 in relation to Section 203 of the Tax Code, an interpretation that is contrary to law, existing jurisprudence and outside of the purpose and intent for which they were enacted.16 We find merit in the appeal. The first assigned error relates to the jurisdiction of the CTA over the issues in this case. The Court of Appeals ruled that only decisions of the BIR denying a request for reconsideration or reinvestigation may be appealed to the CTA. Since the petitioner did not file a request for reinvestigation or reconsideration within thirty (30) days, the assessment notices became final and unappealable. The petitioner now argue that the case was brought to the CTA because the warrant of distraint or levy was illegally issued and that no assessment was issued because it was based on an invalid waiver of the statutes of limitations. We agree with petitioner. Section 7(1) of Republic Act No. 1125, the Act Creating the Court of Tax Appeals, provides for the jurisdiction of that special court: SEC. 7. Jurisdiction. ± The Court of Tax Appeals shall exercise exclusive appellate jurisdiction to review by appeal, as herein provided ±

(1) Decisions of the Commissioner of Internal Revenue in cases involving disputed assessments, refunds of internal revenue taxes, fees or other charges, penalties imposed in relation thereto, or other matters arising under the National Internal Revenue Code or other laws or part of law administered by the Bureau of Internal Revenue; (Emphasis supplied). The appellate jurisdiction of the CTA is not limited to cases which involve decisions of the Commissioner of Internal Revenue on matters relating to assessments or refunds. The second part of the provision covers other cases that arise out of the NIRC or related laws administered by the Bureau of Internal Revenue. The wording of the provision is clear and simple. It gives the CTA the jurisdiction to determine if the warrant of distraint and levy issued by the BIR is valid and to rule if the Waiver of Statute of Limitations was validly effected. This is not the first case where the CTA validly ruled on issues that did not relate directly to a disputed assessment or a claim for refund. In Pantoja v. David,17 we upheld the jurisdiction of the CTA to act on a petition to invalidate and annul the distraint orders of the Commissioner of Internal Revenue. Also, in Commissioner of Internal Revenue v. Court of Appeals,18 the decision of the CTA declaring several waivers executed by the taxpayer as null and void, thus invalidating the assessments issued by the BIR, was upheld by this Court. The second and fifth assigned errors both focus on Revenue Memorandum Circular No. 20-90 (RMO No. 20-90) on the requisites of a valid waiver of the statute of limitations. The Court of Appeals held that the requirements and procedures laid down in the RMO are only formal in nature and did not invalidate the waiver that was signed even if the requirements were not strictly observed. The NIRC, under Sections 203 and 222,19 provides for a statute of limitations on the assessment and collection of internal revenue taxes in order to safeguard the interest of the taxpayer against unreasonable investigation.20Unreasonable investigation contemplates cases where the period for assessment extends indefinitely because this deprives the taxpayer of the assurance that it will no longer be subjected to further investigation for taxes after the expiration of a reasonable period of time. As was held in Republic of the Phils. v. Ablaza:21 The law prescribing a limitation of actions for the collection of the income tax is beneficial both to the Government and to its citizens; to the Government because tax officers would be obliged to act promptly in the making of assessment, and to citizens because after the lapse of the period of prescription citizens would have a feeling of security against unscrupulous tax agents who will always find an excuse to inspect the books of taxpayers, not to determine the latter¶s real liability, but to take advantage of every opportunity to molest peaceful, lawabiding citizens. Without such a legal defense taxpayers would furthermore be under obligation to always keep their books and keep them open for inspection subject to harassment by unscrupulous tax agents. The law on prescription being a remedial measure should be interpreted in a way conducive to bringing about the beneficent purpose of affording protection to the taxpayer within the contemplation of the Commission which recommend the approval of the law.(Emphasis supplied) RMO No. 20-90 implements these provisions of the NIRC relating to the period of prescription for the assessment and collection of taxes. A cursory reading of the Order supports petitioner¶s argument that the RMO must be strictly followed, thus: In the execution of said waiver, the following procedures should be followed:

1. The waiver must be in the form identified hereof. This form may be reproduced by the Office concernedbut there should be no deviation from such form. The phrase "but not after __________ 19___" should be filled up« 2. « Soon after the waiver is signed by the taxpayer, the Commissioner of Internal Revenue or the revenue official authorized by him, as hereinafter provided, shall sign the waiver indicating that the Bureau has accepted and agreed to the waiver. The date of such acceptance by the Bureau should be indicated« 3. The following revenue officials are authorized to sign the waiver. A. In the National Office « 3. Commissioner For tax cases involving more than P1M

B. In the Regional Offices 1. The Revenue District Officer with respect to tax cases still pending investigation and the period to assess is about to prescribe regardless of amount. « 5. The foregoing procedures shall be strictly followed. Any revenue official found not to have complied with this Order resulting in prescription of the right to assess/collect shall be administratively dealt with. (Emphasis supplied)22 A waiver of the statute of limitations under the NIRC, to a certain extent, is a derogation of the taxpayers¶ right to security against prolonged and unscrupulous investigations and must therefore be carefully and strictly construed.23 The waiver of the statute of limitations is not a waiver of the right to invoke the defense of prescription as erroneously held by the Court of Appeals. It is an agreement between the taxpayer and the BIR that the period to issue an assessment and collect the taxes due is extended to a date certain. The waiver does not mean that the taxpayer relinquishes the right to invoke prescription unequivocally particularly where the language of the document is equivocal. For the purpose of safeguarding taxpayers from any unreasonable examination, investigation or assessment, our tax law provides a statute of limitations in the collection of taxes. Thus, the law on prescription, being a remedial measure, should be liberally construed in order to afford such protection. As a corollary, the exceptions to the law on prescription should perforce be strictly construed.24 RMO No. 20-90 explains the rationale of a waiver: ... The phrase "but not after _________ 19___" should be filled up. This indicates the expiry date of the period agreed upon to assess/collect the tax after the regular three-year period of prescription. The period agreed upon shall constitute the time within which to effect the

assessment/collection of the tax in addition to the ordinary prescriptive period. (Emphasis supplied) As found by the CTA, the Waiver of Statute of Limitations, signed by petitioner¶s comptroller on September 22, 1997 is not valid and binding because it does not conform with the provisions of RMO No. 20-90. It did not specify a definite agreed date between the BIR and petitioner, within which the former may assess and collect revenue taxes. Thus, petitioner¶s waiver became unlimited in time, violating Section 222(b) of the NIRC. The waiver is also defective from the government side because it was signed only by a revenue district officer, not the Commissioner, as mandated by the NIRC and RMO No. 20-90. The waiver is not a unilateral act by the taxpayer or the BIR, but is a bilateral agreement between two parties to extend the period to a date certain. The conformity of the BIR must be made by either the Commissioner or the Revenue District Officer. This case involves taxes amounting to more than One Million Pesos (P1,000,000.00) and executed almost seven months before the expiration of the threeyear prescription period. For this, RMO No. 20-90 requires the Commissioner of Internal Revenue to sign for the BIR. The case of Commissioner of Internal Revenue v. Court of Appeals,25 dealt with waivers that were not signed by the Commissioner but were argued to have been given implied consent by the BIR. We invalidated the subject waivers and ruled: Petitioner¶s submission is inaccurate« « The Court of Appeals itself also passed upon the validity of the waivers executed by Carnation, observing thus: We cannot go along with the petitioner¶s theory. Section 319 of the Tax Code earlier quoted is clear and explicit that the waiver of the five-year26 prescriptive period must be in writing and signed by both the BIR Commissioner and the taxpayer. Here, the three waivers signed by Carnation do not bear the written consent of the BIR Commissioner as required by law. We agree with the CTA in holding "these µwaivers¶ to be invalid and without any binding effect on petitioner (Carnation) for the reason that there was no consent by the respondent (Commissioner of Internal Revenue)." « For sure, no such written agreement concerning the said three waivers exists between the petitioner and private respondent Carnation. « What is more, the waivers in question reveal that they are in no wise unequivocal, and therefore necessitates for its binding effect the concurrence of the Commissioner of Internal Revenue«. On this basis neither implied consent can be presumed nor can it be contended that the waiver required under Sec. 319 of the Tax Code is one which is

unilateral nor can it be said that concurrence to such an agreement is a mere formality because it is the very signatures of both the Commissioner of Internal Revenue and the taxpayer which give birth to such a valid agreement.27 (Emphasis supplied) The other defect noted in this case is the date of acceptance which makes it difficult to fix with certainty if the waiver was actually agreed before the expiration of the three-year prescriptive period. The Court of Appeals held that the date of the execution of the waiver on September 22, 1997 could reasonably be understood as the same date of acceptance by the BIR. Petitioner points out however that Revenue District Officer Sarmiento could not have accepted the waiver yet because she was not the Revenue District Officer of RDO No. 33 on such date. Ms. Sarmiento¶s transfer and assignment to RDO No. 33 was only signed by the BIR Commissioner on January 16, 1998 as shown by the Revenue Travel Assignment Order No. 14-98.28 The Court of Tax Appeals noted in its decision that it is unlikely as well that Ms. Sarmiento made the acceptance on January 16, 1998 because "Revenue Officials normally have to conduct first an inventory of their pending papers and property responsibilities."29 Finally, the records show that petitioner was not furnished a copy of the waiver. Under RMO No. 2090, the waiver must be executed in three copies with the second copy for the taxpayer. The Court of Appeals did not think this was important because the petitioner need not have a copy of the document it knowingly executed. It stated that the reason copies are furnished is for a party to be notified of the existence of a document, event or proceeding. The flaw in the appellate court¶s reasoning stems from its assumption that the waiver is a unilateral act of the taxpayer when it is in fact and in law an agreement between the taxpayer and the BIR. When the petitioner¶s comptroller signed the waiver on September 22, 1997, it was not yet complete and final because the BIR had not assented. There is compliance with the provision of RMO No. 2090 only after the taxpayer received a copy of the waiver accepted by the BIR. The requirement to furnish the taxpayer with a copy of the waiver is not only to give notice of the existence of the document but of the acceptance by the BIR and the perfection of the agreement. The waiver document is incomplete and defective and thus the three-year prescriptive period was not tolled or extended and continued to run until April 17, 1998. Consequently, the Assessment/Demand No. 33-1-000757-94 issued on December 9, 1998 was invalid because it was issued beyond the three (3) year period. In the same manner, Warrant of Distraint and/or Levy No. 33-06-046 which petitioner received on March 28, 2000 is also null and void for having been issued pursuant to an invalid assessment. WHEREFORE, premises considered, the instant petition for review is GRANTED. The Decision of the Court of Appeals dated August 5, 2003 and its Resolution dated March 31, 2004 are REVERSED and SET ASIDE. The Decision of the Court of Tax Appeals in CTA Case No. 6108 dated May 14, 2002, declaring Warrant of Distraint and/or Levy No. 33-06-046 null and void, is REINSTATED. SO ORDERED. Davide, Jr., C.J. (Chairman), Quisumbing, Carpio, and Azcuna, JJ., concur. Republic of the Philippines SUPREME COURT Baguio City EN BANC

G.R. No. 167756

April 8, 2008

THE PEOPLE OF THE PHILIPPINES, appellee, vs. JERRY NAZARENO, appellant. DECISION REYES, R.T., J.: IN this rape case, the Court is confronted with remedial questions on (a) specificity of dates in the Information; (b) quantum of proof; and (c) concurrence of allegation and proof. For Our final review is the Decision1 of the Court of Appeals (CA) affirming with modification appellant's conviction for rape of his two minor daughters. The Facts In line with Our ruling in People v. Cabalquinto,2 the real names of the rape victims will not be disclosed. We will instead use fictitious initials to represent them throughout the decision. The personal circumstances of the victims or any other information tending to establish or compromise their identities will likewise be withheld. Private complainants AAA and BBB are the legitimate daughters of appellant Jerry Nazareno with CCC. AAA was born on April 30, 1983.2-a BBB, the second child of the union, was born on June 24, 1984.2-b At that time, appellant and CCC were yet to wed. It was only in 1987 that the couple formally tied the knot in simple church ceremonies. Three more children sprang from the marriage since then.3 Sometime in 1990, AAA was inside a room in their house located at Barangay Codon, Municipality of San Andres, Province of Catanduanes. All of her siblings were playing in their yard. Unexpectedly, appellant entered the room, and without saying a word, held AAA tightly. He then directed AAA to crouch on the floor and raise her buttocks (baka-bakahan). While in that position, appellant removed the girl's short pants and underwear. He then proceeded to remove his own undergarments. Subsequently, appellant forcibly entered AAA from behind, inserting his penis into the girl's vagina. She was seven.4 Appellant threatened AAA not to reveal what happened to her to anyone; or else, she and the rest of her family would be killed. Expectedly, AAA suffered in silence. She feared for her life as well as that of her mother and siblings.5 AAA's ordeal with her father became a regular fare. Appellant would rape her whenever they were left alone in the house.6 CCC was rarely home because she attended to farm work and accepted laundry jobs from neighbors to support the family. Appellant was jobless and stayed at home.7 On March 25, 1996, appellant again imposed his bestial urges on AAA. AAA distinctly remembered the incident because she graduated from primary school on that day. At around 2:00 p.m., appellant and AAA were left alone in the house. He told AAA to remove her shorts and panty. Appellant then asked her to crouch on the floor and raise her buttocks. Just as he did before, appellant positioned himself behind the girl and then inserted his penis into her vagina. All that time, appellant's hands

were clutching the girl's back.8 Coincidentally, AAA's graduation from elementary school also marked the end of appellant's sexual abuses. BBB suffered the same fate as her older sister AAA. Sometime in January 1992, appellant and BBB were left alone in their house. Suddenly, appellant told BBB to kneel on all fours (pig baka-baka).9 Appellant then removed BBB's shorts and panties. He then removed his maong pants. Appellant positioned himself at BBB's rear and then inserted his penis into the young girl's vagina. At the time of the rape, BBB was only seven years old and was a Grade II pupil.10 Appellant continued raping BBB, using the girl for his sexual gratification every other day. From BBB's account, appellant would rape her fifteen times in a month. Every time, appellant would threaten her that he would kill all of them should she tell anyone what was happening between them.11 On October 27, 1998, AAA and BBB found the courage to tell their mother CCC what appellant had been doing to them. AAA accidentally found that BBB was likewise being subjected to sexual abuses by their father. Gathering strength from one another, AAA and BBB tearfully recounted to their mother their individual ordeals. CCC was devastated.12 On December 6, 1998, appellant again attempted to force himself on BBB. He inserted his finger into BBB's vagina. BBB felt extreme pain from the nails protruding from her father's fingers. That was the last time appellant abused BBB.13 On February 16, 1999, CCC, with AAA and BBB, secretly went to the Municipal Building of San Andres, Catanduanes to file a complaint against appellant for the rape of AAA and BBB. AAA and BBB were immediately attended to by personnel from the Department of Social Welfare and Development. The two were later examined at the JMA District Hospital by Dr. Erlinda H. Arcilla. CCC testified as to the age of the victims AAA and BBB at the time of the commission of the crimes. She affirmed that AAA was born on April 30, 1983 while BBB was born on June 24, 1984.14 CCC narrated that she was shocked when she heard her two daughters complain that they were raped by their own father. She knew appellant to be temperamental. He would hit AAA and BBB at the slightest provocation. She failed to act immediately on her daughters' plight for fear of her husband. CCC was convinced that appellant might make good his threats to kill all of them.15 Dr. Arcilla narrated that she examined both AAA and BBB on February 16, 1999. During her examination, she uncovered old healed hymenal lacerations on both AAA and BBB at the 3 o'clock, 6 o'clock and 9 o'clock positions. The lacerations suggested that the two girls were no longer in a virgin state.16 On March 17, 1999, appellant Jerry Nazareno was indicted for violation of Article 266-A of the Revised Penal Code in Criminal Case No. 2638 for the rape of BBB. The information reads: That sometime and between January 1992 up to December 06, 1998, in Barangay Codon, Municipality of San Andres, Province of Catanduanes, Philippines, and within the jurisdiction of this Honorable Court, the above-named accused by means of force, violence and intimidation did then and there willfully, unlawfully, feloniously and repeatedly made sexual intercourse with his daughter BBB at the age of 7 through 14 years old against her will. CONTRARY TO LAW.17

On May 3, 1999, another Information docketed as Criminal Case No. 2650, for the rape of AAA, was levelled against appellant. The indictment is worded thus: That from sometime in January 1990 up to December 1998 in barangay Codon, municipality of San Andres, Catanduanes, and within the jurisdiction of the Honorable Court, the said accused, being the father of the complainant, did then and there willfully, feloniously and criminally repeatedly had sexual intercourse with her daughter AAA, then five years old up to the time when she was 15-years-old against her will. CONTRARY TO LAW.18 The case for the People, which portrayed the foregoing facts, revolved around the combined testimonies of AAA, BBB, CCC, and Dr. Erlinda Arcilla of the JMA District Hospital in San Andres, Catanduanes. The defense, anchored on denial, was summed up by the trial court in this wise: The defense presented JERRY NAZARENO, the accused himself who testified that he is 34 years old, married, fisherman, a resident of Codon, San Andres, Catanduanes. He denied having raped his daughters. He said that he sometimes beat his children because he is strict with them in their studies especially during weekdays. He did not want them to watch television during schooldays. Though he is strict, he could not molest the complainants because they are his daughters. He said that the reason why his daughters filed these cases against him was because his father-in-law wants him to be incarcerated for the reason that from the very start, he was opposed to his marriage to CCC, his daughter. He also said that in December 1998, the last molestation of BBB, he was in the motor launch that plies the San Andres and Caramoran route.19 RTC and CA Dispositions On October 25, 2002, the trial court handed down a joint judgment of conviction, imposing upon appellant the capital punishment of death in both cases. The fallo of the RTC decision reads: WHEREFORE, in view of all the foregoing, the prosecution having proved the guilt of the accused beyond reasonable doubt, he is sentenced to suffer the extreme penalty of DEATH for raping BBB in Criminal Case No. 2638 and the same penalty for raping AAA in Criminal Case No. 2650 in accordance with Article 335 of the Revised Penal Code as amended by R.A. 7659. The accused is further ordered to indemnify both complainants the amount of Fifty Thousand Pesos (P50,000.00) each, to pay each of them the amount of Fifty Thousand Pesos (P50,000.00) as moral damages and the cost of suit. SO ORDERED.20 Conformably with the pronouncement in People v. Mateo21 providing for an intermediate review by the CA of cases in which the penalty imposed is death, reclusion perpetua or life imprisonment, the Court issued a Resolution dated September 21, 2004,22 transferring the case to the appellate court for appropriate action and disposition.

On February 22, 2005, the CA affirmed with modification the RTC judgment, disposing as follows: WHEREFORE, finding the accused guilty beyond reasonable doubt of the crime of rape as defined and penalized under Art. 335 of the Revised Penal Code as amended by Anti Rape Law of 1997, with the aggravating circumstance of relationship and minority, the decision of the court a quo sentencing him to death in both Criminal Cases Nos. 2638 and 2650 is hereby AFFIRMED. The award of civil indemnity is MODIFIED and INCREASED to P75,000.00 each, in both cases. The award of moral damages ofP50,000.00 for each case is AFFIRMED. We also award P25,000.00 as exemplary damages in each case. Let the records of this case be transmitted to the Supreme Court for appropriate action. SO ORDERED.23 Issues On September 27, 2005, the Court resolved to require the parties to submit their respective supplemental briefs, if they so desired, within thirty (30) days from notice. In a manifestation dated December 6, 2005, the Public Attorney's Office, representing appellant Jerry Nazareno, informed the Court that it is adopting its main brief on record.24 The Office of the Solicitor General, for the People, similarly opted to dispense with the filing of a supplemental brief in its manifestation dated March 9, 2006.25 Appellant stands by the same lone error he raised before the appellate court: THE TRIAL COURT ERRED (IN) NOT FINDING THAT THE INFORMATION(S) IN CRIMINAL CASE NO[S]. 2638 AND 2650 ARE INSUFFICIENT TO SUPPORT A JUDGMENT OF CONVICTION FOR ITS (SIC) FAILURE TO STATE THE PRECISE DATES OF THE COMMISSION OF THE OFFENSE CHARGED.26 (Corrections and underscoring supplied) Our Ruling In the main, appellant argues that the Informations charging him with the rape of AAA and BBB are defective for failure to state with specificity the approximate date of the commission of the offenses. According to him, the twin convictions have no basis in law because the People violated his constitutional right to be informed of the nature and cause of the accusations against him. The argument is specious. An information is intended to inform an accused of the accusations against him in order that he could adequately prepare his defense. Verily, an accused cannot be convicted of an offense unless it is clearly charged in the complaint or information. Thus, to ensure that the constitutional right of the accused to be informed of the nature and cause of the accusation against him is not violated, the information should state the name of the accused; the designation given to the offense by the statute; a statement of the acts or omissions so complained of as constituting the offense; the name of the offended party; the approximate time and date of the commission of the offense; and the place where the offense has been committed.27 Further, it must embody the essential elements of the crime charged by setting forth the facts and circumstances that have a bearing on the culpability and liability of the accused, so that he can properly prepare for and undertake his defense.28

However, it is not necessary for the information to allege the date and time of the commission of the crime with exactitude unless time is an essential ingredient of the offense.29 In People v. Bugayong,30 the Court held that when the time given in the information is not the essence of the offense, the time need not be proven as alleged; and that the complaint will be sustained if the proof shows that the offense was committed at any time within the period of the statute of limitations and before the commencement of the action. In People v. Gianan,31 the Court ruled that the time of the commission of rape is not an element of the said crime as it is defined in Article 335 of the Revised Penal Code. The gravamen of the crime is the fact of carnal knowledge under any of the circumstances enumerated therein, i.e.: (1) by using force or intimidation; (2) when the woman is deprived of reason or otherwise unconscious; and (3) when the woman is under twelve years of age or is demented. In accordance with Rule 110, Section 11 of the 2000 Rules of Criminal Procedure, as long as it alleges that the offense was committed "at any time as near to the actual date at which the offense was committed," an information is sufficient. The doctrine was reiterated with greater firmness in People v. Salalima32 and in People v. Lizada.33 In the case under review, the information in Criminal Case No. 2638 alleged that the rape of BBB transpired "sometime and between January 1992 up to December 6, 1998 in Barangay Codon, Municipality of San Andres, Province of Catanduanes." In Criminal Case No. 2650, the information averred that "from sometime in January 1990 up to December 1998 in Barangay Codon, Municipality of San Andres, Province of Catanduanes," AAA was raped by appellant. To the mind of the Court, the recitals in the informations sufficiently comply with the constitutional requirement that the accused be informed of the nature and cause of the accusation against him. In People v. Garcia,34 the Court upheld a conviction for ten counts of rape based on an Information which alleged that the accused committed multiple rapes "from November 1990 up to July 21, 1994." In People v. Espejon,35 the Court found the appellant liable for rape under an information charging that he perpetrated the offense "sometime in the year 1982 and dates subsequent thereto" and "sometime in the year 1995 and subsequent thereto." Indeed, this Court has ruled that allegations that rapes were committed "before and until October 15, 1994,"36"sometime in the year 1991 and the days thereafter,"37 and "on or about and sometime in the year 1988"38constitute sufficient compliance with Rule 110, Section 11 of the 2000 Rules of Criminal Procedure. More than that, the Court notes that the matter of particularity of the dates in the information is being raised for the first time on appeal. The rule is well-entrenched in this jurisdiction that objections as to matter of form or substance in the information cannot be made for the first time on appeal.39 Appellant failed to raise the issue of defective informations before the trial court. He could have moved to quash the informations or at least for a bill of particulars. He did not. Clearly, he slumbered on his rights and awakened too late. Too, appellant did not object to the presentation of the evidence for the People contending that the offenses were committed "sometime and between January 1992 up to December 6, 1998" for Criminal Case No. 2632 and "sometime in January 1990, up to December 1998" in Criminal Case No. 2650. On the contrary, appellant actively participated in the trial, offering denial and alibi as his defenses. Simply put, he cannot now be heard to complain that he was unable to defend himself in view of the vagueness of the recitals in the informations. We now tackle appellant's convictions for the multiple rape of AAA and BBB.

In an effort to exculpate himself, appellant contends that the charges for rape are mere fabrications and lies. He insists his daughters were instigated by his father-in-law to file the complaints. According to appellant, his father-in-law has an axe to grind against him. His in-law disdained him from the very beginning and wanted him out of CCC's life. In reviewing rape cases, the Court is guided by the following jurisprudential guidelines: (a) an accusation of rape can be made with facility; it is difficult to prove but more difficult for the person accused, though innocent, to disprove; (b) due to the nature of the crime of rape in which only two persons are usually involved, the testimony of complainant must be scrutinized with extreme caution; and (c) the evidence for the prosecution must stand or fall on its own merits and cannot be allowed to draw strength from the weakness of the evidence for the defense.40 Tersely put, the credibility of the offended party is crucial in determining the guilt of a person accused of rape. By the very nature of this crime, it is usually only the victim who can testify as to its occurrence. Thus, in rape cases, the accused may be convicted solely on the basis of the testimony of the victim, provided that such testimony is credible, natural, convincing and consistent with human nature and the normal course of things. Elsewise stated, the lone testimony of the offended party, if credible, suffices to warrant a conviction for rape.41 In her testimony before the trial court, AAA narrated: Q Why, when were you particularly raped by your father? A Since 1990, when I was in Grade I until I was in Grade VI, Sir. Q When you were in Grade I, how old were you then? A Seven (7) years old, Sir. Q Can you remember the first time, you said your father raped you in 1990? A I could no longer remember the date, Sir. Q But how did your father rape you, do you remember how he raped you in 1990, the first time? A Yes, Sir. Q Could you please tell us how he raped you for the first time? A I was croaching with raised buttocks, Sir. Q Do you remember where did he tell you to make that position? A No, Sir. Q Where particularly in your house? A In our room, Sir.

Q Do you still remember the date, the first time he raped you? A No, Sir. Q Who were with you in your house during that time? A No one, Sir, because all my other siblings are playing outside the house, and my mother was at work. Q When you were in that position with your buttocks raised and hands and knees on the floor, what did your father do next? A He positioned behind me and s[tar]ted raping me, Sir. Q When you used the term "rape," what do you mean? A He inserted his penis into my vagina, Sir. Q You mean your father inserted his penis to your vagina? A Yes, Sir. Q Now after that first time, do you remember the second time that he did it to you? A I could not remember anymore, Sir. Q Do you remember how long the period was between the first and the second time he raped you? A I could not longer remember, Sir. COURT Fiscal, we are only trying here the rape that occurred on March 25, so if you can prove to us really, maybe several times before that, the court cannot do something about that, because it is not included in the information. AYO Q So when was the last time that your father raped you? A When I graduated from the elementary school, Sir. Q When was that? A March 24, 1996, Sir. Q Between the first time that your father raped you and the last time that your father raped you, did you not report this to anybody, the thing that your father had been doing to you?

A I did not report this to anybody, Sir. Q Why? A Because I was threatened by my father that if we tell this matter to anybody, he would not only kill me but the rest of us, Sir. Q What other things did your father do when you said that he raped you, whenever your father raped you, you said you have been raped by your father in the time that you are in Grade I up to the time that you were in Grade VI, what did your father do to you? CABRERA The question is vague, because there is no definite date. COURT Recess for ten (10) minutes. COURT (After ten minutes) Court session resumed. AYO Q Do you remember the last time that your father raped you? A March 25, 1996, Sir. Q Where? A In our house, Sir. Q How old were you then? A Thirteen (13) years old, Sir. Q How did he rape you? A I was croaching with raised buttocks, Sir. Q And what did he do again when you are in that position? A He told me to remove my shorts and my panty, Sir. Q And did you do it? A Yes, Sir. Q Then what did he do next?

A He positioned behind me and he raped me, Sir. Q In that position while he was raping you, where was (sic) his hands? A His hands were on my back, Sir.42 Upon the other hand, BBB testified thus: AYO Q Now, Miss witness, you said your first rape by your father in 1992, do you remember the specific time when he first raped you? A I could no longer recall the date, because that has been sometime already, Sir. Q In 1992, were you already in school then? A Yes, Sir. Q What grade were you in when your father first raped you? A Grade II, Sir. Q Do you recall the circumstances when you were first raped by your father in 1992? A I was made to lie on top of my father, Sir. Q When you used the term "Pig baka-baka," will you please demonstrate to us how it is done? A (Witness demonstrating by kneeling and had her two hands on the floor, a position similar to four-legged animal, and she stated that her father is at her rear portion). Q And that was the first time your father raped with that position? A Yes, Sir. Q And what clothes were you wearing at that time when you were at that position, if you can still remember? A Yes, Sir, I can remember, I was wearing shorts. Q How about your father, do you remember what clothes he was wearing in that position? A He was wearing maong pants, Sir. Q And what was your father doing aside from having that position? A He removed my shorts and panty, Sir.

Q And after removing your shorts and panty, what did he do? A My father inserted his penis in my vagina, Sir. Q That was the first time you said he raped you? A Yes, Sir. Q Do you remember the date again, the first time that he raped you? A I could no longer remember the exact date, Sir. Q You could only remember the month and the year? A Yes, Sir, I could not remember the date, but I remember only the month and the year. Q How about the second time, do you remember when he raped you, the second time? A I could not, Sir.43 On cross-examination, BBB stated that: CABRERA Q You said you were allegedly raped by your own father, sometime in 1992, will you tell us what time is that alleged incident committed to you? A About 2:00 p.m., Sir. Q And who were the persons in the house, at around 2:00 o'clock in the afternoon? A The two of us only, Sir. Q Where were your companions in the home? A By that time, my mother is working in the farm, my ate is in school, and the rest of my siblings are playing outside, Sir. Q What was your age then at the time you were allegedly raped? A I was eight years old, Sir. Q You were never forced to have that position of "baka-bakahan"? A I was forced to do that, Sir. Q You were only told in mild manner, correct? A He kepts (sic) on telling me that I should do that position, although I don't like it, he kepts (sic) on prodding me, Sir.

Q At that time your father was telling you on a very low voice, because you were near to the children who are playing? A They were playing, Sir. Q Will you describe to us your house, what is the elevation of your house from the ground floor? A The flooring of our house is quite elevated. (Witness is demonstrating a height of about one [1] foot). Q Who were those children playing outside the house? A My three (3) siblings are playing outside the house, Sir. Q Your house has a window fronting the yard, correct? A Yes, Sir. Q And that yard was the playing ground of the children while your father was telling you that position of "baka-bakahan"? A They were playing in our yard, but they are playing near the house of our neighbor, Sir. Q How far is the house of your neighbor to your house? A (Witness demonstrating a distance of one two-arms length). Q And those children could hear what your father is saying? A They could not have heard what my father said, because they were playing, Sir. Q Why, what kind of game they are playing? A They were playing hide and seek, Sir. Q What time did you eat your lunch? A I took my lunch at 11:00 o'clock a.m., Sir. Q Will you tell us what was the nature of your father's work at that time you were allegedly raped? A He is jobless, Sir. Q Who is the one providing for your subsistence? A My mother, Sir. Q From where does your mother get your subsistence?

A She is doing some laundry works and works in the farm, Sir. Q If your story is correct that you were allegedly raped, will you tell us what happened to your vagina after the alleged rape? A My vagina became painful, Sir. Q Was there blood that oozed in your vagina? A I do not know if there was blood, what I could feel was the pain, Sir. Q After the alleged intercourse, did you wear your panty? A Yes, Sir. Q After the rape, what time did your mother arrive in your home? A My mother arrived at about 4:00 o'clock in the afternoon, Sir. Q Since you were still a child, if your story is correct, why did you not tell your mother that you were allegedly raped at 2:00 o'clock in the afternoon? A I did not tell my mother because he threatened me, Sir. Q Were you threatened before, during, or after the rape? A Before I was raped, Sir. Q And you were silent after the rape, he did not threaten you anymore? A Yes, Sir, he threatened me again after he committed the rape. Q Would you tell us the exact words, what did your father tell you? A He told me that if you will tell anybody, I am going to kill all of you, Sir. Q Was there any occasion on the part of your mother and you that you were alone without the presence of your father, after the rape? A There was none, Sir. Q You mean your father was always in your house? A There are times that he stays outside the house, he is jobless, he hangs around, Sir. Q After you were allegedly raped, did you have any occasion in the evening to talk to your mother immediately after this alleged rape? A There are, but then I could not tell my mother, because I was afraid of my father, Sir.

Q But there was an occasion that you were together with your mother and you could have told her what happened to you, is that correct? A Yes, there were occasions and opportunities that I could tell my mother, but I could not because of the threat of my father, Sir. Q Was there any occasion that actually happened after that threat when you were harmed by your father? A Yes, Sir. Q When was that? A Right after that evening, I did not do anything wrong, he just punished us, because he is not tempered, Sir. Q Your father is not insane, he will not do anything to you without any reason? A Yes, because every time he has no money, he becomes ill tempered, because he wanted to gamble, Sir. Q You are a young child then, is it not a fact that as a loving father he tried to discipline you, because of your mischievous acts? A We do not considered that a discipline, although we feel we did not do anything wrong, he keeps on punishing us, because he is ill tempered, Sir. Q Where was your mother when your father is trying to harm you? A She is at work, Sir. Q You mean he tried to harm you when your mother is out? A When my mother is around, he punishes us every time we did something wrong, but then he does that too when my mother is not around, Sir. Q Do you tell that to your mother that your father punished you without any reason? A Yes, Sir. Q Will you tell us the date, the first you were abused by your father in the year 1992? A I could no longer remember the date, Sir. Q But you can recall the fifteen (15) times? A Yes, Sir. Q What is important to you is the fifteen (15) times, but the first rape is not important to you?

A Yes, Sir. Q You said you were last raped on February 16, 1998, is that correct? A No, Sir, December 16, 1998. February 16 was when we reported to the police. Q This last incident, did you tell your mother about this? A Yes, Sir. Q And what did your mother say? A My mother told us that we report the matter, but we told her that we could not manage to do it, Sir. Q How were you raped on December 6, 1998? A He used his finger, Sir. Q Was there any nail in the finger? A Yes, Sir. Q And how did you feel when your father used his finger? A It is painful, Sir. Q What he used is finger only? A Yes, Sir. Q Could it be possible that there was inside your vagina and your father is trying to remove it? A There is none, Sir.44 (Underscoring supplied) The trial court observed that AAA's and BBB's testimonies bear the hallmarks of truth. Their testimonies are "spontaneous, convincing and highly-credible."45 We find no cogent reason not to apply here the oft-repeated rule that the matter of assigning values to the declaration of witnesses on the stand is a matter best left to the discretion of the trial court. The trial court has the advantage of observing the witnesses through the different indicators of truthfulness or falsehood, such as the angry flush of an insisted assertion or the sudden pallor of a discovered lie or the tremulous mutter of a reluctant answer or the forthright tone of a ready reply; or the furtive glance, the blush of conscious shame, the hesitation, the sincere or the flippant or sneering tone, the heat, the calmness, the yawn, the sigh, the candor or lack of it, the scant or full realization of the solemnity of an oath, the carriage and mien.46 This doctrine assumes greater significance when the determination of the trial court on the credibility of a witness has been affirmed by the appellate court.47 The Court has consistently ruled that no young girl would concoct a sordid tale of defloration at the hands of her own father, undergo medical examination, then subject herself to the stigma and

embarrassment of a public trial, if her motive were other than a fervent desire to seek justice.48 A rape victim's testimony against her parent is entitled to great weight since Filipino children have a natural reverence and respect for their elders. These values are so deeply ingrained in Filipino families, and it is unthinkable for a daughter to brazenly concoct a story of rape if such were not true.49 Certainly, a rape victim or any other member of her family would not dare to publicly expose the dishonor of the family, more specifically, if such accusation is against a fellow member of the family, unless the crime was, in fact, committed.50 We sustain the trial court and the CA's rejection of appellant's defense founded on denial and alibi. Denial and alibi, being weak defenses, cannot overcome the positive testimonies of the offended parties and their witnesses. As this Court has reiterated often enough, denial and alibi cannot prevail over positive identification of the accused by the prosecution witnesses.51 The positive, consistent and straightforward testimonies of the victims and the other witnesses for the People sufficiently established appellant's culpability. In order to merit credibility, alibi must be buttressed by strong evidence of non-culpability. Verily, for the said defense to prosper, accused must prove not only that he was at some other place at the time of the commission of the crime, but also that it was physically impossible for him to be at the locus criminis or its immediate vicinity.52Appellant dismally failed to discharge this onus. The trial court and the CA, however, both blundered in convicting appellant of multiple rape of AAA and BBB, from January 1990 to December 1998 and from January 1992 up to December 6, 1998, respectively. The RTC and the CA convicted appellant of multiple rapes under two separate informations, Criminal Cases Nos. 2638 and 2650. However, both the trial and appellate courts erroneously sentenced him to a single death penalty for each information. We find that appellant is guilty of two qualified rapes, instead of multiple rapes under Criminal Case No. 2650, and only one qualified rape, not multiple, under Criminal Case No. 2638. The legal basis for conviction for as many offenses as are charged and proved is Section 3, Rule 120 of the 2000 Rules of Criminal Procedure.53 It is axiomatic that each and every charge of rape is a separate and distinct crime. Verily, each of the alleged incidents of rape charged should be proven beyond reasonable doubt.54 In People v. Matugas,55 the Court aptly ruled: This Court cannot thus sustain the conviction of accused-appellant for 29 counts of rape because only two incidents were sufficiently proven by the prosecution. While we do not doubt that she was raped on other dates, we cannot ascertain the exact number of times she was actually raped. It must be remembered that each and every charge of rape is a separate and distinct crime so that each of the 27 other alleged incidents of rape charged should be proven beyond reasonable doubt. If, as complainant claimed, the number could be more, the possibility that it could be much less than 27 cannot be discounted.56 In People v. De la Torre,57 the Court held that: Each and every charge of rape is a separate and distinct crime; hence, each of the eight other rape charges should be proven beyond reasonable doubt. The prosecution is required to establish, by the necessary quantum of proof, the elements of rape for each charge. Baby Jane's testimony on the first rape charge was explicit, detailing the participation of each appellant in the offense and clearly illustrating all the elements of the offense of rape.

However her simple assertion that the subsequent rapes occurred in exactly the same manner as in previous incidents is clearly inadequate and grossly insufficient to establish to a degree of moral certainty the guilt of the appellants insofar as the eight rape charges are concerned. Her testimony was too general as it failed to focus on material details as to how each of the subsequent acts was committed. Even her testimony on cross-examination did not add anything to support her accusations of subsequent rape. Thus, only the rape alleged to have been committed on September 1992 was proven beyond reasonable doubt and the appellants may be penalized only for this offense.58 In the case under review, the evidence bear out that what were proved by the People beyond reasonable doubt in Criminal Case No. 2650 were the rapes committed by appellant on AAA sometime in 1990 and then again on March 25, 1996. AAA was categorical that she was first raped by appellant sometime in 1990. Her account of the first rape was vivid, candid and straightforward. She further disclosed that appellant repeatedly abused her. However, when asked by the court to clarify her claim that the sexual abuses were repeated, AAA failed to supply the details. But she was able to recount the last incident of rape on March 25, 1996. According to her, that day was of significance to her since she graduated from primary school on that day.59 Applying De la Torre, We hold that AAA's assertion that the subsequent rapes occurred in exactly the same manner as in previous incidents is clearly inadequate and grossly insufficient to establish to a degree of moral certainty the guilt of appellant insofar as the other rape incidents are concerned. Her testimony was too general as it failed to focus on material details as to how each of the subsequent acts was committed. In fine, appellant should have been convicted, in Criminal Case No. 2650, only of the qualified rape of AAA sometime in 1990 and then again on March 25, 1996. With respect to private complainant BBB in Criminal Case No. 2638, what is extant from the records is that appellant succeeded in raping her in January 1992. BBB, like AAA, failed to give an account of the alleged rape subsequent to January 1992 when she testified in the court below.60 As with AAA, We hold that BBB's account of the rapes subsequent to January 1992 but before December 6, 1998 is too general and unconvincing. Likewise borne by the records is the insertion of appellant's finger into BBB's vagina on December 6, 1998. BBB testified that appellant raped her for the last time on December 6, 1998. When asked by the court to clarify what she meant, BBB disclosed that appellant inserted his finger into her vagina.61 What appellant did was rape by sexual assault, punishable under Article 266-A, paragraph 2 of the Revised Penal Code, as amended by Republic Act (R.A.) No. 8353. The said law provides: Art. 266-A. Rape; when and how committed. ± Rape is committed ± 1) By a man who shall have carnal knowledge of a woman under any of the following circumstances: a) Through force, threat or intimidation; b) When the offended party is deprived of reason or otherwise unconscious; c) By means of fraudulent machination or grave abuse of authority; and d) When the offended party is under twelve (12) years of age or is demented, even though none of the circumstances mentioned above be present.

2) By any person who, under any of the circumstances mentioned in paragraph 1 hereof, shall commit an act of sexual assault by inserting his penis into another person's mouth or anal orifice, or any instrument or object, into the genital or anal orifice of another person.62 (Underscoring supplied) Rape by sexual assault was introduced into our penal system via the amendatory Anti-Rape Law of 1997 (R.A. No. 8353), which took effect on October 22, 1997. With these amendments, rape was reclassified as a crime against person and not merely a crime against chastity.63 Considering that the law was already in force at the time of the insertion of appellant's finger into BBB's vagina on December 6, 1998, he should have been prosecuted and tried for rape by sexual assault and not under the traditional definition of rape. The People, however, failed in this regard. That is fatal. Sections 8 and 9 of the 2000 Rules of Criminal Procedure state: Sec. 8. Designation of the offense. ± The complaint or information shall state the designation of the offense given by the statute, aver the acts or omissions constituting the offense, and specify its qualifying and aggravating circumstances. If there is no designation of the offense, reference shall be made to the section or subsection of the statute punishing it. Sec. 9. Cause of the accusation. ± The acts or omissions complained of as constituting the offense and the qualifying and aggravating circumstances must be stated in ordinary and concise language and not necessarily in the language used in the statute but in terms sufficient to enable a person of common understanding to know what offense is being charged as well as its qualifying and aggravating circumstances for the court to pronounce judgment. Under the new rules, the information or complaint must state the designation of the offense given by the statute and specify its qualifying and generic aggravating circumstances. Otherwise stated, the accused will not be convicted for the offense proved during the trial if it was not properly alleged in the information. Although the rule took effect on December 1, 2000, the same may be applied retroactively because it is a cardinal rule that rules of criminal procedure are given retroactive application insofar as they benefit the accused.64 In sum, in Criminal Case No. 2638, appellant should have been convicted only of the qualified rape of BBB in January 1992. The rape by sexual assault committed on December 6, 1998, although proven, should not have been considered by the trial and appellate courts for lack of a proper allegation in the information. We go now to the penalty and the award of damages. Appellant is liable for the rape of AAA sometime in 1990 and on March 25, 1996. He is also guilty of raping BBB in January 1992. At that time, the law penalizing rape was still Article 335 of the Revised Penal Code, as amended by R.A. No. 7659. The said law provides: Art. 335. When and how rape is committed. xxxx

The death penalty shall also be imposed if the crime of rape is committed with any of the following attendant circumstances: 1. When the victim is under eighteen (18) years of age and the offender is a parent, ascendant, step-parent, guardian, relative by consanguinity or affinity within the third civil degree, or the common-law spouse of the parent of the victim. In view of the passage of R.A. No. 9346 entitled, "An Act Prohibiting the Imposition of Death Penalty in the Philippines," the death penalty should be downgraded. Pursuant to Section 2 of the said law, the penalty to be meted out to appellant shall be reclusion perpetua. Said section reads: Section 2. In lieu of the death penalty, the following shall be imposed: (a) the penalty of reclusion perpetua, when the law violated makes use of the nomenclature of the penalties of the Revised Penal Code; or (b) the penalty of life imprisonment, when the law violated does not make use of the nomenclature of the penalties of the Revised Penal Code. Notwithstanding the reduction of the penalty imposed on appellant, he is not eligible for parole following Section 3 of the said law, which provides: Section 3. Persons convicted of offenses punished with reclusion perpetua, or whose sentences will be reduced to reclusion perpetua, by reason of this Act, shall not be eligible for parole under Act No. 4103, otherwise known as the Indeterminate Sentence Law, as amended. With regard to the award of damages, the same must be modified. The CA correctly increased the amount of indemnity from P50,000.00 to P75,000.00 each for AAA and BBB. Civil indemnity of P75,000.00 is warranted if the crime is qualified by circumstances which warrant the imposition of the death penalty.65 The award of additionalP25,000.00 each by way of exemplary damages deserves affirmance due to the presence of the qualifying circumstances of minority and relationship.66 However, the CA erred in affirming the RTC award of moral damages of P50,000.00 which should be increased toP75,000.00 without need of pleading or proof of basis.67 WHEREFORE, the appealed judgment is AFFIRMED WITH MODIFICATION, as follows: (1) In Criminal Case No. 2650, appellant Jerry Nazareno is hereby found GUILTY of two counts of qualified rape and is sentenced to reclusion perpetua for each felony, without eligibility for parole. He is further ordered to indemnify the victim in the amount of P75,000.00, another P75,000.00 in moral damages and P25,000.00 in exemplary damages, for each count. (2) In Criminal Case No. 2638, appellant is found GUILTY of one count of qualified rape and is sentenced toreclusion perpetua without eligibility for parole. He is likewise ordered to pay the complainant P75,000.00 as civil indemnity, P75,000.00 as moral damages and P25,000.00 as exemplary damages. SO ORDERED.

Puno, C.J., Quisumbing, Ynares-Santiago, Carpio, Austria-Martinez, Corona, Carpio-Morales, Azcuna, Tinga, Chico-Nazario, Velasco, Jr., Nachura, Leonardo-de Castro, Brion, JJ., concur. Republic of the Philippines SUPREME COURT SECOND DIVISION G.R. No. 139736 October 17, 2005 BANK OF THE PHILIPPINE ISLANDS, Petitioner, vs. COMMISSIONER OF INTERNAL REVENUE, Respondent. DECISION CHICO-NAZARIO, J.: This Petition for Review on Certiorari, under Rule 45 of the 1997 Rules of Civil Procedure, assails the Decision of the Court of Appeals in CA-G.R. SP No. 51271, dated 11 August 1999,1 which reversed and set aside the Decision of the Court of Tax Appeals (CTA), dated 02 February 1999,2 and which reinstated Assessment No. FAS-5-85-89-002054 requiring petitioner Bank of the Philippine Islands (BPI) to pay the amount of P28,020.00 as deficiency documentary stamp tax (DST) for the taxable year 1985, inclusive of the compromise penalty. There is hardly any controversy as to the factual antecedents of this Petition. Petitioner BPI is a commercial banking corporation organized and existing under the laws of the Philippines. On two separate occasions, particularly on 06 June 1985 and 14 June 1985, it sold United States (US) $500,000.00 to the Central Bank of the Philippines (Central Bank), for the total sales amount of US$1,000,000.00. On 10 October 1989, the Bureau of Internal Revenue (BIR) issued Assessment No. FAS-5-85-89002054,3 finding petitioner BPI liable for deficiency DST on its afore-mentioned sales of foreign bills of exchange to the Central Bank, computed as follows ± 1985 Deficiency Documentary Stamp Tax Foreign Bills of Exchange««««««««««.. P 18,480,000.00 Tax Due Thereon: P18,480,000.00 x P0.30 (Sec. 182 NIRC). P200.00 Add: Suggested compromise penalty««««.«« TOTAL AMOUNT DUE AND COLLECTIBLE«.

27,720.00

300.00 P 28,020.00

Petitioner BPI received the Assessment, together with the attached Assessment Notice,4 on 20 October 1989.

Petitioner BPI, through its counsel, protested the Assessment in a letter dated 16 November 1989, and filed with the BIR on 17 November 1989. The said protest letter is reproduced in full below ± November 16, 1989 The Commissioner of Internal Revenue Quezon City Attention of: Mr. Pedro C. Aguillon Asst. Commissioner for Collection Sir: On behalf of our client, Bank of the Philippine Islands (BPI), we have the honor to protest your assessment against it for deficiency documentary stamp tax for the year 1985 in the amount of P28,020.00, arising from its sale to the Central Bank of U.S. $500,000.00 on June 6, 1985 and another U.S. $500,000.00 on June 14, 1985. 1. Under established market practice, the documentary stamp tax on telegraphic transfers or sales of foreign exchange is paid by the buyer. Thus, when BPI sells to any party, the cost of documentary stamp tax is added to the total price or charge to the buyer and the seller affixes the corresponding documentary stamp on the document. Similarly, when the Central Bank sells foreign exchange to BPI, it charges BPI for the cost of the documentary stamp on the transaction. 2. In the two transactions subject of your assessment, no documentary stamps were affixed because the buyer, Central Bank of the Philippines, was exempt from such tax. And while it is true that under P.D. 1994, a proviso was added to sec. 222 (now sec. 186) of the Tax Code "that whenever one party to a taxable document enjoys exemption from the tax herein imposed, the other party thereto who is not exempt shall be the one directly liable for the tax," this proviso (and the other amendments of P.D. 1994) took effect only on January 1, 1986, according to sec. 49 of P.D. 1994. Hence, the liability for the documentary stamp tax could not be shifted to the seller. In view of the foregoing, we request that the assessment be revoked and cancelled. Very truly yours, PADILLA LAW OFFICE By: (signed) SABINO PADILLA, JR.5 Petitioner BPI did not receive any immediate reply to its protest letter. However, on 15 October 1992, the BIR issued a Warrant of Distraint and/or Levy6 against petitioner BPI for the assessed deficiency DST for taxable year 1985, in the amount of P27,720.00 (excluding the compromise penalty of P300.00). It served the Warrant on petitioner BPI only on 23 October 1992.7

Then again, petitioner BPI did not hear from the BIR until 11 September 1997, when its counsel received a letter, dated 13 August 1997, signed by then BIR Commissioner Liwayway VinzonsChato, denying its "request for reconsideration," and addressing the points raised by petitioner BPI in its protest letter, dated 16 November 1989, thus ± In reply, please be informed that after a thorough and careful study of the facts of the case as well as the law and jurisprudence pertinent thereto, this Office finds the above argument to be legally untenable. It is admitted that while industry practice or market convention has the force of law between the members of a particular industry, it is not binding with the BIR since it is not a party thereto. The same should, therefore, not be allowed to prejudice the Bureau of its lawful task of collecting revenues necessary to defray the expenses of the government. (Art. 11 in relation to Art. 1306 of the New Civil Code.) Moreover, let it be stated that even before the amendment of Sec. 222 (now Sec. 173) of the Tax Code, as amended, the same was already interpreted to hold that the other party who is not exempt from the payment of documentary stamp tax liable from the tax. This interpretation was further strengthened by the following BIR Rulings which in substance state: 1. BIR Unnumbered Ruling dated May 30, 1977 ± "x x x Documentary stamp taxes are payable by either person, signing, issuing, accepting, or transferring the instrument, document or paper. It is now settled that where one party to the instrument is exempt from said taxes, the other party who is not exempt should be liable." 2. BIR Ruling No. 144-84 dated September 3, 1984 ± "x x x Thus, where one party to the contract is exempt from said tax, the other party, who is not exempt, shall be liable therefore. Accordingly, since A.J.L. Construction Corporation, the other party to the contract and the one assuming the payment of the expenses incidental to the registration in the vendee¶s name of the property sold, is not exempt from said tax, then it is the one liable therefore, pursuant to Sec. 245 (now Sec. 196), in relation to Sec. 222 (now Sec. 173), both of the Tax Code of 1977, as amended." Premised on all the foregoing considerations, your request for reconsideration is hereby DENIED.8 Upon receipt of the above-cited letter from the BIR, petitioner BPI proceeded to file a Petition for Review with the CTA on 10 October 1997;9 to which respondent BIR Commissioner, represented by the Office of the Solicitor General, filed an Answer on 08 December 1997.10 Petitioner BPI raised in its Petition for Review before the CTA, in addition to the arguments presented in its protest letter, dated 16 November 1989, the defense of prescription of the right of respondent BIR Commissioner to enforce collection of the assessed amount. It alleged that respondent BIR Commissioner only had three years to collect on Assessment No. FAS-5-85-89002054, but she waited for seven years and nine months to deny the protest. In her Answer and subsequent Memorandum, respondent BIR Commissioner merely reiterated her position, as stated in her letter to petitioner BPI, dated 13 August 1997, which denied the latter¶s protest; and remained silent as to the expiration of the prescriptive period for collection of the assessed deficiency DST. After due trial, the CTA rendered a Decision on 02 February 1999, in which it identified two primary issues in the controversy between petitioner BPI and respondent BIR Commissioner: (1) whether or not the right of respondent BIR Commissioner to collect from petitioner BPI the alleged deficiency

DST for taxable year 1985 had prescribed; and (2) whether or not the sales of US$1,000,000.00 on 06 June 1985 and 14 June 1985 by petitioner BPI to the Central Bank were subject to DST. The CTA answered the first issue in the negative and held that the statute of limitations for respondent BIR Commissioner to collect on the Assessment had not yet prescribed. In resolving the issue of prescription, the CTA reasoned that ± In the case of Commissioner of Internal Revenue vs. Wyeth Suaco Laboratories, Inc., G.R. No. 76281, September 30, 1991, 202 SCRA 125, the Supreme Court laid to rest the first issue. It categorically ruled that a "protest" is to be treated as request for reinvestigation or reconsideration and a mere request for reexamination or reinvestigation tolls the prescriptive period of the Commissioner to collect on an assessment. . . ... In the case at bar, there being no dispute that petitioner filed its protest on the subject assessment on November 17, 1989, there can be no conclusion other than that said protest stopped the running of the prescriptive period of the Commissioner to collect. Section 320 (now 223) of the Tax Code, clearly states that a request for reinvestigation which is granted by the Commissioner, shall suspend the prescriptive period to collect. The underscored portion above does not mean that the Commissioner will cancel the subject assessment but should be construed as when the same was entertained by the Commissioner by not issuing any warrant of distraint or levy on the properties of the taxpayer or any action prejudicial to the latter unless and until the request for reinvestigation is finally given due course. Taking into consideration this provision of law and the aforementioned ruling of the Supreme Court in Wyeth Suaco which specifically and categorically states that a protest could be considered as a request for reinvestigation, We rule that prescription has not set in against the government.11 The CTA had likewise resolved the second issue in the negative. Referring to its own decision in an earlier case,Consolidated Bank & Trust Co. v. The Commissioner of Internal Revenue,12 the CTA reached the conclusion that the sales of foreign currency by petitioner BPI to the Central Bank in taxable year 1985 were not subject to DST ± From the abovementioned decision of this Court, it can be gleaned that the Central Bank, during the period June 11, 1984 to March 9, 1987 enjoyed tax exemption privilege, including the payment of documentary stamp tax (DST) pursuant to Resolution No. 35-85 dated May 3, 1985 of the Fiscal Incentive Review Board. As such, the Central Bank, as buyer of the foreign currency, is exempt from paying the documentary stamp tax for the period above-mentioned. This Court further expounded that said tax exemption of the Central Bank was modified beginning January 1, 1986 when Presidential Decree (P.D.) 1994 took effect. Under this decree, the liability for DST on sales of foreign currency to the Central Bank is shifted to the seller. Applying the above decision to the case at bar, petitioner cannot be held liable for DST on its 1985 sales of foreign currencies to the Central Bank, as the latter who is the purchaser of the subject currencies is the one liable thereof. However, since the Central Bank is exempt from all taxes during 1985 by virtue of Resolution No. 35-85 of the Fiscal Incentive Review Board dated March 3, 1985, neither the petitioner nor the Central Bank is liable for the payment of the documentary stamp tax for the former¶s 1985 sales of foreign currencies to the latter. This aforecited case of Consolidated Bank vs. Commissioner of Internal Revenue was affirmed by the Court of Appeals in its decision dated March 31, 1995, CA-GR Sp. No. 35930. Said decision was in turn affirmed by the Supreme Court in

its resolution denying the petition filed by Consolidated Bank dated November 20, 1995 with the Supreme Court under Entry of Judgment dated March 1, 1996.13 In sum, the CTA decided that the statute of limitations for respondent BIR Commissioner to collect on Assessment No. FAS-5-85-89-002054 had not yet prescribed; nonetheless, it still ordered the cancellation of the said Assessment because the sales of foreign currency by petitioner BPI to the Central Bank in taxable year 1985 were tax-exempt. Herein respondent BIR Commissioner appealed the Decision of the CTA to the Court of Appeals. In its Decision dated 11 August 1999,14 the Court of Appeals sustained the finding of the CTA on the first issue, that the running of the prescriptive period for collection on Assessment No. FAS-5-85-89002054 was suspended when herein petitioner BPI filed a protest on 17 November 1989 and, therefore, the prescriptive period for collection on the Assessment had not yet lapsed. In the same Decision, however, the Court of Appeals reversed the CTA on the second issue and basically adopted the position of the respondent BIR Commissioner that the sales of foreign currency by petitioner BPI to the Central Bank in taxable year 1985 were subject to DST. The Court of Appeals, thus, ordered the reinstatement of Assessment No. FAS-5-85-89-002054 which required petitioner BPI to pay the amount of P28,020.00 as deficiency DST for taxable year 1985, inclusive of the compromise penalty. Comes now petitioner BPI before this Court in this Petition for Review on Certiorari, seeking resolution of the same two legal issues raised and discussed in the courts below, to reiterate: (1) whether or not the right of respondent BIR Commissioner to collect from petitioner BPI the alleged deficiency DST for taxable year 1985 had prescribed; and (2) whether or not the sales of US$1,000,000.00 on 06 June 1985 and 14 June 1985 by petitioner BPI to the Central Bank were subject to DST. I The efforts of respondent Commissioner to collect on Assessment No. FAS-5-85-89-002054 were already barred by prescription. Anent the question of prescription, this Court disagrees in the Decisions of the CTA and the Court of Appeals, and herein determines the statute of limitations on collection of the deficiency DST in Assessment No. FAS-5-85-89-002054 had already prescribed. The period for the BIR to assess and collect an internal revenue tax is limited to three years by Section 203 of the Tax Code of 1977, as amended,15 which provides that ± SEC. 203. Period of limitation upon assessment and collection. ± Except as provided in the succeeding section, internal revenue taxes shall be assessed within three years after the last day prescribed by law for the filing of the return, and no proceeding in court without assessment for the collection of such taxes shall be begun after the expiration of such period: Provided, That in a case where a return is filed beyond the period prescribed by law, the three-year period shall be counted from the day the return was filed. For the purposes of this section, a return filed before the last day prescribed by law for the filing thereof shall be considered as filed on such last day.16 The three-year period of limitations on the assessment and collection of national internal revenue taxes set by Section 203 of the Tax Code of 1977, as amended, can be affected, adjusted, or suspended, in accordance with the following provisions of the same Code ±

SEC. 223. ± Exceptions as to period of limitation of assessment and collection of taxes. ± (a) In the case of a false or fraudulent return with intent to evade tax or of failure to file a return, the tax may be assessed, or a proceeding in court for the collection of such tax may be begun without assessment, at any time within ten years after the discovery of the falsity, fraud, or omission: Provided, That in a fraud assessment which has become final and executory, the fact of fraud shall be judicially taken cognizance of in the civil or criminal action for the collection thereof. (b) If before the expiration of the time prescribed in the preceding section for the assessment of the tax, both the Commissioner and the taxpayer have agreed in writing to its assessment after such time the tax may be assessed within the period agreed upon. The period so agreed upon may be extended by subsequent written agreement made before the expiration of the period previously agreed upon. (c) Any internal revenue tax which has been assessed within the period of limitation aboveprescribed may be collected by distraint or levy or by a proceeding in court within three years following the assessment of the tax. (d) Any internal revenue tax which has been assessed within the period agreed upon as provided in paragraph (b) hereinabove may be collected by distraint or levy or by a proceeding in court within the period agreed upon in writing before the expiration of the three-year period. The period so agreed upon may be extended by subsequent written agreements made before the expiration of the period previously agreed upon. (e) Provided, however, That nothing in the immediately preceding section and paragraph (a) hereof shall be construed to authorize the examination and investigation or inquiry into any tax returns filed in accordance with the provisions of any tax amnesty law or decree.17 SEC. 224. Suspension of running of statute. ± The running of the statute of limitation provided in Section[s] 203 and 223 on the making of assessment and the beginning of distraint or levy or a proceeding in court for collection, in respect of any deficiency, shall be suspended for the period during which the Commissioner is prohibited from making the assessment or beginning distraint or levy or a proceeding in court and for sixty days thereafter; when the taxpayer requests for a reinvestigation which is granted by the Commissioner; when the taxpayer cannot be located in the address given by him in the return filed upon which a tax is being assessed or collected: Provided,That, if the taxpayer informs the Commissioner of any change in address, the running of the statute of limitations will not be suspended; when the warrant of distraint and levy is duly served upon the taxpayer, his authorized representative, or a member of his household with sufficient discretion, and no property could be located; and when the taxpayer is out of the Philippines.18 As enunciated in these statutory provisions, the BIR has three years, counted from the date of actual filing of the return or from the last date prescribed by law for the filing of such return, whichever comes later, to assess a national internal revenue tax or to begin a court proceeding for the collection thereof without an assessment. In case of a false or fraudulent return with intent to evade tax or the failure to file any return at all, the prescriptive period for assessment of the tax due shall be 10 years from discovery by the BIR of the falsity, fraud, or omission. When the BIR validly issues an assessment, within either the three-year or ten-year period, whichever is appropriate, then the BIR has another three years19 after the assessment within which to collect the national internal revenue tax due thereon by distraint, levy, and/or court proceeding. The assessment of the tax is deemed made and the three-year period for collection of the assessed tax begins to run on the date the assessment notice had been released, mailed or sent by the BIR to the taxpayer.20

In the present Petition, there is no controversy on the timeliness of the issuance of the Assessment, only on the prescription of the period to collect the deficiency DST following its Assessment. While Assessment No. FAS-5-85-89-002054 and its corresponding Assessment Notice were both dated 10 October 1989 and were received by petitioner BPI on 20 October 1989, there was no showing as to when the said Assessment and Assessment Notice were released, mailed or sent by the BIR. Still, it can be granted that the latest date the BIR could have released, mailed or sent the Assessment and Assessment Notice to petitioner BPI was on the same date they were received by the latter, on 20 October 1989. Counting the three-year prescriptive period, for a total of 1,095 days,21 from 20 October 1989, then the BIR only had until 19 October 1992 within which to collect the assessed deficiency DST. The earliest attempt of the BIR to collect on Assessment No. FAS-5-85-89-002054 was its issuance and service of a Warrant of Distraint and/or Levy on petitioner BPI. Although the Warrant was issued on 15 October 1992, previous to the expiration of the period for collection on 19 October 1992, the same was served on petitioner BPI only on 23 October 1992. Under Section 223(c) of the Tax Code of 1977, as amended, it is not essential that the Warrant of Distraint and/or Levy be fully executed so that it can suspend the running of the statute of limitations on the collection of the tax. It is enough that the proceedings have validly began or commenced and that their execution has not been suspended by reason of the voluntary desistance of the respondent BIR Commissioner. Existing jurisprudence establishes that distraint and levy proceedings are validly begun or commenced by the issuance of the Warrantand service thereof on the taxpayer.22 It is only logical to require that the Warrant of Distraint and/or Levy be, at the very least, served upon the taxpayer in order to suspend the running of the prescriptive period for collection of an assessed tax, because it may only be upon the service of the Warrant that the taxpayer is informed of the denial by the BIR of any pending protest of the said taxpayer, and the resolute intention of the BIR to collect the tax assessed. If the service of the Warrant of Distraint and/or Levy on petitioner BPI on 23 October 1992 was already beyond the prescriptive period for collection of the deficiency DST, which had expired on 19 October 1992, then what more the letter of respondent BIR Commissioner, dated 13 August 1997 and received by the counsel of the petitioner BPI only on 11 September 1997, denying the protest of petitioner BPI and requesting payment of the deficiency DST? Even later and more unequivocally barred by prescription on collection was the demand made by respondent BIR Commissioner for payment of the deficiency DST in her Answer to the Petition for Review of petitioner BPI before the CTA, filed on 08 December 1997.23 II There is no valid ground for the suspension of the running of the prescriptive period for collection of the assessed DST under the Tax Code of 1977, as amended. In their Decisions, both the CTA and the Court of Appeals found that the filing by petitioner BPI of a protest letter suspended the running of the prescriptive period for collecting the assessed DST. This Court, however, takes the opposing view, and, based on the succeeding discussion, concludes that there is no valid ground for suspending the running of the prescriptive period for collection of the deficiency DST assessed against petitioner BPI. A. The statute of limitations on assessment and collection of taxes is for the protection of the taxpayer and, thus, shall be construed liberally in his favor.

Though the statute of limitations on assessment and collection of national internal revenue taxes benefits both the Government and the taxpayer, it principally intends to afford protection to the taxpayer against unreasonable investigation. The indefinite extension of the period for assessment is unreasonable because it deprives the said taxpayer of the assurance that he will no longer be subjected to further investigation for taxes after the expiration of a reasonable period of time.24 As aptly explained in Republic of the Philippines v. Ablaza25 ± The law prescribing a limitation of actions for the collection of the income tax is beneficial both to the Government and to its citizens; to the Government because tax officers would be obliged to act promptly in the making of assessment, and to citizens because after the lapse of the period of prescription citizens would have a feeling of security against unscrupulous tax agents who will always find an excuse to inspect the books of taxpayers, not to determine the latter¶s real liability, but to take advantage of every opportunity to molest peaceful, law-abiding citizens. Without such a legal defense taxpayers would furthermore be under obligation to always keep their books and keep them open for inspection subject to harassment by unscrupulous tax agents. The law on prescription being a remedial measure should be interpreted in a way conducive to bringing about the beneficent purpose of affording protection to the taxpayer within the contemplation of the Commission which recommend the approval of the law. In order to provide even better protection to the taxpayer against unreasonable investigation, the Tax Code of 1977, as amended, identifies specifically in Sections 223 and 22426 thereof the circumstances when the prescriptive periods for assessing and collecting taxes could be suspended or interrupted. To give effect to the legislative intent, these provisions on the statute of limitations on assessment and collection of taxes shall be construed and applied liberally in favor of the taxpayer and strictly against the Government. B. The statute of limitations on assessment and collection of national internal revenue taxes may be waived, subject to certain conditions, under paragraphs (b) and (d) of Section 223 of the Tax Code of 1977, as amended, respectively. Petitioner BPI, however, did not execute any such waiver in the case at bar. According to paragraphs (b) and (d) of Section 223 of the Tax Code of 1977, as amended, the prescriptive periods for assessment and collection of national internal revenue taxes, respectively, could be waived by agreement, to wit ± SEC. 223. ± Exceptions as to period of limitation of assessment and collection of taxes. ± ... (b) If before the expiration of the time prescribed in the preceding section for the assessment of the tax, both the Commissioner and the taxpayer have agreed in writing to its assessment after such time the tax may be assessed within the period agreed upon. The period so agreed upon may be extended by subsequent written agreement made before the expiration of the period previously agreed upon. ... (d) Any internal revenue tax which has been assessed within the period agreed upon as provided in paragraph (b) hereinabove may be collected by distraint or levy or by a proceeding in court within the period agreed upon in writing before the expiration of the three-year period. The period so

agreed upon may be extended by subsequent written agreements made before the expiration of the period previously agreed upon.27 The agreements so described in the afore-quoted provisions are often referred to as waivers of the statute of limitations. The waiver of the statute of limitations, whether on assessment or collection, should not be construed as a waiver of the right to invoke the defense of prescription but, rather, an agreement between the taxpayer and the BIR to extend the period to a date certain, within which the latter could still assess or collect taxes due. The waiver does not mean that the taxpayer relinquishes the right to invoke prescription unequivocally.28 A valid waiver of the statute of limitations under paragraphs (b) and (d) of Section 223 of the Tax Code of 1977, as amended, must be: (1) in writing; (2) agreed to by both the Commissioner and the taxpayer; (3) before the expiration of the ordinary prescriptive periods for assessment and collection; and (4) for a definite period beyond the ordinary prescriptive periods for assessment and collection. The period agreed upon can still be extended by subsequent written agreement, provided that it is executed prior to the expiration of the first period agreed upon. The BIR had issued Revenue Memorandum Order (RMO) No. 20-90 on 04 April 1990 to lay down an even more detailed procedure for the proper execution of such a waiver. RMO No. 20-90 mandates that the procedure for execution of the waiver shall be strictly followed, and any revenue official who fails to comply therewith resulting in the prescription of the right to assess and collect shall be administratively dealt with. This Court had consistently ruled in a number of cases that a request for reconsideration or reinvestigation by the taxpayer, without a valid waiver of the prescriptive periods for the assessment and collection of tax, as required by the Tax Code and implementing rules, will not suspend the running thereof.29 In the Petition at bar, petitioner BPI executed no such waiver of the statute of limitations on the collection of the deficiency DST per Assessment No. FAS-5-85-89-002054. In fact, an internal memorandum of the Chief of the Legislative, Ruling & Research Division of the BIR to her counterpart in the Collection Enforcement Division, dated 15 October 1992, expressly noted that, "The taxpayer fails to execute a Waiver of the Statute of Limitations extending the period of collection of the said tax up to December 31, 1993 pending reconsideration of its protest. . ."30 Without a valid waiver, the statute of limitations on collection by the BIR of the deficiency DST could not have been suspended under paragraph (d) of Section 223 of the Tax Code of 1977, as amended. C. The protest filed by petitioner BPI did not constitute a request for reinvestigation, granted by the respondent BIR Commissioner, which could have suspended the running of the statute of limitations on collection of the assessed deficiency DST under Section 224 of the Tax Code of 1977, as amended. The Tax Code of 1977, as amended, also recognizes instances when the running of the statute of limitations on the assessment and collection of national internal revenue taxes could be suspended, even in the absence of a waiver, under Section 224 thereof, which reads ± SEC. 224. Suspension of running of statute. ± The running of the statute of limitation provided in Section[s] 203 and 223 on the making of assessment and the beginning of distraint or levy or a proceeding in court for collection, in respect of any deficiency, shall be suspended for the period during which the Commissioner is prohibited from making the assessment or beginning distraint or levy or a proceeding in court and for sixty days thereafter; when the taxpayer requests for a reinvestigation which is granted by the Commissioner; when the taxpayer cannot be located in the

address given by him in the return filed upon which a tax is being assessed or collected: Provided,That, if the taxpayer informs the Commissioner of any change in address, the running of the statute of limitations will not be suspended; when the warrant of distraint and levy is duly served upon the taxpayer, his authorized representative, or a member of his household with sufficient discretion, and no property could be located; and when the taxpayer is out of the Philippines.31 Of particular importance to the present case is one of the circumstances enumerated in Section 224 of the Tax Code of 1977, as amended, wherein the running of the statute of limitations on assessment and collection of taxes is considered suspended "when the taxpayer requests for a reinvestigation which is granted by the Commissioner." This Court gives credence to the argument of petitioner BPI that there is a distinction between a request for reconsideration and a request for reinvestigation. Revenue Regulations (RR) No. 12-85, issued on 27 November 1985 by the Secretary of Finance, upon the recommendation of the BIR Commissioner, governs the procedure for protesting an assessment and distinguishes between the two types of protest, as follows ± PROTEST TO ASSESSMENT SEC. 6. Protest. The taxpayer may protest administratively an assessment by filing a written request for reconsideration or reinvestigation. . . ... For the purpose of the protest herein ± (a) Request for reconsideration. ± refers to a plea for a re-evaluation of an assessment on the basis of existing records without need of additional evidence. It may involve both a question of fact or of law or both. (b) Request for reinvestigation. ± refers to a plea for re-evaluation of an assessment on the basis of newly-discovered or additional evidence that a taxpayer intends to present in the reinvestigation. It may also involve a question of fact or law or both. With the issuance of RR No. 12-85 on 27 November 1985 providing the above-quoted distinctions between a request for reconsideration and a request for reinvestigation, the two types of protest can no longer be used interchangeably and their differences so lightly brushed aside. It bears to emphasize that under Section 224 of the Tax Code of 1977, as amended, the running of the prescriptive period for collection of taxes can only be suspended by a request for reinvestigation, not a request for reconsideration. Undoubtedly, a reinvestigation, which entails the reception and evaluation of additional evidence, will take more time than a reconsideration of a tax assessment, which will be limited to the evidence already at hand; this justifies why the former can suspend the running of the statute of limitations on collection of the assessed tax, while the latter can not. The protest letter of petitioner BPI, dated 16 November 1989 and filed with the BIR the next day, on 17 November 1989, did not specifically request for either a reconsideration or reinvestigation. A close review of the contents thereof would reveal, however, that it protested Assessment No. FAS-585-89-002054 based on a question of law, in particular, whether or not petitioner BPI was liable for DST on its sales of foreign currency to the Central Bank in taxable year 1985. The same protest letter did not raise any question of fact; neither did it offer to present any new evidence. In its own letter to petitioner BPI, dated 10 September 1992, the BIR itself referred to the protest of petitioner

BPI as a request for reconsideration.32 These considerations would lead this Court to deduce that the protest letter of petitioner BPI was in the nature of a request for reconsideration, rather than a request for reinvestigation and, consequently, Section 224 of the Tax Code of 1977, as amended, on the suspension of the running of the statute of limitations should not apply. Even if, for the sake of argument, this Court glosses over the distinction between a request for reconsideration and a request for reinvestigation, and considers the protest of petitioner BPI as a request for reinvestigation, the filing thereof could not have suspended at once the running of the statute of limitations. Article 224 of the Tax Code of 1977, as amended, very plainly requires that the request for reinvestigation had been granted by the BIR Commissioner to suspend the running of the prescriptive periods for assessment and collection. That the BIR Commissioner must first grant the request for reinvestigation as a requirement for suspension of the statute of limitations is even supported by existing jurisprudence. In the case of Republic of the Philippines v. Gancayco,33 taxpayer Gancayco requested for a thorough reinvestigation of the assessment against him and placed at the disposal of the Collector of Internal Revenue all the evidences he had for such purpose; yet, the Collector ignored the request, and the records and documents were not at all examined. Considering the given facts, this Court pronounced that ± . . .The act of requesting a reinvestigation alone does not suspend the period. The request should first be granted, in order to effect suspension. (Collector vs. Suyoc Consolidated, supra; also Republic vs. Ablaza, supra). Moreover, the Collector gave appellee until April 1, 1949, within which to submit his evidence, which the latter did one day before. There were no impediments on the part of the Collector to file the collection case from April 1, 1949. . . .34 In Republic of the Philippines v. Acebedo,35 this Court similarly found that ± . . . [T]he defendant, after receiving the assessment notice of September 24, 1949, asked for a reinvestigation thereof on October 11, 1949 (Exh. A). There is no evidence that this request was considered or acted upon.In fact, on October 23, 1950 the then Collector of Internal Revenue issued a warrant of distraint and levy for the full amount of the assessment (Exh. D), but there was no follow-up of this warrant. Consequently, the request for reinvestigation did not suspend the running of the period for filing an action for collection. The burden of proof that the taxpayer¶s request for reinvestigation had been actually granted shall be on respondent BIR Commissioner. The grant may be expressed in communications with the taxpayer or implied from the actions of the respondent BIR Commissioner or his authorized BIR representatives in response to the request for reinvestigation. In Querol v. Collector of Internal Revenue,36 the BIR, after receiving the protest letters of taxpayer Querol, sent a tax examiner to San Fernando, Pampanga, to conduct the reinvestigation; as a result of which, the original assessment against taxpayer Querol was revised by permitting him to deduct reasonable depreciation. In another case, Republic of the Philippines v. Lopez,37 taxpayer Lopez filed a total of four petitions for reconsideration and reinvestigation. The first petition was denied by the BIR. The second and third petitions were granted by the BIR and after each reinvestigation, the assessed amount was reduced. The fourth petition was again denied and, thereafter, the BIR filed a collection suit against taxpayer Lopez. When the taxpayers spouses Sison, inCommissioner of Internal Revenue v. Sison,38 contested the assessment against them and asked for a reinvestigation, the BIR ordered the reinvestigation resulting in the issuance of an amended assessment. Lastly, inRepublic of the Philippines v. Oquias,39 the BIR granted taxpayer Oquias¶s request for

reinvestigation and duly notified him of the date when such reinvestigation would be held; only, neither taxpayer Oquias nor his counsel appeared on the given date. In all these cases, the request for reinvestigation of the assessment filed by the taxpayer was evidently granted and actual reinvestigation was conducted by the BIR, which eventually resulted in the issuance of an amended assessment. On the basis of these facts, this Court ruled in the same cases that the period between the request for reinvestigation and the revised assessment should be subtracted from the total prescriptive period for the assessment of the tax; and, once the assessment had been reconsidered at the taxpayer¶s instance, the period for collection should begin to run from the date of the reconsidered or modified assessment.40 The rulings of the foregoing cases do not apply to the present Petition because: (1) the protest filed by petitioner BPI was a request for reconsideration, not a reinvestigation, of the assessment against it; and (2) even granting that the protest of petitioner BPI was a request for reinvestigation, there was no showing that it was granted by respondent BIR Commissioner and that actual reinvestigation had been conducted. Going back to the administrative records of the present case, it would seem that the BIR, after receiving a copy of the protest letter of petitioner BPI on 17 November 1989, did not attempt to communicate at all with the latter until 10 September 1992, less than a month before the prescriptive period for collection on Assessment No. FAS-5-85-89-002054 was due to expire. There were internal communications, mostly indorsements of the docket of the case from one BIR division to another; but these hardly fall within the same sort of acts in the previously discussed cases that satisfactorily demonstrated the grant of the taxpayer¶s request for reinvestigation. Petitioner BPI, in the meantime, was left in the dark as to the status of its protest in the absence of any word from the BIR. Besides, in its letter to petitioner BPI, dated 10 September 1992, the BIR unwittingly admitted that it had not yet acted on the protest of the former ± This refers to your protest against and/or request for reconsideration of the assessment/s of this Office against you involving the amount of P28,020.00 under FAS-5-85-89-002054 dated October 23, 1989 as deficiency documentary stamp tax inclusive of compromise penalty for the year 1985. In this connection, it is requested that the enclosed waiver of the statute of limitations extending the period of collection of the said tax/es to December 31, 1993 be executed by you as a condition precedent of our giving due course to your protest«41 When the BIR stated in its letter, dated 10 September 1992, that the waiver of the statute of limitations on collection was a condition precedent to its giving due course to the request for reconsideration of petitioner BPI, then it was understood that the grant of such request for reconsideration was being held off until compliance with the given condition. When petitioner BPI failed to comply with the condition precedent, which was the execution of the waiver, the logical inference would be that the request was not granted and was not given due course at all. III The suspension of the statute of limitations on collection of the assessed deficiency DST from petitioner BPI does not find support in jurisprudence. It is the position of respondent BIR Commissioner, affirmed by the CTA and the Court of Appeals, that the three-year prescriptive period for collecting on Assessment No. FAS-5-85-89-002054 had not yet prescribed, because the said prescriptive period was suspended, invoking the case of Commissioner of Internal Revenue v. Wyeth Suaco Laboratories, Inc.42 It was in this case in which

this Court ruled that the prescriptive period provided by law to make a collection is interrupted once a taxpayer requests for reinvestigation or reconsideration of the assessment. Petitioner BPI, on the other hand, is requesting this Court to revisit the Wyeth Suaco case contending that it had unjustifiably expanded the grounds for suspending the prescriptive period for collection of national internal revenue taxes. This Court finds that although there is no compelling reason to abandon its decision in the Wyeth Suaco case, the said case cannot be applied to the particular facts of the Petition at bar. A. The only exception to the statute of limitations on collection of taxes, other than those already provided in the Tax Code, was recognized in the Suyoc case. As had been previously discussed herein, the statute of limitations on assessment and collection of national internal revenue taxes may be suspended if the taxpayer executes a valid waiver thereof, as provided in paragraphs (b) and (d) of Section 223 of the Tax Code of 1977, as amended; and in specific instances enumerated in Section 224 of the same Code, which include a request for reinvestigation granted by the BIR Commissioner. Outside of these statutory provisions, however, this Court also recognized one other exception to the statute of limitations on collection of taxes in the case of Collector of Internal Revenue v. Suyoc Consolidated Mining Co.43 In the said case, the Collector of Internal Revenue issued an assessment against taxpayer Suyoc Consolidated Mining Co. on 11 February 1947 for deficiency income tax for the taxable year 1941. Taxpayer Suyoc requested for at least a year within which to pay the amount assessed, but at the same time, reserving its right to question the correctness of the assessment before actual payment. The Collector granted taxpayer Suyoc an extension of only three months to pay the assessed tax. When taxpayer Suyoc failed to pay the assessed tax within the extended period, the Collector sent it a demand letter, dated 28 November 1950. Upon receipt of the demand letter, taxpayer Suyoc asked for a reinvestigation and reconsideration of the assessment, but the Collector denied the request. Taxpayer Suyoc reiterated its request for reconsideration on 25 April 1952, which was denied again by the Collector on 06 May 1953. Taxpayer Suyoc then appealed the denial to the Conference Staff. The Conference Staff heard the appeal from 02 September 1952 to 16 July 1955, and the negotiations resulted in the reduction of the assessment on 26 July 1955. It was the collection of the reduced assessment that was questioned before this Court for being enforced beyond the prescriptive period.44 In resolving the issue on prescription, this Court ratiocinated thus ± It is obvious from the foregoing that petitioner refrained from collecting the tax by distraint or levy or by proceeding in court within the 5-year period from the filing of the second amended final return due to the several requests of respondent for extension to which petitioner yielded to give it every opportunity to prove its claim regarding the correctness of the assessment. Because of such requests, several reinvestigations were made and a hearing was even held by the Conference Staff organized in the collection office to consider claims of such nature which, as the record shows, lasted for several months. After inducing petitioner to delay collection as he in fact did, it is most unfair for respondent to now take advantage of such desistance to elude his deficiency income tax liability to the prejudice of the Government invoking the technical ground of prescription. While we may agree with the Court of Tax Appeals that a mere request for reexamination or reinvestigation may not have the effect of suspending the running of the period of limitation for in such case there is need of a written agreement to extend the period between the Collector and the taxpayer, there are cases however where a taxpayer may be prevented from setting up the defense

of prescription even if he has not previously waived it in writing as when by his repeated requests or positive acts the Government has been, for good reasons, persuaded to postpone collection to make him feel that the demand was not unreasonable or that no harassment or injustice is meant by the Government. And when such situation comes to pass there are authorities that hold, based on weighty reasons, that such an attitude or behavior should not be countenanced if only to protect the interest of the Government.45 By the principle of estoppel, taxpayer Suyoc was not allowed to raise the defense of prescription against the efforts of the Government to collect the tax assessed against it. This Court adopted the following principle from American jurisprudence: "He who prevents a thing from being done may not avail himself of the nonperformance which he has himself occasioned, for the law says to him in effect µthis is your own act, and therefore you are not damnified.¶"46 In the Suyoc case, this Court expressly conceded that a mere request for reconsideration or reinvestigation of an assessment may not suspend the running of the statute of limitations. It affirmed the need for a waiver of the prescriptive period in order to effect suspension thereof. However, even without such waiver, the taxpayer may be estopped from raising the defense of prescription because by his repeated requests or positive acts, he had induced Government authorities to delay collection of the assessed tax. Based on the foregoing, petitioner BPI contends that the declaration made in the later case of Wyeth Suaco, that the statute of limitations on collection is suspended once the taxpayer files a request for reconsideration or reinvestigation, runs counter to the ruling made by this Court in the Suyoc case. B. Although this Court is not compelled to abandon its decision in the Wyeth Suaco case, it finds that Wyeth Suaco is not applicable to the Petition at bar because of the distinct facts involved herein. In the case of Wyeth Suaco, taxpayer Wyeth Suaco was assessed for failing to remit withholding taxes on royalties and dividend declarations, as well as, for deficiency sales tax. The BIR issued two assessments, dated 16 December 1974 and 17 December 1974, both received by taxpayer Wyeth Suaco on 19 December 1974. Taxpayer Wyeth Suaco, through its tax consultant, SGV & Co., sent to the BIR two letters, dated 17 January 1975 and 08 February 1975, protesting the assessments and requesting their cancellation or withdrawal on the ground that said assessments lacked factual or legal basis. On 12 September 1975, the BIR Commissioner advised taxpayer Wyeth Suaco to avail itself of the compromise settlement being offered under Letter of Instruction No. 308. Taxpayer Wyeth Suaco manifested its conformity to paying a compromise amount, but subject to certain conditions; though, apparently, the said compromise amount was never paid. On 10 December 1979, the BIR Commissioner rendered a decision reducing the assessment for deficiency withholding tax against taxpayer Wyeth Suaco, but maintaining the assessment for deficiency sales tax. It was at this point when taxpayer Wyeth Suaco brought its case before the CTA to enjoin the BIR from enforcing the assessments by reason of prescription. Although the CTA decided in favor of taxpayer Wyeth Suaco, it was reversed by this Court when the case was brought before it on appeal. According to the decision of this Court ± Settled is the rule that the prescriptive period provided by law to make a collection by distraint or levy or by a proceeding in court is interrupted once a taxpayer requests for reinvestigation or reconsideration of the assessment. . . ...

Although the protest letters prepared by SGV & Co. in behalf of private respondent did not categorically state or use the words "reinvestigation" and "reconsideration," the same are to be treated as letters of reinvestigation and reconsideration« These letters of Wyeth Suaco interrupted the running of the five-year prescriptive period to collect the deficiency taxes. The Bureau of Internal Revenue, after having reviewed the records of Wyeth Suaco, in accordance with its request for reinvestigation, rendered a final assessment« It was only upon receipt by Wyeth Suaco of this final assessment that the five-year prescriptive period started to run again.47 The foremost criticism of petitioner BPI of the Wyeth Suaco decision is directed at the statement made therein that, "settled is the rule that the prescriptive period provided by law to make a collection by distraint or levy or by a proceeding in court is interrupted once a taxpayer requests for reinvestigation or reconsideration of the assessment."48 It would seem that both petitioner BPI and respondent BIR Commissioner, as well as, the CTA and Court of Appeals, take the statement to mean that the filing alone of the request for reconsideration or reinvestigation can already interrupt or suspend the running of the prescriptive period on collection. This Court therefore takes this opportunity to clarify and qualify this statement made in the Wyeth Suaco case. While it is true that, by itself, such statement would appear to be a generalization of the exceptions to the statute of limitations on collection, it is best interpreted in consideration of the particular facts of the Wyeth Suaco case and previous jurisprudence. The Wyeth Suaco case cannot be in conflict with the Suyoc case because there are substantial differences in the factual backgrounds of the two cases. The Suyoc case refers to a situation where there were repeated requests or positive acts performed by the taxpayer that convinced the BIR to delay collection of the assessed tax. This Court pronounced therein that the repeated requests or positive acts of the taxpayer prevented or estopped it from setting up the defense of prescription against the Government when the latter attempted to collect the assessed tax. In the Wyeth Suaco case, taxpayer Wyeth Suaco filed a request for reinvestigation, which was apparently granted by the BIR and, consequently, the prescriptive period was indeed suspended as provided under Section 224 of the Tax Code of 1977, as amended.49 To reiterate, Section 224 of the Tax Code of 1977, as amended, identifies specific circumstances when the statute of limitations on assessment and collection may be interrupted or suspended, among which is a request for reinvestigation that is granted by the BIR Commissioner. The act of filing a request for reinvestigation alone does not suspend the period; such request must be granted.50 The grant need not be express, but may be implied from the acts of the BIR Commissioner or authorized BIR officials in response to the request for reinvestigation.51 This Court found in the Wyeth Suaco case that the BIR actually conducted a reinvestigation, in accordance with the request of the taxpayer Wyeth Suaco, which resulted in the reduction of the assessment originally issued against it. Taxpayer Wyeth Suaco was also aware that its request for reinvestigation was granted, as written by its Finance Manager in a letter dated 01 July 1975, addressed to the Chief of the Tax Accounts Division, wherein he admitted that, "[a]s we understand, the matter is now undergoing review and consideration by your Manufacturing Audit Division«" The statute of limitations on collection, then, started to run only upon the issuance and release of the reduced assessment. The Wyeth Suaco case, therefore, is correct in declaring that the prescriptive period for collection is interrupted or suspended when the taxpayer files a request for reinvestigation, provided that, as clarified and qualified herein, such request is granted by the BIR Commissioner.

Thus, this Court finds no compelling reason to abandon its decision in the Wyeth Suaco case. It also now rules that the said case is not applicable to the Petition at bar because of the distinct facts involved herein. As already heretofore determined by this Court, the protest filed by petitioner BPI was a request for reconsideration, which merely required a review of existing evidence and the legal basis for the assessment. Respondent BIR Commissioner did not require, neither did petitioner BPI offer, additional evidence on the matter. After petitioner BPI filed its request for reconsideration, there was no other communication between it and respondent BIR Commissioner or any of the authorized representatives of the latter. There was no showing that petitioner BPI was informed or aware that its request for reconsideration was granted or acted upon by the BIR. IV Conclusion To summarize all the foregoing discussion, this Court lays down the following rules on the exceptions to the statute of limitations on collection. The statute of limitations on collection may only be interrupted or suspended by a valid waiver executed in accordance with paragraph (d) of Section 223 of the Tax Code of 1977, as amended, and the existence of the circumstances enumerated in Section 224 of the same Code, which include a request for reinvestigation granted by the BIR Commissioner. Even when the request for reconsideration or reinvestigation is not accompanied by a valid waiver or there is no request for reinvestigation that had been granted by the BIR Commissioner, the taxpayer may still be held in estoppel and be prevented from setting up the defense of prescription of the statute of limitations on collection when, by his own repeated requests or positive acts, the Government had been, for good reasons, persuaded to postpone collection to make the taxpayer feel that the demand is not unreasonable or that no harassment or injustice is meant by the Government, as laid down by this Court in the Suyoc case. Applying the given rules to the present Petition, this Court finds that ± (a) The statute of limitations for collection of the deficiency DST in Assessment No. FAS-5-85-89002054, issued against petitioner BPI, had already expired; and (b) None of the conditions and requirements for exception from the statute of limitations on collection exists herein: Petitioner BPI did not execute any waiver of the prescriptive period on collection as mandated by paragraph (d) of Section 223 of the Tax Code of 1977, as amended; the protest filed by petitioner BPI was a request for reconsideration, not a request for reinvestigation that was granted by respondent BIR Commissioner which could have suspended the prescriptive period for collection under Section 224 of the Tax Code of 1977, as amended; and, petitioner BPI, other than filing a request for reconsideration of Assessment No. FAS-5-85-89-002054, did not make repeated requests or performed positive acts that could have persuaded the respondent BIR Commissioner to delay collection, and that would have prevented or estopped petitioner BPI from setting up the defense of prescription against collection of the tax assessed, as required in the Suyoc case. This is a simple case wherein respondent BIR Commissioner and other BIR officials failed to act promptly in resolving and denying the request for reconsideration filed by petitioner BPI and in enforcing collection on the assessment. They presented no reason or explanation as to why it took them almost eight years to address the protest of petitioner BPI. The statute on limitations imposed by the Tax Code precisely intends to protect the taxpayer from such prolonged and unreasonable assessment and investigation by the BIR.

Considering that the right of the respondent BIR Commissioner to collect from petitioner BPI the deficiency DST in Assessment No. FAS-5-85-89-002054 had already prescribed, then, there is no more need for this Court to make a determination on the validity and correctness of the said Assessment for the latter would only be unenforceable. Wherefore, based on the foregoing, the instant Petition is GRANTED. The Decision of the Court of Appeals in CA-G.R. SP No. 51271, dated 11 August 1999, which reinstated Assessment No. FAS-585-89-002054 requiring petitioner BPI to pay the amount of P28,020.00 as deficiency documentary stamp tax for the taxable year 1985, inclusive of the compromise penalty, is REVERSED and SET ASIDE. Assessment No. FAS-5-85-89-002054 is hereby ordered CANCELED. SO ORDERED. MINITA V. CHICO-NAZARIO Associate Justice WE CONCUR: REYNATO S. PUNO Associate Justice Chairman MA. ALICIA AUSTRIA-MARTINEZ Associate Justice ROMEO J. CALLEJO, SR. Associate Justice

DANTE O. TINGA Associate Justice ATTESTATION I attest that the conclusions in the above Decision were reached in consultation before the case was assigned to the writer of the opinion of the Court¶s Division. REYNATO S. PUNO Associate Justice Chairman, Second Division CERTIFICATION

Pursuant to Article VIII, Section 13 of the Constitution, and the Division Chairman¶s Attestation, it is hereby certified that the conclusions in the above Decision were reached in consultation before the case was assigned to the writer of the opinion of the Court¶s Division. HILARIO G. DAVIDE, JR. Chief Justice

Sponsor Documents

Or use your account on DocShare.tips

Hide

Forgot your password?

Or register your new account on DocShare.tips

Hide

Lost your password? Please enter your email address. You will receive a link to create a new password.

Back to log-in

Close